Vous êtes sur la page 1sur 781

ÍNDICE GENERAL DE PREGUNTAS

No DE
TEMA
PREGUNTAS

GENÉTICA 25

EPIDEMIOLOGÍA DEL CÁNCER 45

TUMORES GINECOLÓGICOS
527
1 - 100 | 101 - 200 | 201 - 300 | 301 - 400 | 401 - 500 | 501 - 527

TUMORES MAMARIOS
209
1-100 | 101-209

TUMORES DIGESTIVOS
166
1-100 | 101-166

LINFOMAS Y LEUCEMIAS
128
1-64 | 65-128

TUMORES UROLÓGICOS 110

TUMORES DE CABEZA Y CUELLO


340
1-100 | 101-200 | 201-300 | 301-340

TUMORES DE TEJIDOS BLANDOS Y ÓSEOS


142
1-70 | 71-142

TUMORES MIXTOS 71

TUMORES DE TÓRAX 40

TUMORES DE PIEL, ANEXOS Y MELANOMA 77

CIRUGÍA RECONSTRUCTIVA ONCOLÓGICA 17

TUMORES INFANTILES
328
1-100 | 101-200 | 201-300 | 301-328

RADIOBIOLOGÍA ONCOLÓGICA 34

ONCOLOGÍA MÉDICA
297
1-100 | 101-201 | 202-297

CASOS CLÍNICOS ONCOLOGÍA QUIRÚRGICA 114


Casos 1 al 40 | 41 al 80 | 81 al 114 cc

7
CASOS CLÍNICOS ONCOLOGÍA MÉDICA
cc
2
CASOS CLÍNICOS ONCOLOGÍA PEDIÁTRICA
cc

GENÉTICA

GE1. Neoplasia que presentan frecuentemente los pacientes con síndrome de Down:

a) Retinoblastoma

b) Tumor de Wilms

c) Leucemia linfoblástica aguda

d) Leucemia granulocítica crónica

e) Linfoma de Hodgkin

GE2. La alteración cromosómica siguiente tiene alta incidencia de presentación de gonadoblastoma:

a) 45,X/46,XY (mosaico de Turner)

b) Deleción de 18p

c) Trisomía 8

d) Monosomía 22

e) Deleción 5p (síndrome de cri-du-chat)

GE3. Los productos de los genes BRCA1 y BRCA2 participan en el proceso de reparación, además:

a) Se encargan de la fosforilación de p53

b) Son reguladores de la transcripción de otros genes

c) Se expresan principalmente durante la apoptosis

d) Inducen la formación de dímeros de timina

e) Realizan fotorreactivación
GE4. Las mutaciones en los genes BRCA1 y BRCA2 se asocian a alteraciones en:

a) p21

b) abl

c) ATM

d) Rb

e) P53

GE5. Son funciones de los oncogenes las siguientes, excepto:

a) Proliferación

b) Diferenciación

c) Apoptosis

d) Inhibición de la proliferación celular

e) Factores de crecimiento

GE6. La característica más importante de los genes supresores tumorales es que:

a) Sus productos inhiben la proliferación celular o facilitan la muerte celular y su inactivación lleva
a la formación de tumor

b) Son factores de crecimiento

c) Tienen la capacidad de auto amplificarse

d) Son moléculas de traducción de señales intracelulares

e) Se autofosforilan

GE7. Para que un gen supresor tumoral lleve a cabo la tumorigénesis en individuos con mutaciones
germinales, hace falta:

a) Activación de un protooncogén

b) Inactivación del alelo nativo residual

c) Interacción con reguladores de transcripción

d) Expresión de Rb

e) Mutación somática de ATM

GE8. Los osteomas de mandíbula ocurren como parte del Síndrome de:
a) Li- Fraumeni

b) Bloom

c) Gardner

d) Down

e) Lynch

GE9. Las siguientes son características de tumores de mama relacionados con mutaciones germinales de
BRCA1, excepto:

a) Exceso de histopatología medular

b) Alto grado histológico

c) Receptores hormonales negativos

d) Receptores hormonales positivos

e) Alta frecuencia de mutaciones en p53

GE10. El síndrome de Peutz-Jeghers presenta:

a) Pigmentación mucocutánea y poliposis gastrointestinal

b) Forma de herencia autosómica recesiva

c) Quistes epidermoides y sebáceos

d) Fibromatosis del mesenterio

e) Osteomas de cráneo, mandíbula y huesos largos

GE11. El cáncer gástrico familiar se asocia con mutaciones en el gen:

a) RET

b) NF-1

c) E-caderina/CDH1

d) PTEN

e) C-jun

GE12. El modelo de Gail se utiliza para:

a) Valorar una mastografía


b) Estimar la probabilidad de que una mujer con factores de riesgo pueda desarrollar cáncer de
mama

c) Estimar el riesgo de padecer síndrome de Li-Fraumeni

d) Valorar el uso de radioterapia en cáncer de mama

e) Estimar el riesgo de metástasis en el cáncer de mama

GE13. Se sospecha de cáncer familiar cuando existe el mismo cáncer en dos o más familiares, cáncer
bilateral en órganos pares, múltiples primarios, edad temprana de presentación así como:

a) Individuos que además del cáncer presenten dismorfias y un patrón de herencia mendeliano

b) Forma más agresiva del tumor

c) Familiares de primer grado con Síndrome de Li-Fraumeni

d) Múltiples metástasis

e) Afección orgánica múltiple

GE14. El gen responsable de la enfermedad de Cowden es:

a) BRCA2

b) MLH1

c) PTEN

d) P53

e) PMS2

GE15. El siguiente es un síndrome de inestabilidad cromosómica de predisposición a cáncer

a) Cowden

b) Gardner

c) Bloom

d) Li-Fraumeni

e) Menkes
GE16. Mutaciones en el gen de Ataxia-Telangiectasia (ATM) causa un síndrome en homocigotos y a los
heterocigotos los predispone a:

a) Retinoblastoma, alteraciones inmunológicas y anemia

b) Sarcomas en adolescencia y acortamiento de miembros inferiores

c) Infecciones frecuentes, hiperpigmentación y osteosarcomas

d) Ataxia, ceguera y leucemia

e) Cáncer, enfermedad isquémica cardiaca y mortalidad temprana

GE17. La proteína químerica que resulta de la translocación entre los cromosomas (9;22) tiene una actividad
de:

a) Topoisomerasa

b) Polimerasa

c) Tirosin-cinasa

d) Ciclina

e) Cisteína

GE18. El siguiente es un tumor maligno particularmente agresivo y frecuentemente fatal en la


neurofibromatosis I:

a) Hamartoma de iris

b) Neurofibromas plexiformas

c) Manchas café con leche

d) Nódulos de Lish

e) Neurofibrosarcoma

GE19. Como criterios para diagnosticar síndrome de Li-Fraumeni están: Sarcoma óseo o de tejidos blandos
diagnosticado antes de los 45 años en el individuo que se designa como caso índice, un familiar de primer
grado del caso índice con cáncer diagnosticado antes de los 45 años y

a) Dos o más familiares de segundo grado con cáncer de mama antes de los 40 años

b) Un familiar de primer o segundo grado del caso índice en la misma generación con cáncer
diagnosticado antes de los 45 años o con sarcoma diagnosticado a cualquier edad
c) Un familiar de cualquier grado con cáncer de sistema nervioso central diagnosticado antes de
los 45 años

d) Dos o más familiares con sarcoma de partes blandas diagnosticado a cualquier edad

e) Tres o más familiares de segundo y tercer grado con diagnóstico de cáncer renal antes de los
45 años

GE20. El síndrome de Von Hippel-Lindau tipo II se diferencia del tipo I por presentar:

a) Carcinoma renal

b) Osteosarcoma

c) Neurofibromas

d) Retraso mental

e) Feocromocitoma

GE21. Mutaciones en el gen ret se involucran en la forma hereditaria de:

a) Carcinoma pancreático

b) Carcinoma renal

c) Cáncer de ovario

d) Feocromocitoma aislado

e) Neoplasia endocrina múltiple tipo II

GE22. La siguiente es una alteración citogenética que se observa en más de la mitad de las veces en células
de melanoma:

a) Deleción de 9p21

b) Duplicación de 11q13

c) Pérdida de heterocigocidad de 17q

d) Deleción de 4p12

e) Inversión de 16p11

GE23. El STI571 funciona como:


a) Activador de fosforilación

b) Inhibidor de actividad tirosin-cinasa

c) Coactivador celular selectivo

d) Señal intracelular de fosforilación

e) Inhibidor del oncogén ras

GE24. Además del cuadro clínico, el síndrome de Bloom se diagnostica mediante:

a) Serie ósea completa

b) Alteraciones en la biometría hemática

c) Hiperdiploidias

d) Presencia de dobles minutas en el cariotipo

e) Aumento de intercambio de cromátides hermanas

GE25. La anemia de Fanconi es una enfermedad autosómica recesiva que tiene alta predisposición a
malignidad, especialmente a:

a) Linfoma no Hodgkin

b) Cáncer de colon

c) Carcinoma basocelular

d) Melanoma

e) Leucemia mielocítica aguda

EPIDEMIOLOGÍA DEL CÁNCER

EP1. Desde el punto de vista epidemiológico ¿Qué lugar ocupa mundialmente el Ca Cu?

a) Primero

b) Segundo

c) Tercero

d) Cuarto
e) Quinto

EP2. Las investigaciones epidemiológicas actuales sugieren como agente de transmisión en el Ca Cu a un


virus del tipo:

a) Herpes simple tipo II

b) Herpes simple tipo I

c) Del papiloma humano

d) Epstein-Barr

e) HIV

EP3. En los países en desarrollo el cáncer más frecuente en la mujer es:

a) El de mama

b) El de pulmón

c) El de ovario

d) Los linfomas

e) El cérvicouterino

EP4. El síndrome de esclerosis tuberosa se caracteriza por asociarse con:

a) Sarcoma de células clara de riñón

b) Astrocitomas de células gigantes

c) Sarcoma de partes blandas

d) Meduloblastoma

e) Carcinoma de colon

EP5. Metodología utilizada en el análisis del DNA:

a) Northern blot

b) Western blot

c) Southern blot

d) RT-PCR

e) Inmunohistoquímica
EP6. Evento que distingue a los genes supresores:

a) Apoptosis

b) Pérdida de heterocigocidad

c) Recombinación mitótica

d) Ninguno de los anteriores

e) Todos los anteriores

EP7. Factor promotor de la fase de mitosis:

a) Complejo DNA-DNA polimerasa

b) La ciclina de la mitosis

c) Una ciclina y la ciclina dependiente de cinasa

d) La maquinaria de reparación de DNA

e) Nutricional

EP8. El factor promotor de la mitosis debe estar presente en mayor concentración:

a) Durante todo el ciclo celular

b) En las fases G1 y M

c) En la fase M

d) En la interfase

e) En la fase G1 tardía

EP9. La citometría de flujo permite cuantificar los cambios del RNA nuclear que acompañan a la
transformación neoplásica y que se interpretan en forma de grados establecidos; por ejemplo, un sarcoma de
alto grado de malignidad se expresa como:

a) Diploide

b) Tetraploide

c) Aneuploide

EP10. Medida epidemiológica para conocer el impacto del cáncer:

a) Frecuencia

b) Incidencia y mortalidad
c) Morbilidad y mortalidad

d) Sobrevida a cinco años

e) Incidencia, mortalidad y sobrevida

EP11. En los países desarrollados la incidencia y mortalidad del Ca Cu disminuyeron debido a:

a) El uso masivo de la colposcopía

b) El uso masivo del condón

c) La toma de biopsia de manera temprana

d) Citología cérvicovaginal realizada a todas las mujeres de alto riesgo

e) El uso de la histerectomía

EP12. ¿Cuál de las neoplasias siguientes tienen un factor hereditario conocido?

a) Leucemia aguda y linfoma no Hodgkin

b) Leucemia aguda linfoblástica y no linfoblástica

c) Enfermedad de Hodgkin y neuroblastoma

d) Tumor de Wilms y retinoblastoma

e) Sarcoma de Ewing y rabdomiosarcoma

EP13. La teoría de Knudson y herencia del cáncer se describió en:

a) Rabdomiosarcoma

b) Neuroblastoma

c) Granuloma letal de la línea media

d) Histiocitosis de las células de Langerhans

e) Retinoblastoma

EP14. La hipótesis del cáncer hereditario según la teoría de Knudson se basa en:

a) Doble mutación

b) Alteración autosómica dominante

c) Alteración ligada al cromosoma sexual

d) Alteración de penetrancia variable


e) Alteración fenotípica

EP15. Las entidades que predisponen la formación del tumor de Wilms son las siguientes, excepto el (la):

a) Síndrome de WAGR

b) Síndrome de Denys-Drash

c) Hemihipertrofia

d) Síndrome de Beckwith-Wiedeman

e) Síndrome de Poland

EP16. El riesgo de desarrollar leucemia en un niño con síndrome de down es uno de cada:

a) 150

b) 2880

c) 95

d) 10

e) 3500

EP17. El gen p53:

a) Es un gen facilitador tumoral

b) Es un gen supresor tumoral

c) Es un gen inhibidor de la enfermedad de injerto contra huesped

d) Denota una translocación

e) Denota efecto de citotoxicidad

EP18. El Li-Fraumeni es un síndrome:

a) Paraneoplásico

b) Premaligno

c) Dismielopoyético

d) De cáncer familiar

e) Preleucémico
EP19. En casos de retinoblastoma hereditario la alteración cromosómica común es:

a) d 13q14

b) d 11q21

c) t 9:22

d) t 8:14

e) t 8:2

EP20. El gen RTB1 del retinoblastoma se encuentra en el locus del cromosoma:

a) 16

b) 19

c) 21

d) 22

e) 13

EP21. La asociación de neurofibromatosis tipo 1 y cáncer es más frecuente en casos de:

a) Sarcoma de Ewing

b) Linfoma no Hodgkin

c) Meduloblastoma

d) Tumor de Wilms

e) Glioma óptico

EP22. La asociación de neurofibromatosis tipo 2 y cáncer es más frecuente en pacientes con:

a) Meduloblastoma

b) Ependimoma

c) Astrocitoma

d) Meningioma

e) Glioma

EP23. El síndrome de Von Hippel-Lindau se asocia frecuentemente con:

a) Carcinoma de células claras de riñón


b) Sarcoma de partes blandas

c) Carcinoma de tiroides

d) Carcinoma de colon

e) Osteosarcoma

EP24. La exposición a radiación ionizante puede inducir las neoplasias siguientes, excepto:

a) Leucemia aguda

b) Leucemia mielocítica crónica

c) Carcinoma de tiroides

d) Carcinoma de glándulas salivales

e) Histiocitosis de células de Langerhans

EP25. La exposición prolongada a asbesto condiciona:

a) Leucemia aguda no linfoblástica

b) Carcinoma hepático

c) Carcinoma de colon

d) Mesotelioma

e) Carcinoma de tiroides

EP26. La ingestión de dietilestilbestrol durante el embarazo induce en el producto del sexo femenino:

a) Adenocarcinoma de vagina

b) Carcinoma gástrico

c) Carcinoma de colon

d) Adenocarcinoma de mama

e) Adenocarcinoma suprarrenal

EP27. La ingestión de andrógenos anabólicos esteroideos puede inducir:

a) Leucemia aguda linfoblástica

b) Leucemia linfocítica crónica

c) Carcinoma hepático
d) Enfermedad de Hodgkin

e) Carcinoma de colon

EP28. El carcinoma hepatocelular puede estar inducido por:

a) Traumatismo

b) Infección por virus de la hepatitis B

c) Infección por adenovirus

d) Rubéola

e) Infección por el virus de la varicela-zoster

EP29. El riesgo de adquirir cáncer está aumentado en pacientes con:

a) Trasplante de órganos

b) Síndrome de Wiskott-Aldrich

c) Ataxia telangiectasia

d) Tratamiento previo con alquilantes

e) Todo lo enunciado

EP30. La alfafetoproteína está presente en las situaciones siguientes, excepto:

a) Seminoma

b) Carcinoma embrionario de testículo

c) Teratoma inmaduro de ovario

d) Hepatoblastoma

e) Hepatitis

EP31. El mecanismo farmacológico caracterizado por rompimiento de los enlaces de ADN se observa con:

a) Prednisona

b) Dexametasona

c) Vincristina

d) Metotrexate

e) Etopósido
EP32. El índice de ADN es un reflejo de:

a) Mitosis

b) Fase S

c) Fase G0

d) Fase G1

e) Fase G2

EP33. Mecanismo mediante el cual un protooncogén se transforma en oncogén:

a) Pérdida de heterocigocidad

b) Mutación con pérdida de función

c) Deleción

d) Ganancia de homocigocidad

e) Mutación que genera sobreexpresión

EP34. ¿Qué efecto puede tener una mutación sobre la expresión de un oncogén?

a) Pérdida de función

b) Sobreexpresión

c) Regulación positiva

d) Regulación negativa

e) Ninguna de las anteriores

EP35. Función de los protooncogenes:

a) Receptores

b) Transductores de señales

c) Factores de transcripción

d) Todas las anteriores

e) Ninguna de las anteriores

EP36. Según su función dentro del ciclo celular ¿cuál es la diferencia entre un gen supresor y un oncogén?
a) Un gen supresor es la forma normal de un oncogén

b) Los genes supresores promueven la recombinación mitótica y los oncogenes la división


mitótica

c) Un gen supresor puede dirigir la célula a apoptosis y un oncogén a la diferenciación celular

d) Un oncogén promueve la división celular y un gen supresor la inhibe

e) Ninguna de las anteriores

EP37. De qué manera la proteína pRB regula negativamente el ciclo celular:

a) pRB forma complejos activos con el factor E2F que promueven la expresión de p53 para
conducir a la célula a apoptosis

b) pRB forma complejos inactivos con el factor E2F evitando la expresión de genes cuya función
es promover la división celular

c) pRB se une al factor E2F y reconocen secuencias de genes como c-myc, c-fos

d) c-Jun evitando su expresión

e) Ninguna de las anteriores

EP38. ¿Qué funciones son características de p53?

a) Conducir al proceso de apoptosis, pérdida de heterocigocidad, deleciones de gran tamaño

b) Conducir al proceso de apoptosis, pérdida de heterocigocidad, deleciones de tipo puntual

c) Conducir al proceso de apoptosis, regular el ciclo celular de manera negativa, promover


mecanismos de reparación

d) Conducir al proceso de apoptosis, regular el ciclo celular de manera negativa, promover


mecanismos de recombinación

e) Ninguna de las anteriores

EP39. ¿Qué efecto puede tener una mutación sobre la expresión de un gen supresor?

a) Pérdida de la función reguladora

b) Activación de los mecanismos de reparación

c) Expresión y división celular sin control

d) Ninguno de los anteriores

e) Todos los anteriores


EP40. En esta fase del ciclo celular se requiere que la célula adquiera mayor tamaño, que el ambiente sea
favorable y que la maquinaria de síntesis de DNA esté completa:

a) G0 b) G2 c) G1 tardía d) S e) M

EP41. En esta fase la célula permanece quiescente porque el ambiente no reúne las condiciones adecuadas,
el metabolismo celular es mínimo:

a) G0 b) G2 c) G1 tardía d) S e) M

EP42. Para iniciar la fase de mitosis se requiere de:

a) Ciclina

b) Ribonucleótido reductasa

c) p53

d) Nada de lo anterior

e) Todo lo anterior

EP43. ¿Cuál de los complejos siguientes regula negativamente al ciclo celular?

a) Cinasa dependiente de ciclina

b) Myc, Fos, Jun

c) Fosfatasas y proteínas inactivadoras del complejo de la ciclina

d) Myc

e) Ninguno de los enunciados

EP44. El proceso de muerte celular programada (apoptosis) es un proceso normal y se produce cuando la
célula:

a) Duplica su DNA

b) Tiene su DNA dañado y no reparado

c) Reparó su DNA

d) Está en la fase G2-M

e) Está en fase G1 temprana

EP45. Las alteraciones numéricas que presentan las células cancerosas se pueden producir durante:
a) La fase S

b) La mala reparación de la molécula de DNA después de fase S

c) La fase M

d) La pérdida de los telómeros

e) El inicio del ciclo celular

TUMORES GINECOLÓGICOS

GN1. Histopatológicamente el cáncer más frecuente del cérvix es el:

a) Adenocarcinoma

b) Adenoacantoma

c) Sarcoma

d) Epidermoide

e) Linfomas

GN2. Cuántas veces aumenta el riesgo de desarrollar Ca Cu, por arriba de lo normal, en mujeres con más de
diez parejas sexuales:

a) Cuatro veces

b) Tres veces

c) Cinco veces

d) Diez veces

e) Dos veces

GN3. En la actualidad existen reportes de que el VPH se encuentra asociado al Ca Cu en un:

a) 50%

b) 70%

c) 30-40%

d) 80-100%

e) 75%
GN4. Los subtipos de VPH que se encuentran con más frecuencia en lesiones premalignas o malignas son:

a) 6 y 11

b) 11 y 16

c) 16,18,31,33 y 35

d) 16, 18, 43 y 45

e) 62, 63 y 64

GN5. El Ca Cu invasor confinado al cérvix con lesión menor o igual a 4 cm corresponde a una etapa clínica:

a) Ib

b) Ia

c) Ia1

d) Ia2

e) Ib1

GN6. Lugar que ocupa el cáncer de ovario como causa de muerte entre los cánceres del tracto genital en la
mujer norteamericana:

a) 1°

b) 2º

c) 3°

d) 4º

e) 5º

GN7. Frecuencia porcentual en México del cáncer de ovario entre las neoplasias del tracto genital:

a) 80

b) 60

c) 40

d) 20

e) 6
Correlacione las columnas siguientes en relación al lugar que ocupa cada uno de los subtipos
histopatológicos del cáncer de ovario (en orden de importancia).

GN08. Seroso papilar (a) a) 1er. lugar

GN09. Células claras (d) b) 2º lugar

GN10. Mucinoso (b) c) 3er. lugar

GN11. Endometroide (c) d) 4º lugar

GN12. T. de Brener (e) e) 5º lugar

GN13. El riesgo para desarrollar cáncer de ovario cuando dos familiares de primera línea han padecido la
enfermedad es de:

a) 2%

b) 10%

c) 20%

d) 40%

e) 50%

GN14. Marcador para tumor de senos endodérmicos:

a) HGC

b) ACE

c) AFP

d) Estradiol

GN15. ¿A qué edad es más frecuente el tumor epitelial de ovario?

a) 10 años

b) 15 años

c) 20 años

d) 50 años

GN16. El marcador CA-125 es útil en:

a) El carcinoma epitelial de ovario

b) El carcinoma de colon
c) El cáncer gástrico

d) Ninguno de los tumores antes mencionados

GN17. A partir de qué etapa clínica y grado histológico de cáncer endometrial se debe dar tratamiento
complementario con radioterapia:

a) Ia G1

b) Ia G2

c) Ia G3

d) Ib G1

e) Ninguna

GN18. Edad en años más frecuente de presentación del carcinoma de endometrio:

a) 30

b) 60

c) 50

d) 10

e) 80

Enumere en relación a la frecuencia el lugar que ocupan los cánceres del tracto genital en los Estados
Unidos y México:

Estados Unidos México

__GN19. Cáncer cervico uterino ( ) GN23. Cáncer cervico uterino( )

__GN20. Cáncer del endometrio ( ) GN24. Cáncer del endometrio( )

__GN21. Cáncer de vulva () GN25. Cáncer de vulva ()

__GN22. Cáncer de ovario () GN26. Cáncer de ovario ()

GN27. La sobrevida a cinco años libre de enfermedad para el cáncer de endometrio etapa Ia es del:

a) 100%

b) 90%

c) 75%

d) 60%
e) 50%

GN28. La sobrevida a cinco años del Ca de vulva tratado con cirugía es del:

a) 100%

b) 50%

c) 80%

d) 67%

e) 90%

GN29. Según publicaciones a nivel nacional, la cifra anual de muertes por Ca Cu asciende aproximadamente
a:

a) 4500

b) 2500

c) 6000

d) 10000

e) 20000

GN30. El registro histopatológico de tumores señala que el Ca Cu cuenta con el porciento global siguiente,
dentro de la totalidad de neoplasias que incluye dicho registro:

a) 60%

b) 20%

c) 30%

d) 40%

e) 10%

GN31. ¿Cuál es el porciento de tumores endometroides de ovario asociados con carcinomas del endometrio?

a) 10

b) 25

c) 40

d) 50

e) 15
GN32. El carcinoma verrucoso de la vagina es una variante de:

a) Carcinoma epidermoide bien diferenciado

b) Carcinoma epidermoide moderadamente diferenciado

c) Carcinoma epidermoide poco diferenciado

d) Adenocarcinoma

e) Carcinoma de células basales

GN33. El marcador alfafetoproteína se encuentra elevado en casos de:

a) Adenocarcinoma

b) Coriocarcinoma

c) Senos endodérmicos

d) Disgerminoma

e) Gonadoblastoma

GN34. ¿Qué elementos contiene un ginandroblastoma?

a) Disgerminoma y Sertoli-Leydig

b) Disgerminoma y células granulosa

c) Sertoli-Leydig y células granulosa

d) Coriocarcinoma y disgerminoma

e) Disgerminoma y senos endodérmicos

GN35. La principal vía de diseminación del Ca de endometrio es:

a) Transcelómica

b) Por contiguidad

c) Hematógena

d) Linfática

e) A través de las trompas de falopio

GN36. La variedad histológica más frecuente del Ca de endometrio es:


a) Adenocarcinoma papilar

b) Adenocarcinoma escamoso

c) Carcinoma indiferenciado

d) Adenocarcinoma seroso

e) Adenocarcinoma endometroide

GN37. Un tumor endometrial que invade el estroma endocervical se clasifica de acuerdo a la FIGO en:

a) IIb

b) Ic

c) IIIa

d) IIa

e) IIIb

GN38. Marcador tumoral útil en Ca de endometrio:

a) CA-125

b) CA 19-9

c) Antígeno carcinoembrionario

d) DHL

e) CA 15:3

GN39. Región cromosómica asociada a neoplasias malignas epiteliales del ovario:

a) 13 q

b) 11 p

c) 9 p

d) 17 q

e) Ninguna de las anteriores

GN40. Factores para desarrollo de cáncer de ovario, excepto:

a) Nuliparidad

b) Embarazo
c) Hormonal

d) Medio ambiente

e) Genéticos

GN41. ¿Cuáles son los factores de riesgo para tener cáncer de endometrio más significativos (riesgo
relativo)?

a) Obesidad, hiperestrogenismo, nuliparidad, uso de tamoxifén tiempo prolongado

b) Hipertensión, nuliparidad

c) Diabetes

d) Ca de mama y de ovario

e) Todos

En cáncer de endometrio, mencione tres tipos histológicos de mal pronóstico:

GN42. Células claras____________________

GN43. Seroso papilar____________________

GN44. Adenoescamoso___________________

GN45. Marcador para coriocarcinoma:

a) Alfafetoproteína

b) ACE

c) CA 125

d) Fracción beta de GCH

e) Deshidrogenasa láctica

GN46. Señale los tipos de virus de papiloma humano considerados de alto riesgo para el desarrollo de
cáncer de cuello uterino:

a) 31 y 33

b) 35 y 23

c) 18 y 16

d) 6 y 11

e) 52 y 58
GN47. Factor(es) pronóstico(s) en cáncer de endometrio etapa I en que se justifica radioterapia:

a) Ic

b) Los de histología desfavorable

c) G3

d) Todos los anteriores

e) Sólo a y b

GN48. La laparoscopía en el Ca Cu es:

a) Útil para etapificación

b) Cambia la etapificación

c) Confirma el diagnóstico

d) Sirve para conocer el pronóstico

e) Aún no está aceptado por la FIGO

GN49. El porciento de cáncer epidermoide del cérvix es de:

a) 100%

b) 80%

c) 85-90%

d) Menos del 80%

e) 50-70%

GN50. Datos clínicos del Ca Cu invasor que indican infiltración tumoral a pared pélvica:

a) Dolor pélvico y sangrado

b) Dolor pélvico, sangrado y leucorrea

c) Dolor pélvico, edema de una pierna e hidronefrosis (Tríada)

d) Dolor pélvico, pérdida de peso y sangrado

e) Edema de ambas piernas, sangrado y fístula vesical

GN51. La sobrevida a cinco años del cáncer ovárico temprano tratado adecuadamente es del:

a) 100%
b) 90%

c) 70%

d) 60%

e) 50%

GN52. La presencia de metástasis ganglionares paraórticas clasifica un cáncer ovárico en etapa:

a) IIc

b) IVb

c) IIIb

d) IIIc

e) IVa

GN53. Cáncer ovárico bilateral más células malignas en lavado peritoneal clasifica un cáncer ovárico en
etapa:

a) Ib

b) Ic

c) IIb

d) IIc

e) IIIa

GN54. Tumor germinal de ovario, considerado de historia natural favorable:

a) Senos endodérmicos

b) Germinal mixto

c) Carcinoma embrionario

d) Teratoma inmaduro

GN55. Proporción de pacientes con involucro de ganglios linfáticos en etapa Ib1 en el carcinoma de cérvix:

a) 5-10%

b) 10-15%

c) 27-45%
d) 20-25%

GN56. Incidencia de ganglios paraórticos metastásicos en etapa Ib1 de carcinoma de cérvix:

a) 2%

b) 20%

c) 6%

d) 35%

GN57. Localización anatómica más frecuente del carcinoma invasor de vulva:

a) Labios mayores

b) Labios menores

c) Clítoris

d) Perineo

GN58. Vía más común de metástasis del carcinoma de ovario:

a) Por continuidad

b) Por exfoliación

c) Linfática

d) Hematógena

GN59. Son factores pronósticos de cáncer de endometrio los siguientes, excepto:

a) Tipo histológico y diferenciación

b) Etapa de la enfermedad

c) Tamaño uterino

d) Invasión al miometrio

e) Citología peritoneal

GN60. Síntoma o signo principal del cáncer de endometrio:

a) Dolor abdominal

b) Leucorrea
c) Crecimiento uterino

d) Sangrado transvaginal, 85%

e) Hipertensión arterial

GN61. En el manejo de la enfermedad trofoblástica, lo más importante es:

a) El diagnóstico de mola

b) El diagnóstico de mola invasora

c) El diagnóstico de coriocarcinoma

d) Los factores de alto y bajo riesgo

e) La edad

GN62. El coriocarcinoma de ovario que involucra a uno o ambos ovarios con extensión a las trompas es:

a) IIa

b) IIb

c) Ic

d) IIc

e) IIIa

GN63. Los tecomas tienen unilateralidad cercana al:

a) 100%

b) 80%

c) 70%

d) 85%

e) 75%

GN64. Grupo de edad en el que ocurre con mayor frecuencia el carcinoma de células claras del cervix.

a) 15 a 20

b) 25 a 30

c) 30 a 35

d) 40 a 45
e) 50 a 55

GN65. Gran parte de los teratomas ováricos son:

a) Malignos a bajo grado

b) Benignos

c) Borderline

d) Impredecibles

e) Malignos en alto grado

GN66. Tipo de tumor ovárico que se asocia con regresión progresiva de la obesidad posoperatoria:

a) Células de Sertoli-Leydig

b) Células teca de la granuloso

c) Células lipoídicas

d) Endometroide

e) Ginandroblastoma

GN67. Porciento de sobrevida a cinco años sin enfermedad para el cáncer endometrial Ic:

a) 100

b) 90

c) 80

d) 70

e) 50

GN68. La presencia de ganglios pélvicos con cáncer en una histerectomía más muestreo ganglionar por Ca
endometrial, lo clasifica en etapa:

a) Ic

b) IIa

c) IIb

d) IIIb

e) IIIc
GN69. La invasión a la serosa del útero en el cáncer endometrial, clasifica a la lesión en etapa:

a) Ic

b) IIa

c) IIIb

d) IIIa

e) IIIc

Correlacione las columnas siguientes:

GN70. Cáncer cervicouterino IIIb ( )

GN71. Cáncer de ovario IIIc ( )

GN72. Cáncer de endometrio IIIc ( )

GN73. Cáncer cervicouterino IIa ( )

GN74. Cáncer de vulva EIII ( )

a. Ganglios inguinales con cáncer

b. Ganglios pélvicos positivos

c. Metástasis paraórticas

d. Cura en el 75%

e. Hidronefrosis

GN75. ¿Cuál es la frecuencia con la que se presentan metástasis diafragmáticas en el cáncer epitelial del
ovario aparentemente temprano?

a) 0%

b) 15%

c) 33%

d) 50%

e) 70%

GN76. La presencia de ganglios paraórticos en un cáncer endometrial lo clasifica en etapa:

a) IIb
b) IIIa

c) IIIb

d) IIIc

e) IVb

GN77. Las metástasis ocultas en ovario por un cáncer del endometrio aparentemente temprano ocurren con
la frecuencia siguiente:

a) 0%

b) 10%

c) 20%

d) 40%

e) No ocurren

GN78. Las cifras de curación en el cáncer ovárico etapa III tratado correctamente son cercanas al:

a) 0%

b) 10%

c) 20%

d) 40%

e) 70%

GN79. Las cifras de curación para un disgerminoma Ia son:

a) 90%

b) 80%

c) 60%

d) 50%

e) 40%

GN80. ¿Qué se entiende por citorreducción óptima en cáncer de ovario?

a) Completar panhisterectomía

b) Residual tumoral menor de 2 cm

c) Residual tumoral menor de 3 cm


d) Panhisterectomía omentectomía y apendicectomía

e) Residual tumoral mayor de 2 cm

GN81. Complicación más frecuente de la histerectomía radical:

a) Infección de herida

b) Absceso pélvico

c) Fístula ureteral

d) Fístula vesicovaginal

e) Disfunción vesical

GN82. ¿Cuál de las siguientes complicaciones de la radioterapia suele tener mayor morbilidad y requerir
incluso de cirugía?

a) Estrechez vaginal

b) Supresión hormonal en mujer joven

c) Enteritis proximal

d) Proctitis

e) Ninguna de las mencionadas

GN83. El resultado global de sobrevida a cinco años en pacientes con Ca Cu IIIb tratados con radioterapia
radical es del orden de:

a) 40 a 45%

b) 10 a 15%

c) 25 a 48%

d) 50 a 70%

e) Ninguno de los anteriores

GN84. Clínicamente la tríada de falla pélvica postratamiento de RT y cirugía es:

a) Anemia, dolor ciático e hipoalbunemia

b) Dolor ciático, hidronefrosis y edema de piernas

c) Edema de piernas, anemia y dolor

d) Ganglios paraórticos positivos, tumor central y dolor


e) Ganglios pélvicos, ganglios supraclaviculares y dolor

GN85. En Ca de vagina, cuando la localización del tumor es en pared posterior, ¿Cuál es el drenaje
linfático?

a) Ilíacos

b) Obturadores

c) Paraórticos

d) Inguinales y crunales

e) Hemorroidales superiores y presacros

GN86. En cáncer de endometrio ¿Cuáles son los tres factores pronósticos más importantes?

a) Ploidia, invasión a espacio vascular, citología positiva

b) Metástasis pélvicas, etapa clínica, obesidad

c) Etapa clínica, grado de diferenciación, tipo histológico

d) Tipo histológico, amplificación oncogén her 2/neu, metástasis anexial

e) Receptores progesterona, ploidia, edad

GN87. Factor pronóstico más importante en Ca de ovario epitelial:

a) Tipo histológico

b) Titulación o cifra de CA-125

c) Etapa clínico-quirúrgica

d) Edad

e) El Karnofsky

GN88. El cáncer microinvasor del cuello uterino debe diagnosticarse mediante:

a) Colposcopía

b) Citología

c) Biopsia

d) Conización

e) Legrado endocervical
GN89. En el estudio de la pieza quirúrgica de histerectomía radical el factor pronóstico más importante es:

a) Profundidad de invasión

b) Número de ganglios positivos

c) Tamaño tumoral

d) Invasión a vagina (márgenes libres)

e) Invasión a endometrio

GN90. Manifestación paraneoplásica más frecuentemente asociada a cáncer de ovario:

a) Dermatomiositis

b) Hipercalcemia

c) Acantosis

d) Alopecia

e) Ninguna de las anteriores

GN91. Valores de sensibilidad y especificidad del CA-125 para Ca epitelial de ovario en mujeres
posmenopáusicas:

a) 99% y 50%

b) 90% y 95%

c) 100% y 80%

d) 95% y 40%

e) 97% y 78%

GN92. Porciento de pacientes que se encuentran en etapas clínicas III-IV al momento de diagnosticarse un
tumor ovárico limítrofe:

a) 60

b) 20

c) 40

d) 10

e) 80

GN93. El tumor conocido como struma ovárico corresponde a:


a) Un tumor maligno del estroma ovárico

b) Un tumor maligno de células germinales

c) Un teratoma monodérmico formado por tejido tiroideo

d) Un tumor ovárico de origen epitelial

e) Ninguna de las respuestas antes enunciadas

GN94. El grado de inmadurez de un teratoma depende de:

a) Presencia de elementos embrionarios en el tumor

b) Presencia de elementos malignos en el tumor

c) Presencia de neuroepitelio inmaduro

d) Ninguno de ellos

GN95. Las metástasis pulmonares en cáncer de ovario están predominantemente localizadas en:

a) Parénquima pulmonar

b) Pulmón derecho

c) Espacio pleural

d) Pulmón izquierdo

e) Lóbulos inferiores de ambos pulmones

GN96. ¿Qué porciento de pacientes con cáncer de endometrio con diseminación extrauterina, tienen
citología peritoneal positiva?

a) 100

b) 90

c) 50

d) 35

e) 20

GN97. ¿Cuál es el porciento de pacientes con cáncer de endometrio con lavado peritoneal positivo que no
presentan evidencia de enfermedad extrauterina?

a) 15

b) 5
c) 10

d) 0

e) 20

GN98. ¿Cuál es el porciento de pacientes, de acuerdo al GOG, con lavado peritoneal positivo, en cáncer de
endometrio, que desarrollan recurrencias intra-abdominales?

a) 10

b) 15

c) 40

d) 50

e) 80

GN99. En el cáncer de endometrio, uno de los factores más adversos es la invasión del miometrio, y puede
evaluarse en el preoperatorio mediante:

a) Examen histeroscópico

b) Legrado fraccionado

c) Resonancia magnética

d) Gamagrafía con Estroncio 87

e) Ultrasonografía transvaginal

GN100. Un paciente con cáncer endometrial temprano que desarrolla recurrencia tumoral, tiene el siguiente
porciento de posibilidades de curación con nuevas opciones de manejo:

a) 80%

b) 60%

c) 40%

d) 20%

e) 0%

GN101. La sobrevida total a cinco años en pacientes con exenteración pélvica total por Ca Cu recurrente a
radioterapia es de:

a) 10-15%

b) 20-46%

c) Más del 50%


d) 50-60%

e) 5-10%

GN102. Porciento de sobrevida total a cinco años en pacientes con exenteración pélvica anterior por Ca Cu
recurrente a radioterapia:

a) 20

b) 30

c) 33-60

d) 10-15

e) 15-20

GN103. En una paciente con evisceración pélvica total, que hace una fístula intestinal ¿Cuál es el porciento
de mortalidad?

a) 20

b) 30

c) 53

d) 60

e) 80

GN104. En pacientes portadoras de Ca Cu en todas las etapas radiadas ¿Cuál es el porciento de recurrencia a
nivel central?

a) 0

b) Menos del 10

c) Más del 15

d) 15-20

e) Más del 20

GN105. ¿Cuál es el porciento, en general, de metástasis óseas en el Ca Cu?

a) 5

b) 0-1

c) 3-4
d) 5-6

e) 6-10

GN106. Antecedente más relacionado con el cáncer de vagina:

a) Diabetes mellitus

b) Histerectomía

c) Hipertensión arterial

d) Menopausia tardía

e) Menarca temprana

GN107. De todos los melanomas de la mujer, el de los genitales ocupa el:

a) 10%

b) 25%

c) 3%

d) 15%

e) 40%

GN108. Son variantes histológicas de cáncer de ovario las siguientes, excepto:

a) Adenocarcinoma

b) Carcinoma de células claras

c) Carcinoma endometrial

d) Fibroma

e) Tumor de Brenner

GN109. Qué porciento de cáncer epitelial de ovario se presenta en etapas III y IV:

a) 20

b) 40

c) 55

d) 70

e) 90
GN110. ¿En qué etapa se presenta el 70% del cáncer de ovario germinal?

a) I

b) II

c) IIIa

d) IIIc

e) IV

GN111. Los tumores de células de la granulosa se presentan durante:

a) El embarazo

b) La premenarca

c) La premenopáusica

d) La perimenopáusica

e) La posmenopáusica

GN112. Son factores pronósticos en el carcinoma epitelial de ovario los siguientes, excepto:

a) Etapa

b) Tumor residual

c) Histología

d) Tamaño tumoral

e) Grado de diferenciación

GN113. Frecuencia de tumores borderline de las neoplasias malignas de ovario:

a) 5-10%

b) 35%

c) 4-14%

d) 1-3%

e) 20%

GN114. Frecuencia de tumores de ovario, del estroma gonadal:


a) 10%

b) 1%

c) 20%

d) 30%

e) 5%

GN115. El tumor ovárico de células granulosas, se asocia con Ca de:

a) Endometrio

b) Cérvix

c) Trompa de falopio

d) Colon

d) Vagina

GN116. ¿Qué porciento de tumores localizados en ovario, presentan metástasis ganglionares paraórticos?

a) 5

b) 10

c) 15

d) 20

e) 25

GN117. Principal síntoma en Ca de ovario epitelial:

a) Sangrado transvaginal

b) Obstrucción intestinal

c) Malestar abdominal

d) Síntomas urinarios

e) Diseña

GN118. Cómo se clasifica un tumor de ovario que invade órganos pélvicos:

a) IV

b) Ic
c) II

d) IIc

e) IIIa

GN119. Son estudios indicados para diagnóstico de un probable Ca de ovario los siguientes, excepto:

a) Marcador tumoral

b) Ultrasonograma pélvico

c) TAC

d) Paracentesis

e) Laparotomía

GN120. ¿Qué efecto provoca la ruptura de cáncer de ovario de mal pronóstico?

a) Disminuye la sobrevida

b) No hay efecto en la sobrevida

c) Aumenta la sobrevida

d) Disminuye el intervalo libre de enfermedad

e) Aumenta el intervalo libre de enfermedad

GN121. Tipo histológico de Ca de ovario epitelial desfavorable:

a) Adenocarcinoma

b) Tumor de Brenner

c) Disgerminoma

d) Fibroma

e) Células claras

GN122. Incidencia de second look negativo en pacientes sin enfermedad residual:

a) 17%

b) 34%

c) 50%

d) 60%
e) 77%

GN123. Sobrevida a cinco años del cáncer de ovario etapas I y II:

a) 50%

b) 60%

c) 70%

d) 80%

e) 90%

GN124. Sobrevida en general del cáncer de ovario:

a) 70%

b) 60%

c) 50%

d) 40%

e) 30%

GN125. Son factores de alto riesgo en Ca de endometrio, excepto:

a) Grado 3

b) Carcinoma papilar seroso

c) Receptor estrogénico positivo

d) Carcinoma de células claras

e) Carcinoma indiferenciado

GN126. Un factor de riesgo para el desarrollo de Ca de endometrio es:

a) Multiparidad

b) Dieta baja en grasa

c) Deficiencia de vitamina D

d) Dieta baja en fibra

e) Obesidad
GN127. Entre las neoplasias ginecológicas, ¿Qué lugar ocupa el Ca de endometrio en Estados Unidos?

a) Tercero

b) Primero

c) Cuarto

d) Segundo

e) Quinto

GN128. Son factores de riesgo para Ca de endometrio los siguientes, excepto:

a) Nuliparidad

b) Obesidad

c) Uso de tamoxifén

d) Uso de progestágenos

e) Historia familiar de Ca de mama

GN129. Sobrevida a cinco años de pacientes con Ca de endometrio etapa IV al momento del diagnóstico:

a) 1%

b) 5%

c) 10%

d) 15%

e) 20%

GN130. Primer síntoma de Ca de endometrio:

a) Metrorragia

b) Dolor

c) Amenorrea

d) Leucorrea

e) Dispareunia

GN131. ¿En qué década de la vida es más frecuente el Ca de endometrio?

a) Quinta
b) Cuarta

c) Tercera

d) Séptima

e) Segunda

GN132. Son factores pronósticos relacionados con el Ca de endometrio los siguientes, excepto:

a) Grado histológico

b) Edad mayor de 40 años

c) Invasión a miometrio

d) Lavado peritoneal positivo

e) Invasión vascular

GN133. Manifestación de enfermedad avanzada en Ca de endometrio:

a) Dispareunia

b) Leucorrea fétida

c) Hemorragia intermitente

d) Dolor pélvico

e) Amenorrea secundaria

GN134. Tipo de hiperplasia endometrial que tiene mayor riesgo de progresar a Ca de endometrio:

a) Adenomatosa sin atipia

b) Quística con atipia

c) Atípica simple

d) Adenomatosa con atipia

e) Quística sin atipia

GN135. Las etapas tempranas del Ca de endometrio con lesiones de alto grado se asocian a:

a) Invasión miometrial y metástasis ganglionares

b) Diferenciación glandular y sangrado abundante

c) Reacción desmoplásica y adenoacantoma


d) Tumores bien diferenciados y bajo potencial maligno

e) Menor mortalidad y menores recurrencias

GN136. Para utilizar el diagnóstico de tipo celular mixto en Ca de endometrio se requiere que:

a) Se trate de dos tumores diferentes y bien diferenciados

b) Existan células pleomorfas en el estudio

c) Se originen en dos sitios anatómicos diferentes

d) Exista 50% de las células de cada tipo tumoral

e) Se manifiesten dos o más tipos de células diferentes

GN137. ¿Cuál es la frecuencia de afectación ganglionar en etapas tempranas de Ca de endometrio?

a) 1-5%

b) 5-9%

c) 10-15%

d) 16-20%

e) 21-25%

GN138. Porciento de operabilidad en etapas tempranas de Ca de endometrio:

a) 45

b) 58

c) 61

d) 70

e) 87

GN139. La morbilidad quirúrgica de la histerectomía con muestreo ganglionar para Ca de endometrio es de:

a) 10%

b) 20%

c) 30%

d) 40%

e) 50%
GN140. Los pacientes con Ca de endometrio y receptores positivos muestran:

a) Menor sobrevida

b) Menos metastásis pulmonares

c) Alta frecuencia de metástasis a SNC

d) Menor frecuencia de recurrencias locales

e) Altas respuestas a la hormonoterapia

GN141. Sobrevida a cinco años de pacientes con Ca de endometrio etapa I:

a) 50%

b) 60%

c) 70%

d) 80%

e) 90%

GN142. ¿Cuál el es papel actual en nuestro país (de acuerdo a nuestra situación), de la hibridación o del
PCR, en el manejo del VPH?

a) Ninguno ya que el precio del estudio es muy cara para población abierta

b) Ninguno porque es de poca sensibilidad

c) Importante ya que tiene una sensibilidad del 100%

d) a y d son ciertas

GN143. Porciento de metástasis a ganglios pélvicos en el Ca Cu etapa IIb:

a) 10 a 20%

b) 15 a 25%

c) 27 a 45%

d) 59 a 60%

e) 50 a 75%

GN144. ¿Cuál es la incidencia promedio de metástasis ganglionares en Ca Cu microinvasor?

a) 1.2%
b) 5%

c) 8%

d) 10%

e) 20%

GN145. Dentro del sistema Bethesda ¿Se deben tratar las lesiones clasificadas como bajo grado? ¿Sí, no y
por qué?

a) Sí, por ser lesiones premalignas o malignas iniciales

b) Sí, porque requieren tratamiento conservador y son potencialmente invasoras

c) No, por ser lesiones premalignas

d) No, porque los tratamientos conservadores no son efectivos en el 100%

e) Sólo a y b son ciertas

GN146. La causa más común de irresecabilidad en un Ca Cu recurrente a radiación limitado clínicamente a


la pelvis:

a) Invasión vesical

b) Invasión rectal

c) Invasión a esqueleto de la pelvis

d) Metástasis ganglionares fijas a vasos pélvicos

e) Metástasis paraórticas

GN147. Los factores pronósticos quirúrgicos en el Ca Cu son:

a) Tipo histológico tumor

b) Tipo y grado tumorales

c) Infiltración a parametrios y vasos linfáticos

d) Tamaño tumoral invasión vascular y linfática, metástasis ganglionares, infiltración a parametrios


y márgenes quirúrgicos positivos

e) Tipo de cirugía empleada

GN148. Desde el punto de vista histopatológico el carcinoma de cervix de mayor supervivencia corresponde
a:

a) Adenocarcinoma poco diferenciado


b) Carcinoma de células claras

c) Carcinoma adenoescamoso

d) Carcinoma epidermoide de células grandes no queratizante

e) Carcinoma epidermoide moderadamente diferenciado queratizante

GN149. Edad de mayor incidencia del carcinoma epitelial de ovario:

a) 50-54 años

b) 55-59 años

c) 60-64 años

d) 65-69 años

e) 70-74 años

GN150. Manejo adyuvante en cáncer de ovario etapa 1a, bien y moderamente diferenciado:

a) Hormonoterapia

b) Quimioterapia

c) Radioterapia

d) Intraperitoneal

e) Vigilancia

GN151. Son tumores epiteliales del ovario (clasificación histológica) todos, excepto:

a) Endometroide

b) Seroso

c) Células claras

d) Mucinoso

e) Tumor de células de la granulosa

GN152. Se reconoce como coriocarcinoma primario del ovario a:

a) Tumores de células germinales que presentan diferenciación trofoblástica

b) Este diagnóstico se establece solo en mujeres en edad premenárquica o posmenopáusica


c) Este diagnóstico se puede establecer en cualquier tumor trofoblástico localizado en el ovario

d) Son verdaderos a y b

e) Ninguno es cierto

GN153. Los tumores borderline del ovario se caracterizan por:

a) Ausencia de invasión del estroma ovárico

b) Tienen capacidad para dar metástasis

c) 15 a 70% son bilaterales

d) Muestran atipias celulares y actividad mitótica elevada

e) Todo es verdadero

GN154. A la exploración clínica una mujer con diagnóstico de cáncer de ovario se encuentran ganglios
inguinales, en qué etapa se incluyen:

a) IIc

b) IIIa

c) IIIb

d) IIIc

e) IV

GN155. ¿Cuál de los siguientes no es tumor germinal de ovario?

a) Coriocarcinoma

b) Senos endodérmicos

c) Disgerminoma

d) Ginandroblastoma

e) Embrionario

GN156. Ante el diagnóstico de tumor germinal de ovario, en qué casos se justifica efectuar biopsia del
ovario contralateral:

a) Cuando se identifican folículos en el ovario

b) En presencia de anomalidad macroscópica

c) Cuando el tumor está en contacto con el ovario


d) Si al excidirse el tumor se rompe la cápsula

e) Ninguna de las anteriores

GN157. Manejo más apropiado en pacientes premenopáusicas con diagnóstico de tumor germinal operadas
inicialmente por cirujanos no oncólogos:

a) Relaparotomía para completarse la rutina de ovario sin apendicectomía

b) Revisión de laminillas, TAC de abdomen y pelvis

c) Revisión de laminillas, telerradiografía de tórax y marcadores tumores

d) Durante la reintervención de ser posible preservar la función ovárica

e) Todas las anteriores

GN158. ¿En qué etapas clínicas de Ca de ovario epitelial puede estar indicado el second look?

a) Etapa III

b) Etapa Ic

c) Etapa II

d) Todas

e) Ninguno

GN159. ¿Cuál es el porciento de negatividad en la cirugía de segunda vista (second look)?

a) 20

b) 30

c) 40

d) 50

e) 0

GN160. ¿Cuál es el porciento de second look negativos según la etapa?

a) Etapa I 80%, II 70%, III y IV 33%

b) Etapa I 100%, II 80%, III 40% y IV 20%

c) Etapa I 90%, II 80%, III 40% y IV 25%

d) Etapa I 80%, II 60%, III 50% y IV 30%


GN161. Paciente posoperada de 23 años de un tumor de ovario de células de Sertoli-Leydig deberá
mantenerse en control subsecuente con base en:

a) Cuantificación de CA-125

b) Cuantificación de deshidrogenasa láctica

c) USG anual

d) Cuantificación de alfafetoproteína

e) Cuantificación de gonadotrofina coriónica fracción beta

GN162. Las cifras de curación en un cáncer ovárico Ic:

a) 100%

b) 90%

c) 80%

d) 50%

e) 40%

GN163. ¿Cuál de los siguientes no es un factor pronóstico en el carcinoma epitelial de ovario?

a) Etapa

b) Tumor residual

c) Histología

d) Tamaño tumoral

e) Grado de diferenciación

GN164. ¿Cuáles pacientes con cáncer de ovario son los que presentaran mayor recurrencia después de un
second look negativo?

a) QT previa

b) Volumen tumoral

c) CA-125 elevado

d) Adenocarcinoma seroso

e) Gonadoblastoma

GN165. Porciento de recurrencia del disgerminoma etapa Ia:


a) 0-5

b) 5-15

c) 15-25

d) 25-35

e) 35-45

GN166. La asociación del cáncer del endometrio con sus típicos factores de riesgo, está presente en el
porciento de casos siguiente:

a) 10

b) 20

c) 50

d) 70

e) 100

GN167. Son factores de riesgo para cáncer de endometrio los siguientes, excepto:

a) Obesidad

b) Nuliparidad

c) Menopausia tardía

d) Estrógenos por tiempo prolongado

e) Tabaquismo intenso

GN168. ¿Qué medicamentos a grandes dosis y después de cinco años de uso pueden causar Ca de
endometrio condicionando un riesgo relativo?

a) Platino

b) Taxol

c) Tamoxifén

d) Todos

e) Ninguno

GN169. En el síndrome de Lynch II, se asocia el cáncer endometrial con:

a) Cáncer de mama

b) Cáncer de ovario
c) Cáncer de colon

d) Enfermedad del trofoblasto

e) Cáncer de páncreas

GN170. Factor de riesgo para el desarrollo de Ca de endometrio:

a) Multiparidad

b) Dieta baja en grasa

c) Deficiencia de vitamina D

d) Dieta baja en fibra

e) Obesidad

GN171. El grupo llamado GOG de la unión americana denomina actualmente etapa III con enfermedad
óptima a la entidad siguiente:

a) Ca Cu IIIa

b) Ca endometrio IIIa

c) Ca ovárico IIIc

d) Trofoblasto alto riesgo

e) Ca vulva III

GN172. La presencia de células neoplásicas en el lavado peritoneal de pacientes con cáncer endometrial
limitado al cuerpo uterino lo clasifica en:

a) Etapa Ib

b) Etapa Ic

c) Etapa IIIa

d) Etapa IIIc

e) Etapa IVb

GN173. En etapas tempranas de Ca de endometrio el estudio preoperatorio escencial para indicar la cirugía
es:

a) Ultrasonografía

b) Urografía excretora
c) TAC pélvica

d) Histerometría

e) Biopsia endometrial

GN174. ¿En países desarrollados, qué porciento de pacientes con Ca de endometrio presentan enfermedad
confinada al útero al momento del diagnóstico?

a) 90%

b) 50%

c) 35%

d) 75%

e) 100%

GN175. Los siguientes son órganos frecuentemente afectados por metástasis de Ca de endometrio:

a) Pulmón

b) Riñón

c) Hígado

d) Hueso

e) Sistema nervioso central

GN176. A que etapa clínica se le da tratamiento con quimioterapia y hormonoterapia, hablando de cáncer de
endometrio:

a) Etapa I

b) Etapa II

c) Etapas III y IV

d) Ninguna de las anteriores

e) Todas de las anteriores

GN177. Señale el porciento de respuestas al tratamiento paliativo con hormonoterapia en el cáncer


endometrial avanzado o recurrente:

a) 100

b) 75
c) 50

d) 40

e) 20

GN178. En cáncer de endometrio, son indicaciones para muestreo ganglionar retroperitoneal:

a) Invasión del miometrio mayor al 50%, crecimiento ganglionar

b) Extensión al cérvix y diseminación extrauterina

c) Carcinoma de endometrio de tipo seroso

d) Carcinoma de endometrio de tipo células claras, de tipo escamoso o indiferenciado

e) Todos las anteriores

GN179. De la lista siguiente, señale el cáncer endometrial de peor pronóstico:

a) Adenoacantoma endometroide

b) Adenoacantoma

c) Adenoacantoma moderadamente diferenciado

d) Cáncer seroso papilar

e) Adenoacantoma bien diferenciado

GN180. Los resultados terapéuticos menos favorables para el cáncer endometrial en la población negra
norteamericana (1) y en la población abierta de nuestro país (2), se relacionan con:

a) Factores genéticos adversos

b) Un mayor número de etapas III y IV

c) Un menor número de receptores positivos a la progesterona

d) Predominio de lesiones menos diferenciadas

e) Predominio de lesiones en pacientes añosas

GN181. Anote el factor pronóstico más importante en el Ca endometrial etapa I:

a) Cáncer moderadamente diferenciado

b) Cáncer poco diferenciado endometroide

c) Invasión a la mitad del miometrio


d) Ausencia de receptores de progesterona

e) Invasión profunda del miometrio

GN182. Factor(es) pronóstico(s) más importante(s) en el cáncer de endometrio:

a) Etapa clínica

b) Tamaño del tumor

c) Grado de diferenciación

d) Etapa clínica, grado de diferenciación e invasión al miometrio

e) Citología peritoneal positiva

Anote verdadero o falso en las siguientes aseveraciones relacionadas con el cáncer endometrial:

GN183. ( V ) Su pronóstico es peor en ancianas

GN184. ( V ) El Ca papilar requiere de omentectomia para su manejo

GN185. ( F ) Las cifras de curación en el Ca de células claras se aproximan al 50%

GN186. ( V ) La población negra de los Estados Unidos tiene peor pronóstico

GN187. ( V ) La mujer joven tiene mejor pronóstico

GN188. En cáncer de endometrio pertenecen al grupo de alto riesgo para recurrencia:

a) Las pacientes en etapa Ib, Ic, IIa, IIb y IIIa

b) Las pacientes en etapa IIIa, IIIb, IIIc

c) Las pacientes en etapa IVa, IVb

d) Sólo b y c

e) Sólo a y b

GN189. La invasión profunda del miometrio en el cáncer endometrial etapa Ic, se acompaña de cifras de
seguimiento a cinco años sin tumor del orden del:

a) 90%

b) 80%

c) 60%

d) 40%
e) 20%

GN190. Las recurrencias tumorales en el cáncer endometrial etapa Ic son eventos que ocurren con la
frecuencia siguiente:

a) 0%

b) 5 a 10%

c) 10 a 20%

d) 21 a 40%

e) 41 a 60%

Anote verdadero o falso en las aseveraciones siguientes:

GN191. En el cáncer endometrial, los antecedentes de obesidad y de nuliparidad, se acompañan de un mejor


pronóstico.

a) Verdadero

b) Falso

GN192. Los pacientes con enfermedad subóptima de cáncer endometrial avanzado, son aquellas que cursan
con residuales poscirugía mayores de 1 cm.

a) Verdadero

b) Falso

GN193. La sobrevida libre de enfermedad a cinco años de pacientes con Ca de endometrio y metástasis a
ganglios paraórticos tratados con radioterapia es de:

a) 15-20%

b) 60-70%

c) 80-90%

d) 30-40%

e) 50-60%

GN194. Signo(s) más frecuente(s) de la mola hidatiforme:

a) Náuseas y vómito

b) Sangrado transvaginal
c) Manifestaciones de hipertiroidismo

d) Enfermedades respiratorias

GN195. Tratamiento de elección en paciente de 43 años gesta-III, para-II, con mola hidatiforme recurrente:

a) Esquema VAC

b) Esquema AC

c) Histerectomía

d) Sobredosis metotrexate

e) Legrados múltiples

GN196. Síntoma(s) y/o signo(s) más frecuente(s) en pacientes con cáncer invasor de vulva:

a) Sangrado y flujo

b) Flujo y tumor

c) Sangrado y prurito

d) Prurito y tumor

GN197. Incidencia de ganglios pélvicos en caso de encontrar metástasis inguinales por cáncer de vulva:

a) 10%

b) 5%

c) 0%

d) 25%

GN198. Un tumor de vulva de cualquier tamaño, con extensión a uretra, vagina o ano, o con ganglios
regionales metastásicos unilaterales, se clasifica según la FIGO en:

a) Etapa IVa

b) Etapa IVb

c) Etapa III

d) Etapa II

e) Etapa I

GN199. De acuerdo a la etapificación anatómica el coriocarcinoma EC-III involucra:


a) Enfermedad confinada al cuerpo uterino

b) Metástasis pélvicas

c) Metástasis en vagina

d) Metástasis en pulmón

e) Metástasis hepáticas

GN200. La posibilidad de curación en pacientes con adenocarcinoma de endometrio con recurrencia pélvica
puede lograrse hasta en un 50% si se realiza lo siguiente:

a) Radioterapia

b) Quimioterapia

c) Quimioterapia-radioterapia

d) Excenteración pélvica

e) Histerectomía

GN201. ¿Cuál es la sobrevida a cinco años en pacientes con cáncer epitelial de ovario etapa III con tumor
voluminoso?

a) 10%

b) 20%

c) 30%

d) 40%

e) 50%

GN202. Falsa en relación con cáncer de ovario:

a) La detección temprana con CA-125 ha mejorado la sobrevida a largo plazo

b) El uso de anticonceptivos orales reduce el riesgo de tener la enfermedad

c) El 10% de los casos tienen una predisposición hereditaria

d) El tratamiento con fármacos que induce la ovulación aumenta el riesgo de tener la enfermedad

e) Los casos hereditarios aparecen a edad más temprana del promedio y son de estirpe papilar
seroso

GN203. En una paciente portadora de Ca in situ tratada previamente con método quirúrgico conservador y
citologías de control anormales, el riesgo de presentar cáncer invasor es (cuántas) veces mayor que en los
pacientes con citología normal:
a) 5

b) 10

c) 15

d) 20

e) 25

GN204. La histerectomía radical clásica o tipo III, fue implantada por:

a) Clark (1895)

b) Schauta (1902)

c) Wertheim (1912)

d) Bonny-Meigs (1951)

e) Clark-Wertheim

GN205. Factor(es) pronóstico(s) más importante(s) en el Ca Cu:

a) Tipo histológico del tumor

b) Tipo y grado tumorales

c) Infiltración a parametrios y vasos linfáticos

d) Tamaño tumoral, invasión vascular y linfática, metástasis ganglionar, infiltración a parametrios y


márgenes quirúrgicos positivos

e) Tipo de cirugía empleada

GN206. La probabilidad de que una paciente presente hidronefrosis por un Ca Cu etapa clínica II es:

a) 0%

b) 5%

c) 15%

d) 20%

e) 35%

GN207. ¿Cuál es la probabilidad que una paciente presente hidronefrosis por Ca Cu en etapa clínica III?

a) 15%
b) 20%

c) 35%

d) 40%

e) 50%

GN208. Tratamiento de elección en tumores Ca germinales de ovario cuando el tumor está limitado a un
solo ovario:

a) Cirugía conservadora: ooforectomía unilateral

b) Ooforectomía más biopsia en cuña del ovario contralateral

c) Biopsia del tumor

d) Histerectomía más salpingooforectomía bilateral

e) Rutina de ovario

GN209. Grupo de pacientes con tumor no disgerminoma exento de tratamiento adyuvante:

a) Pacientes con EC Ia, grado I y componente de teratoma inmaduro

b) Casos en EC Ia, en un 76%

c) Pacientes en EC Ia, grado 2 y 3 de teratoma inmaduro y ECI de senos endodérmicos

d) Pacientes en EC Ia resecados completamente

e) e y b

Correlacione el tratamiento anotando en el paréntesis la letra que corresponda a cada una de las pacientes
siguientes:

GN210. Femenino de 47 años con cirugía subóptima de Ca de ovario etapa III, sin evidencia clínica y
radiológica de actividad tumoral al término de seis ciclos de quimioterapia. ( )

GN211. Femenino de 62 años de edad con células neoplásicas en líquido ascítico a tensión y evidencia
clínica y ultrasonográfica de carcinomatosis peritoneal por Ca de ovario. ( )

GN212. Femenino de 35 años de edad nulípara con cáncer endometroide de ovario limitado a un ovario.
( )

GN213. Femenino de 45 años con cáncer de ovario etapa IIIa. ( )

GN214. Femenino de 48 años con cirugía óptima de cáncer de ovario etapa III sin evidencia clínica de
tumor al cabo de seis ciclos de quimioterapia y con determinaciones en ascenso de CA-125 ( )

a) Salpingooforectomía unilateral

b) Panhisterectomía, omentectomía y biopsias


c) Cirugía de intervalo

d) Second look

e) Cirugía de citorreducción secundaria

GN215. Tratamiento primario para los sarcomas uterinos:

a) Radioterapia

b) Quimioterapia

c) Radioterapia seguida de cirugía

d) Cirugía

e) Inmunoterapia

GN216. Procedimiento realizado mediante laparotomía exploradora después de un tratamiento previo a los
tumores epiteliales cuando no hay datos clínicos, de laboratorio ni radiológicos, de positividad:

a) Cirugía de intervalo

b) Second look

c) Cirugía citorreductora subóptima

d) Cirugía óptima citorreductora

GN217. En el cáncer de endometrio etapa clínica Ia G1, la radioterapia posoperatoria se inicia en caso de
que se reporte:

a) Pieza quirúrgica con invasión superficial de la mucosa

b) Pieza quirúrgica sin invasión a la mucosa

c) Pieza quirúrgica con invasión tumoral a más del 50% del miometrio

d) Que el tumor sólo infiltra incipientemente al miometrio

e) Que el tumor infiltra menos del 50% del miometrio

GN218. El tratamiento quirúrgico de elección para el cáncer de endometrio es:

a) Histerectomía radical

b) Histerectomía extrafascial+SOB+muestreo ganglionar y lavado peritoneal

c) Histerectomía extrafascial sin SOB

d) Histerectomía intrafascial
e) Histerectomía extrafascial más ooforectomía unilateral

GN219. En el cáncer de ovario el factor(es) pronóstico(s) más importante(s) son:

a) Etapa clínica

b) Etapa clínica y grado de diferenciación

c) Etapa clínica, grado de diferenciación y tamaño del tumor residual

d) Etapa clínica, grado de diferenciación, tamaño del tumor residual y DNA (ploidia tumoral)

e) Tipo de cirugía efectuada

GN220. Las fístulas urinarias después de una histerectomía radical van de:

a) 0 a 0.5%

b) 1 a 2%

c) 2.5 a 3%

d) 3 a 3.5%

e) 4 a 5%

GN221. Tratamiento de paciente femenina de 27 años de edad, con un solo producto con NIC III:

a) Histerectomía con SOD

b) Criocirugía

c) Cono de cérvix con asa de diatermia o rayo láser y vigilancia

d) Panhisterectomía

e) Histerectomía radical con SOD

GN222. Tratamiento indicado a una paciente de 24 años de edad con disgerminoma de ovario etapa Ib:

a) Laparotomía con S/O bilateral

b) Laparotomía-biopsia-quimioterapia

c) Salpingooforectomía unilateral y radioterapia

d) Salpingooforectomía con omenectomía

GN223. En Ca Cu avanzado la complicación más frecuente de los esquemas utilizados con ifosfamida es
hematuria. Esta complicación se evita usando:
a) Hidratación intravenosa

b) Líquidos por vía oral

c) Mesna simultáneo con la ifosfamida

d) Infusión de 24 horas con ifosfamida

e) Dosis reducida de ifosfamida

GN224. El uso de modificadores de la respuesta biológica en Ca Cu avanzado están indicados:

a) En combinación con QT

b) Simultánea con RT

c) Simultáneamente con RT no, ya que son inútiles

d) Como monoterapia

e) Sólo en protocolos de investigación

GN225. En neoplasias de ovario, el esquema paclitaxel/CDDP frente al tradicional CDDP/ciclofosfamida,


induce respuestas en:

a) 77 versus 64%

b) 90 versus 50%

c) 50 versus 90%

d) 35 versus 40%

e) 65 versus 50%

GN226. ¿Cuál de los citotóxicos siguientes escogería usted de base para el tratamiento paliativo del Ca
endometrial?

a) 5 fluorouracilo

b) Ciclofosfamida

c) Dtic

d) Vinblastina

e) Adriamicina

GN227. Tratamiento quirúrgico efectivo en el Ca embrionario de ovario etapa III:

a) Panhisterectomía
b) Panhisterectomía y omentectocomía

c) Salpingooferectomía bilateral

d) Salpingooferectomía unilateral

e) Detumorización conservando útero

GN228. En un tumor de senos endodérmicos etapa Ia, la conducta poscirugía es:

a) Expectante

b) Second look a los seis meses

c) Cirugía de intervalo

d) Radioterapia radical

e) Quimioterapia

GN229. La curación en un cáncer ovárico etapa Ia es de:

a) 100%

b) 90%

c) 80%

d) 50%

e) 40%

GN230. Paciente femenino de 36 años de edad que ingresa al servicio con antecedente de salpingooforectomía
por Ca endometroide de ovario. La conducta es:

a) Radioterapia radical

b) Quimioterapia y cirugía de intervalo

c) Second look

d) Nueva laparotomía

e) Observación

Correlacione las columnas siguientes respecto al tratamiento médico del cáncer ginecológico:

GN231. ( ) Enf. trofoblasto alto riesgo a) Metotrexate

GN232. ( ) Tumor de senos endodérmicos b) Progestágenos


GN233. ( ) Cáncer de endometrio c) Platino+ciclofosfamida

GN234. ( ) Enf. trofoblasto etapa I d) Esquema VAC

GN235. ( ) Cáncer ovárico epitelial e) Paclitaxel

GN236. ( ) Cáncer ovárico recurrente f) Esquema MAC

GN237. Tratamiento de elección en un Ca de vulva T2 N0 M0:

a) Vulvectomía simple y observación

b) Escisión amplia y radioterapia a ingles

c) Vulvectomía radical

d) Radioterapia

e) Radioterapia y moduladores

Correlacione las columnas siguientes. Las intervenciones quirúrgicas se han asociado a los autores que se
mencionan:

GN238. ( ) Vulvectomía radical a) A. Brunshwing

GN239. ( ) Histerectomía radical b) Patey

GN240. ( ) Histerectomía radical vía vaginal c) Shauta

GN241. ( ) Exenteración pélvica d) J. V. Meigs

GN242. ( ) Mastectomía radical modificada e) Operación de Way

GN243. Paciente con Ca de ovario operada con información insuficiente respecto a la cirugía. Karnofsky 90%,
cirugía hace dos semanas; la indicación es:

a) Pasa a radioterapia

b) Conducta según resultados de laboratorio y gabinete

c) Cirugía de intervalo

d) Platino+ciclofosfamida

e) Nueva laparotomía estadificadora

GN244. La radioterapia radical a la pelvis en el posoperatorio de una histerectomía radical por metástasis
ganglionares en la pieza quirúrgica:

a) Disminuye significativamente las recurrencias locales


b) Aumenta las cifras de curación en un 30%

c) Está contraindicado por excesiva morbilidad

d) Aumenta las cifras de curación en un 50%

e) No afecta la evolución de la enfermedad

GN245. Indicaciones de radioterapia en cáncer de endometrio:

a) Citología positiva

b) Metástasis pélvicas y paraórticas

c) Tumor residual, G2-3, invasión a miometrio

d) A y B

e) Ninguna de las anteriores

GN246. Para cáncer del cuello uterino, el mejor método de detección es:

a) Colposcopía

b) Biopsia al azhar

c) Citología cervical

d) Prueba de Shiller

e) Hibridación de VPH

GN247. La cirugía de segunda vista en cáncer de ovario epitelial está indicada en:

a) Toda paciente que reciba completa su quimioterapia adyuvante

b) En aquellos pacientes con progresión de la enfermedad a la quimioterapia adyuvante

c) En pacientes sin enfermedad (por exploración física, imagen y por marcador tumoral) y que
estén en protocolo de estudio

d) En pacientes que reciben QT neoadyuvante o de inducción y existe evidencia de enfermedad

e) Pacientes que por laparoscopia existe evidencia de enfermedad

GN248. En el tumor borderline epitelial de ovario, el tratamiento es:

a) Igual que cualquier cáncer epitelial avanzado de ovario

b) Tratado con cirugía conservadora


c) Útil la QT adyuvante

d) Cirugía de segunda vista

e) El tratamiento con RT adyuvante mejora la sobrevida

GN249. El tratamiento con QT se inicia en paciente con tumor abdominal y fuerte sospecha que sea
primario de ovario cuando exista:

a) Elevación del CA-125>1000

b) Ultrasonido pélvico con diagnóstico de tumor de ovario

c) Citológico positivo a Ca en derrame pleural

d) Tomografía con múltiples ganglios retroperitoneal

e) Edad mayor de 80 años

GN250. La radioterapia adyuvante en pacientes con cáncer de endometrio etapa Ia G1 y 2:

a) Mejora la sobrevida a cinco años

b) Reduce la incidencia de recurrencias

c) No tiene indicaciones en este grupo de pacientes

d) Se inicia dentro de los dos primeros meses poscirugía

e) Se relacionan con a, b y c

GN251. La omentectomía infracólica es un procedimiento a efectuarse en el manejo del cáncer del


endometrio:

a) De células claras seroso papilar

b) Poco diferenciado

c) En etapa clínica IIIc

d) No tiene indicación en este cáncer

GN252. En una paciente sometida a evisceración pélvica total por Ca Cu recurrente, la complicación
quirúrgica más frecuente es:

a) Fístula intestinal

b) Obstrucción de intestino delgado

c) Absceso pélvico
d) Pielonefritis

e) Tromboembolia pulmonar

GN253. En una paciente joven con Ca Cu radiado y recurrencia central, el procedimiento de rescate más
indicado es:

a) Sobredosis de radioterapia

b) Derivación urinaria

c) Histerectomía clase III

d) Evisceración pélvica total

e) Tratamiento con quimioterapia

GN254. En un paciente con Ca Cu radiada sin actividad tumoral, con fístula rectovaginal, la conducta
correcta es:

a) Tratamiento conservador

b) Resección de la fístula

c) Colostomía derivativa

d) Resección intestinal

e) Exenteración posterior

GN255. Tratamiento de elección para metástasis óseas por Ca Cu sintomáticas:

a) Cirugía

b) QT más RT

c) QT sola

d) RT sola

e) Tratamiento médico sintomático

GN256. Los siguientes son factores que influyen en la sobrevida, al presentar persistencia o recurrencia de
Ca de ovario y someterse a citorreducción secundaria, excepto:

a) La cirugía previa óptima

b) La ascitis

c) El tamaño tumoral

d) El intervalo libre de enfermedad


e) La edad

GN257. Quimioterapia indicada para Ca de ovario epitelial recurrente:

a) Melfalan

b) Hexametimelamina

c) Paclitaxel

d) Ciclofosfamida

e) Etopósido

GN258. Tratamiento de rutina de los tumores del estroma gonadal:

a) Salpingooferectomía

b) Cirugía completa

c) QT

d) RT

e) Ninguno

GN259. El tratamiento quirúrgico para Ca de endometrio etapas I y II es histerectomía:

a) Simple

b) Radical

c) Total con biopsia ganglionar

d) Extrafascial con SOB

e) Radical ampliada

GN260. La radioterapia como tratamiento inicial en Ca de endometrio está indicada cuando:

a) La enfermedad es avanzada

b) El sangrado es abundante

c) El tumor es de bajo grado

d) La paciente es mayor de 65 años

e) La histerometría es mayor de 8 cm
GN261. La radioterapia posoperatoria está indicada a partir de la etapa:

a) IIaG3

b) IIbG2

c) IaG3

d) IIIaG1

e) IbG2

GN262. En Ca de endometrio la dosis de radioterapia externa adyuvante a la pelvis es de:

a) 2500-3500 cGy

b) 3000-4000 cGy

c) 4000-5000 cGy

d) 6000-7000 cGy

e) 7000-8000 cGy

GN263. Las pacientes con invasión profunda del miometrio son candidatas a tratamiento a base de:

a) RT pélvica y vaginal

b) Disección ganglionar pélvica

c) QT sistémica adyuvante

d) Hormonoterapia profiláctica

e) Vigilancia estrecha posoperatoria

GN264. Con el tratamiento adyuvante del Ca de endometrio con medroxiprogesterona se ha observado que:

a) Hay mejoría en la sobrevida

b) Se logra disminución de las recurrencias vaginales

c) No hay beneficio demostrable

d) Mejora la calidad de vida

e) Se evitan metástasis pulmonares

GN265. La exenteración pélvica por Ca de endometrio está indicada en:

a) Enfermedad metastásica a ingle


b) Recurrencia central después de radioterapia

c) Presencia de tumor indiferenciado

d) Recurrencia vaginal posquirúrgica

e) Etapa IVa al momento del diagnóstico

GN266. Cuál es la conducta terapéutica ante una enfermedad preinvasora del cuello uterino en un centro
oncológico:

a) Expectante

b) Colposcopía y prueba de Schiller con toma de Biopsias

c) Citologías cervicovaginales seriadas

d) Cono cervical

e) Histerectomía simple

GN267. El tratamiento de elección en pacientes portadores de Ca Cu Ib2 es:

a) Histerectomía radical clase III

b) Histerectomía clase II

c) Radioterapia radical

d) Histerectomía radical más radioterapia

e) Radioterapia más histerectomía clase III

GN268. La linfadenectomía en la histerectomía radical (Meigs o clase III), consiste en:

a) Disección de ganglios paraórticos

b) Disección de ganglios paraaórticos y paracavales

c) Disección de ganglios obturadores e ilíacos

d) Disección de ganglios ilíacos primitivos, ilíacos externos e hipogástricos y obturadores

e) Sólo disección de ganglios obturadores

GN269. En una paciente premenopáusica, portadora de Ca Cu Ib1 epidermoide, el procedimiento quirúrgico


adecuado es:

a) Histerectomía radical clase II

b) Histerectomía radical clase III más SOB


c) Histerectomía radical clase III, conservando uno o ambos ovarios

d) Histerectomía clase I

e) Histerectomía clase II más radioterapia

GN270. En una paciente de más de 40 años de edad portadora de Ca Cu Ib1 tipo adenocarcinoma, el
tratamiento quirúrgico correcto es:

a) Histerectomía radical clase III

b) Histerectomía radical clase III más SOB

c) Histerectomía radical clase III más radioterapia

d) Histerectomía radical clase II más radioterapia

e) Histerectomía radical clase I más radioterapia

GN271. Las pacientes con histerectomía radical clase III, presentan con cierta frecuencia disfunción vesical,
por lo que se indica:

a) Sondeo vesical por cuatro semanas

b) Sondeo vesical y pruebas de micción intermitente por cuatro semanas

c) Se da tratamiento médico

d) Se indica sondeo por tres meses

e) Se retira la sonda vesical una semana después de la intervención quirúrgica

GN272. En una paciente con embarazo del primer trimestre y un Ca Cu Ib1, la conducta correcta es:

a) Vigilancia estrecha

b) Histerectomía radical clase III

c) Se deja evolucionar el embarazo hasta alcanzar la madurez pulmonar del producto

d) Se indica radioterapia preparatoria

e) Se hace histerectomía clase II más radioterapia

GN273. Paciente femenino de 42 años de edad G-6, P-4, A-l, con reporte histopatológico de Ca epidermoide
con invasión al estroma de más de 4 mm. con embarazo de 15 semanas ¿que tratamiento indicaría?

a) Histerectomía radical

b) Radioterapia
c) Cono con asa diatérmica

d) Cono con láser

e) Asa diatérmica y seguimiento

GN274. El agente antineoplásico más utilizado en Ca Cu es:

a) Cisplatino

b) Vincristina

c) Leucovorin

d) CCNU

e) Melfalán

En relación con el cáncer cervicouterino, marque verdadero o falso en las aseveraciones siguientes:

GN275. La histerectomía radical es el tratamiento de elección en el Ca Ib2.

a) Verdadero b ) Falso

GN276._ Mujer de 35 años con Ca Cu IIa. Se trata electivamente con RT.

a) Verdadero b ) Falso

GN277. Mujer de 40 años con masa anexial y etapa IIa. Se trata electivamente con histerectomía radical.

a) Verdadero b ) Falso

GN278. El Ca Ia2 en mujer anciana puede tratarse exitosamente con RT.

a) Verdadero b ) Falso

GN279. Reporte de tres ganglios con Ca en histerectomía radical es indicación de radioterapia posoperatoria

a) Verdadero b ) Falso

GN280. ¿Cuál de las siguientes complicaciones de la radioterapia suele tener mayor morbilidad y requerir
incluso de cirugía?

a) Estrechez vaginal
b) Supresión hormonal en mujer joven

c) Enteritis proximal

d) Proctitis

e) Ninguna de las mencionadas

GN281. ¿Cuáles son los procedimientos quirúrgicos aceptados para tratamiento del carcinoma
microinvasor?

a) Histerectomía radical

b) Panhisterectomía

c) Radioterapia

d) Resección con asa electroquirúrgica

e) En la actualidad la mejor elección es b y d

GN282. La radioterapia adyuvante posterior a una histerectomía radical en Ca Cu invasor está indicada en:

a) Todos los casos

b) Presencia de márgenes positivos

c) Aquellos casos con metástasis ganglionares pélvicas

d) Nunca debe indicarse ya que se trata de un segundo procedimiento radical

e) Las situaciones indicadas en b y c

GN283. En cáncer del cuello uterino tratado quirúrgicamente (histerectomía radical) en etapas Ibl-Ib2 y IIa
¿Cuál es la complicación más frecuente?

a) Fístula vesicovaginal

b) Fístula úterovaginal

c) Linfoquiste

d) Atonía vesical

e) Trombosis pulmonar

En relación con las exenteraciones pélvicas por Ca Cu recurrente a radiación, marque verdadero o falso en las
aseveraciones siguientes:

GN284. El hallazgo quirúrgico de tumor fijo a pelvis contraindica su realización.


a) Verdadero b ) Falso

GN285. La exenteración total tiene igual morbimortalidad que la anterior.

a) Verdadero b ) Falso

GN286. La hidronefrosis previa determina cifras de curación del 15% aproximadamente.

a) Verdadero b ) Falso

GN287. Con ganglios metastásicos las cifras de curación son del 0%.

a) Verdadero b ) Falso

GN288. Las cifras globales de curación son de aproximadamente 40%.

a) Verdadero b ) Falso

GN289. La exenteración total tiene mejor pronóstico que la anterior.

a) Verdadero b ) Falso

GN290. Cuál es el impacto en el pronóstico de la radioterapia adyuvante poshisterectomía radical en Ca Cu


invasor:

a) No modifica la sobrevida

b) No modifica la tasa de recurrencias locales ni a distancia

c) Reduce la tasa de recurrencias locales

d) Mejora la sobrevida

e) Son ciertos c y d

GN291. Alternativa de tratamiento en mujeres con disgerminoma resecado completamente:

a) Si se corrobora ECIA mantenerse en observación

b) Si se corrobora ECIB administrar quimioterapia adyuvante

c) Administrar tres ciclos de quimioterapia con base en VAC

d) a y b

e) Rtx retroperitoneal más 3 ciclos de quimioterapia con base en PVB


GN292. Se llama al procedimiento realizado mediante laparatomía exploradora después de un tratamiento
previo a los tumores epiteliales cuando no hay datos clínicos laboratoriales ni radiologicos de positividad:

a) Cirugía de intervalo

b) Second look

c) Cirugía citorreductora suboptima

d) Cirugía optima citorreductora

GN293. Seleccione la indicación que considere más apropiada para enviar a su paciente con Ca de ovario
recurrente a cirugía citorreductora:

a) Pacientes que no responden a quimioterapia inicial se benefician de cirugía citorreductiva en


relación con masas tumorales refractarias

b) Cirugía citorreductiva completa solo es posible en 10% de las mujeres con cáncer de ovario
recurrente

c) La mejoría en sobrevida se debe a quimioterapia posoperatoria, más que a la cirugía


citorreductiva

d) Las cifras de sobrevida favorables de cirugía citorreductiva son uniformes en todos los estudios
publicados

e) La cirugía citorreductiva secundaria no debe sugerirse en pacientes con Karnofsky bajo

Correlacione las siguientes columnas respecto al Ca epitelial del ovario:

GN294. Cirugía estadificadora ( )

GN295. Second look ( )

GN296. Cirugía de citorreducción secundaria ( )

GN297. Cirugía conservadora ( )

GN298. Cirugía de intervalo ( )

a) Ca recurrente o residual

b) Ca virgen de tratamiento

c) Ca avanzado sin evidencia de actividad tumoral

d) Karnofsky 60% en Ca avanzado

e) Etapa I en paciente virgen

GN299. ¿Cuál es la probabilidad promedio de realizar una citorreducción óptima en etapas III y IV de
cáncer de ovario?

a) 50%
b) 60%

c) 70%

d) 35%

e) 25%

GN300. ¿Cuáles de los siguientes son los requisitos para cirugía conservadora de ovario?

a) Etapa Ia, con tumor bien diferenciado

b) Mujer joven, con deseo de embarazo, con pelvis normal, con posibilidad de seguimiento
estrecho

c) No antecedentes de cáncer de ovario hereditario

d) Mujer con vida sexual

e) Sólo a y b son requisitos

GN301. Tratamiento de quimioterapia, con mayor efectividad para tumores germinales:

a) Platino

b) VIP

c) BEP

d) VAC

e) Vinblastina

GN302. Cuál de los siguientes medicamentos se asocia a Ca de endometrio:

a) Progesterona

b) Tamoxifén

c) Anticonceptivos

d) Carotenos

e) Testosterona

GN303. El tratamiento quirúrgico para el Ca de endometrio en etapa clínica I es:

a) Histerectomía radical

b) Histerectomía extrafascial más SOB

c) Histerectomía extrafascial sin SOB

d) Histerectomía intrafascial
e) Histerectomía extrafascial más ooforectomía unilateral

GN304. En el cáncer de endometrio, son indicaciones para radioterapia adyuvante las siguientes:

a) Pacientes con metástasis glanglionares, intraperitoneales y en anexos

b) Pacientes con etapificación incompleta

c) Pacientes con tumores con grado histológico 3, citología peritoneal positiva e invasión al
espacio linfovascular

d) Pacientes con invasión al miometrio y cérvix

e) Todo lo anterior es verdadero

GN305. Son indicaciones de muestreo ganglionar retroperitoneal en Ca de endometrio las siguientes,


excepto:

a) Invasión a más de la mitad del miometrio

b) Variedad histológica endometrioide

c) Extensión extrauterina

d) Ganglios aumentados de tamaño

e) Extensión al estroma cervical

GN306. El tratamiento de elección para un paciente con Ca de endometrio etapa IIbG3 es:

a) RT intracavitaria + cirugía

b) Cirugía + hormonoterapia

c) Cirugía radical únicamente

d) Cirugía + quimioterapia adyuvante

e) Cirugía + RT pélvica y vaginal

GN307. La terapia hormonal paliativa para el cáncer endometrial avanzado y/o recurrente se acompaña de
respuestas objetivas en el porciento siguiente:

a) 0 a 10

b) 10 a 30

c) 30 a 60

d) 50 a 70
e) 100

GN308. La exenteración pélvica en el cáncer endometrial avanzado o recurrente:

a) Tiene una contraindicación absoluta

b) Indicada en casos muy seleccionados

c) Se acompaña de curaciones en menos del 10%

d) Se acompaña de curaciones en el 40%

e) b y d son correctas

f) a y c son correctas

GN309. La radioterapia adyuvante ha reportado beneficios en términos de mejoría en la sobrevida global y


sobrevida libre de recurrencias en pacientes con:

a) Metástasis ganglionares pélvicas y paraórticas

b) Pacientes con diseminación a los anexos y estructuras intraperitoneales

c) Pacientes con invasión al miometro, cervix y espacio linfovascular

d) Pacientes con citología peritoneal positiva (utilizando fósforo radioactivo)

e) Pacientes con etapificación quirúrgica incompleta

GN310. Paciente femenino de 60 años, con Ca de vulva izquierda T2 N1 M0 con invasión a labio mayor y
menor se trata con:

a) Vulvectomía simple más radioterapia a la ingle izquierda

b) Hemivulvectomía con radioterapia ingle izquierda

c) Vulvectomía más DR ingle izquierda

d) Vulvectomía radioterapia y quimioterapia

e) Vulvectomía radical con DRI bilateral

GN311. Además de la ooforectomía profiláctica, la única otra acción bien documentada que reduce la
probabilidad de desarrollar un cáncer de ovario es:

a) Mediciones frecuentes del marcador CA-125

b) Ultrasonografía transvaginal

c) Dieta baja en grasas


d) Anticonceptivos orales

e) Ejercicio tipo aerobics

GN312. Drogas de elección para la enfermedad del trofoblasto de alto riesgo:

a) Etopósido y actinomicina

b) Metotrexate y cisplatino

c) Etopósido, actinomicina, metotrexate

d) Vinblastina, metotrexate

e) Cisplatino y mostaza nitrogenada

GN313. Tratamiento de elección para la enfermedad trofoblástica no metastásica:

a) Etopósido solo

b) Metotrexate y cisplatino

c) Cisplatino y etopósido

d) Metotrexate y rescate con ácido folínico

e) Etopósido más metotrexate

GN314. En enfermedad metastásica del trofoblasto con mal pronóstico por metástasis cerebrales, el
tratamiento es:

a) Quimioterapia sola

b) Cirugía sola

c) Quimioterapia y radioterapia

d) Radioterapia sola

e) Ninguno de los anteriores

GN315. Cuando una paciente tiene una enfermedad de trofoblasto no metastásico y desea preservar su
fertilidad, el tratamiento inicial debe ser:

a) Resección uterina local

b) Quimioterapia de combinación

c) Legrados uterinos repetidos


d) Quimioterapia con monodroga con base en taxol o cisplatino

e) Quimioterapia con monodroga base en metotrexate o actinomicina D

GN316. Después de una remisión completa con quimioterapia para enfermedad de trofoblasto, a las pacientes se
les debe informar en las próximas consultas que:

a) Tendrán mayor riesgo de abortos espontáneos

b) Deben esperar una vida reproductiva normal

c) Van a tener mayor riesgo de requerir operación cesárea

d) Tendrán mayor número de anormalidades congénitas

e) Tendrán mayor riesgo de embarazo extrauterino

GN317. Femenina de 60 años de edad con diagnóstico de carcinoma de ovario etapa IIIa a la cuál se le
practicó cirugía radical. Recibe después su primer ciclo de quimioterapia a base de
Carboplatino+Ciclofosfamida y como terapia antiemética Ondansetron, Metoclopramida y Dexametasona
cada 8 h. Posteriormente presenta desviación de su cabeza hacia el lado izquierdo y desviación ocular del
mismo lado. La exploración neurológica no demuestra focalización. ¿Cuál sería el siguiente paso
recomendado?

a) Toma de ECG

b) Tomografía de encéfalo

c) Iniciar tratamiento con difenhidramina 25 mg IV

d) Inicio de Lorazepan 2 mg IV

e) Suspender metoclopramida

GN318. Tratamiento quirúrgico del Ca Cu Ia1 en mujer joven nuligesta sin factores de riesgo:

a) Histerectomía simple clase I

b) Histerectomía clase II

c) Histerectomía clase III

d) Cono cervical

e) Histerectomía vaginal

GN319. El tratamiento quirúrgico correcto para el Ca Cu Ia2 es:

a) Histerectomía clase II

b) Histerectomía clase III (Meigs)

c) Histerectomía vaginal
d) Cono cervical

GN320. El tratamiento de elección para las pacientes jóvenes portadoras de Ca Cu Ib1 sin contraindicación
quirúrgica es:

a) Histerectomía clase I

b) Histerectomía clase II

c) Histerectomía clase III (Meigs)

d) Radioterapia radical

e) Radioterapia más cirugía

GN321. El tratamiento de elección en pacientes menores de 50 años portadores de Ca Cu Ib que mide menos
de 4 cm de diámetro es:

a) Histerectomía clase I

b) Histerectomía clase II

c) Histerectomía clase III (Meigs)

d) Radioterapia radical

e) Radioterapia más histerectomía radical

GN322. Tipos histológicos más frecuentes de neoplasias malignas de ovario:

a) Tumores epiteliales, germinales y del estroma gonadal

b) Carcinomas, sarcomas y linfomas

c) Tumores mesequimatosos, neuroendocrinos y epiteliales

d) Tumores germinales, neuroendocrinos y del estroma gonadal

e) Ninguna de las anteriores

GN323. Constituyen más del 80% de todas las neoplasias malignas de ovario:

a) Sarcomas

b) Tumores germinales

c) Tumores del estroma gonadal

d) Tumores epiteliales

e) Tumores de Krukenberg
GN324. Subtipo histológico más frecuente de cáncer epitelial de ovario:

a) Tumor de Brenner

b) Adenocarcinoma seroso

c) Adenocarcinoma mucinoso

d) Adenocarcinoma endometrioide

e) Carcinoma de células claras

GN325. Dos principales vías de diseminación del cáncer epitelial de ovario:

a) Hematógena y linfática

b) Linfática y transperitoneal

c) Transperitoneal y hematógena

d) Hematógena y por contiguidad

e) Por contiguidad y linfática

GN326. Porciento de pacientes con tumor aparentemente confinado a ovario que tienen metástasis ocultas
en los ganglios paraórticos:

a) 60 a 80

b) 45

c) 10 a 25

d) 60

e) menos de 1

GN327. Se considera que se realizó una citorreducción óptima, en pacientes con cáncer epitelial de ovario,
cuando el tamaño del residual es:

a) Mayor a 5 cm

b) Entre 3 y 5 cm

c) Entre 2 y 3 cm

d) Menor a 2 cm

e) Ninguna es correcta
GN328. Corresponden a subtipos histológicos de hiperplasia endometrial dentro de la clasificación de
Kurman, excepto:

a) Simple sin atipias

b) Simple con atipias

c) Hiperplasia quística

d) Compleja sin atipias

e) Compleja con atipias

GN329. Porciento de pacientes con hiperplasia endometrial compleja con atipias del endometrio, en las que se
puede encontrar cáncer de la pieza quirúrgica?

a) 10

b) 60

c) 20

d) 80

e) 1

GN330. Porciento de pacientes con cáncer de endometrio que se diagnostican antes de la menopausia:

a) 5

b) 65

c) 25

d) 85

e) menos del 1

GN331. ¿Qué porciento de las pacientes con carcinoma de endometrio se diagnostican antes de los 40 años?

a) 50

b) 80

c) 40

d) 0

e) 5
GN332. Localización más frecuente del cáncer endometrial:

a) Segmento uterino

b) Cuerpo del útero

c) Fondo uterino

d) Paredes laterales

e) Pared anterior

GN333. ¿Qué porciento de las pacientes con cáncer de endometrio se diagnostican en etapa IV?

a) 30

b) 60

c) Menos de 5

d) Menos de 1

e) 20

GN334. Sitio más frecuente de recurrencia en cáncer de endometrio estadios I y II:

a) Hígado

b) Ganglios linfáticos

c) Vagina

d) Ingle

e) Pulmón

GN335. Segundo primario asociado con mayor frecuencia al carcinoma endometrioide del endometrio:

a) Cáncer broncogénico de células pequeñas

b) Carcinoma endometrioide de ovario

c) Cáncer cervicouterino invasor

d) Cáncer de glándula mamaria

e) Cáncer de vulva
GN336. Femenino de 35 años de edad, con tumor de 3x3 cm en el cuello uterino y con biopsia positiva para
carcinoma epidermoide invasor de células grandes no queratinizante. Se programa para histerectomía radical
con linfadenectomía pélvica bilateral y durante el transoperatorio se encuentra un ganglio en la cadena iliaca
interna de 3cm de diámetro mayor, necrótico y fijo a tejidos adyacentes. El patólogo informa que se trata de
metástasis de carcinoma epidermoide. ¿Qué conducta seguirá usted y a qué estadio pTNM correspondería el
caso?

a) Histerectomía radical con linfadenectomía pélvica bilatreral y radioterapia posoperatoria.


pTNM:IIB

b) Histerectomía radical con LPB y quimioterapia posoperatoria pTNM:IIIa

c) Histerectomía radical y LPB+Quimio y radioterapia posoperatorias IIIB pTNM

d) Cerrar a la paciente y enviarla a radioterapia IIIB pTNM

e) Cerrar a la paciente y enviarla a quimioterapia paliativa IVB pTNM

GN337. Constituyen el primer relevo ganglionar en caso de Ca Cu:

a) Paracervicales, parametriales y obturadores

b) Inguinales, retrocavos y presacros

c) Inguinales, paracervicales y paraórticos

d) Paracervicales, ilíacos internos y paraórticos

e) Parametriales, ilíacos externos e inguinales

GN338. Femenino de 25 años de edad, nulípara, hemorragia genital anormal intermestrual y colposcopía
satisfactoria con imagen sospechosa de CaCu microinvasor. Usted le efectúa una conización cervical y el
reporte de patología es de carcinoma epidermoide microinvasor, con profundidad de 1.5 mm, extensión
horizontal de 2 mm, sin patrón confluente y sin invasión al espacio linfovascular. ¿Cuál sería su conducta?

a) Histerectomía radical con linfadenectomía pélvica bilateral

b) Histerectomía extrafascial con linfadenectomía pélvica

c) Traquelectomía con linfadenectomía pélvica bilateral

d) Vigilancia estrecha con colposcopía semestral

e) Radioterapia ciclo pélvico completo

GN339. Son factores pronósticos para recurrencia local después de histerectomía radical por cáncer del
cuello uterino, excepto:

a) Invasión de más de 50% del estroma cervical

b) Invasión a más de dos tercios del estroma cervical

c) Parametrio positivo
d) Límite vaginal menos de 0.5 cm

e) Variedad epidermoide de células grandes

GN340. ¿Qué porciento de las pacientes con cáncer cervicouterino estadio 1B, programadas para
histerectomía radical con linfadenectomía pélvica bilateral, tienen evidencia histológica de metástasis en
ganglios paraórticos en el momento de la cirugía?

a) 15

b) 16 a25

c) 30

d) 2 a 5

e) menos de 1

GN341. Contraindicación absoluta para efectuar exenteración pélvica en pacientes con cáncer cervicouterino
tratado con radioterapia:

a) Parametrio fijo a pared pélvica

b) Tumor de más de 4 cm de diámetro mayor

c) Invasión a vejiga y a recto

d) Extensión hasta el tercio medio de la vagina

e) Intervalo libre de enfermedad menor a un año

GN342. El porciento de pacientes a los que se diagnostica adenocarcinoma del cuello uterino, a pesar de ser
una patología rara, es del:

a) 25

b) 5-10

c) 20

d) 0-1

e) 40

GN343. Los estudios aceptados por la FIGO para la etapificación del Ca Cu son:

a) Exploración física y biopsia

b) Exploración física, citología y biopsia

c) Resonancia magnética nuclear, TAC y telerradiografía de tórax


d) Exploración física y laparoscopía

e) Examen físico, telerradiografía de tórax, urografía excretora, rectoscopía y cistoscopía

GN344. Etapa clínica del Ca Cu donde la invasión al estroma no supera los 3 mm de profundidad y la
extensión no es mayor de 7 mm:

a) Ia

b) Ib

c) Ib1

d) Ia1

e) Ia2

GN345. Tumor germinal más frecuente que se presenta bilateralmente:

a) Teratoma inmaduro

b) Tumor de senos endodérmicos

c) Coriocarcinoma

d) Disgerminoma

e) Carcinoma embrionario

GN346. Tumor que se asocia frecuentemente al disgerminoma en el ovario contralateral:

a) Quiste endometriósico

b) Quiste folicular

c) Teratoma quístico maduro

d) Luteoma

e) b y c

GN347. Frecuencia que ocupa el cáncer del ovario entre los cánceres del tracto genital femenino en la mujer
americana:

a) 1er. lugar

b) 2º. lugar

c) 3er. lugar

d) 4º. lugar
e) 5º. lugar

GN348. La frecuencia del cáncer de ovario entre los cánceres del tracto genital femenino en la mujer
mexicana ocupa el:

a) 1er. lugar

b) 2º. lugar

c) 3er. lugar

d) 4º. lugar

e) 5º. lugar

GN349. La graduación histológica es primordial en la evaluación del siguiente tumor germinal de ovario:

a) Disgerminoma

b) Teratoma inmaduro

c) Carcinoma embrionario

d) Tumor de senos endodérmicos

e) Tumor germinal mixto

Anote en la línea o paréntesis la letra que corresponda a cada una de las aseveraciones siguientes
relacionándolas con los tumores germinales del ovario:

GN350. Cursa con elevación de alfafetoproteínas ( )

GN351. Cursa con elevación de gonadotrofina coriónica ( )

GN352. Cursa con elevación de Deshidrogenasa láctica ( )

a) Disgerminoma puro

b) Teratoma inmaduro

c) Tumor de senos endodérmicos

GN353. La deficiencia nutricional que más frecuentemente se asocia con molahidatiforme es la carencia de
la(s) vitamina(s):

a) A

b) B

c) C
d) D y K

GN354. Tipo de VPH más frecuentemente encontrado en carcinoma invasivo de vulva:

a) 6

b) 11

c) 16

d) 40

GN355. Cariotipo más fecuente para mola hidadiforme completa:

a) XY

b) XX

c) XXX

d) XYY

e) X0

GN356. Cariotipo más frecuente en mola parcial:

a) XY

b) XX

c) XYY, XXX y XXY

d) YY/X0

GN357. Factor NO relevante para el pronóstico del cáncer endometrial:

a) Edad menor de 60 años

b) Células positivas en el lavado peritoneal

c) Invasión de espacio perivascular

d) Invasión profunda del miometrio

e) Sobreexpresión de proteína P53

GN358. La sensibilidad de la determinación del CA-125 trimestral para control en el cáncer de ovario
tratado es:
a) 50%

b) 65%

c) 80%

d) 100%

e) 0-20%

GN359. El tumor de Brenner es considerado como:

a) Teratoma maduro

b) Tecoma

c) Derivado del epitelio celómico

d) Teratoma inmaduro

e) Fibroma

GN360. Frecuencia de metástasis ganglionares en el cáncer de vulva etapa I:

a) 0%

b) 20%

c) 40%

d) 60%

e) 80%

GN361. Frecuencia de metástasis ganglionares en el cáncer de vulva etapa III:

a) 0%

b) 20%

c) 50%

d) 80%

e) 100%

GN362. Nuestro porciento de error al diagnósticar las metástasis inguinales en el Ca de vulva se aproxima
al:

a) 0%

b) 20%
c) 50%

d) 80%

e) 100%

GN363. Factores pronósticos más significativos en Ca de vagina:

a) Etapa clínica, edad de la paciente e histología

b) Etapa clínica, grado de diferenciación y hematócrito

c) Etapa clínica, profundidad de la invasión tumoral y localización en vagina

d) Queratización, edad e histología

e) Grosor de la lesión, histología, no queratización y grado de diferenciación

GN364. En el cáncer del cuello uterino microinvasor E.C. Ia1 de la FIGO, son factores pronósticos
importantes los siguientes, excepto:

a) Profundidad de invasión

b) Invasión a espacios vasculares

c) Confluencia

d) Tipo histológico

e) Invasión a espacios linfáticos

GN365. El marcador tumoral CA-125 se encuentra elevado en:

a) El Ca epitelial de ovario

b) La endometriosis, adenomiosis, leimiomatosis uterina, enfermedad inflamatoria pélvica,


hepatitis alcohólica, falla renal

c) La menstruación

d) Los procesos inflamatorios de pleura, peritoneo y pericardio

e) Todos los anteriores

GN366. En melanoma de vulva, el sistema de etapificación que mejor define el pronóstico de la paciente es:

a) El de Breslow

b) El de Clark

c) El de Chung
d) El de Shah

e) Ninguno de los anteriores

GN367. Los casos tempranos de cáncer endometrial con contraindicación quirúrgica pueden tratarse
exitosamente con radioterapia. Las cifras de seguimiento a cinco años con tumor para estas enfermas son:

a) 100%

b) 85%

c) 60%

d) 40%

e) 30%

GN368. Las siguientes son características clínicas de tumores del estroma gonadal, excepto la (el):

a) Virilización

b) Pubertad precoz

c) Amenorrea

d) Sangrado posmenopáusico

e) Galactorrea

GN369. ¿Qué porciento del Ca de ovario es hereditario?

a) 5

b) 10

c) 15

d) 20

e) 25

GN370. Gen causante en la mayoría de los casos del síndrome hereditario de Ca de mama-ovario:

a) RAS

b) MYC

c) BRCA1

d) ERB
e) VHL

GN371. ¿Qué tumor eleva el marcador HGC-B?

a) Coriocarcinoma

b) Adenocarcinoma

c) Senos endodérmicos

d) Poliembrioma

e) Ginandroblastoma

GN372. La etapa IbG2 de la FIGO para el Ca de endometrio se define como tumor:

a) Moderadamente diferenciado sin infiltración miometrial

b) Que invade la mitad del miometrio y es moderadamente diferenciado

c) Bien diferenciado que se extiende hasta las glándulas endocervicales

d) Que invade la vagina y es moderadamente diferenciado

e) Moderadamente diferenciado con invasión mayor a la mitad del miometrio

GN373. ¿Cuales son los subtipos virales más frecuentes y de mayor poder oncogénico relacionados con Ca
Cu?

a) 6, 11

b) 16 y 31

c) 16 y 18

d) 18 y 33

e) 45 y 54

GN374. ¿Cuál es la definición estricta de Ca Cu microinvasor, según la Society Gynecologic Oncologist


(SGO)?

a) Invasión menor de 5 mm

b) Invasión menor de 3 mm

c) Ausencia de permeación linfovascular

d) Es cierto a y c

e) Es cierto b y c
GN375. El gen BRCA1, se ha asociado al cáncer ovárico a través del síndrome siguiente:

a) Síndrome de Lynch l

b) Síndrome de cáncer mamario y ovárico heredado

c) Síndrome de Lynch ll

d) Síndrome de Stein Levethal

e) Síndrome de cáncer de ovario de sitio especifico

GN376. El gen BRCA1 asociado al cáncer de ovario hereditario se ubica en:

a) Brazo largo cromosoma 5

b) Brazo corto cromosoma 13

c) Brazo largo cromosoma 17

d) Brazo largo cromosoma 11

e) Brazo corto cromosoma 6

GN377. La sobrevida actual a cinco años para el cáncer global de ovario en países industrializados
comoEstados Unidos es de alrededor de:

a) 25%

b) 45%

c) 60%

d) 75%

e) 15%

GN378. El uso del tamoxifén en el cáncer mamario, aumenta en el siguiente número de veces, el riesgo para
el desarrollo de cáncer endometrial:

a) 0 veces

b) 2 veces

c) 5 veces

d) 10 veces

e) 20 veces
GN379. Qué droga ingerida en el embarazo puede inducir adenocarcinoma de vagina en la hija?

a) Dietiletilbesterol

b) Clormadinona

c) Prednisona

d) Ranitidina

e) Talidomida

GN380. Factor pronóstico dominante en el Ca Cu epidermoide:

a) Estirpe tumoral

b) Tipo de tratamiento

c) Resistencia al tratamiento con RT

d) Infiltración a parametrios

e) Tamaño tumoral y estado ganglionar

GN381. En las neoplasias intraepiteliales del cérvix uterino, la clasificación de Behesda, se refiere a:

a) Displasia I, II y III

b) Displasia moderada y severa

c) NIC 1, 2 y 3

d) Lesiones intraepiteliales de alto y bajo grado

e) La citología cérvicovaginal

GN382. El Ca Cu neuroendocrino de células pequeñas tiene un pronóstico:

a) Bueno

b) Igual al epidermoide

c) Mejor que el adenocarcinoma

d) Muy malo

e) Desconocido

GN383. Ante el diagnóstico de tumor germinal de ovario, ¿En qué casos se justifica la biopsia del ovario
contralateral?

a) Cuando se identifican folículos en el ovario


b) En presencia de anormalidad macroscópica

c) Cuando el tumor está en contacto con el ovario

d) Si al excederse el tumor se rompe la cápsula

e) Ninguno de los anteriores

GN384. Conducta más apropiada en mujer joven con tumor germinal bilateral:

a) Rutina de ovario

b) Salpingooforectomía bilateral

c) Biopsia con ETO y quimioterapia

d) Ooforectomía unilateral más quimioterapia

e) c y d

GN385. Fármaco básico en el tratamiento de tumores de ovario tipo epitelial:

a) Ciclofosfamida

b) Platino

c) Bleomicina

d) Etopósido

e) Todos

GN386. Desde el punto de vista histopatológico el carcinoma de cérvix de mayor supervivencia corresponde
al:

a) Adenocarcinoma poco diferenciado

b) Carcinoma de células claras

c) Carcinoma adeno escamoso

d) Carcinoma epidermoide de células grandes no queratizante

e) Carcinoma epidermoide moderadamente diferenciado queratizante

GN387. Paciente portadora de Ca Cu etapa IIb, con urografía excretora + hidronefrosis izquierda, se
clasifica como:

a) Etapa IIb

b) Etapa IVa
c) Etapa IIIb

d) Etapa IIa

e) Fuera de clasificación

GN388. Paciente femenina de 30 años con tumor carcinoide del cérvix en etapa Ib se trata con:

a) Cono de cérvix y vigilancia

b) Laparotomía y Panhisterectomía

c) Criocirugía

d) Exenteración anterior

e) Histerectomía radical con SOU y Linfadenectomía Pélvica

GN389. La determinación de antígeno CA-125 suele reportarse negativa predominantemente en el cáncer:

a) Endometroide ovárico

b) Seroso ovárico

c) Mucinoso ovárico

d) Ovárico de células claras

e) De endometrio

GN390. ¿Cuál de los parámetros siguientes es más fiel para sospechar recurrencia en Ca epitelial de ovario?

a) Evaluación clínica periódica

b) TAC de pelvis y abdomen

c) Urografía excretora

d) Papanicolaou vaginal

e) Determinación de CA-125 con cifras anormales

GN391. En pacientes con Ca Cu temprano, operadas con histerectomía radical y lifadenectomía pélvica y las
cuales posteriormente recibieron radioterapia complementaria, ¿Cuáles fueron los factores pronósticos más
importantes que determinaron esta conducta?

a) No. de ganglios positivos y reportados más histología

b) Tamaño del tumor, edad y no. de ganglios positivos

c) Edad, extensión endometrial y VPH 16-18


d) Tamaño del tumor, HLA y VPH 16-18

e) Ninguno de los anteriores

GN392. En qué mujeres estaría recomendado realizar ooforectomía profiláctica:

a) Si la madre de ésta murió por cáncer de ovario

b) Si la madre, tía materna y hermana presentaron cáncer de ovario

c) Si padece ovarios poliquísticos

d) Si es hipotiroidea y nulípara

e) Mujer portadora del síndrome Lynch II

GN393. En un teratoma ovárico se pueden originar neoplasias malignas, entre las cuales se pueden
mencionar:

a) Carcinoma de células escamosas, adenocarcinomas

b) Sarcomas, tumores neuroectodérmicos

c) Melanomas malignos, carcinomas de células basales

d) Se pueden desarrollar cualquiera de los tumores mencionados

e) No se desarrollan neoplasias malignas

GN394. Incidencia de Ca Cu invasor en el embarazo:

a) 1: 2205 (0.045%)

b) 1: 500 (0.20%)

c) 1: 200 (0.50%)

d) 1: 20000 (0.005%)

e) 1: 15000 (0.006%)

GN395. La RT adyuvante se indica después de una histerectomía radical por un Ca Cu Ib1,cuando:

a) Los bordes quirúrgicos están libres de lesión

b) Existen parametrios libres

c) Un ganglio pélvico en positivo

d) Dos ganglios pélvicos son positivos

e) Tres o más ganglios pélvicos son positivos


GN396. Neoplasia vaginal maligna más frecuente en niñas:

a) Variante botrioide del rabdomiosarcoma

b) Melanoma

c) Linfoma

d) Carcinoma de células claras

e) Carcinoma de células basales

GN397. QT con mayor efectividad para tumores germinales:

a) Platino

b) VIP

c) BEP

d) VAC

e) Vinblastina

GN398. Síntoma más común asociado al Ca de ovario en edad pediátrica:

a) Dolor

b) Distensión

c) Sangrado

d) Masa pélvica

e) Obstrucción intestinal

GN399. La frecuencia de recurrencias vaginales en Ca de endometrio etapa I de bajo riesgo tratado con
cirugía es de:

a) Menor al 5%

b) 10-15%

c) 5-10%

d) 15-20%

e) Más de 20%

GN400. ¿Cuál es la importancia de la colposcopía, en relación con otros estudios de sospecha en las lesiones
cervicales?
a) Con una sensibilidad diagnóstica del 60%

b) Con una sensibilidad diagnóstica del 80%

c) Con experiencia y biopsia dirigida, 95% de especificidad diagnóstica

d) De poca sensibilidad

e) Ninguno

GN401. Los linfoquistes en una paciente con histerectomía radical clase III se presentan en 0.4-2% ¿Cuál es
la indicación para drenarlos?

a) Cuando hay dolor importante en pelvis

b) Cuando hay infección

c) Hay sensación de plenitud, urgencia urinaria, obstrucción ureteral y pielonefritis

d) Cuando hay fiebre y dolor

e) Sólo cuando hay urgencia urinaria

GN402. En el Ca Cu Ib y IIa, tratados con histerectomía radical con reporte de más de tres ganglios
positivos, la RT se indica porque:

a) La recurrencia es igual

b) La sobrevida es la misma

c) Aumenta la morbilidad

d) Disminuye la recurrencia pélvica hasta en un 27%, comparada con 67% de pacientes que no
reciben RT

e) No se deben indicar dos tratamientos radicales

GN403. Factor(es) que favorece(n) la baja respuesta a la QT en el Ca Cu:

a) Karnofki 50% + anemia

b) Anemia

c) Desnutrición

d) Cambios locales posradioterapia y cirugía

e) Diabetes

GN404. En un disgerminoma ovárico de 15 cm unilateral:

a) Es necesario completar panhisterectomía


b) Debe completarse tratamiento con radioterapia

c) Debe mantenerse conducta expectante

d) Debe darse quimioterapia

e) Debe resecarse el otro anexo por riesgo de recurrencía

GN405. En una paciente con 24 semanas de gestación y diagnóstico de Ca Cu etapa Ia1, la conducta
adecuada es:

a) Interrumpir el embarazo de inmediato

b) Practicar cesárea e indicar radioterapia

c) Vigilar al producto hasta que alcance la madurez pulmonar, cesárea y tratamiento oncológico
adecuado

d) Practicar histerectomía clase I

e) Indicar radioterapia con el útero gestante

GN406. ¿Qué grupos de pacientes con tumor germinal de ovario se benefician del second-look?

a) Los casos con citorreducción óptima inicial

b) Casos con citorreducción óptima y hallazgo de teratoma

c) Pacientes con resección incompleta durante la citorreducción primaria y que contenían


elementos de teratoma

d) En presencia de componentes mixtos de más del 50% con disgerminoma y menos del 50% de
teratoma

e) En presencia de combinación de tumor de senos endodérmicos, coriocarcinoma y


disgerminoma

GN407. Paciente de 25 años con estudio histopatológico que reporta invasión al estroma de más de 5 mm,
con extensión superficial que abarca todo el labio anterior. ¿Qué tratamiento se indicaría?

a) Cono con asa diatérmica

b) Cono con bisturí frío

c) Cono con láser

d) Histerectomía radical

e) Panhisterectomía

GN408. En una mujer joven nulípara con cáncer de ovario de células germinales etapa clínica temprana, el
tratamiento quirúrgico es:
a) Cirugía radical

b) Histerectomía total más SOB

c) Histerectomía más ooforectomía del ovario afectado

d) Ooforectomía del ovario tumoral y exploración macroscópica del ovario contralateral

e) Salpingooforectomía bilateral

GN409. Factor que favorece la pobre respuesta a la quimioterapia:

a) Karnofki

b) Anemia

c) Desnutrición

d) Cambios locales posradioterapia y cirugía

e) Diabetes

GN410. La terapia que ha tenido valor significativo en análisis multivariados, en relación a que influye en la
sobrevida de las pacientes con Ca de cérvix, es:

a) Taxol

b) Taxol + platino

c) 5FU+ leuvocorín

d) Platino + radioterapia

e) Gemcitabina

GN411. ¿En qué etapificación está indicada la radioterapia durante el embarazo?

a) Etapas avanzadas

b) De IIb a IVb

c) De Ib a IIa

d) Ninguna de las anteriores

e) Sólo a y b son correctos

GN412. De los siguientes, cuál es el paso inicial que debe realizarse dentro de la estrategia terapéutica de las
enfermedades malignas de ovario:

a) TAC
b) Punción de médula ósea

c) Paracentesis

d) Laparotomía exploradora

e) Quimioterapia neoadyuvante

GN413. En paciente de 60 años con ascitis y ecosonograma positivo para masas peritoneales se reportó
adenocarcinoma en la citología de la paracentesis el CA-125 es de 350 y demás estudios normales, el paso
siguiente sería:

a) Repetir la paracentesis para estudios de inmunoperoxidasa y microscopía electrónica

b) Laparoscopía con biopsia

c) Laparotomía con citorreducción máxima

d) Estudios endoscópicos de tubo digestivo alto

e) Quimioterapia con taxol y cisplatino

GN414. Los dos relevos ganglionares hacia los que drena primariamente el cáncer de ovario son:

a) Inguinales e ilíacos externos

b) Paraórticos y presacros

c) Parametriales e hipogástricos

d) Paraórticos e ilíacos externos

e) Inguinales y presacros

GN415. Etapa a la que corresponde el caso de una paciente con cáncer epitelial de ovario, con un reporte
histopatológico de invasión a la cápsula ovárica, lavado peritoneal con células malignas, citología de cúpula
diafragmática positiva para células malignas y ganglio inguinal metastásico de 2 cm:

a) IV

b) Ic

c) IIIc

d) Ivb

e) IIIa

GN416. Factor pronóstico más importante en pacientes con cáncer epitelial de ovario en estadio IIIc,
sometidas a cirugía de citorreducción:
a) Grado de diferenciación del tumor

b) Invasión linfovascular

c) Tamaño del tumor primario

d) Tamaño del residual

e) Subtipo histológico

GN417. ¿Cuál sería su conducta ante el caso siguiente? Femenino de 40 años de edad, dos meses de evolución con
adenopatía supraclavicular izquierda de 1.5 cm de diámetro mayor, aumento del perímetro abdominal, ECOG 1,
CA125=800 ng/mL, TAC abdominopélvica con reporte de tumor de ovario derecho y ascitis. Telerradiografía de
tórax normal. Reporte de la biopsia excisional de ganglio supraclavicular izquierdo: Adenocarcinoma seroso
metastásico.

a) Radioterapia a región supraclavicular izquierda y quimioterapia paliativa

b) Citología de ascitis y si es positiva para malignidad: tres ciclos de quimioterapia y cirugía de


intervalo

c) Citorreducción primaria y seis ciclos de quimioterapia adyuvante

d) Cirugía de máximo esfuerzo y quimioterapia intraperitoneal

e) Cirugía de máximo esfuerzo y tamoxifén paliativo

GN418. ¿Cuál sería su conducta ante el caso siguiente? Femenino de 21 años, gesta O, inicio de vida sexual
activa hace un año. 4 meses de evolución con dolor pélvico de baja intensidad. US pélvico tumor sólido de
ovario derecho, de 10 x 10 cm. Usted somete a la paciente a salpingooforectomía derecha con DHL
preoperatoria de 500 u/mL y con RHP transoperatorio de: Disgerminoma puro. El único hallazgo de
diseminación extraovárica en ese momento es un implante en el epiplón mayor de 1 cm, que es reportado
como metástasis de disgerminoma, el ovario contralateral es macroscópicamente normal.

a) Cirugía citorreductora radical y quimioterapia adyuvante

b) Salpingooforectomía derecha ya efectuada y omentectomía infracóloca, quimioterapia


adyuvante

c) Salpingooforectomía derecha ya efectuada, omentectomía infracólica lavado peritoneal y


biopsia de ganglios pélvicos y paraórticos, citología de cúpula diafragmática y quimioterapia
adyuvante

d) Salpingooforectomía derecha, biopsia de epilón (ya efectuados) y quimioterapia. Valoración


después de tres ciclos con TAC abdominopélvica y citorreducción radical de intervalo

e) Salpingooforectomía derecha, biopsia de epiplón (ya efectuados) y radioterapia pélvica y


paraórtica

GN419. Subtipo histológico más frecuente de adenocarcinoma de endometrio:

a) Papilar-seroso

b) Mucinoso
c) De células claras

d) Endometrioide

e) Poco diferenciado

GN420. El reporte histológico de la pieza quirúrgica de una laparotomía etapificadora por adenocarcinoma
de endometrio es el siguiente: Adenocarcinoma moderadamente diferenciado que infiltra menos del 50% del
miometrio, sin evidencia de enfermedad extrauterina (lavado peritoneal y biopsias de ganglios linfáticos:
negativos). ¿A qué etapa corresponde, según la clasificación de la FIGO?

a) 1AG1

b) 1BG2

c) 1CG3

d) 1AG2

e) Ninguna de las anteriores

GN421. Constituyen los dos factores pronóstico más importantes, en casos de cáncer endometrial confinado
al útero:

a) Invasión vascular y linfática

b) Profundidad de invasión endocervical y subtipo histológico

c) Profundidad de invasión miometrial y tamaño uterino

d) Grado histológico y profundidad de invasión miometrial

e) Grado e invasión linfovascular

GN422. ¿En cuál de los casos siguientes efectuaría usted una histerectomía radical tipo III de Piver con
linfadenectomía pélvica bilateral, en un paciente con diagnóstico de adenocarcinoma del endometrio,
clínicamente confinado a útero?

a) Invasión miometrial mayor al 50%

b) Extensión a glándulas endocervicales

c) Metástasis al ovario

d) Invasión al estroma cervical

e) Permeación vascular y linfática

GN423. Porciento de sobrevida global a cinco años para pacientes con cáncer del cuello uterino en estadio
clínico 1B1:

a) 90
b) 78

c) 60

d) 50

e) menos de 50

GN424. Para que un carcinoma de muñón cervical, en caso de histerectomía supracervical previa, sea
clasificado como verdadero, ¿Cuánto tiempo mínimo debe de haber pasado de la histerectomía?

a) 6 meses

b) 1 año

c) 2 años

d) 5 años

e) 10 años

De acuerdo a la clasificación de los tumores uterinos, correlacione las columnas siguientes:

GN425. Sarcoma del estroma endometrial ( ) a) Tumor epitelial


GN426. Papilar endometrioide ( ) b) Tumor mesenquimatoso
GN427. Tumor muleriano mixto maligno ( ) c) Tumor mixto
GN428. Adenosarcoma ( )
GN429. Papilar seroso ( )

GN430. Del marcador tumoral CA-125 la cifra más alta considerada como normal es:

a) 25 U/mL

b) 35 U/mL

c) 50 U/mL

d) 75 U/mL

e) 125 U/mL

GN431. Un tumor vulvar confinado a vulva y periné, de 2 cm y con invasión del estroma de más de 1mm, se
clasifica como T... según el sistema TNM:

a) TX

b) T1a

c) T1b

d) T2
e) T3

GN432. ¿Qué porciento de todos los rabdomiosarcomas de vagina, se presentan en las niñas de 5 años?

a) 5

b) 15

c) 35

d) 75

e) 95

GN433. En relación con el carcinoma de cuello uterino, cuando existe una infiltración tumoral de 12 mm, la
posibilidad de presentar metástasis ganglionar es de:

a) cero

b) 16

c) 26

d) 45

e) 65

GN434. El carcinoma cervicouterino Ib presenta una sobrevida a 5 años del 92.4%, pero si existe infiltración
al endometrio la sobrevida a 5 años es del:

a) 17.3%

b) 30.2%

c) 53.8%

d) 92.4%

e) 98.5%

En el carcinoma cervicouterino avanzado tratado únicamente con radioterapia las recurrencias se presentan
con la distribución porcentual siguiente:

GN435. Pulmón ( ) a) 3
GN436. Ganglios paraórticos ( ) b) 4
GN437. Ganglios supraclaviculares ( ) c) 7
GN438. Hígado ( ) d) 11
GN439. Ganglios inguinales ( ) e) 21
GN440. La nuliparidad incrementa el riesgo de carcinoma endometrial en:

a) 2 veces

b) 10 veces

c) 15 veces

d) 25 veces

e) 50 veces

GN441. El teratoma quístico maduro se presenta en ovario bilateralmente en un:

a) 5%

b) 12%

c) 24%

d) 36%

e) 50%

GN442. Cuándo una lesión epitelial maligna de endometrio afecta hasta las glándulas endocervicales ¿qué
etapa clínica es? (FIGO 1988)

a) 1B

b) 1C

c) IIA

d) IIB

e) IIIA

GN443. Cuándo una lesión epitelial maligna de endometrio afecta hasta el estroma endocervical qué etapa
clínica es? (FIGO 1988)

a) 1B

b) 1C

c) IIA

d) IIB

e) IIIA

GN444. La incidencia de este tumor comprende sólo alrededor del 3-5% de todos los tumores uterinos:
a) Sarcomas

b) Adenocarcinoma

c) Melanoma

d) Linfoma

e) Leiomioma celular

GN445. Es la clasificación más empleada para los sarcomas uterinos:

a) FIGO

b) GOG

c) Ober

d) TNM

e) Chung

GN446. La estadificación de los sarcoma uterinos se realiza:

a) Por clínica

b) Quirúrgicamente

c) Por patología

d) Radiológicamente

e) Con inmunohistoquímica

GN447. Es el sarcoma uterino más frecuente:

a) Leiomiosarcoma

b) Sarcoma del estroma endometrial

c) Adenosarcoma

d) Carcinosarcoma

e) Sarcoma mixto Mulleriano

GN448. Es el segundo cáncer invasivo más frecuente de la vulva:

a) Epidermoide

b) Melanoma

c) Adenocarcinoma
d) Sarcoma

e) Indiferenciado

GN449. Sitio anatómico más frecuente de presentación del melanoma vulvar:

a) Labios menores y clítoris

b) Labios mayores

c) Horquilla

d) En espejo

e) Periuretral

GN450. Son los factores pronósticos más importantes en melanoma vulvar:

a) Tamaño de la lesión y profundidad de invasión

b) Edad

c) Localización

d) Tiempo de evolución

e) Tratamiento

GN451. Neoplasia ginecológica maligna con mayores posibilidades de curación:

a) Cáncer de ovario epitelial

b) Cáncer cervicouterino

c) Neoplasia trofoblástica gestacional

d) Cáncer de endometrio

e) Cáncer de vulva

GN452. Son características de la mola completa, excepto:

a) Proliferación trofoblástica

b) Degeneración hidrópica

c) Ausencia de vascularización

d) 90% tiene cariotipo normal

e) Se identifica placenta y feto


GN453. Son indicaciones de quimioterapia en la neoplasia trofoblástica gestacional:

a) HCG mayor de 20,000 UI/L por más de cuatro semanas posterior a evacuación

b) Evidencia histológica de coriocarcinoma

c) Evidencia de metástasis

d) Hemorragia uterina aguda

e) Valores crecientes de HCG

GN454. En el seguimiento de la neoplasia trofoblástica gestacional se realiza lo siguiente, excepto:

a) Determinación semanal de HCG hasta obtener dos títulos normales

b) HCG bimensual por 6-12m

c) Anticonceptivos 6-12m

d) Tele de tórax mensual

e) Exploración física, incluyendo pélvica cada 2 sem hasta remisión

GN455. Porciento que ocupan los tumores germinales en los tumores ováricos:

a) 85-90

b) 20%

c) 2

d) 3-5

e) 1

GN456. Porciento de bilateralidad de disgerminoma:

a) 50

b) 10-15

c) 20 a 30

d) 1

e) 38 a 50
GN457. Son características de los disgerminomas, excepto:

a) Diseminación linfática

b) Sensibilidad a radioterapia

c) Elevación de AFP

d) Hasta un 10-15% eleva HCG

e) Elevación de DHL

GN458. Tumor germinal que eleva AFP:

a) Disgerminoma

b) Tumor de seno endodérmico y carcinoma embrionario

c) Coriocarcinoma

d) Poliembrioma

e) Teratoma

GN459. Son características del teratoma inmaduro, excepto:

a) Representa menos de 1% de los teratomas ováricos

b) Es más frecuente en las primeras dos décadas

c) Prácticamente nunca es bilateral

d) Contiene únicamente elementos neurales maduros

e) La cantidad de tejido neural inmaduro determina por sí sola el grado

GN460. Es el tratamiento de elección en disgerminoma:

a) Cirugía radical

b) HTA+SOU

c) Tumorectomía

d) Oforectomia y exploración contralateral

e) SOB

GN461. Los sarcomas uterinos son lesiones con pronóstico desfavorable, ¿Cuál es la incidencia de este tipo
de tumores dentro de todos los tumores uterinos?

a) 3-5%
b) 10-15%

c) 15-30%

d) 30-40%

e) 40-50%

GN462. El sarcoma uterino heterólogo se refiere a una lesión:

a) Una estirpe celular

b) Con varias estirpes celulares

c) Sólo con tejido de origen uterino

d) Con tejido diferente al tejido uterino

e) Con diferente tejido al uterino y varias estirpes celulares

GN463. Único factor etiológico relacionado con los sarcomas uterinos:

a) Exposición al dietiletilbestrol

b) Ingesta crónica a hormonas

c) Exposición a radioterapia

d) Manejo previo con platino

e) Exposición crónica a imidazoles

GN464. No es factor pronóstico para desarrollar enfermedad metastásica en los sarcomas uterinos:

a) Actividad mitósica

b) Histología

c) Grado Nuclear

d) Tamaño tumoral

e) Profundidad de la lesión

GN465. La clasificación de Ober divide a los sarcomas uterinos en:

a) Simples y mixtos

b) Autólogos y heterólogos

c) Homólogos y heterólogos
d) Puros y mixtos

e) Células grandes y pequeñas

GN466. La supervivencia a 5 años en etapa clínica I es de:

a) 10%

b) 30%

c) 50%

d) 70%

e) 80%

GN467. Es el sarcoma uterino más frecuente:

a) Sarcoma del estroma endometrial

b) Leiomiosarcoma

c) Sarcoma mesodérmico mixto

d) Hemangiosarcoma

e) Coriosarcoma

GN468. ¿Cuál es la etapa clínica más frecuente de los sarcomas uterinos?

a) I

b) II

c) III

d) IV

e) IVb

GN469. ¿Cuál es el papel de la RT en los sarcomas uterinos?

a) Ninguno

b) Aumentar la supervivencia (SV)

c) Aumento del período libre de enfermedad (PLE)

d) Curación

e) Aumenta la SV y el PLE
GN470. ¿Cuáles son las drogas más activas en los sarcomas uterinos?

a) Adriamicina y ciclofosfamida

b) Platino, doxorrubicina e ifosfamida

c) Mitomicina C

d) Ciclofosfamida

e) Metrotexato

GN471. En el leiomiosarcoma uterino ¿cuáles son los factores más importantes para predecir su malignidad?

a) Pleomorfismo e infiltrado linfocitario

b) Atipias celulares, necrosis e índice mitótico

c) Mitosis y pleomorfismo nuclear

d) Mitosis solamente

e) infiltrado linfocitario

GN472. ¿A qué edad es más frecuente el leiomiosarcoma uterino?

a) 30 años

b) 40 años

c) 50 años

d) 70 años

e) 80 años

GN473. No es factor de buen pronóstico en el leiomiosarcoma uterino:

a) Posmenopausia

b) Tumor en un mioma

c) Premenopausia

d) Menos de 5 mitosis

e) Seudocápsula

GN474. El pronóstico a 5 años en el leiomiosarcoma uterino grado 1-2 es de:

a) 10%
b) 30%

c) 45%

d) 60%

e) 85%

GN475. ¿Qué porciento de metástasis ganglionares presenta el leiomiosarcoma uterino?

a) 10

b) 15

c) 25

d) 4

e) 20

GN476. ¿Qué porciento de ganglios positivos presenta un tumor mesodérmico maligno de útero?

a) 2

b) 5

c) 9

d) 18

e) 35

GN477. ¿Cuál es el papel de la ploidia del ADN en los leiomiosarcomas uterinos?

a) Ninguno

b) Aumenta el riesgo de recaída

c) Mayor respuesta a la QT

d) mayor supervivencia

e) Pronóstico

GN478. Los tumores mixtos mullerianos del útero son:

a) Menos agresivos que los leiomiosarcomas

b) De comportamiento más local

c) Más agresivos que el leiomiosarcoma

d) Igual de agresivos que el leiomiosarcoma


e) De comportamiento desconocido

GN479. Los elementos heterólogos del sarcoma uterino confieren a la lesión:

a) Ser más benigna

b) Ser más agresiva

c) Indistinto

d) Hacer menos metastasis ganglionares

e) Mayor sobrevida

GN480. ¿Cuál es la supervivencia a cinco años del tumor mixto maligno del útero?

a) 30%

b) 40%

c) 20%

d) 50%

e) 0%

GN481. ¿Cuál es el factor pronóstico más importante en el tumor mixto mulleriano del útero?

a) Infiltración vascular

b) Tamaño tumoral

c) Profundidad de invasión

d) Elementos homólogos

e) Invasión linfática

GN482. ¿Cuál es el estado actual del tamizaje para cáncer de ovario?

a) En la población general no debe realizarse más que en protocolos de estudio

b) En mujeres de riesgo con historia familiar tienen un RR de 3.1

c) Es posible detectar una población cautiva realizando pruebas de BRCA1-2

d) La combinación del examen clínico, Ca-125 y US doppler transvaginal permiten identificar a la


población de riesgo

e) Las pacientes con cáncer de ovario curiosamente son las que menos factores de riesgo poseen
GN483. La ooforectomía profiláctica tiene un papel preponderante en uno de los casos siguientes:

a) Se recomienda en paciente que poseen uno o más familiares con cáncer de ovario, pues
poseen un RR de 3.6 de presentar cáncer de ovario

b) El dejar los ovarios después de una histerectomía implica un 20% de riesgo de cáncer en el
futuro

c) En la mujer posmenopáusica debe realizarse siempre que se realice una laparotomía por
cualquier otra causa

d) Debe practicarse ooforectomía bilateral siempre que se realice histerectomía a partir de los 40
años de edad

GN484. Se conocen como factores de riesgo para justificar quimioterapia adyuvante en etapas tempranas:

a) En todos los casos con grado de diferenciación II y III

b) Siempre que se realice cirugía preservadora de la fertilidad

c) Si hubo ruptura de la cápsula del tumor

d) En las estirpes mucinoso, células claras, indiferenciados

e) En pacientes operadas de manera incompleta y reintervenidas para completarse la rutina de


ovario

GN485. Son pacientes en quienes se justifica reestadificarlos a pesar de previa laparotomía exploradora:

a) A quien únicamente se le realizó biopsia

b) A quien se le intentó realizar una rutina completa

c) Quien clínicamente y por TAC no evidencia de AT

d) Con Ca-125 arriba de 1000uI/dl, TAC positiva y ascitis masiva

e) Con presencia de tumor palpable, ascitis masiva y fondo de saco de Douglas positivo

GN486. ¿En qué caso se justificaría la apendicectomía en la rutina de ovario?

a) Por subtipo histológico mucinoso

b) Si existe presencia de implantes mesentéricos

c) Cáncer de ovario epitelial bilateral

d) Tumor germinal del ovario derecho

e) Líquido de ascitis positivo


GN487. Situación en la que sería factible cirugía conservadora por cáncer epitelial de ovario:

a) Cáncer de ovario epitelial localizado a un ovario y cápsula íntegra

b) Estadio clínico IA grado I

c) Tumor limitado a un ovario, lavado peritoneal negativo

d) Mujer en etapa reproductiva, estadio IA, grado I

e) Mujer premenopáusica, tumor circunscrito a un ovario y cápsula íntegra

GN488. Condiciones por imagen que contraindican un intento de cito-reducción primaria:

a) Actividad hepática intraparenquimatosa y derrame pleural

b) Derrame pleural, ganglios peritoneales e inguinales

c) Actividad tumoral en hilio esplénico, retroperitoneal y hepático

d) Tumor pélvico, tacto vaginal positivo, ascitis loculada y derrame pleural

e) Tumor en mesenterio, tejidos blandos, hepático y ganglios inguinales

GN489. Indicaciones de omentectomía supramesocólica:

a) Presencia de tumor en el epiplón inframesocólico

b) Invasión por tumor en el epiplón supramesocólico

c) Ascitis masiva, tumor en la cápsula de Glisson

d) Ascitis masica, “omental cake”.

e) Ascitis masiva, actividad en el epiplón inframesocólico c/s en el supramesocólico

GN490. La presencia de metástasis hepáticas intraparenquimatosas es sinónimo del estadio:

a) IIIA

b) IV

c) IIIB

d) IIIC

e) IVB

GN491. La probabilidad de citorreducción óptima en un estadio IV por derrame pleural es de alrededor del:
a) 35%

b) 30% pero puede alcanzar el 76%

c) 40 %, y con quimioterapia de inducción se consigue hasta el 76%

d) 35% pero conlleva hasta un 48% de morbilidad

e) 35%, pero con un esfuerzo quirúrgico máximo se logra hasta el 90%

GN492. Se considera criterio de alto riesgo y pobre pronóstico en tumores epiteliales de ovario:

a) Estirpes endometrioide, células claras e indiferenciado

b) Pobremente diferenciados, Ca-125 con cifras superiores a los 2000 mUI/L

c) Tumores de alto grado, extensión a cérvix y ganglios positivos

d) Pobre respuesta al esquema de CDDP/CFM

e) Citorreducción primaria subóptima

GN493. ¿Existe algún papel para la laparotomía de segunda vista?

a) Fuera de protocolos de estudio es un procedimiento que no se justifica

b) Con intento de ensayos de nuevas líneas de quimioterapia

c) Si existe enfermedad residual posterior a quimioterapia de primera línea

d) No ha demostrado beneficio en la supervivencia

e) Como consolidación de QT con estudios de imagen y marcador negativos

GN494. ¿En la ruptura de la cápsula en un tumor de ovario cambia la estadificación?

a) Se modifica únicamente el pronóstico y debe señalarse el incidente

b) Un tumor IA se reestadifica en IC

c) Se le considera ECIIC

d) La paciente debe recibir quimioterapia adyuvante

e) La paciente es candidata a quimioterapia intraperitoneal

GN495. En relación a laprogresión a quimioterapia adyuvante y la cito-reducción secundaria, se puede decir


lo siguiente:
a) La citorreducción secundaria no tiene beneficio alguno

b) Las pacientes deben someterse a segunda línea de quimioterapia

c) Deben administrarse cuando menos tres ciclos de QT y decidir conducta

d) Administrarse los seis ciclos de QT y dejarse en vigilancia

e) Normar la conducta en base al descenso del Ca-125

GN496. ¿Cuál es el argumento a favor de la laparotomía de intervalo?

a) Optimiza el efecto de la quimioterapia

b) Permite la citorreducción óptima en el 76% y morbilidad del 20%

c) Evita el retraso en la administración de QT con la citorreducción primaria

d) Ofrece un periodo libre de enfermedad superior al compararse con aquellas pacientes


citorreducidas óptimamente desde la laparotomía primaria

e) La supervivencia es similar a la de pacientes citorreducidas óptimamente

GN497. ¿Qué tipo de pacientes se benefician con la relaparotomía en pacientes operadas inicialmente en un
centro no oncológico?

a) Las referidas con aparente enfermedad circunscrita a un ovario o con intento de rutina
subóptima

b) Las pacientes con intento de rutina de ovario y resultado subóptimo

c) Quienes únicamente se les practicó biopsia diagnóstica

d) Todas las pacientes deben pasar a quimioterapia de inducción

e) Todas las pacientes deben reestadificarse quirúrgicamente

GN498. ¿Se justifica la cirugía para descomprensión gastrointestinal por enfermedad masiva?

a) Únicamente debe practicarse en casos de vómito incohersible

b) La cirugía derivativa de bypass es la primera opción

c) La expectativa de vida es menor a los 30 días

d) El consenso es que no ofrece beneficio alguno

e) Acentúa el deterioro y gravedad del estado general de la paciente

GN499. Son indicaciones de la laparoscopía en tumores epiteliales de ovario


a) Tumor pélvico, con ecos en su interior y presencia de ascitis escasa por US

b) Tumor de ovario bilateral

c) Carcinomatosis peritoneal, ascitis y masa pélvica

d) Ca-125 elevado, masa ovárica con presencia de papilas en la superficie

e) Masa pélvica única menor a 10cm, superficie lisa, sin excrecencias

GN500. Características de sospecha de malignidad en un tumor de ovario

a) Tumor sólido ó mixto superior a los 10cm de diámetro, papilas, pared engrosada y ascitis

b) Tumor de ovario bilateral

c) Tumor con excrecencias en la superficie y ascitis masiva

d) Tumor sólido, de cualquier dimensión

e) Quiste mayor a 10cm, con necrosis en su interior y papilas en la superficie

GN501. Paciente con ndicación más apropiada para citorreducción secundaria:

a) Con recurrencia después de completar una primera línea de QT

b) Con persistencia tumoral al concluir su primera línea de QT

c) Con recurrencia después de un período libre de enfermedad mayor a seis meses

d) Resistentes a tratamiento con segundas líneas de QT

e) Con enfermedad residual después de tres ciclos de QTx

GN502. Concepto de citorreducción óptima:

a) Enfermedad residual menor o igual a 2cm.

b) Enfermedad residual menor o igual a 1.5cm

c) Presencia de implantes múltiples en “granos de mijo”

d) Ausencia total de enfermedad macroscópica

e) Reporte histológico definitivo de enfermedad microscópica

GN503. Supervivencias por estadio clínico I, II, III y IV, globales a cinco años:

a) 80%, 65%, 45%, 15%

b) 78%, 70%, 33% y 13%

c) Con cirugía como modalidad única 80%, 70%,30% y 10%


d) Modalidades combinadas, 90%,80%,60%,30%

e) 100%, 80%,40% y 10%

GN504. Características histológicas para considerarse tumor borderline:

a) Estratificación del recubrimiento epitelial de las papilas,papilas microscópicas que surgen del
recubrimiento epitelial de las papilas, pleomorfismo, atipias, actividad mitótica y ausencia de
invasión al estroma

b) Presencia de papilas lineales, recubrimiento sin ruptura de la cápsula y ausencia de mitosis

c) Ausencia de invasión al estroma, atipias pobres, mitosis escasas

d) Pleomorfismo nuclear y mitosis escasas

e) Presencia de cuando menos dos de estos parámetros

GN505. Características biológicas propias del tumor borderline:

a) Gran parte son tumores serosos y se presentan en estadios I con supervivencias superiores al
50%

b) El 30% se presentan en estadios III y IV, hasta 50% con tumor extraovárico

c) La supervivencia global es del 92% y en estadios I y II

d) En estadios I están el 70-85%, supervivencias superiores al 85%

e) Son tumores circunscritos a un ovario

GN506. Son características propias del tumor de Kruckemberg

a) Tumor de ovario bilateral

b) Ascitis masiva, tumor de ovario bilateral en la superficie

c) Metástasis a ovarios por un cáncer del tracto gastrointestinal

d) El páncreas es el segundo lugar en frecuencia

e) Ca19-9 por arriba de 1000 es diagnóstico

GN507. ¿El cáncer cervicouterino que infiltra hasta 3 mm de profundidad estromal y mide hasta 7 mm en
superficie corresponde a una etapa clínica?

a) Ib

b) Ib1

c) Ia1
d) Ia2

e) In situ

GN508. ¿El Ca Cu con invasión estromal de 3-5 mm y con invasión superficial hasta 7 mm, corresponde a
la etapa clínica?

a) Ib2

b) Ib1

c) Ia1

d) Ia2

e) Ia1

GN509. ¿Se ha observado que el porciento de recurrencia en el Ca Cu Ia1 y Ia2 tratados correctamente es?

a) 0-1%

b) 2%

c) 0-3%

d) 1-2%

e) más del 4%

GN510. ¿En paciente joven con diagnóstico de Ca Cu Ia1, con paridad de 0. El tratamiento de elección es?

a) HTA

b) Histerectomía tipo III

c) Histerectomía tipo I

d) Conización cervical

e) Histerectomía tipo II

GN511. ¿En el Ca Cu Ia2 la incidencia de metástasis gangliores regionales es de?

a) 3%

b) 5%

c) 0-3%

d) 4%
e) 7%

GN512. El Ca Cu neuroendocrino de células pequeñas en etapas tempranas se trata con:

a) Histerectomía radical

b) Radioterapia radical

c) Histerectomía más radioterapia

d) Histerectomía más radioterapia más quimioterapia

e) Quimioterapia

GN513. El Ca Cu de células vítreas en etapa clínica Ib, tratados con histerectomía tipo III tiene una
supervivencia a 5 años de:

a) 90%

b) 78%

c) 45%

d) 60%

e) 30%

GN514. Se ha demostrado que el Ca Cu asociado al embarazo tiene un pronóstico:

a) Peor que la paciente no embarazada

b) Mejor que la paciente no embarazada

c) Igual a la paciente no embarazada

d) El pronostico es incierto

e) No es posible evaluarlo

GN515. Ca Cu tratado con radioterapia radical, con edema unilateral de miembro pélvico, dolor ciático y
obstrucción ureteral, ¿qué indica esta situación?

a) Recurrencia central

b) Recurrencia parametrial

c) Recurrencia e Irresecabilidad

d) Tromboflebitis
e) Fibrosis posradioterapia

GN516. Es una de las complicaciones post quirúrgicas más serias en la exenteración pélvica por Cacu
recurrente

a) Infección de vías urinarias

b) Obstrucción del intestino delgado

c) Absceso pélvico

d) Tromboembolia pulmonar

e) Infarto agudo al miocardio

GN517. El cáncer más frecuente de la vagina histológicamente es el epidermoide ¿cuál es la frecuencia de


presentación?

a) 50%

b) 70%

c) 85%

d) 45%

e) 100%

GN518. El tratamiento de primera elección para el carcinoma de vagina es:

a) Colpectomía

b) Exenteración anterior

c) Radioterapia radical

d) Cirugía más radioterapia

e) Radioterapia más cirugía

GN519. En una paciente con Ca de tercio superior de vagina etapa clínica II, que conserva el útero ¿Qué
tipo de cirugía le haría?

a) Vaginectomía radical

b) Histerectomía radical con colpectomía

c) Vaginectomía radical con disección Inguinal bilateral

d) Resección amplia
e) Resección amplia más disección inguinal bilateral

GN520. ¿Cuál es la incidencia de enfermedad en los ganglios pélvicos, en Ca de vagina etapa clínica I?

a) 5%

b) 6-14%

c) 10%

d) 0-5%

e) 3%

GN521. En el Ca estadificado TI N0 M0 de vagina en su tercio inferior, el tipo de cirugía que se indica es:

a) Vaginectomía radical

b) Exenteración total

c) Vaginectomía más vulvectomía radical con disección inguinal bilateral

d) Vaginectomía con vulvectomía simple

e) Exenteración anterior

GN522. Histológicamente, ¿Qué porciento ocupa el Ca epidermoide de vulva?

a) 80

b) 90-95

c) 70

d) 65

e) 50

GN523. ¿En qué porciento se asocia el Ca epidermoide de vulva con una displasia tipo III?

a) 40

b) 25-50

c) 30

d) 25

e) 15-20
GN524. ¿La vía de diseminación linfática del Ca epidermoide de vulva que infiltra a clítoris es a través de
los ganglios?

a) Inguinales superficiales

b) Inguinales profundos

c) Pélvicos

d) Femorales

e) Paraórticos

GN525. ¿El tratamiento de elección del Ca epidermoide de vulva es?

a) Radioterapia

b) Cirugía

c) Radioterapia más cirugía

d) Cirugía más radioterapia

e) Quimioterapia más radioterapia

GN526. ¿En qué casos se debe hacer disección inguinal bilateral en el Ca epidermoide de vulva?

d) Tumor de 1 mm

a) Tumor menor de 1 cm y mayor de 2 mm

b) Tumor mayor de 2 cm

c) Tumor mayor de 2 cm localizado en la línea media

e) Tumor de 5 cm con localización parasagital

GN527. El tratamiento posoperatorio más apropiado para pacientes con cáncer de ovario etapa Ia G1 es:

a) Observación estrecha solamente

b) Quimioterapia con monodroga

c) Quimioterapia en combinación

d) Irradiación a la pelvis

e) Irradiación a la pelvis y quimioterapia de combinación

TUMORES MAMARIOS
MA1. En la paciente con cáncer de mama tratada con cirugía conservadora en su seguimiento como examen
rutinario se recomienda:

a) Sólo exploración física

b) Tele de tórax

c) Exploración física y mastografía

d) TAC de mama

e) Ultrasonido mamario

MA2. ¿Cuál de los siguientes tipos de carcinoma mamario es considerado como de mejor pronóstico?

a) Ductal infiltrante sin patrón específico

b) Lobulillar infiltrante

c) Medular atípico

d) Coloide

e) Metaplásico

MA3. Está contraindicada la reconstrucción mamaria posmastectomía en:

a) Mujeres mayores de 50 años

b) Quimioterapia adyuvante

c) Enfermedad diseminada

d) Etapa IIIb

e) Cáncer mamario multicéntrico

MA4. La sobrevida a 10 años de cáncer mamario inicialmente operable cuando hay invasión al grupo
ganglionar 1 es de:

a) 70%

b) 60%

c) 50%
d) 40%

e) 30%

MA5. ¿Cuál es el órgano principalmente afectado por metástasis de cáncer de mama en la mujer
posmenopausica?

a) Pulmón

b) Hueso

c) Sistema nervioso central

d) Hígado

e) Ovarios

MA6. Se considera una lesión como carcinoma ductal in situ cuando:

a) No existe evidencia de ruptura de la membrana basal e invasión de células malignas hacia el


estroma periférico

b) Comparte el potencial biológico del carcinoma lobulillar in situ

c) En general se presenta como una lesión palpable

d) Mantiene una histopatología homogénea

e) El subtipo comedo se relaciona generalmente con microcalcificaciones finas, granulares

MA7. La principal vía de diseminación del cáncer mamario es:

a) Infiltración directa al parénquima mamario

b) A través de los conductos mamarios

c) A través de los linfáticos

d) Todas las anteriores

e) Ninguna de las anteriores

MA8. Ante un tumor phyllodes:

a) Debe ser resecado con márgenes amplios, generalmente mayores a 2 cm

b) La paciente deberá recibir terapia sistémica con esquema para carcinoma mamario
c) Son más frecuentes en mujeres blancas de estrato socioeconómico superior

d) Puede controlarse con tratamiento antiestrogénico

e) El pleomorfismo y la vascularidad son factores que deben tomarse en cuenta para designar su
grado de agresividad

MA9. ¿Qué porciento de pacientes con cáncer de mama presenta factores de riesgo?

a) Del 5 al 10

b) Del 20 al 30

c) Del 40 al 50

d) 60

e) Del 70 al 80

MA10. Se considera a la enfermedad de Paget como manifestación de:

a) Carcinoma medular mamario

b) Carcinoma inflamatorio

c) Carcinoma ductal

d) Entidad neoplásica independiente

e) Carcinoma lobulillar in situ

MA11. Es considerado primariamente como marcador de riesgo aumentado para cáncer mamario invasor:

a) Carcinoma tubular

b) Enfermedad de Paget de la mama

c) Enfermedad de Bowen

d) Carcinoma lobular in situ

e) Carcinoma medular

MA12. El tratamiento de la hiperplasia ductal atípica es conservador, siendo la incidencia del cáncer invasor
a 20 años de seguimiento el:

a) 50%
b) 70%

c) 30%

d) 20%

e) 10%

MA13. El concepto cáncer mamario mínimo se ha dejado de usar porque:

a) Se sustituyó por cáncer oculto

b) Se sustituyó por primario desconocido

c) Se sustituyó por cáncer invasivo de extensión limitada

d) Es preferible usar T1 a, T1 b, T1 c

e) Comprende entidades con pronóstico diferente

MA14. Es contraindicación para la cirugía conservadora de la mama:

a) Tumor en C.S.I

b) Tumor de 4 cm

c) Paciente mayor de 65 años

d) Embarazo (1er trimestre)

e) Hiperplasia lobular atípica

MA15. En la escala histológica del cáncer mamario hay tipos más agresivos que otros, de los siguientes el
más agresivo es el:

a) Mucinoso

b) Ductal

c) Lobular

d) Papilar

e) En anillo de sello

MA16. Se considera nivel ganglionar II a los ganglios ubicados:

a) Por fuera del nervio torácico largo

b) Por dentro de los vasos subescapulares

c) Entre los músculos pectorales


d) Por detrás del músculo pectoral menor

e) Por dentro del músculo pectoral menor

MA17. Estudio indispensable para poder realizar tratamiento conservador de cáncer mamario operable en
etapa I-II:

a) Dopller color

b) Resonancia magnética

c) Determinación de CA 15.3

d) Determinación de ACE

e) Mastografía bilateral

MA18. La posibilidad de recurrencia local o enfermedad metastásica a distancia en un cáncer mamario de 2.5 cm
etapa I-II con ganglios negativos es de:

a) 0-10%

b) 10-30%

c) 50-60%

d) 70-80%

e) 80-90%

MA19. Riesgo de asociación de papilomatosis múltiple y cáncer de mama:

a) 2%-5%

b) 10%-20%

c) 30%-40%

d) 50%-60%

e) 70%-80%

MA20. Los tumores más sensibles a la terapia endócrina se caracterizan por un apariencia histológica:

a) Anaplásica

b) Bien diferenciada
c) Poco diferenciada

d) Moderadamente diferenciada

e) Indiferenciada

MA21. En etapas I y II, la mastectomía radical modificada comparada con la cirugía conservadora en los
estudios de la NSABP BO6 y Milán, demostró que:

a) Tiene una mayor sobrevida

b) No hay diferencia en sobrevida

c) Tiene mayor número de recurrencias locales

d) Las recurrencias locales de la cirugía conservadora son más frecuentes

e) La enfermedad sistémica se presenta en menor tiempo

MA22. El porciento de respuesta al tratamiento hormonal de un tumor mamario que tenga receptores de
estrógenos y de progesterona es de:

a) 20-30

b) 30-40

c) 40-50

d) 50-60

e) 70-80

MA23. El porciento de respuesta al tratamiento hormonal de un tumor mamario que tenga receptores de
estrógenos negativos y de progesterona positivos es de:

a) 30-40

b) 40-60

c) 60-70

d) 70-80

e) 90-100

MA24. El porciento de respuesta al tratamiento hormonal de un tumor mamario que tenga receptores de
estrógenos y de progesterona negativa es de:
a) 0-10

b) 20-30

c) 40-60

d) 60-80

e) 80-100

MA25. ¿En qué consiste el tratamiento conservador para cáncer de mama temprano según Veronesi?

a) Tilectomía+Disección axilar

b) Cuadrantectomía+Disección axilar+Radioterapia posoperatoria

c) Tumorectomía+Radioterapia

d) Tumorectomía+Quimioterapia

e) Todas las anteriores

MA26. Durante la disección axilar, la lesión del nervio tóracodorsal deja como secuela:

a) Escápula alada

b) Debilita la rotación y abducción del brazo

c) Atrofia del músculo pectoral mayor y menor

d) Ninguna secuela

e) a y c son ciertas

MA27. Contraindicación absoluta para efectuar el tratamiento conservador en cáncer mamario:

a) Primer trimestre del embarazo

b) Dos o más tumores en diferentes cuadrantes de la mama

c) Microcalcificaciones difusas sospechosas de malignidad

d) Radioterapia mamaria previa

e) Todas las anteriores

MA28. Factores asociados al desarrollo de linfedema postratamiento quirúrgico:


a) El tratamiento tumoral

b) La presencia de ganglios linfáticos preoperatorios

c) La adición de radioterapia posterior a la disección radical de axila

d) La extención de la disección axilar

e) Edad de la paciente

MA29. ¿Técnica más simple de reconstrucción mamaria inmediata en mujeres con mamas medianas y
pequeñas?

a) Implante de prótesis

b) Expansor tisular

c) Colgajo libre microvascular

d) Colgajo de dorsal ancho

e) Colgajo de recto pectoral del abdomen

MA30. ¿Cuál es la indicación absoluta de mastectomía profiláctica?

a) Antecedente de hermana con cáncer de mama

b) Presencia de hiperplasia severa atípica

c) No existe indicación absoluta de mastectomía profiláctica

d) Antecedente de cáncer de mama lobulillar in situ

e) Cancerofobia

MA31. Tratamiento estándar para la enfermedad de Paget con tumor palpable de 1.5 cm:

a) Resección del complejo areola pezón + radioterapia

b) Mastectomía total

c) Mastectomía radical modificada

d) Quimioterapia neoadyuvante y mastectomía total

e) Tamoxifén por 5 años

MA32. El manejo de carcinoma lobulillar in situ es:

a) Escisión local amplia


b) Escisión local amplia más radioterapia

c) Radioterapia radical

d) Mastectomía simple

e) Vigilancia estrecha

MA33. A la mastectomía radical modificada en donde se conservan ambos músculos pectorales se le conoce
como tipo:

a) Halsted

b) Warren

c) Madden

d) Patey

e) Mc Whiter

MA34. La posibilidad de recurrencia local después de tratamiento quirúrgico conservador sin radioterapia
posoperatoria es de:

a) 0-5%

b) 10-15%

c) 15-20%

d) 30-40%

e) 70-80%

MA35. En el tratamiento quirúrgico de cualquier patología benigna de la mama, la incisión más indicada es:

a) Subareolar

b) Circular

c) Radial

d) Areolar

e) Siguiendo los surcos mamarios

MA36. Complicación más frecuente inmediata de la mastectomía radical modificada:


a) Dehisencia de sutura

b) Hematoma

c) Seroma

d) Necrosis de colgajos

e) Linfedema

MA37. En una mujer mastectomizada lo recomendable para retirar la succión o drenaje es:

a) A los 5 días

b) A los 8 días

c) Gasto menor de 25 mL

d) Gasto menor de 50 mL

e) Gasto menor de 100 mL

MA38. En una paciente con cáncer de mama EC. I-II operada de una MRM con tumor en una localización
medial, axila positiva con más del 20% de ganglios linfáticos ¿Cuál es la indicación de radioterapia?

a) Cadena mamaria

b) Linfáticos axilares

c) Ciclos mamarios completos

d) Linfáticos + pared torácica

e) Campos tangenciales

MA39. ¿Con cuál procedimiento terapéutico es mayor la incidencia de linfedema?

a) Muestreo axilar

b) Radioterapia

c) Cirugía conservadora + RT a mama

d) Mastectomía radical

e) Disección axilar radical + radioterapia


MA40. Un cáncer mamario de 3 cm de diámetro situado en el cuadrante superior interno de la mama se trata
con:

a) Mastectomía simple

b) Radioterapia

c) Mastectomía super–radical

d) Mastectomía radical modificada

e) Mastectomía radical

MA41. El tumor phyllodes de 3 cm de diámetro se trata inicialmente con:

a) Mastectomía simple

b) Escisión local + DRA

c) Escisión local amplia tridimensional

d) Mastectomía subcutánea

e) Mastectomía radical modificada

MA42. En la mastectomía radical modificada de tipo Patey se resecan además de la mama los siguientes
elementos anatómicos:

a) Músculos pectorales

b) Músculo pectoral mayor y ganglios axilares

c) Ganglios axilares niveles I-II-III

d) Músculo pectoral menor y ganglios axilares

e) Músculos pectorales, ganglios axilares y mamarios internos

MA43. Un tumor phyllodes mayor de 8 cm se debe tratar con:

a) Escisión local

b) Mastectomía radical modificada

c) Mastectomía simple

d) Cirugía conservadora

e) Mastectomía radical clásica

MA44. De las modalidades de tratamiento para el carcinoma inflamatorio, el que mejores resultados ha dado
es:
a) Radioterapia sola

b) Radioterapia + quimioterapia

c) Quimioterapia y cirugía con radioterapia

d) Radioterapia + cirugía

e) Endocrinoterapia, radioterapia y cirugía

MA45. Revisando los tratamientos del cáncer mamario localmente avanzado se ha observado una mejor
respuesta con uno de los tratamientos siguientes:

a) Radioterapia sola

b) Quimioterapia, cirugía y radioterapia

c) Cirugía más radioterapia

d) Cirugía más quimioterapia

e) Cirugía precedida de quimioterapia y seguida de quimioterapia

MA46. Antes del tamoxifén, nuevos progestágenos y la aminoglutetimida, se usaron los estrógenos en las
pacientes posmenopáusicas con cáncer mamario con respuesta de:

a) 5%

b) 15%

c) 25%

d) 35%

e) 45%

MA47. Investigaciones sistemáticas sobre la ingesta prolongada de tamoxifén han demostrado:

a) Aumento no significante de segundo primario de todos los tipos

b) Aumento del riesgo de padecer un segundo primario contralateral mamario

c) Disminución del riesgo de cáncer hepatocelular

d) Aumenta el riesgo de cáncer ovárico

e) Aumenta el riesgo de cáncer endometrio


MA48. El tratamiento del tumor phyllodes benigno de 4 cm en mama de tamaño medio consiste en:

a) Enucleación

b) Excisión local

c) Cuadrantectomía más disección radical

d) Mastectomía simple

e) Excisión local con márgenes de tejido sano

MA49. El tratamiento del tumor phyllodes maligno de 3 cm en una mama de tamaño medio es:

a) Escisión local amplia con márgenes

b) Enucleación

c) Mastectomía simple

d) Mastectomía Halsted

e) Mastectomía radical modificada

MA50. La frecuencia de remisión completa del cáncer de mama diseminado en pacientes tratadas con
quimioterapia es del:

a) 1%

b) 15%

c) 30%

d) 50%

e) 70%

MA51. Bisfosfonato que ha demostrado actividad antitumoral para las metástasis viscerales y de tejidos
blandos:

a) Pamidronato

b) Etidronato

c) Risedronato

d) Clodronato

e) Ninguno de los anteriores


MA52. El pamidronato reduce el dolor óseo, las fracturas y la compresión radicular por metástasis en hueso
por cáncer de mama en un:

a) 5%

b) 20%

c) 35%

d) 50%

e) 70%

MA53. En mujeres menores de 30 años con cáncer de mama ¿qué porciento tiene cáncer de mama
hereditario?

a) 10

b) 10 a 20

c) 25 a 30

d) 36 a 85

e) 100

MA54. ¿En qué sitio de la glándula mamaria es más frecuente la aparición de un carcinoma?

a) Cuadrante superior interno

b) Cuadrante inferior interno

c) Retroareolar

d) Cuadrante superior externo

e) Cuadrante inferior externo

MA55. ¿Qué porciento de los pacientes con carcinoma mamario presentan como signo inicial la secreción a
través del pezón?

a) 2 a 5

b) 10 a 15

c) 25 a 30

d) 40 a 50
e) 75 a 80

MA56. Son características que deben hacer sospechar que una secreción por el pezón esté asociada a un
posible carcinoma:

a) Bilateral, serosa y escasa

b) Unilateral, lechosa y abundante

c) Unilateral sanguinolenta y espontánea

d) Unilateral, serosa y espontánea

e) Bilateral, escasa y pastosa

MA57. Debe considerarse como un marcador histológico asociado a incremento en el riesgo de presentar a
futuro un carcinoma invasor, la siguiente lesión histológica:

a) Metaplasia apocrina

b) Fibroadenomatosis

c) Carcinoma lobulillar in situ

d) Microquistes múltiples

e) Engrosamiento de piel

MA58. ¿A qué etapa clínica de mama comprende T3 NI M0?

a) IIa

b) IIb

c) IIIa

d) IIIb

e) IIIc

MA59. Un tumor clasificado como T3 N0 M0 se etapifica como:

a) IIa

b) IIb

c) IIIa

d) IIIb

e) Ninguna de las anteriores


MA60. ¿Cómo se estadifica una paciente con enfermedad de Paget con un tumor subyacente de 3 cm fijo al
pectoral mayor y axilar negativa?

a) Tis No Mx

b) TI No Mx

c) T2 No Mx

d) T3 No Mx

e) T4 No Mx

MA61. La enfermedad de Paget sin tumor subyacente se estadifica:

a) EC 0

b) EC 1

c) EC 11

d) EC IIB

e) EC IIIB

MA62. ¿Cuál de los enunciados siguientes no es un factor predisponente para el desarrollo de un cáncer
mamario?

a) Edad de menarquia

b) Multiparidad

c) Edad de menopausia

d) Nuliparidad

e) Uso de estrógenos

MA63. ¿Qué porciento de lesiones no palpables mastográficamente sospechosas de malignidad son cáncer?

a) Menos del 10

b) Del 20 al 30

c) 50

d) Del 75 al 80
e) Más del 95

MA64. La combinación del examen físico, mastografía y BAAF da un diagnóstico preciso del 100%. La
sensibilidad de la biopsia aspiración con aguja fina es del:

a) 95%

b) 70%

c) 85%

d) 80%

e) 100%

MA65. La frecuencia de carcinoma lobulillar en el varón es de:

a) Menos 5%

b) 10%

c) 25%

d) 50%

e) 70%

MA66. Un cáncer de mama clasificado como TON2MO se incluye en la etapa:

a) IIa

b) IIb

c) IIIa

d) IIIb

e) IVa

MA67. El tipo histopatológico del cáncer de mama asociado más frecuente a un fibroadenoma es:

a) Carcinoma ductal in situ

b) Carcinoma lobulillar in situ

c) Carcinoma ductal infiltrante


d) Carcinoma lobulillar infiltrante

e) Carcinoma medular

MA68. El signo clínico de mayor frecuencia, en la exploración mamaria de la mujer con cáncer de mama es
la tumoración y su porciento es del:

a) 40

b) 50

c) 60

d) 70

e) 80

MA69. La característica más importante para hacer el diagnóstico entre un fibroadenoma mamario y tumor
phyllodes es:

a) El tamaño

b) La presencia de zonas de necrosis y calcificaciones

c) Los bordes infiltrantes

d) La celularidad del estroma y presencia de mitosis en éste

e) La celularidad y presencia de atipias y mitosis en el componente epitelial

MA70. Lesión que se presenta como una masa polipoide intraluminal, en conductos galactóforos principales
y clínicamente como un tumor subareolar con secreción sanguinolento por el pezón:

a) Adenoma de pezón

b) Papiloma intraductal

c) Fibroadenoma mamario

d) Tumor Phyllodes

e) Adenoma de la lactancia

MA71. El diagnóstico definitivo de fibroadenoma mamario se obtiene por:

a) Mastografía

b) Ultrasonido
c) Estudio histopatológico

d) Clínica

e) Citología

MA72. Un cáncer mamario de 2 cm ulcerado se clasifica como:

a) TI

b) T4

c) TIIb

d) TIII

e) TIIa

MA73. Las metástasis óseas de un cáncer mamario ocurren más frecuentemente en:

a) Húmero y radio

b) Fémur

c) Cráneo y columna

d) Tibia y peroné

e) Arcos costales

MA74. Un cáncer mamario con metástasis ganglionares supraclaviculares del mismo lado se clasifica como:

a) Etapa IIIa

b) Etapa IIa

c) Etapa IV

d) Etapa IIIb

e) Etapa I

MA75. Un cáncer mamario T1 N-2 corresponde a una:

a) Etapa clínica I

b) Etapa clínica IIIa

c) Etapa clínica IIb

d) Etapa clínica IIIb


e) Etapa clínica IVa

MA76. La enfermedad de Paget se diagnostica con:

a) Mastografía

b) Ultrasonido

c) Biopsia escisional

d) Biopsia insicional de piel

e) Biopsia por aspiración con aguja fina

MA77. El cáncer mamario se presenta con menor frecuencia en la glándula mamaria en el cuadrante:

a) Inferior externo

b) Inferior interno

c) Sector retroareolar

d) Superior externo

e) Superior interno

MA78. Enfermedad de Mondor es sinónimo de:

a) Trombos tumorales

b) Tromboflebitis de venas toracoepigástricas

c) Papilomatosis intraductal

d) Galactocele

e) Carcinoma intraquístico

MA79. Generalmente hay sospecha de cáncer mamario por mastografía cuando se observan
microcalcificaciones y éstas son en número de:

a) Dos

b) Tres

c) Cuatro

d) Cinco

e) Uno
MA80. La edad media de presentación del tumor phyllodes oscila entre:

a) 15-24 años

b) 25-34 años

c) 35-44 años

d) 45-54 años

e) 55-64 años

MA81. Las características microscópicas del tumor phyllodes que sugieren malignidad son:

a) Necrosis y hemorragia

b) Volumen tumoral (más de 15 cm)

c) Zonas firmes que alteran con zonas quísticas

d) Color gris y consistencia firme al corte con hendiduras

e) Consistencia suave y gelatinosa con zonas polipoides

MA82. El carcinoma mamario que con mayor frecuencia crece (65%) dentro de un fibroadenoma es de la
histología:

a) Ductal infiltrante

b) Tubular

c) Lobular

d) Papilar

e) Medular

MA83. De las patologías mamarias siguientes, señale la que más frecuentemente curse con secreción
serosanguinolenta por el pezón:

a) Papiloma intraductal

b) Fibroadenoma

c) Ectasia ductal

d) Mastitis

e) Carcinoma canalicular
MA84. Son causas frecuentes de telorrea, excepto:

a) Anovulatorios orales

b) Mastitis puerperal

c) Carcinoma mamario

d) Ectasia ductal

e) Ninguna de las anteriores

MA85. Porciento de pacientes con carcinoma lobulillar in situ que no desarrollan un carcinoma invasor, aún
a largo plazo:

a) 50-60

b) 40-50

c) 90-95

d) 70-80

e) 60-70

MA86. Son diagnósticos diferenciales con la enfermedad de Paget los siguientes, excepto:

a) Eczema del pezón

b) Papiloma intraductal

c) Enfermedad de Bowen

d) Carcinoma basocelular

e) Micosis fungoide

MA87. Este tipo de cáncer se define como el carcinoma de mama detectado entre dos controles
mamográficos de escrutinio, habiendo resultado negativo el primero de ellos:

a) Cáncer de intervalo

b) Cáncer oculto

c) Segundo cáncer primario

d) Cáncer no detectable

e) Ninguna de las anteriores


MA88. Son signos mastográficos indirectos de carcinoma ductal infiltrante en mama los siguientes, excepto:

a) Microcalcificaciones

b) Neodensidad

c) Asimetría de densidad

d) Distorsión arquitectural localizada

e) Conducto dilatado único

MA89. Carcinoma de mama originalmente descrito por Velpau en 1853:

a) Inflamatorio

b) De Paget

c) Lobulillar in situ

d) Canalicular infiltrante

e) Tubular

MA90. Los tumores malignos de mama de origen mesenquimatoso (sarcomas) en conjunto no llegan al:

a) 5%

b) 1%

c) 10%

d) 15%

e) 8%

MA91. En qué tipo de mastectomía radical se conserva el músculo pectoral menor:

a) Tipo Halsted

b) Tipo Patey

c) Tipo Madden

d) Tipo Clásica

e) Tipo Prudente
MA92. Se considera como factor de alto riesgo para desarrollar cáncer de mama, el antecedente
histopatólogico de:

a) Mastopatía fibroquística

b) Papilomatosis intraductal

c) Fibroadenoma mamario

d) Hiperplasia ductal atípica

e) Adenosis esclerosante

MA93.¿Cuál de los siguientes factores de riesgo no eleva entre dos y cuatro veces la posibilidad de cáncer
mamario?

a) Historia familiar de cáncer de mama en pacientes de primer grado

b) Enfermedad fibroquística mamaria

c) Edad del primer parto a término por arriba de los 30 años

d) Radiaciones ionizantes

e) Cáncer de ovario previo

MA94. Señale cuál de los siguientes factores pronósticos tiene más significancia:

a) Índice mitótico

b) Grado nuclear

c) Invasión vascular

d) Ganglios metastásicos regionales

e) Necrosis tumoral

MA95. ¿Cuál es el tamaño tumoral máximo permitido para la realización de cirugía conservadora en cáncer
mamario temprano (según el Consenso Nacional de Cáncer)?

a) 2 cm

b) 3 cm

c) 4 cm

d) 5 cm
e) 10 cm

MA96. ¿Cuál de las siguientes variedades histológicas del cáncer de mama ductal in situ tiene un
comportamiento biológico más agresivo?

a) Sólido

b) Comedo

c) Cribiforme

d) Papilar

e) Micropapilar

MA97. En México ¿Qué lugar ocupa el cáncer de mama en morbimortalidad en pacientes del sexo
femenino?

a) 1er lugar

b) 2º lugar

c) 3er lugar

d) 5º lugar

e) 10º lugar

MA98. ¿Cuál de las afirmaciones siguientes sobre factores de riesgo para cáncer de mama es falso?

a) Existe un aumento de riesgo relacionado al consumo de alcohol

b) La enfermedad no proliferativa benigna no aumenta el riesgo de neoplasia maligna

c) Las radiaciones ionizantes posteriores a los 40 años aumentan el riesgo

d) La edad de manarquia, menopausia y primer embarazo son factores que están relacionados a
riesgo de cáncer mamario

e) El cáncer mamario hereditario constituye del 5 al 10% del total de casos de cáncer mamario

MA99. En lo que se refiere a genes y oncogenes en cáncer mamario:

a) Existe una sobreexpresión de p53 en el 45% de los casos

b) La sobreexpresión de p53 se asocia a un mal pronóstico


c) El Her-2 neu codifica una cinasa de proteína que es homóloga al factor de crecimiento
epidérmico humano

d) Su sobreexpresión se relaciona a un mejor pronóstico

e) b y c son correctas

MA100. En cuanto a los estudios del NSABP BO6 y Milán (Veronesi) se demostró que la cirugía
conservadora con radioterapia con respecto a la mastectomía:

a) Tiene una mayor sobrevida libre de enfermedad

b) Tiene mayor número de recurrencias locales

c) No hay diferencia en sobrevida total

d) Las recurrencias locales de la cirugía conservadora afectan adversamente la sobrevida

e) b y c son correctas

MA101. El pronóstico de la enfermedad de Paget es asociado a:

a) Presencia de células de Paget

b) Porciento de cambios eczematoides en areola

c) Edad del paciente

d) Etapa clínica

e) Antecedente familiar de cáncer de mama

MA102. ¿En qué cromosomas se encuentra la mutación del gen BRCA 1?

a) Cromosoma X

b) Cromosoma 10

c) Cromosoma 22

d) Cromosoma 17

e) Cromosoma 27

MA103. ¿Uno de los siguientes síndromes familiares NO se asocia a cáncer de mama?

a) Síndrome de Li-Fraumeni

b) Síndrome de Cowden
c) Síndrome de Muir

d) Síndrome de Lynch 1

e) Ataxia-telangiectasia

MA104. ¿Qué papel juega en la actualidad la disección radical de axila en el tratamiento del cáncer mamario
en tumores inicialmente operables etapa I-II?

a) Para etapificación

b) Para emitir pronóstico

c) Para finalidad terapéutica

d) Con fin pronóstico-etapificadora

e) Para todas las anteriores

MA105. Lesión mamaria, considerada como de alto riesgo, y la más factible de ser premaligna:

a) Hiperplasia epitelial atípica

b) Adenosis

c) Esclerosis

d) Papiloma intraductal

e) Necrosis grasa

MA106. Cuando el protooncogén C-erb-B-2 está sobreexpresado se relaciona con:

a) Cáncer mamario hereditario

b) Buen pronóstico

c) Cáncer mamario esporádico

d) Mal pronóstico

e) Elevados receptores estrogénicos

MA107. Los focos multicéntricos del carcinoma en la misma mama, ocurren:

a) Rara vez

b) En 25 a 50% de los especímenes de mastectomía

c) En más del 80% de los casos


d) Sólo en pacientes con carcinoma lobulillar

e) Sólo en pacientes con ganglios axilares positivos para metástasis

MA108. La existencia de receptor estrogénico positivo (mayor a 10 Fentomolas) en el tejido tumoral de un


carcinoma se asocia a:

a) Pronóstico muy adverso

b) Pronóstico bueno

c) No se asocia a pronóstico

d) Son pacientes que requerirán quimioterapia a altas dosis

e) La hormonoterapia no tiene utilidad

MA109. Porciento de respuesta al tratamiento hormonal de tumor mamario con receptores de estrógenos
positivos y de progesterona negativos:

a) 20-30

b) 30-40

c) 40-50

d) 50-60

e) 70-80

MA110. Indicación(es) de mastectomía radical modificada en nuestro medio:

a) Componente intraductal extenso mayor a 25%

b) Tumores multicéntricos

c) Preferencia de la enferma por este tratamiento

d) Imposibilidad de seguimiento adecuado o de efectuar RT posoperatoria

e) Todas son correctas

MA111. ¿Cuál de los siguientes tipos de reconstrucción posmastectomía tiene mayor tiempo de estancia
hospitalaria?

a) Implantes

b) Expansor tisular

c) Colgajo de dorsal ancho


d) Colgajo de recto anterior abdominal

e) Colgajo de mama contralateral

MA112. Indicación absoluta de mastectomía radical modificada en cáncer mamario inicialmente operable
etapa clínica I-II:

a) Tumor de 1.5 cm en área retroareolar

b) Adenopatía axilar ipsilateral de 1.0 cm

c) Componente de extensión intraductal del 50%

d) Subtipo histológico lobulillar

e) Tumor con índice mitótico elevado

MA113. Indicación de mastectomía radical clásica en cáncer mamario en etapa inicialmente operable:

a) Cáncer inflamatorio de la mama

b) Cáncer en el varón

c) Cáncer y embarazo

d) Cáncer y dermatomiositis

e) Enfermedad de Paget

MA114. Actualmente en el tratamiento de radioterapia ¿qué es lo que incluye el ciclo mamario completo?

a) Campos tangenciales al seno + mamario interno

b) Campos axilosupraclavicular + tangenciales al seno

c) Campos tangenciales al seno + axila

d) Campos tangenciales y sobredosis al lecho

e) Mamario interno + sobredosis al lecho

MA115. El carcinoma canalicular in situ de tipo comedo de 2.5 cm se trata preferentemente con:

a) Mastectomía simple

b) Mastectomía radical modificada

c) Escisión local + DRA


d) Escisión local

e) Radioterapia

MA116. Un cáncer mamario etapa clínica I con relación mama-tumor favorable se trata preferentemente
con:

a) Tumorectomía

b) Cuadrantectomía

c) Segmentectomía + DRA + radioterapia posoperatoria

d) Mastectomía radical modificada tipo maden

e) Cuadrantectomía + DRA

MA117. La disección radical de axila no está indicada en el tratamiento del tumor phyllodes a menos que:

a) El tumor sea mayor de 10 cm

b) La mama albergue un carcinoma

c) Que haya recurrido más de una vez

d) Histológicamente presente más de un elemento heterólogo

e) Que haya un ganglio palpable

MA118. El tratamiento para el cáncer mamario invasor que crece dentro de un fibroadenoma es:

a) Enucleación del fibroadenoma

b) Escisión del fibroadenoma con márgenes del tejido sano

c) Cuadrantectomía

d) El mismo que para cualquier cáncer fuera del fibroadenoma

e) Mastectomía simple

MA119. Paciente femenina que recibe quimioterapia con el esquema de Fac por cáncer mamario se queja de
dolor y molestia en el sitio de la aplicación. La conducta adecuada debe ser:

a) Administrar analgésicos y continuar la quimioterapia

b) Consultar al cirujano plástico de inmediato

c) Probar el sitio de la punción con otra inyección de QT. Si no aumenta la molestia, continuar con
el tratamiento de acuerdo a lo planeado

d) Inyectar corticoesteroides
e) Suspender inmediatamente la quimioterapia, aspirar y retirar la solución; aplicar compresas
frías en ese sitio, documentar el evento y vigilar al paciente estrechamente

MA120. La frecuencia de sobrevida sin tumor a cinco años, de las pacientes con cáncer de mama
metástasico que obtuvieron remisión completa con quimioterapia es del:

a) 0%

b) 20%

c) 40%

d) 60%

e) 80%

MA121. La clasificación de Scarff-Bloom-Richardson toma en cuenta:

a) La formación de túbulos, pleomorfismo nuclear

b) El pleomorfismo nuclear, comedo carcinoma

c) El índice mitósico, formación de túbulos

d) Formación de vacuolas, índice mitótico

e) Formación de túbulos, pleomorfismo nuclear, mitosis

MA122. El dato radiológico (mastográfico) más común en el carcinoma mamario invasor es:

a) Microcalcificaciones

b) Imagen de mayor densidad irregular

c) Ductos mamarios dilatados y prominentes

d) Aumento de la vascularidad

e) Engrosamiento de piel

MA123. De los antígenos tumorales que pueden ser útiles en el seguimiento de valoración de respuesta a tratamiento
de metástasis en cáncer de mama el mejor es:

a) Ca-125

b) Antígeno carcino-embrionario

c) Gonadotrofina corionica humana


d) CA 15:3

e) AFP

MA124. La edad media de presentación de cáncer mamario en el varón es:

a) 30 años

b) 40 años

c) 50 años

d) 60 años

e) 70 años

MA125. ¿A qué corresponde el N3 en la etapificación del cáncer de mama?

a) Ganglio único ipsilateral móvil

b) Conglomerado ganglionar ipsilateral fijo

c) Ganglios bilaterales

d) Ganglios positivos en la cadena mamaria interna

e) Ganglios supraclaviculares

MA126. Una paciente de 45 años con un enrojecimiento generalizado de la mama, piel de naranja presencia
de un conglomerado ganglionar axilar de más de 2cm y un ganglio supraclavicular de 0.5 cm y que en la
biopsia presenta infiltrado linfático subdérmico ¿Cómo se clasifica?

a) IIb b) IIc c) IIIb d) IIIa e) IV

MA127. ¿Cuál es el principal método de pesquisa en cáncer de mama?

a) Tomografía axial computada

b) Resonancia magnética nuclear

c) Tomografía

d) Mamografía y exámen físico

e) Ultrasonido
MA128. De los siguientes hallazgos mastográficos en pacientes con protocolo de pesquisa ¿Cuál no es
sugestivo de malignidad?

a) Microcalcificaciones agrupados

b) Distorsión de la arquitectura

c) Lesiones estelares

d) Macrocalcificaciones

e) Asimetría del parénquima mamario

MA129. Según el Consenso Nacional de cáncer mamario, la pesquiza mastográfica deberá iniciarse a la
edad de:

a) 35 años

b) 40 años

c) 45 años

d) 50 años

e) No existe consenso con respecto con la edad

MA130. El grupo de Rotter se encuentra localizado en:

a) Nivel ganglionar I

b) Nivel ganglionar II

c) Supraclaviculares

d) Mamaria interna

e) Subclaviculares

MA131. Son factores de riesgo para el cáncer mamario:

a) Lactancia negativa y multiparidad

b) Nuliparidad y promiscuidad sexual

c) Dieta rica en grasa y ser de raza oriental

d) Menarca temprana y menopausia tardía

e) Antecedente familiar de CaCu y alcoholismo positivo


MA132. El cáncer mamario es más frecuente en pacientes con antecedentes de:

a) Cáncer de glándulas salivales

b) Cáncer pulmonar

c) Cáncer de colon

d) Cáncer de vejiga

e) Melanoma maligno

MA133. El escurrimiento de líquido sanguinolento a través del pezón está dado por distintas patologías
mamarias. La lesión más frecuente es:

a) Carcinoma papilar

b) Papiloma

c) Ectasia ductal

d) Cambios fibroquísticos

e) Enfermedad de Paget

MA134. Las mastografías exponen a la glándula mamaria a radiación ionizante, la dosis estimada para una
glándula de tamaño medio por cada placa es de:

a) 1000 mrad b) 500 mrad c) 100 mrad d) 75 mrad e) 40 mrad

MA135. Uno de los tumores que a continuación se mencionan no corresponde a la etapa III:

a) TO N2 M0

b) T4 N0 M0

c) T3 N0 M0

d) T1 N2 M0

e) T0 N3 M0

MA136. En el estudio mastográfico siempre que se hable de masa de deben describir las características
siguientes, excepto:

a) Forma

b) Volumen
c) Tamaño

d) Densidad

e) Márgenes

MA137. La(s) característica(s) de malignidad asociadas a carcinoma in situ de las microcalcificaciones en


un estudio mastográfico son, excepto:

a) Asociadas a una densidad radiológica

b) Agrupales

c) Irregulares

d) Heterogéneas

e) Ninguna de las anteriores

MA138. En estudios recientes se ha demostrado que los pacientes con sobreexpresión del oncogén Her 2-
neu presentan cierto grado de resistencia a los fármacos siguientes, excepto:

a) Metrotexate

b) Adriamicina

c) 5 fluoruracilo

d) Taxotere

e) Tamoxifén

MA139. La localización más frecuente del cáncer de mama en el hombre es:

a) Cuadrante superior externo

b) Cuadrante inferior externo

c) Cuadrante inferior interno

d) Cuadrante superior interno

e) Subareolar

MA140. Es un gen supresor tumoral asociado al cáncer de mama:

a) Her-2/neu

b) p53
c) Ca 125

d) BrE-3

e) Bcl-2

MA141. En cáncer de mama tratado médicamente (quimioterapia u hormonoterapia) se define como la aparición
de cualquier nueva lesión no identificada previamente, o aumento superior o igual al 25% en lesiones ya
existentes:

a) Respuesta completa

b) Estabilización

c) Segundo primario

d) Progresión

e) Respuesta parcial

MA142. Se consideran factores de pronóstico de riesgo elevado en el cáncer de mama, excepto:

a) Fase S<5%

b) pT> 3cm

c) Grado nuclear 3

d) Her-2/neu+

e) Aneuploide

MA143. Se utiliza en metástasis óseas como un método más rápido y efectivo en cuanto a control del dolor
y recalcificación de las zonas afectadas:

a) 5 fluoracilo

b) Placlitaxel

c) Radioterapia

d) Vinorelbina

e) Antifolatos

MA144. Característica del carcinoma inflamatorio de mama que se presenta con menos frecuencia:

a) Masa palpable

b) Retracción del pezón


c) Edema (piel de naranja)

d) Eritema

e) Aumento de volumen

MA145. Una paciente con tumor mamario de 4 cm, piel de naranja y un conglomerado axilar fijo
homolateral, se clasifica:

a) T2 N2 M0

b) T2 N1 M0

c) T3 N2 M0

d) T4b N2 M0

e) T4d N2 M0

MA146. Gen que establece progresión del cáncer mamario, excepto:

a) erbB2

b) myc

c) BRCA-2

d) Cyclin D1

e) Rb-1

Correlacione las siguientes columnas referentes a riesgo relativo de carcinoma mamario invasor, de acuerdo a
estudio histologico en tejido mamario con:

MA147. Adenosis esclerosante ( ) a) Sin riesgo aumentado


MA148. Carcinoma lobulillar in situ ( ) b) Riesgo aumentado 1.5 a 2
veces
MA149. Hiperplasia ductal atípica ( ) c) Riesgo aumentado 4 a 5
veces
MA150. Cambio apócrino ( ) d) Riesgo aumentado 8 a 10
veces
MA151. Papiloma ( )

Correlacione el tipo histológico del cáncer de mama con su frecuencia de presentación.

MA152. Carcinoma ductal infiltrante ( ) a) Menos de 1%


MA153. Carcinoma medular ( ) b) 2%
MA154. Enfermedad de Paget ( ) c) 5%
MA155. Tumor maligno del estroma ( ) d) 50%
MA156. Carcinoma lobulillar infiltrante ( )

MA157. En relación con el carcinoma mamario, el síndrome de Stewart-Treves se refiere a:

a) Tumor que se presenta posradioterapia en la pared torácica

b) Manera de diseminación por sistema ácigos hacia la columna

c) Angiosarcoma pos-mastectomía

Linfedema de la extremidad superior ipsilateral al carcinoma

d) Tipo de carcinoma que forma una coraza en el tórax y lo inmoviliza

MA158. El índice pronóstico de Van Nuys relaciona los factores siguientes:

a) Grado nuclear, presencia de ganglios axilares, tamaño del tumor

b) Tamaño del tumor, márgenes de resección, clasificación patológica

c) Grado nuclear, número de mitosis por campo en seco fuerte, tamaño del tumor

d) Tamaño del tumor, presencia de ganglios axilares, expresión de receptores hormonales

e) Expresión de receptores hormonales, edad de la paciente, clasificación patológica

MA159. El índice pronóstico de Van Nuys indica:

a) Progresión de la enfermedad

b) Posibilidad de metástasis

c) Respuesta al tratamiento

d) Recurrencia local

e) Sobrevida global mejor que sobrevida libre de enfermedad

MA160. En cáncer de mama, las pacientes con presencia de ganglios axilares clínicamente metastásicos y que
reciben quimioterapia neoadyuvante, al ser intervenidas quirúrgicamente y el reporte de patología informa
ganglios axilares histológicamente negativos a enfermedad metastásica, se espera una evolución:

a) Con tendencia a la recurrencia local


b) Igual a la que hubiera tenido con ganglios negativos antes de la quimioterapia

c) Con tendencia a la recurrencia a distancia

d) Sin cambios en su evolución respecto a su clasificación inicial

e) Peor a lo que corresponde según su clasificación inicial

MA161. El diagnóstico de carcinoma inflamatorio es:

a) Clínico

b) Histopatológico

c) Por mastografia

d) Por clínica e histopatología

e) Por clínica, histopatología y mastografía

MA162. La enfermedad de Paget se puede manifestar en:

a) Piel

b) Región anogenital

c) Hueso

d) Mama

e) Todos los sitios anteriores

MA163. El esquema de graduación histológica para el cáncer de mama Bloom-Richardson está basado en:

a) Cantidad de formación de túbulos, pleomorfismo nuclear, número de mitosis

b) Pleomorfismo nuclear y citoplasmático

c) Ruptura de la membrana celular

d) Pleomorfismo citoplasmático

MA164. La disección de los ganglios axilares en el cáncer de mama:

a) Incrementa el efecto terapéutico quirúrgico

b) Se practica para saber la presencia o ausencia de receptores hormonales


c) Tiene significancia pronóstica

d) No modifica el efecto terapéutico quirúrgico

MA165. Se habla de micrometástasis cuando los focos de metástasis en cáncer mamario son:

a) De 1.0 cm de diámetro

b) Entre 0.5 cm y 1.0 cm

c) Entre 2 mm y 5 mm

d) Menores de 2 mm

e) Ninguno

MA166. La enfermedad de Paget del pezón se presenta en:

a) 5% de los cánceres de mama

b) 3% de los cánceres de mama

c) 10% de los cánceres de mama

d) 3 al 5% de los cánceres de mama

e) Ninguno

MA167. Más del 90% son carcinomas con receptores estrogénicos y receptores progestacionales negativos
típicamente aneuploides, con alta fase S:

a) Tubular

b) Mucinoso

c) Adenoideoquístico

d) Medular

e) Papilar

MA168. Porciento en el que ocurre el cáncer mamario inflamatorio:

a) 1 a 2

b) 5 a 7

c) 8 a10
d) 15 a 16

e) 20 a 23

MA169. Gen tumoral supresor más importante en el cáncer mamario localizado en el cromosoma 17 p21:

a) BRCA 1

b) BRCA 2

c) P53

d) 17q

e) 13q

MA170. Carcinoma mamario que representa el 1 al 2% de los cánceres mamarios, indolente y con bajas
metástasis:

a) Papilar

b) Medular

c) Mucinoso

d) Tubular

e) Ninguno

MA171. La mama es sitio de metástasis de primarios en:

a) Melanoma

b) Pulmón

c) Sarcomas

d) Próstata

e) Todos

MA172. El azul de isosulfan al 1% se emplea para:

a) Delimitar el área por irradiar

b) Delimitar el área quirúrgica

c) Localizar el ganglio centinela


d) Aplicación de la quimioterapia

e)Ninguno

MA173. Dosis de radiación recibida en las dos tomas en una mamografía:

a) 1 Gy

b) 1.4 Gy

c) 1.5 Gy

d) 0.40 cGy

e) Ninguna

MA174. El Colegio Americano de Radiología clasifica las imágenes mamográficas en categorías. Cuando se
indica categoría simple significa que:

a) El paciente no tiene ninguna imagen sospechosa

b) La lesión es benigna

c) Se requiere de una biopsia

d) Existe alta probabilidad de malignidad

e) Ninguna

MA175. ¿Qué ventajas tiene la reconstrucción mamaria inmediata en la mastectomía por cáncer de mama?

a) Aumenta el intervalo libre de enfermedad

b) Aumenta la sobrevida global

c) Tiene efecto cosmético y mejora la esfera psicológica

d) No tiene ninguna ventaja

e) Eleva el riesgo quirúrgico

MA176. ¿Qué significado tiene la linfopatía axilar contralateral en el cáncer mamario?

a) Un nuevo cáncer primario (oculto con metástasis axilares)

b) Metástasis de lado opuesto


c) Metástasis de un sitio extramamario

d) Todo lo anterior

e) Nada de lo anterior

MA177. ¿Cuál es el principal factor de riesgo en un cáncer mamario contralateral?

a) Edad

b) Historia familiar

c) Genético

d) Radiación previa

e) Haber tenido cáncer en la otra mama

MA178. ¿Cómo se define al cáncer mamario asociado a embarazo?

a) Al que ocurre durante el embarazo

b) Al que ocurre durante el embarazo y la lactancia

c) Al que ocurre durante el embarazo y hasta un año después

d) Al que ocurre al inicio del embarazo

e) Al que ocurre hasta dieciocho meses después del embarazo

MA179. El tratamiento local de elección del cáncer mamario en la mujer embarazada es:

a) Mastectomía radical modificada

b) Mastectomía radical

c) Tumorectomía

d) Tumorectomía y radioterapia

e) Mastectomía radical modificada y radioterapia

MA180. Qué características tiene el carcinoma mamario mínimo:

a) Lesión de 1 cm de diámetro

b) Lesión de 1.5 cm de diámetro


c) Lesión de 2 cm de diámetro

d) Lesión de 0.5 cm de diámetro

e) No existe una dimensión definida

MA181. Referente al carcinoma mamario, con un tumor primario T3 (>51 mm) ¿qué porciento presentan
metástasis ganglionares axilares ipsilaterales?

a) 100

b) 80

c) 60

d) 30

e) 10

MA182. En el carcinoma mamario, ¿hasta qué grado se considera bajo riesgo, según la Escala de Scarff-
Bloom-Richardson modificada?

a) 2

b) 3

c) 4

d) 5

e) No inferire el grado de riesgo

MA183. Factor que aumenta el riesgo de recurrencia en el carcinoma mamario, después de escisión tumoral
completa, más radioterapia en lesiones iguales o menores a 5.0 cm:

a) Márgenes muy cercanos al corte

b) Invasión vascular linfática

c) Glándulas mamarias pequeñas

d) Paciente premenopáusicas

e) Reacción linfocítica peritumoral discreta

MA184. En referencia al carcinoma ductal in situ, según el índice de Van Nuys ¿hasta que puntaje se
consideran de bajo riesgo?
a) 5

b) 6

c) 3

d) 4

e) 7

MA185. El tratamiento de una recurrencia local con invasión de la pared costal posterior a tratamiento de
mastectomía, radioterapia y quimioterapia en ausencia de enfermedad sistémica es:

a) Resección de parrilla costal

b) Nuevo ciclo de radioterapia

c) Nuevo esquema de quimioterapia

d) Observación, medicación sistomática

e) Curaciones de la lesión

MA186. El diagnóstico de recurrencia local después de una cirugía conservadora de mama se puede
establecer:

a) Solamente con mastografía

b) Solamente por clínica

c) Por clínica y mastografía

d) Por ultrasonido mamario

e) PET

MA187. El tratamiento más aceptado en el manejo del cáncer de mama con recurrencia local poscirugía
conservadora sin evidencia de enfermedad sistémica es:

a) Nueva excisión del tumor localmente

b) Tratamiento de radioterapia

c) Mastectomía radical modificada

d) Quimioterapia sistémica

e) RT y QT concomitante
MA188. Contraindicación para efectuar la resección de recurrencia en parrilla costal posmastectomía,
radioterapia y quimioterapia

a) Metástasis cerebrales

b) Invasión de médula ósea

c) Tumor en corasa

d) Todas las anteriores

e) Ninguna de las anteriores

MA189. La persistencia de metástasis pulmonar única, sin evidencia de actividad tumoral sistémica en
paciente con buen estado general, postratamiento de cirugía, hormonoterapia y más de dos esquemas de
quimioterapia es:

a) Observación

b) Resección quirúrgica de la metástasis

c) Radiación

d) Quimioterapia

MA190. La resección de parrilla costal por recurrencia de cáncer de mama da como resultado los efectos
siguientes, excepto:

a) Mejora la calidad de vida

b) Prolonga el tiempo de sobrevida

c) Algunas enfermas pueden ser curadas

d) Disminuye el tiempo de sobrevida

MA191. Cuántos nuevos casos de cáncer de mama se presentan anualmente en el país?

a) 1000–2000

b) 3000–5000

c) 9000–10000

d) 100–500

e) 18000–20000
MA192. Es un factor de pronóstico adverso para cirugía conservadora en carcinoma canalicular in situ.

a) Tumor infiltrante

b) Más de 10 mitosis por campo de alto poder

c) Invasión vascular

d) Patrón comedocarcinoma

e) Antecedentes familiares de cáncer de mama

MA193. Es una contraindicación para realizar cirugía conservadora en carcinoma mamario:

a) Tercer trimestre de embarazo

b) Tumor de 3 cm de diámetro mayor

c) Uso de quimioterapia neoadyuvante

d) Etapa clínica IIA

e) Relación mama – tumor desfavorable

MA194. Criterios de Van Nuys:

a) Formación de túbulos, pleomorfismo nuclear, mitosis

b) Tamaño del tumor, márgenes, citopatología

c) Tumor central, más de 4 ganglios positivos, tumor de más de 4 cm

d) HER2/neu, p53, bcl-2

e) Patrón comedo, microcalcificaciones, formación de glándulas

MA195. Concepto de microinvasión en carcinoma mamario

a) Tumor menor de 5 mm de diámetro mayor

b) Tumor con infiltración menor de 1 mm de diámetro mayor

c) Tumor menor de 10 mm de diámetro mayor

d) Tumor menor de 20 mm de diámetro mayor

e) Tumor con infiltración mayor de 1 mm de diámetro mayor


MA196. Gen implicado en el cáncer de mama familiar:

a) APC

b) WTN

c) p26

d) BRCA1

e) MYCN

MA197. En qué porciento se presentan recurrencias óseas por cáncer de mama:

a) 5–10

b) 15–20

c) 30–40

d) 50–60

e) 80–90

MA198. Es un inhibidor de aromatasa:

a) Megestrol

b) Droloxifén

c) Goserelina

d) Raloxifén

e) Anastrozole

MA199. El raloxifén es:

a) Progestina

b) Modulador selectivo de receptores estrogénicos

c) Análogo LH-RH

d) Inhibidor de aromatasa

e) Andrógeno
MA200. Es un bifosfonato:

a) Trastuzumab

b) Aminoglutetimida

c) Megestrol

d) Pamidronato

e) Capecitabine

MA201. En el TNM, un tumor con invasión a piel se etapifica como:

a) T1B

b) T2A

c) T4D

d) T4B

e) T3A

MA202. Es un dato sugestivo de la presencia de mutación en BRCA2:

a) Tumor poco diferenciado

b) Antecedente familiar de carcinomas y sarcomas

c) Antecedente familiar de cáncer de mama en hombres

d) Antecedente familiar de sarcomas de partes blandas

e) Antecedente familiar de leucemia

MA203. Riesgo anual de cáncer endometrial con uso de tamoxifén:

a) 2 en 1000

b) 2 en 10000

c) 5%

d) 10%

e) 50%
MA204. Se considera una etapa localmente avanzada:

a) I

b) IIA

c) IV

d) IIIB

e) IS

MA205. Procedimiento resectivo super-radical:

a) Patey

b) Halsted

c) Madden

d) Wangensteen

e) Auchincloss

MA206. El músculo serrato anterior está inervado por el:

a) Intercostobraquial

b) Toracodorsal

c) Perforante

d) Pectoral medial

e) Torácico largo

MA207. Se considera una lesión precursora o marcadora de malignidad en mama:

a) Mastopatía fibroquística

b) Ectasia ductal

c) Papiloma intraductal solitario

d) Metaplasia apócrina

e) Hiperplasia atípica
MA208. Incidencia media del cáncer mamario en hombres:

a) 5% de los cánceres en hombres

b) 1% de los cánceres en hombres

c) 10% de los cánceres mamarios

d) 1% de los cánceres mamarios

e) 4% de los cánceres en población general

MA209. El tratamiento en la enfermedad de Paget mamaria se basa en:

a) El grado de destrucción del pezón

b) La extensión de la enfermedad a la piel periareolar

c) La edad de la paciente

d) El tumor subyacente

e) La carga genética de la paciente

TUMORES DIGESTIVOS

DG1. Se aprecian heces melénicas con deficiencia de hierro y masa tumoral en el cáncer de colon:

a) Sigmoide

b) Descendente

c) Ascendente

d) Transverso

DG2. Proporciona información específica acerca de la profundidad de invasión del cáncer rectal:

a) Rectoscopía

b) Colon por enema

c) Colonoscopía

d) Ultrasonido endorrectal

DG3. En el cáncer del recto localizado en su tercio superior, el tratamiento es:


a) Hemicolectomía izquierda

b) Resección anterior

c) Resección abdominoperineal

d) Escisión local

DG4. Se origina en el tejido neuroectodérmico (células APUD), asintomático en gran parte de los casos, en
menos del 5% puede dar metástasis:

a) Feocromocitoma

b) Adenoma velloso

c) Carcinoide

d) Astrocitoma

DG5. Tratamiento de elección en cáncer del canal anal de tipo cloacogénico:

a) Inmunoterapia

b) Radioterapia + quimioterapia

c) Quimioterapia

d) Cirugía

DG6. Adenoma rectal, sesil, plano, aterciopelado, blando, menor de 4 cm que en un 90% de casos puede
malignizarse:

a) Adenoma velloso

b) Adenoma tubular

c) Adenoma mixto

d) Fibroadenoma

DG7. La elevación del marcador tumoral ACE en el cáncer del colon:

a) Indica recidiva tumoral o metástasis

b) Predice el tamaño tumoral

c) Se relaciona con la etapa


d) Indica pronóstico pretratamiento

DG8. Se entiende como prevención secundaria en el cáncer colorrectal la:

a) Extirpación de lesiones precanceosas o completa del órgano

b) Administración de vitamina C y calcio

c) Alimentación rica en avena y colesterol

d) Alimentación rica en grasa y colesterol

DG9. Múltiples adenocarcinomas colónicos y extracolónicos:

a) Poliposis juvenil

b) Síndrome Peutz Jehers

c) Síndrome de Lynch I

d) Síndrome de Lynch II

DG10. Márgenes en los que se considera al adenocarcinoma como el tipo más frecuente de cáncer colónico:

a) 40 a 50%

b) 50 a 60%

c) 70 a 80%

d) 90 a 95%

DG11. Sobrevida a cinco años para pacientes de cáncer colorrectal con invasión a mucosa y submucosa es:

a) Menos del 50%

b) Más del 90%

c) Menos del 80%

d) Entre 60 y 70%

DG12. Sitio de mayor recurrencia en el cáncer rectal:

a) Local

b) Ovario
c) Hígado

d) Pulmón

DG13. Complicación más frecuente del cáncer en colon izquierdo:

a) Infección

b) Obstrucción

c) Perforación

d) Hemorragia

DG14. Los carcinomas del márgen anal deben tratarse como cáncer:

a) De piel

b) Colónico

c) De recto

d) Apocrino

DG15. Principal sitio de diseminación del cáncer anal por vía hematógena:

a) Hueso

b) Hígado

c) Cerebro

d) Pulmón

DG16. Principal signo-síntoma en el cáncer anal:

a) Prurito

b) Tenesmo

c) Cambios en el hábito intestinal

d) Sangrado

DG17. Factores pronósticos principales en el cáncer colorrectal:


a) Etapa clínica, tamaño, tipo histológico

b) Edad del paciente y tratamiento

c) Antecedentes neoplásicos familiares

d) Estado físico del paciente y ploidia

DG18. Tratamiento de elección en la enfermedad de Paget del ano:

a) Escisión local

b) RAP

c) RT

d) QT

DG19. El involucro de ganglios celíacos en el momento del diagnóstico del cáncer de tercio medio de
esófago:

a) No existe

b) Es menor en el cáncer epidermoide

c) Es mayor en el adenocarcinoma

d) Es alrededor de 5%

e) Es mayor al 40%

DG20. Factor predisponente más frecuente en el cáncer de vesícula biliar:

a) Litiasis

b) Parasitosis

c) Pólipos

d) Pancreatitis

e) Colangitis

DG21. Juicio correcto de la terapia adyuvante en el cáncer colorrectal:

a) 5FU + NCCNU mejora intervalo de enfermedad pero no la sobrevida


b) 5FU + Metil CCNI son drogas de elección en el tratamiento adyuvante

c) El 5FU y levamisol han demostrado mejoría en la sobrevida a cinco años en el adenocarcinoma


etapa Dukes C

DG22. Los pacientes con enfermedad inflamatoria intestinal tienen riesgo de desarrollar carcinoma
colorrectal:

a) 4 a 20 veces mayor que la población

b) 40 veces mayor

c) No existe relación

d) En un doble porciento de la población general

DG23. Son ventajas de la RT preoperatoria en recto las siguientes, excepto para:

a) Aumentar el control locorregional

b) Para esterilizar micrometástasis ganglionares

c) Para esterilizar extensión microscópica del tumor

d) Para disminuir el porciento de metástasis a distancia

DG24. Son factores predisponentes a cáncer gástrico los siguientes, excepto:

a) Pólipos adenomatosos gástricos

b) Irradiación previa al estómago

c) Cirugía previa de estómago

d) Presencia de Helicobacter pylori

e) Gastritis atrófica

DG25. El riesgo de cáncer pancreático aumenta en pacientes que:

a) Tienen una historia de exposición a las nitrosaminas y benzidina

b) Tienen una historia prolongada de abuso de tabaco y alcohol

c) No tienen historia familiar de cáncer de páncreas

d) Tienen expresión del protooncogén K-Ras en sangre


e) Tienen expresión del gen Mts-1 (Gen supresor tumoral múltiple en cromosoma 9 p. 21)

DG26. Se consideran precancerosos los pólipos colorrectales siguientes, excepto:

a) Adenoma velloso

b) Adenoma tubular

c) Hiperplásico

d) Adenoma tubulovelloso

e) Adenoma de 4 cm

DG27. Son factores asociados con mayor riesgo de cáncer de esófago todos los siguientes, excepto:

a) Ingesta elevada de alcohol

b) Tabaquismo

c) Esófago de Barrett

d) Antecedentes familiares de cáncer esofágico

e) Nitrosaminas

DG28. Manifestación más frecuente en pacientes con cáncer de esófago:

a) Pérdida de peso

b) Disfagia

c) Dolor o ardor retroesternal o epigástrico

d) Vómito o regurgitación

e) Disfonía

DG29. Porciento de pacientes con cáncer hepatocelular y anticuerpos para hepatitis C positivo:

a) 5%

b) 10%

c) 20%

d) 30%
e) 50%

DG30. Los adenocarcinomas del recto localizados a 5 cm del márgen anal se tratan con:

a) RAP

b) Resección local

c) Resección anterior

d) Resección sacra

DG31. Tratamiento del cáncer rectal con invasión a vejiga, sin metástasis a distancia:

a) RAP

b) Exenteración pélvica

c) Resección anterior baja

d) Radioterapia y quimioterapia

DG32. La radioterapia preoperatoria en el cáncer rectal tiene como objeto:

a) Quitar el dolor

b) Evitar la hemorragia

c) Disminuir la recurrencia local

d) Evitar la colostomía

DG33. Actualmente la colangiopancreatografía retrógrada debe usarse para cáncer de páncreas como:

a) Método auxiliar de diagnóstico

b) Método auxiliar para paliación biliar

c) Estudio que sustituye una TAC

d) En el preoperatorio inmediato

e) Auxiliar en la evaluación de resecabilidad

DG34. En la cirugía de ascenso gástrico para sustitución esofágica debe preservarse la:

a) Arteria gástrica izquierda

b) Mayor parte de los vasos gástricos cortos


c) Arteria gastroepiploica izquierda

d) Arteria gástrica derecha, rama de la gastroduodenal

e) Arteria esplénica

DG35. El cáncer avanzado de esófago se trata mejor con:

a) Esofagectomía transhiatal tipo Orriger

b) Puenteo retroesternal con colon

c) QT y RT combinadas

d) Gastrostomía

e) Hormonoterapia e inmunoterapia

DG36. El esófago de Barret es quirúrgico cuando:

a) Presenta datos de esofagitis 611-111

b) Existe en menores de 40 años

c) Hay displasia moderada-severa

d) Persiste a pasar del tratamiento médico por un año

e) Tiene más de 3 cm de extensión

DG37. Las estadísticas muestran que en el primer año posterior al diagnóstico de cáncer de páncreas la
mortalidad es de:

a) 50%

b) 60%

c) 70%

d) 80%

e) 90%

DG38. La incidencia de metástasis en el carcinoma de intestino delgado de más de 2 cm es de:

a) 0%
b) 10%

c) 20 a 40%

d) 50 a 70%

e) Más del 80%

DG39. El síndrome carcinoide atípico es más común en:

a) Pulmón y bronquios

b) Yeyuno-íleon

c) Colon

d) Recto

e) Apéndice vermiforme

DG40. ¿Qué ventajas tiene el procedimiento clásico de Whipple sobre la preservación del píloro en la
supervivencia?

a) Existen más recidivas en la preservación pilórica

b) Ninguna

c) En la preservación del píloro la mortalidad es mayor

d) Los pacientes sometidos a pancreatoduodenectomía clásica sobreviven mejor a cinco años

e) En la preservación pilórica existen más complicaciones posoperatorias

DG41. Mejor paliación en pacientes con cáncer avanzado de vías biliares que cursan con ictericia
obstructiva:

a) Nada

b) Prótesis (endoscópica)

c) Derivación biliodigestiva

d) Resección local amplia

e) Operación de Whippe

DG42. Factor pronóstico más importante en el cáncer gástrico:


a) Grado de diferenciación

b) Extensión local de la lesión

c) Tipo histológico

d) Presencia de ganglios metastásicos

DG43. Diagnóstico histopatológico más frecuente en el cáncer gástrico:

a) Epidermoide

b) Adenocarcinoma

c) De células en anillo de sello

d) Linfoma

e) Sarcoma

DG44. Son características de la poliposis familiar las siguientes, excepto:

a) El patrón de herencia autosómico dominante

b) La presencia de 100 o más pólipos

c) Que el 50% de los afectados desarrollarán cáncer en la 3ª o 4ª décadas de la vida, a menos


que se realice colectomía

d) La penetrancia del 95%

DG45. Factores de mal pronóstico para el carcinoma de colon:

a) Edad de inicio del carcinoma colorrectal antes de los 40 años y perforación

b) Edad de inicio del carcinoma colorrectal antes de los 40 años, obstrucción y perforación

c) Localización primaria del carcinoma por arriba de la reflexión peritoneal y obstrucción

d) Obstrucción y perforación

DG46. El esófago de Barret se encuentra en pacientes con esofagitis en un margen de:

a) 0 a 10%

b) 5 a 25%

c) 8 a 20%
d) 3 a 15%

DG47. El tratamiento en pacientes que desarrollan adenocarcioma en un esófago de Barret es:

a) Omeprazol más amoxicilina más sales de bismuto

b) Funduplicatura de Nissen

c) Esofagogastrectomía

d) Funduplicatura laparoscópica más tratamiento médico

DG48. No es un factor de riesgo para el cáncer de hígado y vías biliares:

a) El contacto continuo con Asperguillus flavus

b) Los esteroides anabólicos y anticonceptivos

c) El tabaquismo

d) La infestación por Cionorchis

e) La hepatitis

DG49. ¿Cuál de los estudios o procedimientos siguientes no se considera primario en el diagnóstico de


tumores hepáticos?

a) Transaminasas hepáticas

b) Fosfata alcalina

c) Antígeno carcinoembrionario

d) Albúmina

e) Alfafetoproteína

DG50. Agentes quimioterápicos de mayor utilidad en el tratamiento del cáncer hepático:

a) Fluouracílico

b) VP-16

c) Mitrozantron

d) Doxorrubicina

e) Iomustine
DG51. El tumor de Krukenberg corresponde a metástasis de un adenocarcinoma:

a) Primario de estómago

b) De células en anillo de sello

c) De páncreas

d) De colon

e) De esfínteres

DG52. Opción de tratamiento para paciente con carcinoma hepatocelular de 2 cm:

a) La radiación hepática

b) La QT con fluorouracilo y adriamicina

c) La resección quirúrgica

d) La ablación crioquirúrgica

DG53. En el melanoma del canal anal sin evidencia de metástasis el tratamiento de elección es:

a) Escisión local

b) Resección abdominoperineal

c) Exenteración total

d) RT

DG54. Efecto tóxico más significativo del UFT:

a) Mielosupresión

b) Vómito

c) Síndrome mano-pie

d) Neurotoxicidad

e) Diarrea

DG55. Combinación de fármacos no recomendable en el tratamiento del cáncer gástrico avanzado:

a) 5FU, adriamicina, mitomicina C


b) Cisplatino, BCNU

c) 5FU, BCNU

d) 5FU, epirubicina, cisplatino

e) Etopósido, adramicina, cisplatino

DG56. Tratamiento de un tumor carcinoide apendicular de más de 2 cm en su base:

a) Apendicectomía

b) Tumorectomía

c) Escisión local + RT

d) Hemicolectomía derecha

e) Observación

DG57. Tratamiento más adecuado del carcinoide gástrico de más de 2 cm con hipergastrinemia:

a) Escisión endoscópica

b) Hemigastrectomía

c) Tumorectomía por laparoscopía

d) Gastrectomía más linfadenectomía

e) Observación

DG58. Lesión(es) cardiaca(s) más común(es) por síndrome carcinoide:

a) Cavidades derechas

b) Cavidades izquierdas

c) Válvula pulmonar y tricuspídea

d) Válvula mitral

e) Aorta

DG59. La ubicación más frecuente del síndrome de Cushing con producción ectópica de ACTH y con tumor
carcinoide asociado, es en:
a) Tráquea y bronquios

b) Estómago

c) Apéndice

d) Colon

e) Recto

DG60. Cuál es el porciento de sobrevida en pacientes con cáncer de páncreas, incluyendo aquellos
sometidos a resección con fines curativos:

a) 5-10

b) 10-20

c) 20-30

d) 30-40

e) 40-50

DG61. La TAC en el cáncer de páncreas sirve para determinar:

a) Extensión e histología

b) Resecabilidad

c) Implantes peritoneales

d) Sitio del tumor

e) Metástasis hepáticas

DG62. La probabilidad de encontrar implantes peritoneales en cáncer del cuerpo y cola del páncreas es de
aproximadamente:

a) 5%

b) 10%

c) 27%

d) 65%

e) 80%
DG63. La dificultad para el diagnóstico de cáncer de páncreas con BAAF es:

a) Que sólo se puede realizar en tumores de más de 4 cm

b) Que requiere siempre guía con US y TAC

c) Cuando el tumor se localiza al nivel del proceso uncinado

d) Que no se puede diferenciar entre el linfoma y cáncer de células insulares

e) Que los pacientes ancianos no toleran el procedimiento

DG64. La mayor utilidad de BAAF en páncreas es:

a) Para realizar cirugía conservadora

b) En el cáncer de cabeza del páncreas

c) En tumores irresecables del cuerpo y cola del páncreas

d) En cáncer menor de 2 cm

e) En pacientes que se someterán a laparotomía exploradora

DG65. Condición predisponente para cáncer de esófago:

a) Divertículo esofágico

b) Acalasia

c) Esófago de Barret

d) Tylosis

e) Todas las anteriores

DG66. Son manifestaciones clínicas del cáncer esofágico:

a) Pérdida de peso, tos, nódulo de Mary Joshef, melena

b) Disfagia, odinofagia, derrame pleural, tromboflebitis migratoria

c) Disfagia, disfonia, neumonía por aspiración, síndrome de Horner

d) Nódulos supracaviculares, disfagia, epistaxis, síndrome de VCS

e) Todas las anteriores

DG67. La clasificación de Child para cáncer hepatocelular incluye los criterios siguientes:

a) Insuficiencia hepática, bilirrubina sérica, ascitis encefalopatía hepática


b) Insuficiencia hepática, transaminasas, bilirrubina, fosfatasa alcalina, albúmina sérica

c) Insuficiencia hepática, antígeno de superficie positivo para hepatitis B, albúmina, ascitis,


encefalopatía hepática

d) Encefalopatía hepática, ascitis, albúmina sérica, bilirrubina sérica, retención de tinción verde
indiocianina y bromosulftaleina

e) Todos los anteriores

DG68. Factores de mejor pronóstico de hepatocarcinoma:

a) Sin metástasis intrahepáticas y asociado a cirrosis micronodular

b) Con metástasis cercanas al primario asociado a cirrosis macronodular y con antecedentes de


hepatitis crónica persistente

c) Tumor menor de 5 cm, cápsula tumoral-grado de retención de tinción de verde indocianina


favorable

d) Con posibilidad de trasplante hepático, no asociado a hepatitis B, sin metástasis intrahepáticas,


asociado a fibrosis y no cirrosis hepática

e) Ninguno de los anteriores

DG69. Sobrevida a cinco años de pacientes con cáncer de vesícula biliar con invasión a la serosa:

a) 0 a 20%

b) 20 a 40%

c) 40 a 60%

d) 60 a 80%

e) 80 a 100%

DG70. Manejo ideal de paciente masculino de 40 años de edad con tumor de 4 cm no fijo al sacro y a 6 cm
del margen anal:

a) Resección abdomineal posterior a RT

b) Resección anterior baja posterior a RT + QT

c) Exentración pélvica posterior a RT + QT

d) Resección abdominoperineal + QT + RT

e) Colostomía + RT a pelvis + RM

DG71. Tratamiento de cáncer gástrico in situ polipoide en un paciente de 80 años de edad, localizado en el
1
/3 medio del estómago:
a) Gastrectomía total

b) Gastrectomía subtotal

c) Gastrectomía ampliada

d) Gastrectomía con disección de ganglios N-1, N-2 y N-3

e) Resección endoscópica

Relacione las columnas siguientes:

DG72. Seminoma ( )

DG73. Ca embrionario ( )

DG74. Teratoma ( )

DG75. Linfoma ( )

DG76. Coriocarcinoma ( )

a) Diseminación hematógena

b) Radiosensible

c) Es frecuente en varones mayores de 50 años

d) Elevación de AFP y gonadotrofina coriónica

e) Resistente a QT y RT

DG77. La resección anterior baja posterior a RT+QT está indicado en paciente masculino de 45 años de
edad con tumor de 3.5 cm no fijo al sacro y a 5 cm del margen anal:

a) Verdadero b) Falso

DG78. ¿En cuál de los siguientes cánceres del complejo pancreaticobiliar, el tratamiento multimodal ha
probado en un estudio randomizado cierto tipo de beneficio?

a) Vesicular

b) Del síndrome de Klatskin de los conductos biliares proximales

c) De los conductos biliares


d) Pancreático

DG79. En la quimioterapia del cáncer gástrico avanzado es cierto que:

a) No mejora la supervivencia comparativamente con mejor soporte médico

b) Los principales factores de respuesTa son el estado general y la extensión de la enfermedad

c) El esquema FAM (5FU, adriamicina y mitomica) es considerado el mejor

d) La quimioterapia intraperitoneal causa mayor toxicidad, pero con respuestas globales del 60 al
80%

e) La sobreexpresión de timidilato sintetasa esta relacionado a alta probabilidad de respuesta al


5FU

DG80. Cáncer gástrico que invade la muscularis propia y la subserosa con seis ganglios linfáticos regionales
positivos para cáncer, sin metástasis. Queda clasificado según la AJCC como:

a) T1 N2 M0–Estadio II

b) T2 N2 M0–Estadio IIIA

c) T2 N1 M0–Estadio II

d) T3 N2 M0–Estadio IIIB

e) T2 N3 M0–Estadio IV

DG81. El cáncer gástrico limitado a la mucosa o estadio 0, manejado con gastrectomía y linfadenectomía
según la experiencia Japonesa y Americana demuestran una sobrevida a cinco años de:

a) 100%

b) Más del 90%

c) 60%

d) Son tan pocas que aún no hay evidencia de sobrevida

e) Entre 70% y 80%

DG82. En pacientes con cáncer de colón en donde el primario está controlado y tiene metástasis hepáticas
limitadas (tres o menos), con la metastasectomía hepática se puede considerar una sobrevida a cinco años
de:

a) 5 a 10%
b) 15%

c) 20 a 40%

d) 45%

e) 60%

DG83. Un cáncer de canal anal de 2 cm, con involucro de esfínter del ano, el tratamiento de elección es:

a) Resección local amplia

b) Resección abdominoperineal

c) Radioterapia con o sin quimioterapia

d) Resección local amplia más disección inguinal

e) Quimioterapia y resección local

DG84. En cáncer gástrico, las lesiones del tercio inferior, se tratan con gastrectomía tipo R1, que incluye los
siguientes relevos ganglionares, excepto:

a) De la curvatura menor

b) De la curvatura mayor

c) Paracardiales

d) Suprapilóricos

e) Infrapilóricos

DG85. Son variedades de poliposis adenomatosa familiar los siguientes, excepto:

a) Síndrome de Gardner

b) Síndrome de Turcot

c) Síndrome de Linch I y II

d) Síndrome de Oldfield

e) c y d

DG86. El tumor de Klatskin se localiza en:


a) El ámpula de Vater

b) El tercio inferior del colédoco

c) La confluencia de los hepáticos

d) En el conducto de Wirsung

e) En la vesícula biliar

DG87. La poliposis adenomatosa familiar, la osteomatosis, el quiste epidermoide y el fibroma de piel


integran el síndrome de:

a) Peutz-Jeghers

b) Carcinoide

c) Gardner

d) Turcot

DG88. Poliposis adenomatosa familiar y malignidades del sistema nervioso central integran el síndrome de:

a) Turcot

b) Peutz-Jeghers

c) Carcinoide

d) Gardner

DG89. Carácter hereditario autosómico dominante, predominancia de colon proximal sincrónico con otros
cánceres de colon, presentación promedio a los 44 años, corresponden al síndrome de:

a) Gardner

b) Lynch I

c) Lynch II

d) Carcinoide

DG90. Quimioterapia adyuvante en el cáncer de colon con metástasis a nódulos linfáticos:

a) Adriamicina

b) 5FU + leucovorin

c) Mitomicina C + cisplatino
d) Leucovorin

DG91. El tumor carcinoide se presenta en el tracto gastrointestinal con mayor frecuencia en:

a) Colon

b) Recto

c) Íleon

d) Apéndice

DG92. Dietas altas en grasa saturada y bajas en fibra se asocian con el cáncer:

a) De estómago

b) De esófago

c) De intestino delgado

d) Colorrectal

DG93. El virus del papiloma humano número 16 se relaciona más con el:

a) Condiloma acuminado

b) SIDA

c) Carcinoma anal de células escamosas

d) Melanoma

DG94. Los ganglios linfáticos del márgen anal drenan a ganglios:

a) Hipogástricos

b) Pararrectales

c) Rectales superiores

d) Inguinofemorales

DG95. La capacidad de malignidad de un tumor carcinoide está determinada por:

a) Índice mitótico
b) Invasión ganglionar o a distancia

c) Carga de aminas y péptidos

d) Origen embriológico

e) Histología

DG96. El evento que explica más frecuentemente la producción del síndrome carcinoide atípico es la
ausencia de la enzima:

a) Dopadescarboxilasa

b) Triptófanohidroxilasa

c) Monoaminooxidasa

d) Aldheido deshidrogenasa

e) Alfarreductasa

DG97. Sitios más frecuentes de metástasis en el cáncer gástrico:

a) Ganglios linfáticos regionales e implantes peritoneales

b) Ganglios linfáticos más hígado

c) Hígado y pulmón

d) Implantes peritoneales y pulmón

DG98. Los criterios de Amsterdam para el diagnóstico de cáncer colorrectal hereditario no polipoideo son
los siguientes, excepto:

a) Familiares con cáncer colorrectal (2 de 1er grado y 1 de 2º grado)

b) 3 familiares con cáncer colorrectal (2 de 2º grado y 1 de 1 er grado)

c) Edad de presentación menor a los 50 años

d) Dos familares afectados

DG99. Patologías con menor riesgo predisponente para cáncer colorrectal:

a) Poliposis familiar múltiple

b) Síndrome de Gardner
c) Síndrome de Turcot

d) CUCI

e) Enfermedad de Crohn

DG100. El porciento de pacientes con esófago de Barret que desarrollan adenocarcinomas fluctúan del:

a) 5 a 15.5

b) 0.1 a 46.5

c) 3 a 20.5

d) 10 a 25

DG101. Los cuatro sitios más frecuentes en que se localizan los tumores carcinoides son:

a) Yeyuno, hueso, tiroides y apéndice

b) Estómago, boca, yeyuno y piel

c) Apéndice, recto, yeyuno y bronquios

d) Hipófisis, cerebro, tiroides y pulmones

e) Ganglios, músculo, páncreas y bronquios

DG102. En relación con el papel de la cirugía en el tratamiento del linfoma gástrico, una de las opciones
siguientes es incorrecta:

a) Previene hemorragias cuando se realiza antes del tratamiento de QT

b) Previene perforaciones si se realiza antes de RT

c) Puede ser curativa en etapas clínicas IE

d) Disminuye las recurrencias y mejora la supervivencia

e) La detumorización no ha probado hasta el momento beneficio en disminuir las recurrencias o


mejorar la supervivencia

DG103. Factor de riesgo para cáncer gástrico:

a) Consumo prolongado de alcohol

b) Infección por Helicobacter pylori restringido al cáncer tipo difuso

c) Uso crónico de omeprazol


d) Alimentos preservados y salados

e) Consumo de frutas y vegetales

DG104. Características del cáncer gástrico temprano:

a) No penetra más allá de la mucosa o submucosa, con presencia o ausencia de metástasis a


ganglios linfáticos

b) La metástasis a ganglios linfáticos lo catalogan en etapa avanzada

c) La duración media entre el diagnóstico y la progresión es de 20 meses

d)Las metástasis a hígado o hueso ocurren en 20% de los casos

e) El 50% de los pacientes con tumores confinados a submucosa sobreviven en promedio cinco
años

DG105. Entre las nuevas drogas para el cáncer de colon se tiene:

a) El nolatrexed que es un fluorinado oral con poca utilidad

b) El oxaliplatino que tiene efecto antagonista cuando se administra con 5 fluorouracilo

c) El raltritexed que es un inhibidor de la timidilato sintetasa y que ha producido respuestas


globales de 15 a 25% en enfermedad metastásica o avanzada

d) El irinotecán, inhibidor específico de la topoisomerasa I, sin toxicidad gastrointestinal

e) El capacitabine que se administra por una intravenosa únicamente

DG106. Actualmente la colangiopancreatografía retrógrada debe usarse para cáncer de páncreas como:

a) Método auxiliar de diagnóstico

b) Método auxiliar para paliación biliar

c) Estudio que sustituye una TAC

d) En el preoperatorio inmediato

e) Auxiliar en la evaluación de resecabilidad

DG107. Candidato a resección de metástasis hepática de cáncer de recto (controlado localmente):

a) Masculino de 55 años con doble primario en pulmón

b) Mayor de 60 años con metástasis única en lóbulo derecho y un implante en serosa de


mesocolon transverso

c) Hombre de 30 años con tres metástasis periféricas, una en cada lóbulo


d) Mujer de 42 años con una metástasis de 6 cm única en hilio hepático

e) Mujer de 85 años con lesión periférica en lóbulo izquierdo, con secuelas de AVC

DG108. Tumor maligno de páncreas con pronóstico favorable:

a) Adenocarcinoma mucinoso

b) Carcinoma de células acinares

c) Carcinoma de células de los islotes

d) Adenocarcinoma quístico y papilar

e) Sarcoma pancreático

DG109. El síndrome carcinoide ocurre con más frecuencia en aquellos localizados en:

a) Región traqueobronquial

b) Estómago

c) Yeyuno-íleon

d) Colon

e) Recto

DG110. El mejor método diagnóstico para el cáncer gástrico es:

a) Serie esófagogastroduodenal

b) Quimismo gástrico

c) Ultrasonografía endoscópica

d) TAC

e) Endoscopía con biopsia

DG111. La incidencia del cáncer gástrico temprano es mayor en:

a) México

b) Japón

c) Asia
d) Países con escrutinio diagnóstico frecuente

DG112. Dentro de las opciones quirúrgicas del paciente con CUCI se encuentra la colectomía total con
iliorrecto anastomosis; con respecto a este procedimiento es verdad que:

a) Los resultados funcionales son comparables a los obtenidos con otros procedimientos
(reservorio ileoanal)

b) No es curativo

c) Elimina el riesgo de carcinoma colorrectal

d) Se correlaciona lo enunciado en a y b

e) Es curativo y elimina el riesgo de cáncer colorrectal

DG113. Enunciado más adecuado asociado al cáncer de colon etapa clínica II:

a) La presencia de perforación se asocia a un menor tiempo de sobrevida libre

b) Incluye tumores Dukes C1 pero no C2

c) En el estudio de intergrupos el 5FU y levamisol disminuyeron significativamente las


recurrencias pero no afectaron la sobrevida

d) Sin terapia adyuvante la sobrevida a cinco años es de 40 a 50%

e) La interrupción de las cifras de los estudios de la terapia adyuvante intergrupal fueron


comprometidas por un número alto de deserciones con intolerancia al régimen

DG114. Son agentes de QT usados junto con RT en el tratamiento de cáncer de conducto anal los siguientes,
excepto:

a) Platino

b) Mitomicina C

c) 5FU

d) Metotrexate

e) Todos los anteriores

DG115. Paciente femenino de 60 años de edad con historia reciente de pérdida de peso, anorexia, ictericia y
sensación de plenitud abdominal. El paso inicial dentro del estudio diagnóstico es:

a) Resonancia magnética sin contraste de abdomen superior.

b) Ultrasonido abdominal superior seguido de tomografía computada

c) Colangiografía transhepática
d) Colangiopancreatografía retrógrada endoscópica (ERCP) seguida de exploración abdominal
inmediata

e) RT externa, solamente

DG116. Droga de elección en el control de los síntomas del síndrome carcinoide:

a) Alfametildopa

b) Fenoxibenzamina

c) Metilsergida

d) Ciproheptadina

e) Octreótido

DG117. Paciente femenino de 48 años de edad con antecedentes heredo familiares patológicos de cáncer del
colon en el abuelo y el padre, con cáncer rectal a 2 cm del margen anal más cáncer sincrónico en colon
izquierdo, obstructivo. ¿Cuál es el tratamiento ideal para este paciente?

a) Resección abdominoperineal (involucrando tumor de colon) + colostomía

b) Histerectomía y ooforectomía bilateral

c) Proctocolectomía + histerectomía + ooforectomía bilateral

d) Colectomía izquierda con reservorio coloanal + anastomosis coloanal + histerectomía +


ooforectomía bilateral

e) Colostomía derivativa + resección en dos tiempos

DG118. Un paciente de 70 años de edad con historia de cáncer de colon resecado 3 años antes, y tratado con
QT adyuvante base de fluorouracilo y leucovorin, se presenta con una TAC de abdomen positiva para
múltiples metástasis hepáticas; el tratamiento que recomendaría es:

a) Hepactomía seguida de transplante de hígado

b) Quimioterapia con Adriamicina y Taxol

c) Quimioterapia con oxiliplatino

d) Interferón tres veces por semana

e) Interleucina 1L-2 a dosis altas por tres semanas

DG119. La quimioterapia adyuvante en cáncer gástrico debe usarse:


a) En jóvenes

b) En paciente con buen estado funcional

c) Sólo en pacientes dentro de un protocolo

d) En todos los pacientes

e) En ningún paciente

DG120. Son factores pronósticos adversos en el cáncer de colón, excepto:

a) Penetración del tumor a través de la pared intestinal

b) Involucro ganglionar

c) Obstrucción y perforación intestinal

d) Niveles séricos elevados pretratamiento de antígeno carcinoembrionario

e) Asociación con cáncer de colón no poliposo hereditario

DG121. Un cáncer de canal anal de 3 cm con metástasis ganglionar inguinal unilateral sin metástasis a
distancia queda clasificado según el sistema TNM de la AJCC como:

a) T2 N2 M0–Estadio IIIB

b) T1 N2 M0–Estadio IIIB

c) T2 N1 M0–Estadio IIIA

d) T1 N1 M0–Estadio IIIA

e) T2 N3 M0–Estadio IIIB

DG122. La clasificación de Nevin se utiliza en:

a) Cáncer de páncreas

b) Colangiocarcinoma

c) Cáncer hepático

d) Cáncer vesicular

e) Cáncer de mama

Correlacione los tumores epiteliales del páncreas exocrino.


DG123. Cistadenoma mucinoso ( ) a) Benigno
DG124. Cistadenoma seroso ( ) b) Potencial maligno incierto
DG125. Neoplasia papilar quística ( ) c) Maligno
DG126. Pancreatoblastoma ( )
DG127. Carcinoma de células pequeñas ( )

DG128. ¿Qué tipo de tumor gastrointestinal se considera como modelo adecuado para el estudio de la
carcinogénesis humana?

a) Gástrico

b) Ovárico

c) Renal

d) Colorrectal

e) Esplénico

DG129. ¿Qué porciento se considera como cierto en relación con la delesión del cromosoma 17p en su
función como gen supresor en cáncer colorrectal?

a) 20

b) 30

c) 50

d) 60

e) 80

DG130. Síndrome familiar condicionado por inactivación del P53:

a) Li-Fraumeni

b) Codwen

c) Goodpasture

d) Turner

e) Anticuerpos antifosfolípidos

DG131. ¿Cuál es el riesgo de desarrollar un carcinoma colorrectal en la poliposis familiar adenomatosa?

a) Ninguno

b) 30%

c) 50%
d) 76%

e) 100%

DG132. La presencia de carcinomas a nivel de intestino grueso, endometrio y ovario como parte de los
trastornos familiares no polipósicos se atribuye al síndrome siguiente:

a) Linch I

b) Linch II

c) Linch III

d) Ataxia-telangiectasia

e) Codwen

DG133. ¿En qué porciento se observa la mutación del RAS en los carcinomas del colon?

a) 10

b) 20

c) 50

d) 70

e) 100

DG134. ¿Cuál de los siguientes factores actúan en forma sinérgica incrementando el riesgo de cáncer
esofágico?

a) Alcohol

b) Verduras

c) Tabaco

d) Plomo

e) A y c

DG135. ¿En que porción del esófago se encuentra la mayor incidencia de cáncer?

a) Tercio proximal

b) Tercio medio
c) Tercio distal

d) Orofaringe

e) Fondo gástrico

DG136. ¿Cuál es el síntoma más frecuente en cáncer esofágico?

a) Dispepsia

b) Disfagia

c) Pérdida de peso

d) B y c

e) Anorexia

DG137. Un tumor esofágico que invade hasta la muscularis propia, con metástasis ganglionares regionales y
sin metástasis a distancia, se clasifica como:

a) T1N0M0

b) T1N1M0

c) T2N0M0

d) T2N1M0

e) T3N2M1

DG138. El T2 N1 M0 de esófago se clasificará como una etapa clínica:

a) I

b) II

c) III

d) IV

e) No clasificable

DG139. En estudios randomizados se ha demostrado que la comparación entre cirugía sola y


quimioradiación en cáncer esofágico resecable es estadísticamente:

a) Similar
b) Superior en el brazo de cirugía

c) Superior en el brazo de quimioradiación

d) Inferior en el brazo de cirugía

e) Inferior en el brazo de quimioradiación

DG140. En qué porciento se encuentra la presencia de esófago de Barrett en endoscopias por esofagitis

a) 3

b) 5

c) 10

d) 15

e) 20

DG141. El esquema actual de quimioterapia para el carcinoma de células escamosas del esófago comprende:

a) Metrotexate

b) Platino

c) 5FU

d) Etoposido

e) b y c

DG142. En el diagnóstico de la profundidad de la neoplasia gástrica y del estado ganglionar regional ¿qué
método diagnóstico se considera más sensible en comparación con la tomografía axial computada?

a) Ultrasonido convencional

b) Gamagrafía con talio

c) Tomografía por emisión de positrones

d) Ultrasonido endoscópico

e) Laparoscopía

DG143. ¿Qué porciento de sobrevida se encuentra posterior a la realización de una cirugía radical de tipo D2
en comparación con la D1, en el tratamiento del cáncer gástrico temprano (T1-3, N-0)?
a) 20

b) 30

c) 50

d) 60

e) 95

DG144. Durante el desarrollo de una gastrectomía distal por cáncer gástrico, ¿qué margen se considera
como adecuado para disminuir la recurrencia local?

a) 1 cm

b) 4 cm

c) 6.5 cm

d) 10 cm

e) 15 cm

DG145. Síndromes hereditarios con predisposición al desarrollo del carcinoma pancreático, excepto:

a) Ataxia-telangiectasia

b) NEM I

c) Codwen

d) Lynch II

e) Von hippel lindau

DG146. ¿En qué porciento se encuentra la presencia de ictericia en cáncer pancreático avanzado como
primer dato clínico?

a) 20

b) 30

c) 50

d) 70

e) 100
DG147. Si durante la estadificación quirúrgica del carcinoma pancreático se encuentra un tumor que invade
el estómago, sin evidencia de crecimientos ganglionares regionales ni metástasis a distancia, éste
corresponde a:

a) T1N0M0

b) T1N1M0

c) T2N0M0

d) T3N1M0

e) T3N0M0

DG148. ¿Cuál es la dosis promedio adecuada para el manejo intraoperatorio con radioterapia en cáncer
pancreático resecable?

a) 10 a 15 Gy

b) 20 a 30 Gy

c) 30 a 40 Gy

d) 40 a 50 Gy

e) 50 a 60 Gy

DG149. ¿Cuál de los siguientes tratamientos se considera como adecuado en el control local del cáncer
pancreático avanzado irresecable?

a) Quimioterapia

b) Radioterapia externa

c) Radioterapia intraoperatoria

d) Braquiterapia de alta tasa de dosis

e) B más c

DG150. ¿En qué porciento se asocia el carcinoma hepatocelular a un proceso de cirrosis hepática?

a) 20 a 30

b) 40 a 50

c) 60 a 80

d) 90 a 95
e) 100

DG151. ¿En qué porciento se encuentra invasión vascular en un carcinoma hepatocelular de 5 cm o más?

a) 90

b) 75

c) 64

d) 54

e) 11

DG152. Se consideran como factores etiológicos del colangiocarcinoma los siguientes, excepto:

a) Hepatolitiasis

b) Torotrast

c) Traumatismos

d) Radiaciones

e) Quiste de colédoco

DG153. Los siguientes son factores que incrementan el riesgo de adenocarcinomas del intestino delgado,
excepto:

a) Enfermedad de Crohn

b) Leishmaniasis

c) Enfermedad celíaca

d) Síndrome de Peutz Jeghers

e) Enfermedad de Von Reckinghausen

DG154. En referencia al carcinoma colorrectal, se han propuesto como marcadores tumorales, aun en
estudio algunos de ellos, excepto:

a) Antígeno carcinoembrionario

b) Ca19-9

c) Tenascina
d) Ploidia nuclear

e) Fosfolipasa

DG155. En el carcinoma gástrico, se consideran factores de mal pronóstico, antes de practicar la


Intervención quirúrgica, excepto:

a) Edad mayor a 70 años

b) Sexo masculino

c) Resecciones gástricas previas (carcinoma en muñón gástrico)

d) Tumores gástricos en cardias versus los antrales

e) Carcinomas gástricos con antígeno carcinoembrionario y Ca19 negativo

DG156. En el carcinoma de páncreas en la escala de Klöppel, se toman en cuenta los siguientes factores,
excepto:

a) La producción de moco

b) El pleomorfismo nuclear

c) La actividad mitótica

d) Las estructuras semejando conductos

e) Los cambios inflamatorios

DG157. El método de diagnóstico en cáncer esofágico que tiene mayor exactitud para determinar grado de
penetración en la pared y metástasis en ganglios linfáticos es:

a) Endoscopía con biopsia escisional

b) TAC de tórax

c) Ultrasonido endoscópico del esófago

d) Resonancia magnética de tórax

e) TAC helicoidal

DG158. El tipo de epitelio en esófago de Barrett con potencial maligno es:

a) Cardial
b) Pilórico

c) Fúndico

d) Columnar especializado

e) Todos tienen potencial maligno

DG159. El procedimiento con mayor exactitud diagnóstica en cáncer de esófago es:

a) Esofagograma

b) Ultrasonido endoscópico

c) Endoscopía con biopsia y cepillado

d) Endoscopía y biopsia escisional

e) Cromoendoscopía y biopsia

DG160. La mejor paliación en un paciente con cáncer esofágico y fístula traqueobronquial se logra con:

a) Gastrostomía endoscópica

b) Radioterapia

c) Quimioterapia

d) Colocación de endoprótesis

e) Fotocoagulación con láser

DG161. La sobrevida a cinco años en carcinoma hepatocelular de menos de 5 cm de diámetro es mayor con:

a) Lobectomía hepática

b) Segmentectomía hepática

c) Inyección del tumor con etanol

d) Criocirugía

e) Trasplante hepático

DG162. Los niveles más bajos de alfafetoproteína se encuentran en:

a) Carcinoma hepatocelular de 5 cm de diámetro


b) Carcinoma hepatocelular menor a 3 cm de diámetro

c) Carcinoma hepático fibrolamelar

d) Carcinoma hepatocelular con diferenciación grado IV

e) Carcinoma hepatocelular multicéntrico

DG163. El potencial maligno de un pólipo colorrectal es mayor en:

a) Adenoma tubular

b) Hamartoma

c) Hiperplásico

d) Adenoma plano

e) Adenoma velloso

DG164. El tratamiento de primera elección en el cáncer epidermoide del conducto anal es:

a) Cirugía radical

b) Escisión local

c) Quimiorradioterapia

d) Radioterapia

e) Braquiterapia

DG165. El porciento de diabetes mellitus que se produce en enfermos con adenocarcinoma pancreático es
de:

a) Menos de 5

b) 10–15

c) Más de 60

d) 30–40

e) Menos de 2

DG166. El tumor gástrico con peor pronóstico es:


a) Linfoma asociado a mucosa

b) Adenocarcinoma del tipo intestinal del antro

c) Adenocarcinoma difuso con células en anillo de sello del cardias

d) Carcinoide del antro

e) Linfoma no Hodgking

LINFOMAS Y LEUCEMIAS

LL1. Indicación para la realización de un trasplante alogénico en un paciente con leucemia mieloblástica
aguda:

a) Progresión de la enfermedad

b) Primera remisión

c) Primera recaída

d) Segunda remisión

e) Segunda recaída

LL2. Los síndromes siguientes se asocian con un aumento en la frecuencia de linfoma no Hodgkin, excepto:

a) Síndrome de Chediak-Higashi

b) Ataxia telangiectasia

c) Síndrome de Wiskott Aldrich

d) Anomalía de Pelger-Huet

e) Síndrome del paciente trasplantado

LL3. Gen de fusión de mal pronóstico en la leucemia aguda linfoblástica:

a) abl-bcr

b) PBX-E2A

c) E2A-PBL

d) TTG1

e) TCR

LL4. ¿Cuál de las leucemias siguientes representa más de la mitad (60%) de las leucemias de linaje B?
a) Pre-B

b) Pre-B temprana

c) Pre-B transicional

d) B

e) Células troncales

LL5. La inmunoglobulina citoplasmática y de superfice es indispensable para el diagnóstico de leucemia:

a) Aguda mieloblástica

b) Mielocítica crónica

c) Aguda linfoblástica T

d) Aguda linfoblástica Pre-B temprana

e) Eritroide

LL6. Para el diagnóstico de una leucemia de doble linaje se utilizan los criterios siguientes, excepto:

a) Rearreglo de cadenas ligeras Ig

b) Inmunoglobulina de superficie

c) Mieloperoxidasa

d) Bastoncillos de Auer

e) CD 10+

LL7. La actividad adenosin deaminasa se encuentra a niveles altos en la leucemia aguda:

a) Linfoblástica T

b) Linfoblástica B

c) Linfoblástica Pre-B

d) Promielocítica

e) Monoblástica

LL8. La deoxinucleotidil transferasa terminal se encuentra comúnmente elevada en la leucemia aguda


linfoblástica:

a) B
b) Pre-B

c) Pre-B temprana

d) Pre-B transicional

e) T

LL9. La elevación de la isoenzima I se encuentra comúnmente elevada en la leucemia aguda:

a) Linfoblástica T

b) Linfoblástica Pre-B

c) Linfoblástica Pre-B transicional

d) Monoblástica

e) Promielocítica

LL10. Un índice de ADN menor de 1.16 en la leucemia aguda linfoblástica significa:

a) Sin valor pronóstico

b) Buen pronóstico para el tratamiento

c) Mal pronóstico para el tratamiento

d) Pronóstico incierto

e) Sin correlación con pronóstico y tratamiento

LL11. El sudán negro en los blastos reacciona intracelularmente con:

a) Peroxidasa

b) Fosfolípidos

c) Catepsinas

d) Lisosimas

e) ARN mensajero

LL12. La tinción de estearasa no específica comúnmente está presente en la leucemia:

a) Leucemia aguda monoblástica


b) Leucemia aguda mieloblástica M1

c) Leucemia aguda linfoblástica T

d) Leucemia aguda linfoblástica Pre-B

e) Leucemia megacarioblástica

LL13. La reacción de PAS identifica en los blastos:

a) Aminoácidos

b) Grasas

c) Glucógeno

d) Núcleo proteínas

e) Purinas

LL14. Translocación más frecuentemente observada en la leucemia T:

a) 8;14

b) 2;8

c) 9;22

d) 11;14

e) 10;14

LL15. En la leucemia mielocítica crónica las alteraciones siguientes son correctas, excepto:

a) t(9;22)

b) Gen bcr/abl

c) Resistencia a la apoptosis

d) Enfermedad clonal

e) Hipogamaglobulinemia

LL16. Alteración cromosómica más frecuente en los síndromes mielodisplástico:

a) t(1;19)

b) Deleción del cromosoma 5 o 7

c) t(8;14)
d) Deleción cromosoma 1

e) t(2.8)

LL17. La leucemia aguda linfoblástica no B, no T se caracteriza por causar:

a) Cuenta de leucocitos baja y ausencia de masa mediastinal

b) Formación de rosetas con eritrocitos de carnero

c) Ausencia de masa mediastinal

d) Marcadores de superficie positivos con inmunoglobulinas

e) Presencia elevada de I2 (antígeno inmune asociado)

LL18. Presentación clínica más frecuente de leucemia meníngea:

a) Fiebre, anorexia y alteraciones visuales

b) Cefalea, vómito y letargia

c) Irritabilidad, convulsiones y coma

d) Ataxia y parálisis de los nervios craneales

e) Parálisis del nervio facial

LL19. Alteración que se puede asociar con enfermedad de Hodgkin:

a) Hiperuricemia

b) Hipercalcemia

c) Hipoalbuminemia

d) Hipofosfatemia

e) Todas las antes enunciadas

LL20. Alteración inmunológica en un paciente con enfermedad de Hodgkin de predominio linfocítico:

a) Alteración de la prueba de azul de tetrazolio

b) Alteración de la quimiotaxis

c) Ninguna
d) Alteración de la transformación blastoide con antígenos de fitohemoaglutinina

e) Ausencia de células plasmáticas en el gánglio linfático

LL21. Factor pronóstico adverso en pacientes con leucemia aguda mieloblástica:

a) Esplenomegalia

b) Variedad M1

c) Monosomía 7

d) t(8;21)

e) t(9;11)

LL22. Factor predisponente para el desarrollo de leucemia aguda mieloblástica:

a) Exposición a benzeno

b) Radiación ionizante

c) Exposición a luz ultravioleta

d) Exposición a cloranfenicol

e) Radiación gama

LL23. De la clasificación de FAB la única correlación que existe entre inmunofenotipo y citomorfología es:

a) L1

b) M1

c) L3

d) M3

e) M4

LL24. ¿Cuál de los siguientes hallazgos no corresponde a leucemia mielocítica crónica?

a) Fase crónica

b) Elevación de la deshidrogenasa láctica

c) Disminución de la fosfatasa alcalina de los leucocitos

d) Fase de metamorfosis

e) Fase blástica
LL25. Son complicaciones frecuentes de la leucemia mielocítica crónica, excepto:

a) Alteraciones metabólicas

b) Hiperleucocitosis

c) Trombocitosis

d) Hemorragias retinianas

e) Priapismo

LL26. Las siguientes son diferencias entre la leucemia mielocítica crónica adulta y juvenil, excepto la
presencia de:

a) Cromosoma Filadelfia

b) Manifestaciones hemorrágicas

c) Adenopatías

d) Hemoglobina fetal

e) Oncogén bcr-abl

LL27. Son síndromes mieloproliferativos los siguientes, excepto la:

a) Leucemia mielomonocítica crónica

b) Policitemia vera

c) Trombocitema esencial

d) Metaplasia mieloide

e) Mielofibrosis aguda

LL28. Los siguientes enunciados respecto a la enfermedad de Hodgkin son ciertos, excepto:

a) Tiene una distribución por edades bimodal

b) Se asocia a un HLA especifico

c) Se asocia a infección por virus herpes 6

d) Se asocia a inmunodeficiencias primarias como la ataxiatelangiectásia

e) El pronóstico en adultos es mejor que en niños


LL29. Las etapas I y IIa/b de Hodgkin son considerados como desfavorables cuando:

a) Existe extensión directa a otros órganos de la enfermedad

b) Hay carga mediastinal mayor de 1/3 del diámetro máximo intratoracico

c) Nunca se considera desfavorable

d) Existe una linfoadenopatía periférica mayor de 6 cm

e) b y d son correctas

LL30. Causa de anemia en los casos avanzados de enfermedad de Hodgkin:

a) Aporte inadecuado de hierro

b) Sangrado crónico microscópico

c) Incapacidad para movilizar los depósitos de hierro hepático

d) Hemólisis asociado a reticulocitosis e hiperplasia normoblástica

e) c y d son correctas

LL31. Los niveles altos de ferritina y la eritrosedimentación indican en la enfermedad de Hodgkin:

a) Activación del sistema reticuloendotelial

b) Afección hepática

c) Gran carga tumoral

d) Presencia de masa mediastinal

e) Ninguna de las anteriores

LL32. En presencia de adenopatia mediastinal:

a) Se debe radiar solo el hilio pulmonar afectado

b) Se debe radiar ambos hilios pulmonares

c) No debe recibir radioterapia a los hilios pulmonares

d) Se dará radioterapia solo si la adenopatia mide más de 6 cm

e) Ninguna de las anteriores

LL33. Existe mal pronóstico en los pacientes con enfermedad de Hodgkin que presentan las siguientes
características:

a) Aquellos que nunca presentan remisión de la enfermedad


b) Aquellos que presentan remisión por menos de 12 meses

c) Aquellos que presentan múltiples recaídas

d) Todas

e) Ninguna

LL34. Sus niveles altos en suero se han asociado a mal pronóstico en la enfermedad de Hodgkin:

a) FNC-B

b) Células de Reed-Stergnberg

c) IL-1

d) IL-2

e) DHL

LL35. Las siguientes son aseveraciones con respecto a la enfermedad de Hodgkin, excepto que:

a) La presencia de síntomas B son un factor de mal pronóstico

b) La presencia de síntomas B se encuentran en relación con altas concentraciones de IL 2

c) La diarrea es causa de la liberación del peptido vasoactivo intestinal

d) El transplante autólogo de médula ósea esta indicado en la enfermedad refractaria

e) El hipotiroidismo se presenta como efecto secundario después de la radioterapia a cuello

LL36. ¿Cuál de los siguientes conceptos relacionados con el tratamiento de la micosis fungoides es
correcto?

a) La aplicación de quimioterapia con mostaza nitrogenada produce regresión tumoral

b) La mielosupresión es una toxicidad importante de la mostaza nitrogenada tópica

c) El ácido Cis-retinóico es inactivo

d) La eritrodermia difusa esta asociada con células circulantes que tiene un diferente perfil genético e
inmunológico que aquellos en las lesiones en piel

e) El síndrome de Sezary responde a la fotoféresis extracorpórea

LL37. El pronóstico de la leucemia aguda mieloblastica depende de muchos factores sin embargo, el factor
determinante es:

a) La edad

b) El tipo citológico (FAB)


c) El cariotipo

d) El régimen de quimioterapia

e) El transplante de médula ósea en la primera remisión

LL38. El valor óptimo de la terapia de mantenimiento para la leucemia aguda no linfoblástica ha demostrado
ser de:

a) 6 meses

b) 12 meses

c) 24 meses

d) 36 meses

e) 40 meses

LL39. En la fase crónica de la leucemia mielocítica crónica el tratamiento convencional óptimo es en base a:

a) Busulfan

b) Alfa interferón

c) Hidroxiurea

d) Ciclofosfamida

e) Adriblastina

LL40. Tratamiento ideal para los síndromes mielodisplásicos:

a) Ciclofosfamida

b) CCNU

c) Trasplante alogeneico de médula ósea

d) Radiación corporal total

e) Ácido retinoico

LL41. Secuela más grave del tratamiento con quimioterapia en la enfermedad de Hodgkin:

a) Talla baja

b) Segundas neoplasias
c) Hipotiroidismo

d) Todas las anteriores

e) Ninguna de las anteriores

LL42. Se utiliza radioterapia de urgencia en la enfermedad de Hodgkin cuando:

a) Existe linfadenopatia mayor a 10 cm

b) Existe compromiso de la vía aérea alta y compresión espinal

c) Existe linfoadenopatia mayor de 6 cm

d) Existe afección pulmonar

e) c y d deben estar presentes

LL43. ¿El objetivo de combinar los esquemas de quimioterapia AVBD-MOPP es?

a) Disminuir el riesgo de efectos secundarios por el uso exclusivo del MOPP

b) Mejorar la sobrevida en etapas avanzadas

c) Disminuir la quimiorresistencia de la enfermedad

d) Evitar el uso de radioterapia en etapas tempranas

e) Disminuir el tiempo del tratamiento

LL44. La laparotomía exploradora en la enfermedad de Hodgkin debe incluir:

a) Esplenectomía y toma de biopsia hepática

b) Esplenectomía, biopsia hepática y exploración de ganglios retroperitoneales

c) Esplenectomía, biopsia hepática y exploración de ganglios pélvicos

d) Esplenectomía y exploración de ganglios pélvicos y retroperitoneales

e) Ninguna de las intervenciones antes mencionadas

LL45. Los siguientes son esquemas utilizados frecuentemente en el tratamiento de la enfermedad de


Hodgkin, excepto:

a) ABVD
b) MOPP

c) OPA

d) VAMP

e) COMP

LL46. En relación con la función gonadal de jóvenes masculinos con enfermedad de Hodking:

a) Antes del tratamiento aproximadamente una tercera parte cursan con oligoespermia

b) Los esquemas de MOPP y AVBD están asociados con posibilidad comparables de disfusión
gonadal postratamiento

c) Las dosis altas de radiación son necesarias para lesionar la espermatogénesis

d) Esta vuelve a la normalidad en la mayoría de los hombres tratados con MOPP

e) Nada de lo anterior es verdadero

LL47. Tipo de leucemia que se observa más comúnmente en pacientes que sobrevivieron a un cáncer:

a) Linfoblástica aguda

b) Mieloblástica aguda

c) Linfocítica crónica

d) Mielocítica crónica

e) De células peludas

LL48. ¿La indicación para la realización de un trasplante alogénico en un paciente con leucemia linfoblástica
aguda se considera cuando éste se encuentra en?

a) Enfermedad progresiva

b) Primera remisión

c) Primera recaída

d) Segunda remisión

e) Fase de consolidación

LL49. Linfoma de peor pronóstico en niños:

a) Indiferenciado no Burkitt

b) Burkitt, pobremente indiferenciado


c) Linfoma linfoblástico

d) Mixto

e) Linfoma anaplásico K1+

LL50. La presencia de un cloroma es sugestivo de leucemia:

a) Aguda linfoblástica T

b) Aguda linfoblástica B

c) Mielocítica crónica

d) Eritroide

e) Aguda monoblástica

LL51. El pronóstico en niños con síndrome de down es mejor cuando padecen leucemia aguda:

a) Linfoblástica B

b) Linfoblástica T

c) Mieloblástica (M1)

d) Megacarioblástica

e) Promielocítica

LL52. ¿Cuál de los anticuerpos monoclonales siguientes sirve para identificar una leucemia aguda
linfoblástica T?

a) D 19

b) CD 21

c) CD 13

d) CD 3

e) CD 15

LL53. ¿Cuál de los anticuerpos monoclonales siguientes sirve para identificar una leucemia aguda
linfoblástica B?

a) CD 10

b) CD 33

c) CD 19
d) CD 7

e) CD 34

LL54. Translocación que se observa en la leucemia aguda linfoblástica B (L3):

a) t(1;19)

b) t(9;22)

c) t(11;14)

d) t(8;14)

e) t(9;11)

LL55. El CD10 es un factor pronóstico de la leucemia:

a) Pre-B

b) De células troncales

c) T temprana

d) Pre-B transicional

e) T madura

LL56. La reacción de fosfatasa ácida de linfoblastos se observa en la leucemia aguda:

a) Linfoblástica B

b) Mieloblástica M1

c) Linfoblástica M2

d) Mielo-monoblástica

e) Linfoblástica T

LL57. Defecto genético básico en el linfoma de Burkitt:

a) Mutación puntual

b) Deleción del cromosoma 8

c) Translocación 8;14

d) Ninguna de las anteriores


e) Todas las anteriores

LL58. En cuanto al origen de las células de Reed-Sternberg, la teoría más aceptada indica que:

a) Se trata de linfocitos T o B activados

b) Son linfocitos infectados por el virus de Ebstein Barr

c) Se trata de células ajenas a la serie linfoide

d) Se trata de linfocitos con mutaciones de novo

e) Ninguna de las opciones anteriores

LL59. Las células de la enfermedad de Hodgkin (Reed-Sternberg) pueden ser positivas para el antígeno:

a) CD 10

b) CD 34

c) ki-1 CD30

d) CD19

e) CD5

LL60. ¿Cuál de las siguientes interleucinas es productor de fiebre en la enfermedad de Hodgkin?

a) IL-1

b) IL-5

c) TNF alfa

d) TNF beta

e) Interferón

LL61. Se asocian frecuentemente con enfermedad de Hodgkin, excepto:

a) Virus del Epstein-Barr

b) Herpes tipo 6

c) Citomegalovirus

d) Virus de la inmunodeficiencia adquirida


e) Hepatitis C

LL62. ¿Cuál de los siguientes estudios radiológicos es inicialmente de más utilidad para evaluar la
enfermedad de Hodgkin infradiafragmática?

a) Gamagrafía con galio

b) Serie gastroduodenal

c) Colon por enema

d) Linfografía pedia bilateral con tomografía computada

e) Angiografía selectiva

LL63. Estudio de imagen con mayor sensibilidad para diagnosticar enfermedad de Hodgkin
intradiafragmática:

a) Gamagrama con galio 67

b) Resonancia magnética abdominal

c) Linfoangiografía

d) Linfoangiografía más tomografía computada

e) Gamagrafía abdominal con galio 67 más linfoangiografía

LL64. Paciente VIH+ con tomografía que muestra múltiples lesiones cerebrales, los títulos de toxoplasmosis
fueron negativos; lo más probable es que la biopsia por estereotaxia revele:

a) Linfoma maligno de linfocitos B de células pequeñas hendidas

b) Linfoma inmunoblástico de células grandes

c) Adenocarcinoma metastásico

d) Toxoplasmosis

e) Linfoma maligno de linfocitos T de células grandes

LL65. Signo-síntoma más común en pacientes con micosis fungoides:

a) Linfadenopatía

b) Fiebre

c) Prurito
d) Pérdida de peso

e) Sudor nocturno

LL66. Síntoma o signo más común en la leucemia aguda linfoblástica:

a) Dolor abdominal y fatigabilidad

b) Fiebre, palidez, astenia y lesiones purpúricas

c) Hematemesis y melena

d) Dolor articular y óseo, con pérdida de peso

e) Pérdida de peso

LL67. Causa más común de insuficiencia renal en niños con leucemia aguda linfoblástica:

a) Infiltración leucémica al riñón

b) Deshidratación severa secundaria al vómito excesivo

c) Hiperuricemia con nefropatía por ácido úrico

d) Pielonefritis

e) Acidosis láctica

LL68. El diagnóstico definitivo de leucemia se establece por medio de:

a) Estudio de líquido cefalorraquídeo

b) Análisis del frotis de sangre periférica

c) Biopsia de gánglio linfático

d) Estudio de médula ósea por aspiración

e) Marcadores inmunológicos

LL69. Causa más importante de mortalidad en niños con leucemia aguda:

a) Insuficiencia cardíaca congestiva secundaria a infiltración del miocardio

b) Infiltración al sistema nervioso central

c) Hemorragia secundaria a trombocitopenia


d) Infección

e) Insuficiencia renal aguda secundaria a hiperuricemia

LL70. Complicación pulmonar más frecuente en niños con leucemia aguda:

a) Infiltrados leucémicos

b) Hemorragia

c) Infección

d) Fibrosis

e) Derrame pleural

LL71. La mayor incidencia de leucemia aguda linfoblástica ocurre:

a) Entre 1 y 2 años de edad

b) Entre 3 y 5 años de edad

c) Entre 7 y 10 años de edad

d) Entre 10 y 15 años de edad

e) Antes del año de edad

LL72. Complicación más frecuente de la leucemia aguda no linfoblástica:

a) Infiltración meníngea

b) Hiperuricemia

c) Hemorragia intracerebral

d) Trombocitopenia

e) Visceromegalia

LL73. La enfermedad de Hodgkin que frecuentemente se asocia a masa en diastino es:

a) Predominio linfocítico

b) Esclerosis nodular

c) Celularidad mixta

d) Depleción linfocítica
e) Ninguno de los niveles antes mencionados

LL74. El síndrome de coagulación intravascular diseminada se asocia más frecuentemente con leucemia:

a) Aguda megacarioblástica

b) Aguda premielocítica

c) Aguda mieloblástica (M2)

d) Eritroide

e) Mielocítica crónica

LL75. Se considera leucemia linfoblástica aguda cuando la presencia de linfoblastos en médula ósea es
mayor del:

a) Más del 50%

b) Más del 40%

c) Más del 25%

d) Más del 80%

e) Más del 10%

LL76. La única manera de destruir el cromosoma Filadelfia en la leucemia mielocítica crónica es con:

a) Trasplante de médula ósea alogénico

b) Infusión de células tallo autólogas

c) Esplenectomía más alfa interferón

d) Citosina de arabinósido a dosis altas

e) Mitoxantrona

LL77. En la población pediátrica la policitemia secundaria es más frecuente que la primaria, debido a las
siguientes causas, excepto:

a) Niveles de eritropoyetina aumentados

b) Niveles de eritropoyetina normales

c) Cardiopatía con corto circuito de derecha a izquierda

d) Anormalidades congénitas renales

e) Enfermedad pulmonar crónica


LL78. En la fisiopatología de los síndromes mieloproliferativos los efectos más frecuentes son debidos a:

a) Trombosis

b) Estimulación de fibroblastos

c) Hipervolemia

d) Aumento de proliferación y hematopoyesis

e) Hipermetabolismo

LL79. Son síndromes mielodisplásicos los siguientes, excepto la:

a) Anemia refractaria con sideroblastos

b) Leucemia mielomonocítica crónica

c) Trombocitosis

d) Anemia refractaria con exceso de blastos

e) Anemia refractaria con exceso de blastos en transformación

LL80. Variedad histológica más frecuente de enfermedad de Hodgkin en pacientes menores de l0 años:

a) Rico en linfocitos

b) Linfoblástica

c) Predominio de linfocitos

d) Anaplásico

e) Deplesión linfocítica

LL81. Principal órgano afectado por las neoplasias (secundarias al tratamiento) de la enfermedad de
Hodgkin:

a) Hígado

b) Pulmón

c) Hueso

d) Tiroides

e) Médula ósea
LL82. Sitio extraganglionar más frecuentemente afectado por la enfermedad de Hodgkin:

a) Tiroides

b) Bazo

c) Hígado

d) Pulmón

e) Corazón

LL83. Entre las complicaciones cardiovasculares más frecuentes, posteriores al tratamiento con radioterapia
se incluye:

a) Derrame pericardico asintomático

b) Pericarditis constrictiva

c) Cardiopatía valvular

d) Arteriopatía coronaria

e) Todas las anteriores

LL84. ¿Cuál de los siguientes conceptos es falso relacionado con el riesgo de cáncer de mama en mujeres
que tuvieron una enfermedad de Hodking en edad pediátrica?

a) El riesgo de cáncer mamario esta aumentado comparado con la población general

b) El más importante factor de riesgo para cáncer mamario es haber utilizado radioterapia de
mantle

c) El riesgo para cáncer mamario es mayor si se trató de la enfermedad de Hodking antes de los
15 años que si lo hizo en edad posterior

d) La quimioterapia combinada con MOPP o AVBD no esta asociada con incremento de cáncer
mamario en niñas tratados por enfermedad de Hodking

e) Más de un 100% de aumento en el riesgo ha sido reportado en jóvenes que recibieron


radioterapia a la pared torácica

LL85. El Tx con índices de duración más altos y menos morbilidad tardía incluyendo segundas neoplasias
para la enfermedad de Hodkin en etapa IIIb en niños es:

a) RT ganglionar total a dosis de 35 Gy

b) QT ciclos de MOOP más RT GT a dosis de 35 Gy

c) QT 4-6 ciclos de ABVD más RTGT a dosis de 20-25 Gy


d) QT 6 ciclos de MOOP más RT campos involucrados a dosis de 35 Gy

e) QT solamente alternando ciclos de MOOP con ABVD

LL86. En leucemia aguda linfoblástica en general, los pacientes con terapia previa responden menos a
tratamientos subsecuentes debido a los factores siguientes, excepto a que causan:

a) Menor reserva medular

b) Pobre función orgánica

c) Frecuencia elevada de infecciones

d) Resistencia de las células leucémicas

e) Reacciones alérgicas frecuentes

LL87. La combinación más efectiva para inducir remisión en la leucemia aguda linfoblástica de riesgo
habitual es:

a) Ciclofosfamida y prednisona

b) Prednisona y 6 mercaptopurina

c) Prednisona y L asparaginasa

d) Vincristina, prednisona y L asparaginasa

e) Vincristina y L asparaginasa

LL88. La radioterapia en manto aplicada a los pacientes con enfermedad de Hodgkin debe incluir los
siguientes campos, excepto:

a) Mediastino

b) Supraclavicular

c) Áreas ilíacas

d) Infraclavicular

e) Axilar

LL89. Un paciente masculino de 9 años de edad que presenta un padecimento de nueve meses de evolución,
fiebre de predominio nocturno de hasta 39 °C, acompañado de afección de tres regiones linfáticas
supradiafragmáticas, de acuerdo a la clasificación de Ann Arbor ¿Cómo queda estadificado?
a) Etapa Ia

b) Etapa IE

c) Etapa IIIb

d) Etapa IIb

e) Etapa IIa

LL90. ¿Las dosis del esquema de MOPP son?

a) Mostaza nitrogenada 6 mg/m2, vincristina 1.5 mg/m2, procarbacina 100 mg/m2/día/14 días,
prednisona 40mg/m2/día/14 días

b) Mostaza nitrogenada 5 mg/m2, vincristina 1.5 mg/m2, procarbacina 100 mg/m2, prednisona 40
mg/m2

c) Mostaza nitrogenada 6mg/m2, vincristina 1.5 mg/m2, procarbacina l00 mg/m2/día/14 días,
prednisona 60 mg/m2/día/14 días

d) Mostaza nitrogenada 5 mg/m2, vincristina 1.5 mg/m2, procarbacina l00 mg/m2, prednisona 60
mg/m2

e) Mostaza nitrogenada 5 mg/m2, vincristina 1.5 mg/m2, procarbacina 150 mg/m2, prednisona 60
mg/m2

LL91. El esquema de AVBD se integra de la siguiente manera:

a) Adriamicina 20 mg/m2, Bleomicina 15 mg/m2, Vinblastina 5 mg/m2, Dacarbacina 275 mg/m2

b) Adriamicina 25 mg/m2, Bleomicina 15 mg/m2, Vinblastina 5 mg/m2, Dacarbacina 375 mg/m2

c) Adriamicina 25 mg/m2, Bleomicina l0 mg/m2, Vinblastina 5 mg/m2, Dacarbacina 375 mg/m2

d) Adriamicina 25 mg/m2, Bleomicina 10 mg/m2, Vinblastina 6 mg/m2, Dacarbacina 375 mg/m2

e) Adriamicina 25 mg/m2, Bleomicina 10 mg/m2, Vinblastina 6 mg/m2, Dacarbacina 275 mg/m2

LL92. Los siguientes conceptos relacionados con los esquemas MOPP y AVBD en el tratamiento de
enfermedades de Hodking son correctos excepto que:

a) Los pacientes que recaen después de quimioterapia con Mopp o AVBD tiene mucho menos
posibilidades de respuesta si se vuelven a tratar con la combinación original a que si lo hacen con
la combinación alternativa

b) El AVBD es superior al MOPP en que tiene menos infertilidad y segundas neoplasias

c) En estudios comparativos, fue más frecuente la disminución de la dosis en MOPP que en AVBD
d) Los regímenes híbridos derivados de MOPP y AVBD así como alternar MOPP-AVBD han
demostrado ser superiores a cualquiera de estos por separado en relación a tiempo libre de
progresión y sobrevida global

e) En estudios comparativos el AVBD es equivalente al MOPP en lo relacionado a sobrevida libre


de progresión

LL93. ¿Cuántas variedades histológicas son las más frecuentes en la enfermedad de Hodgkin?

a) 2

b) 4

c) 5

d) 6

e) 8

LL94. Sitio anatómico más frecuente de inicio de la enfermedad de Hodgkin

a) Inguinal

b) Axilar

c) Esplénico

d) Cervical

e) Hepático

LL95. ¿Qué virus se ha relacionado con la enfermedad de Hodgkin?

a) Citomegalovirus

b) Epstein Barr

c) De la rubéola

d) Varicela zoster

e) Del papiloma humano

LL96. Un paciente con afección a ambos lados del diafragma se estadificaría, según la clasificación de Ann
Arbor como:

a) II
b) I

c) III

d) IV

e) V

LL97. El tratamiento para estadios avanzados consiste en:

a) Radioterapia

b) Monoterapia

c) Poliquimioterapia

d) Poliquimioterapia más radioterapia

LL98. En leucemia linfocítica crónica la alteración cromosómica clonal más frecuente es:

a) t(9;22)

b) inv(16)

c) +12

d) del(7q)

e) i(17q)

LL99. ¿Cuál es el grupo de edad que con mayor frecuencia se ve afectado por la leucemia linfocítica
crónica?

a) 35 – 40

b) 41 – 45

c) 46 – 50

d) 51 – 55

e) 56 – 60

LL100. ¿Cuál de las siguientes es una característica del síndrome de Richter?

a) Pérdida de peso

b) Anemia hemolítica
c) Púrpura trombocitopénica

d) Colestasis

e) Diátesis hemorrágica

LL101. Según la etapificación de Binet para la leucemia linfocítica crónica, ¿qué etapa le corresponde a un
paciente con ganglios aumentados de tamaño a ambos lados del cuello y en la región supraclavicular
derecha, hemoglobina de 9 g/dL y plaquetas de 95,000?

a) A

b) B

c) C

d) D

e) E

LL102. ¿Cuál es la edad media de presentación de la leucemia granulocítica crónica?

a) 35 años

b) 45 años

c) 55 años

d) 65 años

e) 75 años

LL103. ¿Cuál de los siguientes es un síntoma de presentación frecuente en leucemia granulocítica crónica?

a) Priapismo

b) Síndrome hemorrágico

c) Leucopenia

d) Afección de pares craneales

e) Saciedad temprana

LL104. ¿Cuál de las siguientes es la alteración citogenética de la leucemia granulocítica crónica?

a) t(9;22)
b) inv(16)

c) del(7)

d) i(6)

e) +8

LL105. ¿Cuál de las siguientes es una característica de la leucemia granulocítica crónica en fase crónica?

a) Blastos mieloperoxidasa positivos

b) Cuenta plaquetaria entre 50 y 100,000

c) Fosfatasa alcalina de los leucocitos baja

d) Elevación de la vitamina B12

e) Alteración en las pruebas de coagulación

LL106. ¿Cuál de los siguientes es el medicamento de elección para la citorreducción en pacientes con
leucemia granulocítica crónica en fase crónica?

a) 6 mercaptopurina

b) 6 tioguanina

c) clorambucil

d) Hidroxiurea

e) Busulfán

LL107. ¿Cuál de los siguientes es el medicamento de elección para el tratamiento actual de la leucemia
granulocítica crónica?

a) interferón alfa

b) busulfán

c) imatinib

d) azatioprina

e) clorambucil

LL108. ¿Cuál de las siguientes es una alteración citogenética de alto riesgo en leucemia mielcítica crónica?
a) t(8;21)

b) inv(16)

c) del(7)

d) t(15;17)

e) todas las anteriores

LL109.b¿En cuál de las siguientes variantes de leucemia mielocítica crónica es frecuente encontrar
evidencia de coagulación intravascular diseminada?

a) M0

b) M1

c) M2

d) M3

e) M4

LL110. ¿En cuál de las siguientes variantes de leucemia mielocítica crónica es característico encontrar
infiltración leucémica de las encías?

a) M1

b) M5

c) M7

d) M3

e) M0

LL111. ¿Qué porciento de blastos, en la médula ósea, se requiere para el diagnóstico de leucemia mielocítica
aguda?

a) 10

b) 20

c) 30

d) 40

e) 50
LL112. ¿Cuál de los siguientes es un factor pronóstico adverso en leucemia mielocítica aguda?

a) Presencia de inv(16)

b) Cuenta de leucocitos menor a 20 X 10 9 /L

c) Presencia de t(8;21)

d) Edad mayor de 60años

e) Presencia de alteraciones de la coagulación

LL113. ¿Cuál de los siguientes es un agente útil en el tratamiento de inducción en pacientes de nuevo
diagnóstico con leucemia mielocítica aguda?

a) Hidroxiurea

b) Cisplatino

c) Vincristina

d) Lomustina

e) Citarabina

LL114. ¿Cuál de los siguientes agentes debe utilizarse en el tratamiento de la leucemia aguda
promielocítica?

a) 6 tioguanina

b) ácido trans retinoico

c) 6 mercaptopurina

d) Busulfán

e) Lomustina

LL115. ¿Cuál es el grupo de edad en el que con mayor frecuencia se observa leucemia linfocítica aguda?

a) 1 a 5 años

b) 6 a 10 años

c) 11 a 15 años

d) 16 a 20 años
e) 21 a 25 años

LL116. ¿Cuál de las siguientes es la manifestación más frecuente de leucemia linfocítica aguda?

a) Infección

b) Esplenomegalia

c) Hemorragia

d) Crisis convulsivas

e) Fatiga

LL117. ¿Cuál es el inmunofenotipo que con más frecuencia se observa en pacientes con leucemia aguda
linfoblástica?

a) T

b) T temprano

c) Pre B

d) Pre B temprano

e) B

LL118. ¿Cuál de los siguientes es un factor pronóstico desfavorable en leucemia linfocítica aguda?

a) T(9;22)

b) Cuenta de leucocitos mayor a 5 X 109/L

c) Sexo femenino

d) Edad menor a 10 años

e) Subtipo L1 de la FAB

LL119. ¿Cuál de los siguientes es el factor pronóstico más importante en leucemia linfocítica aguda?

a) Sexo

b) Cuenta de leucocitos

c) Tratamiento
d) Edad

e) Masa mediastinal

LL120. ¿Cuál de los siguientes agentes ha mostrado, en estudios comparativos, aumentar la posibilidad de
remisión y sobrevida libre de enfermedad cuando se agrega a la asociación de vincristina y prednisona en el
tratamiento de leucemia linfocítica aguda?

a) Etopósido

b) Ciclofosfamida

c) Citarabina

d) Tenipósido

e) Doxorrubicina

LL121. ¿Cuáles son los sitios en los que con mayor frecuencia se observa recaída en leucemia linfocítica
aguda?

a) Ganglios linfáticos, hígado y piel

b) Sistema nervioso, hígado y ganglios linfáticos

c) Médula ósea, sistema nervioso y testículos

d) Testículos, hígado y sistema nervioso

e) Médula ósea, testículos y ganglios linfáticos

LL122. ¿Cuál de los siguientes agentes se ha asociado a leucemia secundaria con alguna alteración
citogenética en 11q23?

a) Doxorrubicina

b) Ciclofosfamida

c) L-asparaginasa

d) Etopósido

e) Vincristrina

LL123. Características de la biometría hemática. para diagnóstico de leucemia aguda

a) Hb, Normal, leucocitosis, trombocitopenia


b) Hb, baja leucopenia, neutrofilia

c) Hb, baja tromboticopenia, blastos

d) Hb, baja diferencial normal, trombocitopenia

e) Blastos, Leucocitosis, Trombocitosis

LL124. La leucemia más frecuente en la infancia es:

a) Linfócitica crónica

b) Mielomonoblástica

c) Aguda de células peludas

d) Aguda linfoblástica

e) Granulocítica crónica

LL125. Los bastos de Aüer son característicos de los:

a) Mieloblastos

b) Linfoblastos

c) Células en espejo de mano

d) Monoblastos

e) Eritroblastos

LL126. Sitios de mayor recaída en la leucemia agua linfoblástica

a) Piel y mucosas

b) Hígado y piel

c) Sistema Nervioso Central y testículos

d) Testículos y ovarios

e) Sistema Nervioso Central y ovarios

LL127. La leucemia promielocítica aguda responde al Ac-Transretinoico por contar con la alteración
genética siguiente:
a) +(1:19)

b) +(9:22)

c) +(1:14)

d) del 5q

e) +(15:17)

LL128. En que tipo de leucemia aparecen los cloromas:

a) Leucemia aguda linfoblástica

b) Leucemia aguda mieloblástica

c) Leucemia aguda monoblástica

d) Leucemia linfocítica crónica

e) Eritroleucemia

TUMORES UROLÓGICOS

UR1. Marcador tumoral útil en el seguimiento de tumores seminomatosos:

a) CA-125

b) CA 15-3

c) Antígeno carcinoembrionario

d) Deshidrogenasa láctica isoenzima-1

e) Antígeno prostático específico

UR2. El carcinoma de células pequeñas de vejiga tiene una frecuencia de presentación entre todos los tipos
histológicos de cáncer vesical de:

a) Menos del 2%

b) 5%

c) 10%

d) 15%

e) 17%

UR3. En la etiología del cáncer de pene se encuentran los factores siguientes, excepto:
a) El esmegma

b) Higiene deficiente

c) Fimosis

d) Circunsición neonatal

e) Circunsición en la adolescencia

UR4. Lesión premaligna que NO se asocia con el cáncer de pene:

a) Leucoplaquia

b) Eritroplasia de Queyrat

c) Enfermedad de Bowen

d) Queratocantoma

e) Tumor de Buschke-Lowenstein

UR5. Lesión premaligna en pene que involucra con frecuencia el meato-uretra:

a) Leucopaquia

b) Enfermedad de Bowen

c) Tumor de Buschke-Lowenstein

d) Balanitis xerótica obliterans

e) Eritroplasia de Queyart

UR6. El manejo de los ganglios linfáticos regionales en cáncer de pene se justifica en los casos siguientes,
excepto cuando el tumor:

a) Es clínicamente positivo

b) Invade la uretra

c) Es mayor de 5 cm

d) Es menor de 2 cm

e) Invade el escroto

UR7. La relación anatómica posterior de riñón y glándula suprarrenal es con:

a) Músculo psoas
b) Músculo cuadrado lumbar

c) Músculo transverso del abdomen

d) Diafragma

e) Todos los músculos mencionados

UR8. En carcinoma renal, el síndrome de Stauffer se caracteriza por:

a) Anomalías del funcionamiento hepático

b) Resolverse usualmente mediante nefrectomía

c) Presentar hipergamaglobulinemia

d) a y c

e) a, b y c

UR9. El cáncer de próstata se origina en un 60-70% de los casos en la zona:

a) De transición

b) Central

c) Central y periférica

d) Periférica

e) De transición central

UR10. El riego sanguíneo directo de la vejiga proviene de la:

a) Aorta

b) Ilíaca externa

c) Ilíaca interna

d) Pudenda interna

e) Pudenda externa

UR11. El carcinoma vesical in situ muestra frecuentemente:

a) Regresión espontánea

b) Metaplasia
c) Evolución temprana hacia una forma invasiva

d) Regresión espontánea y displasia

e) Regresión espontánea y metaplasia

UR12. Vías de diseminación del cáncer renal:

a) Contiguidad

b) Linfática

c) Hematógena

d) b y c

e) Todas

UR13. Principal causa precursora del cáncer de próstata:

a) La hipertrofia prostática

b) La hiperplasia prostática

c) La prostatitis crónica

d) La neoplasia intraepitelial prostática

e) La prostatitis aguda

UR14. La prostatectomía radical ofrece tasas de curación del:

a) 10 a 20%

b) 20 a 30%

c) 30 a 40%

d) 40 a 50%

e) 60 a 70%

UR15. ¿En el sexo masculino y en Estados Unidos qué lugar ocupa, en mortalidad, el cáncer de próstata?

a) 1°

b) 2º
c) 3º

d) 4º

e) 5º

UR16. El porciento de hombres de 80 años de edad con cáncer de próstata en Estados Unidos, como
hallazgo en autopsias, es de:

a) 20

b) 10

c) 25

d) 30

e) 15

UR17. Complicación a largo plazo más frecuente por RT en el cáncer de próstata:

a) Diarrea

b) Incontinencia urinaria

c) Impotencia

d) Estenosis uretral

e) Cistitis

UR18. El porciento de recurrencia pélvica después de prostatectomía radical es de:

a) 5

b) 7

c) 10

d) 15

e) 20

UR19. El adenocarcinoma de próstata es la neoplasia maligna más frecuente en ese órgano ¿Qué porciento
ocupa?

a) 75

b) 80

c) 85
d) 90

e) 95

UR20. En el adulto, la proliferación y muerte celular de la próstata están reguladas por:

a) Andrógenos

b) Prostaglandinas

c) Factor de crecimiento

d) Linfocitos

e) Factor de gangliogénesis

UR21. ¿Según Mc Neal en cuántas zonas se divide la próstata?

a) 1

b) 2

c) 3

d) 4

e) 5

UR22. La frecuencia y mortalidad en el cáncer de próstata, en los últimos años:

a) Ha disminuido

b) Permanece igual

c) Ha aumentado

d) No ha sido bien estudiada

UR23. Las metástasis óseas en el cáncer de próstata se presentan con mayor frecuencia en:

a) Pelvis

b) Columna vertebral lumbar

c) Húmero

d) Fémur

e) Tibia
UR24. Factor de mayor riesgo para padecer cáncer de próstata:

a) Edad

b) Raza

c) Actividad sexual

d) Lugar de nacimiento

e) Alimentación rica en grasa animal

UR25. La tasa de mortalidad del cáncer de próstata en los últimos años ha:

a) Permanecido sin cambios sustanciales

b) Disminuido ligeramente

c) Aumentado

d) Sido polémica

e) Disminuido drásticamente

UR26. La mortalidad por prostatectomía radical se encuentra cercana al:

a) 1%

b) 2%

c) 2.5%

d) 3%

e) 3.5%

UR27. Son tumores de células germinales del testículo los siguientes, excepto el:

a) Seminoma

b) Carcinoma embrionario

c) Sarcoma

d) Teratoma

e) Coriocarcinoma
UR28. Porciento en que puede presentarse el cáncer de testículo bilateral:

a) 15

b) 10

c) 30

d) 20

e) 5

UR29. Principales factores asociados a la etiología del cáncer de vejiga:

a) Tabaquismo, hidrocarburos policíclicos aromáticos, trabajo con aluminio, limpiadores de


chimenea

b) Promiscuidad sexual

c) Relaciones sexuales tempranas

d) Distenciones frecuentes por no evacuar la vejiga con la frecuencia necesaria

e) Café, alcoholismo

UR30. Estirpe histopatológica presente en el cáncer de vejiga en el 90 a 95% de los casos:

a) Carcinoma epidermoide

b) Adenocarcinoma mucoproductor

c) Adenocarcinoma de células claras

d) Carcinoma de células pequeñas

e) Carcinoma de células transicionales

UR31. El porciento de cáncer de vejiga dentro de todos los cánceres uroteliales es de:

a) 10

b) 40

c) 50

d) 60

e) 90
UR32. Porciento de correlación entre cáncer prostático y presencia de tacto rectal con antígeno prostático
específico anormales:

a) 20

b) 40

c) 60

d) 80

e) 100

UR33. En pacientes con inmunosupresión por trasplantes, el uso de este medicamento puede inducir tumores
ureterales

a) Cisplatino

b) Prednisona

c) Ciclofosfamida

d) Etopósido

e) Dexametasona

UR34. Son opciones de reconstrucción vesical (neovejiga) posterior a cistectomía radical las siguientes,
excepto:

a) Studer

b) Camey

c) Miami

d) Akiyama

e) Kock

UR35. Límite superior de la disección inguinopélvica:

a) Bifurcación de la aorta

b) Ganglio de Cloquet

c) Arco crural

d) Bifurcación de arteria ilíaca primitiva

e) Bifurcación de arteria femoral


UR36. Tratamiento de elección en pacientes con tumores pequeños limitados a prepucio:

a) Penectomía total

b) Falectomía parcial

c) Radioterapia

d) Circunsición

e) Escisión amplia

UR37. Margen de falectomía parcial por cáncer de pene:

a) 0.5 cm

b) 1 cm

c) 2 cm

d) 3 cm

e) 1 cm

UR38. Principal ventaja de la RT en el manejo del tumor primario en el cáncer de pene:

a) Ninguna

b) Menos sangrado transoperatorio

c) Cirugía conservadora

d) Preservación de la función

e) Mejor control local que cirugía

UR39. Droga más efectiva contra cáncer de pene:

a) Meifalan

b) Ciclofosfamida

c) Platino

d) Adramicina

e) Vincristina
UR40. Indicación(es) para la prostatectomía radical:

a) Nódulo pequeño (menor a 1 cm)

b) Induración que comprende los dos lados

c) Próstata no fija

d) Todas las mencionadas

e) Ninguna de las anteriores

UR41. Son posibilidades de tratamiento en el seminoma en etapa avanzada (IIb o III) los siguientes,
excepto:

a) Quimioterapia sistémica (platino, vincristina y blemomicina)

b) Lo anterior más cirugía (linfadenectomía retroperitoneal)

c) Radioterapia más seguimiento con marcadores biológicos

d) Ante marcadores biológicos positivos deben ser tratados como no seminomatosos

UR42. Tratamiento de cáncer de pene in situ en enfermos confiables:

a) Radioterapia externa

b) Amputación distal del pene

c) Biopsia del ganglio de cabañas y linfadenectomía regional inguinal

d) Quimioterapia local y aplicación con láser con neodimio (YAG)

e) Tratamiento sólo de la infección concomitante

UR43. El tratamiento de cáncer de pene en etapa T2 N0 M0 es:

a) Amputación total del pene y emasculación

b) Amputación total del pene, meato perineal y QT sistémica

c) QT sistémica, únicamente

d) Amputación distal del pene con margen de 2 cm, y linfadenectomía ileoinguinal bilateral

UR44. El mejor método para tratar el cáncer de próstata localizado:


a) Dietilestilbestrol

b) Prostatectomía total

c) Bloqueadores de la LHRH

d) Antiandrógenos orales

e) QT citotóxica

UR45. Mejor método para tratar el cáncer de próstata localizado:

a) Quimioterapia

b) Bloqueadores de la LHRH

c) Antiandrógenos orales

d) Inmunoterapia

e) Prostatectomía radical

UR46. Procedimiento terapéutico más efectivo para disminuir el nivel de andrógeno en el cáncer de próstata
en etapa avanzada.

a) Manipulación hormonal

b) Resección transuretral de próstata

c) Prostectomía radical

d) RT

e) QT citotóxica

UR47. La resonancia magnética es un excelente método de escrutinio para establecer de manera definitiva el
diagnóstico de cáncer de próstata:

a) Verdadero

b) Falso

UR48. Tratamiento de elección para el seminoma puro etapa Ib:

a) QT con base en etopósido y bleomicina

b) Orquiectomía más radioterapia con protección a testículo contralateral

c) Orquiectomía más QT seguidos de RT

d) Linfadenectomía retroperitoneal
e) Vigilancia

UR49. Son indicaciones de la linfadenectomía retroperitoneal las siguientes, excepto el:

a) Etapa clínica Ib de cáncer de testículo de células germinales no seminomatoso

b) Etapa clínica II con residual en retroperitoneo posquimioterapia

c) Etapa clínica III tratado con recurrencia a nivel retroperitoneal

d) Manejo del cáncer de testículo de células germinales con residual posquimioterapia en


retroperitoneo

e) Tumor seminomatoso etapa clínica Ib

UR50. Tratamiento de elección en el tumor de células germinales no seminomatosos con enfermedad


retroperitoneal avanzada no resecable:

a) RT

b) QT previa a la linfadenectomía retroperitoneal

c) QT previa a la RT

d) RT seguida de QT

e) Cirugía citorreductora

UR51. Son indicaciones de cistectomía las siguientes, excepto:

a) Tumores que invaden el músculo no adecuados para una resección segmentaria de la vejiga

b) Tumores de etapa temprana multicéntricos o con frecuentes recurrencias, o resistencia a la


instilación de BCG intravesicales

c) Tumores de alto grado T1 G3 asociados a tumor in situ

d) Tumores solitarios de vejiga en el momento de primera consulta, tratado con resección


endoscópica

e) Instilación vesical de BCG sin enfermedad visible a los tres meses de evaluación

UR52. La cistectomía radical consiste en resección:

a) De vejiga y ganglios linfáticos regionales

b) En bloque de vejiga y próstata en el hombre, o resección en bloque de vejiga y útero en la


mujer
c) En bloque de la vejiga, próstata, vesículas seminales, vasos deferentes proximales, uretra
proximal con un margen de tejido adiposo y peritoneo en el hombre. Más linfadenectomía pélvica

d) En bloque de anexos y vejiga con linfadenectomía pélvica en la mujer

e) En bloque de vejiga recto y próstata más linfadenectomía retroperitoneal en el hombre

UR53. Principal objetivo de la radioterapia preoperatoria o posoperatoria en cistectomía radical por cáncer
de vejiga:

a) La sobrevida y período libre de enfermedad es igual con o sin radioterapia

b) Disminución de la sobrevida y del período libre de enfermedad

c) Ninguno

d) Aumento de las metástasis a distancia

e) Disminución del porciento de recurrencias locorregionales

UR54. Tratamiento de elección para cáncer de vejiga invasor en pacientes con contraindicación quirúrgica:

a) Quimioterapia

b) Radioterapia

c) Quimioterapia más radioterapia

d) Radioterapia neoadyuvante

e) Quimioterapia neoadyuvante, radioterapia seguida de más quimioterapia

UR55. Paciente joven con lesión exofítica del glande, se le realiza biopsia reportándose carcinoma
epidermoide invasor; el manejo:

a) Depende del grado histológico de la lesion primaria

b) Requiere de un estudio de extensión antes del tratamiento definitivo

c) Resección limitada o radioterapia externa, dependiendo del tamaño y grosor de la lesión

d) Escisión de la tumoración primaria, y dependiendo del tamaño tumoral, profundidad y


agresividad histológica, una disección profiláctica de los ganglios inguinales

e) QT y RT preoperatorias, seguidas de resección limitada


UR56. Masculino de 75 años de edad con cáncer de próstata metastásico a hueso y dolor importante; debe
de ser tratado con:

a) Leuprolide+quetoconazole

b) Radioterapia a todos los sitios sintomáticos

c) Dietilestilbestrol 1 mg -día

d) Dietilestilbestrol 3 mg-día

e) Leuprolide más flutamida

UR57. Son métodos establecidos de escrutinio del cáncer de próstata los siguientes, excepto:

a) Tacto rectal

b) Resonancia magnética

c) Antígeno prostático específico

d) Ultrasonido transrectal

e) Biopsia transrectal

UR58. Clasificación correspondiente al cáncer verrucoso no invasor de pene en la clasificación TNM:

a) Ta

b) Tl

c) T2

d) T3

e) T4

UR59. En la clasificación TNM la T que corresponde a un tumor de 3 cm que invade a la uretra es:

a) Ta

b) Tl

c) T2

d) T3

e) T4
UR60. Métodos clínicos fundamentales de diagnóstico en el cáncer de próstata:

a) Tacto rectal, ultrasonido y biopsia

b) Tacto rectal, biopsia y urograma excretor

c) Tacto rectal, antígeno prostático específico y ultrasonido transrectal

d) Biopsia, gamagrama óseo y tacto rectal

e) Ninguno de los anteriores

UR61. Son características del cáncer vesical T1 las siguientes, excepto que:

a) Cuando son de bajo grado y pequeños son tratados con RTU+QT intracavitaria

b) Frecuentemente son recurrentes

c) La lesión de bajo grado al nivel de trígono indica cistectomía parcial

d) Cuando son recurrentes y alto grado (III) obligan a cistectomía radical

e) La quimioterapia sistémica poscistectomía radical está indicada ante la recurrencia y el alto


índice de avance

UR62. Método más adecuado para diagnosticar metástasis óseas del cáncer de próstata:

a) Radiografías simples de huesos

b) Gamagrama óseo

c) Fosfatasa alcalina

d) Antígeno prostático específico

e) Tomografía computada de huesos

UR63. La clasificación TNM en el cáncer de próstata tiene correspondencia con la Witmore-Jewett; el T3a
corresponde a:

a) A1

b) B0

c) B1

d) B2
e) C1

UR64. La clasificación TNM en el cáncer de próstata tiene correspondencia con la Witmore Jewett; el T2a
corresponde a:

a) A1

b) B0

c) B1

d) B2

e) C1

UR65. La clasificación TNM en el cáncer de próstata tiene correspondencia con la de Whitmore Jewett; el
T1a corresponde a:

a) A1

b) B0

c) B1

d) B2

e) C

UR66. Método diagnóstico contraindicado en tumores de células germinales del testículo:

a) Orquiectomía inguinal radical

b) Biopsia por vía inguinal

c) Ultrasonografía de testículos

d) Orquiectomía transescrotal

e) Tomografía computada de abdomen

UR67. Corresponde a etapa Ia de cáncer de testículo, según la clasificación de TNM de la AJCC y la UICC
1996:

a) Tumor restringido al retroperitoneo

b) Tumor limitado al testículo y epidídimo sin invasión vascular-linfática, tumor que puede invadir
la túnica albugínea, pero no la vaginalis, no se encuentran adenopatías ni metástasis a distancia
c) Tumor limitado al testículo y epidídimo con invasión vascular-linfática o tumor que se extiende a
través de la túnica albugínea con infiltración a la vaginalis, no se encuentran adenopatía ni
metástasis a distancia

d) Tumor que invade el cordón espermático con invasión vascular-lifática o sin ella, no se
encuentran adenopatías ni metástasis a distancia

e) Tumor que invade el escroto con invasión vascular o linfática, metástasis viscerales no
pulmonares

UR68. Corresponde a etapa Ib de cáncer de testículo según la clasificación deTNM de la AJCC y la UICC
1996:

a) Tumor restringido al retroperitoneo

b) Tumor limitado a testículo y epidídimo sin invasión vascular-linfática, que puede invadir la
túnica albugínea pero no la vaginalis, no se encuentran adenopatías ni metástasis a distancia

c) Tumor limitado a testículos y epidídimo con invasión vascular-linfática o tumor que se extiende
a través de túnica albugínea con infiltración a la túnica vaginalis, no se encuentran adenopatías ni
metástasis a distancia

d) Tumor que invade el cordón espermático con invasión vascular-linfática o sin ella, no se
encuentran adenopatías ni metástasis a distancia

e) Tumor que invade el escroto con invasión vascular o linfática, metástasis viscerales no
pulmonares

UR69. Corresponde a etapa II de cáncer de testículo, según la clasificación de TNM de la AJCC y la UICC
1996:

a) Tumor restringido al retroperitoneo

b) Tumor limitado al testículo y epidídimo sin invasión vascular-linfática. Tumor que puede invadir
la túnica vaginalis, no se encuentran adenopatías ni metástasis a distancia

c) Tumor limitado a testículos y epidídimo con invasión vascular-linfática o tumor que se extiende a
través de la túnica albugínea con infiltración a la vaginalis, no se encuentran adenopatías ni
metástasis a distancia

d) Tumor que invade el cordón espermático con invasión vascular-linfática o sin ella, no se
encuentran adenopatías ni metástasis a distancia

e) Tumor que invade el escroto con invasión vascular o linfática, no hay adenopatía ni metástasis
a distancia

UR70. Marcadores tumorales séricos útiles en el diagnóstico, tratamiento, control y seguimiento para cáncer
de testículo:

a) AFP, deshidrogenasa láctica y hormona gonadrotopina coriónica

b) Fosfatasa alcalina, AFP, antígeno carcinoembrionario, testosterona, deshidrogenasa láctica y


antígeno carcinoembrionario
c) CA 125, deshidrogenasa láctica

d) CA 15:3, CA 125, AFP

UR71. Lo siguiente es verdadero con respecto al cáncer de vejiga, excepto que:

a) El 75% son superficiales al momento del diagnóstico

b) 20 a 25% son invasivos

c) 5 a 20% son metastásicos al momento del diagnóstico

d) La hematuria macroscópica es el signo más frecuente de presentación

e) La variedad histopatológica más común es el carcinoma epidermoide

UR72. Corresponde a etapa I de cáncer de vejiga el tumor que:

a) Invade tejido conectivo subepitelial

b) Invade el músculo

c) Invade la grasa perivesical

d) Invade próstata, útero, vagina, pared pélvica o pared abdominal

e) Presenta metástasis a distancia

UR73. Corresponde a etapa II del cáncer de vejiga el tumor que:

a) Invade tejido conectivo subepitelial

b) Invade músculo

c) Invade grasa perivesical

d) Invade próstata, útero, vagina, pared pélvica o pared abdominal

e) Presenta metástasis a distancia

UR74. Corresponde a etapa IVa del cáncer de vejiga el tumor que:

a) Invade tejido conectivo subepitelial

b) Invade músculo

c) Invade grasa perivesical


d) Invade próstata, útero, vagina, pared pélvica o pared abdominal

e) Presenta metástasis a distancia

UR75. Corresponde a etapa IVb del cáncer de vejiga el tumor que:

a) Invade tejido conectivo subepitelial

b) Invade músculo

c) Invade grasa perivesical

d) Invade próstata, útero, vagina, pared pélvica o pared abdominal

e) Presenta metástasis a distancia

UR76. Porciento de metástasis a distancia en el cáncer de vejiga T1:

a) 5-10

b) 10-30

d) 30-40

e) Más del 50

UR77. Paciente femenino de 52 años de edad con fractura patológica de húmero por lesión lítica, el rastreo
óseo demostró ser negativo en otras regiones y solamente presenta aumento de la capacitación a nivel
periférico de la lesión descrita; el diagnóstico más probable es:

a) Carcinoma de mama metastásico

b) Plasmocitoma

c) Linfoma maligno de hueso

d) Carcinoma metastásico de riñón

e) Displasia fibrosa de hueso

UR78. La zona de recaída en etapa Ia de seminoma testicular no radiado es:

a) Escroto

b) Mediastino

c) Paraaórticos
d) Cuello izquierdo

e) Ninguno de los sitios anteriores

UR79. Tipo de cáncer de mayor incidencia en pene:

a) Melanoma

b) Infiltracion leucémica

c) Basocelular

d) Sarcoma de Kaposi

e) Epidermoide

UR80. La omisión de la terapia a pelvis en el seminoma etapa clínica I resulta en:

a) Cifras de recurrencia global semejantes a la observación

b) Disminución en las cuentas espermáticas en hombres menores de 35 años

c) Mejor tolerancia hematológica durante el tratamiento

d) Riesgo de recurrencia global menor de 5%

e) Aumento en la resistencia a QT de salvamento en 20% de los pacientes que fallan

UR81. El seminoma espermatocítico:

a) Es muy frecuente en niños

b) Envía frecuentemente metástasis a pulmón y huesos

c) Se presenta en mayores de 50 años de edad

d) No requiere tratamiento

e) Nunca da metástasis

UR82. Estructura que tiende a prevenir la invasión del cáncer prostático al recto:

a) Fascia de Denonvilliers

b) Ligamento cardinal

c) Reflexión peritoneal

d) Pared quirúrgica de Guyon

e) Serosa peritoneal
UR83. La incidencia del tumor testicular se incrementa por:

a) Trauma

b) Infección

c) Hidrocele

d) Nada de lo anterior

e) a y b

UR84. El porciento de hombres en programas de detección de cáncer de próstata y con antígeno prostático
específico elevado que tendrán cáncer de próstata es de:

a) 10-15

b) 30-45

c) 20

d) 1.5-4

e) >50

UR85. La etapa A1 de cáncer de próstata tiene un porciento de progresión a 10 años de:

a) 5

b) 10

c) 20

d) 30

e) 40

UR86. Factor que más influye en el pronóstico del cáncer de próstata:

a) Grado de Gleason

b) Nivel pre-tratamiento del antígeno prostático específico

c) Edad del paciente

d) Nivel pre-tratamiento de la fosfatasa ácida

e) Nivel pre-tratamiento de la fosfatasa ácida, fracción prostática

UR87. ¿Qué células prostáticas secretan el antígeno prostático específico?


a) Epiteliales basales

b) Neuroendocrinas

c) Epiteliales secretoras

d) Macrófagos

e) Linfocitos

UR88. En el cáncer de próstata indiferenciado:

a) Se eleva mucho el antígeno prostático específico

b) Tiende a disminuir el antígeno prostático específico producido por gramo de tejido

c) No eleva nunca el antígeno prostático específico

d) Producción de antígeno específico es normal

e) No sucede nada de lo anterior

UR89. Los tumores de células germinales del testículo son más frecuentes en pacientes de:

a) 0-10 años

b) 10-15 años

c) 15-20 años

d) 20-35 años

e) Más de 50 años

UR90. Complicación más frecuente de la disección linfática retroperitoneal:

a) Fístula ureteral

b) Linfocele

c) Oclusión intestinal temprana

d) Ausencia de eyaculación

e) Quilorragia

UR91. Resultado histopatológico más frecuente de las disecciones linfáticas retroperitoneales como manejo
de cáncer de testículo de células germinales con residual posquimioterapia:

a) Necrosis/fibrosis/teratoma

b) Teratoma
c) Persistencia del cáncer

d) Segundo tumor primario maligno

e) Segundo tumor primario benigno

UR92. Secuela tardía más frecuente del tratamiento radioterápico del cáncer de testículo:

a) Radioepitelitis húmeda

b) Necrosis posradiación

c) Esterilidad

d) Segundo tumor primario benigno

e) Segundo tumor primario maligno

UR93. Factor pronóstico de recurrencia más importante del cáncer de vejiga superficial:

a) Etapa clínica

b) Variedad histopatológica

c) Edad del paciente

d) Grado histológico

e) Sexo del paciente

UR94. Factor pronóstico independiente más importante para progresión y sobrevida general del cáncer de
vejiga invasor:

a) Etapa clínica

b) Variedad histopatológica

c) Edad del paciente

d) Grado histológico

e) Sexo del paciente

UR95. El cáncer de vejiga presenta policrontropismo porque:

a) Presenta múltiples metástasis óseas

b) Presenta múltiples lesiones intravasculares


c) Afecta todo el urotelio

d) Las recurrencias se pueden desarrollar en cualquier momento, en el mismo sitio separado del
tejido urotelial y en la misma etapa o en uno más avanzado

e) Recurre fácilmente

UR96. Fármacos que son alternativa en el manejo hormonal en el tratamiento del cáncer de próstata,
excepto:

a) Dietiletilbestrol

b) Ketoconazol

c) Tamoxifén

d) Esteroides

e) Bicalutamida

UR97. El tratamiento intravesical del carcinoma superficial de vejiga incluye lo siguiente, excepto:

a) Thiotepa

b) Adriamicina

c) BCG

d) Fototerapia

e) Taxol

UR98. Tratamiento del cáncer vesical superficial de células trancisionales:

a) Resección transureteral vesical

b) Resección transureteral vesical + quimioterapia intravesical

c) Radioterapia radical

d) Ciatectomía radical

UR99. Tratamiento en cáncer renal localizado:

a) Radioterapia radical

b) Nefrectomía + quimioterapia
c) Quimioterapia, únicamente

d) Nefrectomía radical

e) Quimioterapia + radioterapia

UR100. El modelo actual de tratamiento para el carcinoma renal en etapa I, II y IIIa:

a) Nefrectomía radical y resección quirúrgica de metástasis

b) Nefrectomía radical y resección de ganglios linfáticos regionales

c) Lo anotado en (a) más modificadores de la respuesta biológica

d) Lo anotado en (b) más modificadores de la respuesta biológica

e) Ninguno de los anteriores

UR101. El tumor de células germinales no seminomatosos en etapa avanzada:

a) Se trata mediante combinaciones quimioterápicas posterior a la orquidectomía

b) La linfadenectomía es obligatoria ante marcadores biológicos positivos después de la


orquidectomía

c) Con marcadores biológicos negativos y ausencia de evidencia radiológica de actividad tumoral


sugiere la posibilidad de vigilancia clínica, exclusivamente

d) Se relaciona con todo lo anterior

e) Solamente se relaciona con a y c

UR102. La orquiectomía radical inguinal consiste en la extirpación del testículo y:

a) Disección linfática retroperitoneal

b) Epidídimo, una porcion de los conductos deferentes, una porción de ganglios linfáticos
gonadales y de su aporte sanguíneo

c) Epidídimo, una porción de los conductos deferentes, una porcion de los ganglios linfáticos
gonadales y de su aporte sanguíneo, disección linfática retroperitoneal

d) Epidídimo por vía transescrotal

e) Disección linfática inguinal


UR103. Paciente masculino de 42 años de edad con diagnóstico de seminoma puro de testículo. Se le
encontró una masa retroperitoneal de 8 cm; los marcadores iniciales fueron AFP 56 ng/mL y fracción beta de
la GCH 10 UI/mL, el resto de los exámenes fueron negativos después de tres ciclos; con el esquema BEP sus
marcadores se normalizaron pero persistió una masa retroperitoneal residual de 3 cm; las opciones para el
manejo subsecuente son:

a) Radioterapia a la masa residual

b) Quimioterapia con Cis platino-ifosfamida

c) Aspirado con aguja fina de la masa residual

d) Disección linfática retroperitoneal

e) Observación estrecha

UR104. Son indicaciones de la radioterapia a lecho renal las siguientes, excepto cuando existe:

a) Infiltración a grasa más allá de la cápsula de Gerota

b) Más del 90% de necrosis tumoral

c) Invasión a órganos vecinos

d) Residual macroscópica

UR105. Masculino de 29 años de edad con un tumor testicular derecho de 6 cm. La radiografía de tórax
muestra metástasis pulmonares múltiples, bilaterales. La TAC de abdomen reportó adenopatías
retroperitoneales. Alfafetoproteína y gonadotrofina coriónica negativas. Se le practicó orquiectomía derecha
con diagnóstico de seminoma. Se programa para recibir quimioterapia a base de etopósido, cis platino y
bleomicina. ¿Cuál régimen antiemético seleccionaría?

a) Lorazepan oral antes de la quimioterapia y clopromazina cada 4 a 6 horas, iniciándose con la


primera dosis de quimioterapia

b) Tres dosis orales de Ondanestron, iniciándose la primera 30 minutos antes de la quimioterapia

c) Granisetron IV 30 minutos antes de la quimioterapia

d) Lorazepan IV, Ondansetron y Dexametasona 30 minutos previos a la quimioterapia, seguido


durante 4 días de Metoclopramida, Dexametasona y Difenhidramina

UR106. Frecuencia de respuesta a la interleucina-2 en pacientes con cáncer renal avanzado:

a) 1%

b) 10%

c) 30%
d) 50%

e) 70%

UR107. En cáncer de próstata, la etapa Tic o B0 corresponde a:

a) Metástasis ganglionares regionales

b) Metástasis ganglionares a distancia

c) Tumor palpable en un lóbulo prostático

d) Tumor no palpable detectado sólo por elevación de APE

e) Tumor detectado por resección transuretral

UR108. Signo más frecuente de presentación durante el diagnóstico del cáncer de testículo:

a) Esterilidad

b) Adenomegalias inguinales

c) Criptorquidia

d) Edema escrotal

e) Masa palpable, dura e indolora en el testículo

UR109. Principal signo-síntoma por el que acude el paciente a consulta de primera vez por cáncer de vejiga:

a) Dolor

b) Obstrucción urinaria baja

c) Incontinencia urinaria

d) Hematuria macroscópica

e) Adenomegalias inguinales

UR110. Para la detección del cáncer de próstata en una persona asintomática de 50 años o más, el método
anual recomendable es:

a) Dosificación de testosterona

b) Ultrasonido transrectal

c) Únicamente antígeno prostático específico( APE)


d) Únicamente exploración rectal

e) Exploración rectal y APE

TUMORES DE CABEZA Y CUELLO

CC1. Subtipo histológico más común de tumores malignos en glándulas salivales menores:

a) Células acinares

b) Epidermoide

c) Mucoepidermoide

d) Adenoideo quístico

e) Adenocarcinoma

CC2. Subtipo histológico de neoplasia maligna en glándulas salivales que posee la mayor capacidad de
invasión neural:

a) Adenocarcinoma

b) Epidermoide

c) Adenoideo quístico

d) Células acinares

e) Mucoepidermoide

CC3. Estirpe histológica de tumores malignos parotideos que afecta más al nervio facial:

a) Células acinares

b) De mixto maligno

c) Adenocarcinoma

d) Indiferenciado

e) Adenoideo quístico

CC4. Las siguientes son características del adenoma pleomorfo de la glándula parótida excepto:

a) Es el tumor benigno más frecuente


b) Tiene un origen epitelial en el tejido conectivo

c) Afecta con más frecuencia a mujeres en la tercera y cuarta década de la vida

d) Se trata con biopsia escisional

e) Se localiza frecuentemente en la cola de la parótida

CC5. Cuando se produce regeneración aberrante de las fibras del nervio parasimpático (originadas del
ganglio ótico) a través de las fibras simpáticas hacia las glándulas sudoríparas, se establece:

a) Síndrome de Horner

b) Síndrome de Frey

c) Complicación posoperatoria de cirugía de la glándula submaxilar

d) Síndrome paraneoplásico carcinoide

CC6. ¿Cuál de los ejemplos siguientes no corresponde a cáncer de glándula salival mayor?

a) Adenocarcinoma

b) Carcinoma mucoepidermoide

c) Carcinoma epidermoide

d) Tumor mixto maligno

e) Carcinoma tubular

CC7. ¿Cuál de los estirpes siguientes tiene el mejor pronóstico en tumores malignos de glándulas salivales
mayores?

a) Carcinoma mucoepidermoide de grado intermedio de malignidad

b) Mixto maligno

c) Carcinoma adenoideo quístico

d) Adenocarcinoma

e) Carcinoma de células acinares

CC8. En carcinoma de glándulas salivales mayores de alto grado de malignidad, la posibilidad de tener
metástasis ocultas en las lesiones T1 (menos de 2 cm) es de:
a) 20%

b) 35%

c) 45%

d) 10%

e) menos del 8%

CC9. Lesión mandíbular caracterizada por su alto índice de recurrencia:

a) Quiste dentigero

b) Fibroma central

c) Queratoquiste

d) Osteoma

e) Quiste Nasopalatino

CC10. El carcinoma epidermoide del trígono retromolar drena a los niveles ganglionares siguientes:

a) III y IV

b) I, II y III

c) IV y VI

d) VI y VII

CC11. Tumor de cavidad bucal de crecimiento lento y manejo conservador:

a) Cáncer epidermoide

b) Plasmocitoma

c) Carcinoma verrucoso

d) Melanoma maligno de mucosas

e) Fibrosarcoma

CC12. El cáncer de cavidad bucal más agresivo por localización es:

a) Epidermoide de lengua
b) Epidermoide de encía

c) Epidermoide de trígono retromolar

d) Epidermoide carrillo

e) Epidermoide de paladar

CC13. ¿Cuál es el sitio de presentación más frecuente de carcinoma de lengua?

a) Punta

b) Dorso

c) Base de lengua

d) Borde lateral

e) Cara ventral

CC14. ¿Cuál es el principal factor de riesgo para el cáncer de mucosa bucal?

a) Tabaquismo y alcoholismo

b) Tabaquismo puro

c) Alcoholismo puro

d) Aseo bucal deficiente

e) Ingesta de grasas

CC15. Por su localización e histología los sarcomas de cabeza y cuello son más agresivos y fatales, excepto:

a) Angiosarcoma

b) Rabdomiosarcoma

c) Leiomiosarcoma

d) Schwanoma maligno

e) Osteosarcoma

CC16. ¿Cuál de los siguientes no es un factor predisponente en el cáncer de laringe?

a) Edad
b) Tabaquismo

c) Alcoholismo

d) Desnutrición

e) Sexo

CC17. El porciento de probabilidad de metástasis oculta que justifica una disección electiva en los cuellos es
de:

a) 10%

b) 15-20%

c) 25-30%

d) 60-65%

e) 70%

CC18. El sitio anatómico de la hipofaringe más frecuentemente afectado por cáncer es:

a) Área poscricoidea

b) Seno piriforme

c) Pared posterior de faringe

d) Repliegue faringoepiglótico

e) Repliegue aritenoepiglótico

CC19. Las placas de Rx de senos paranasales son:

a) Caldwell

b) Watters

c) Lateral

d) Son correctas a, b y c

e) Son incorrectas

CC20. El cáncer de tiroides diferenciado más frecuente:

a) Células Hurtle

b) Papilar
c) Folicular

d) Medular

e) Mixto

CC21. ¿Cuál es la vida media de las hormonas tiroideas?

a) 6-8 horas

b) 8-12 horas

c) 12-24 horas

d) 2-5 días

e) 6-8 días

CC22. En el cáncer de tiroides son marcadores biológicos los siguientes, excepto:

a) Tiroglobulina

b) Estimulación de calcitonina con pentagastrina

c) Factor de crecimiento epidermoide

d) Calcitonina

CC23. El pronóstico del cáncer papilar de tiroides:

a) Es mejor en pacientes mayores de 45 años

b) Es mejor en pacientes masculinos menores de 45 años

c) Es mejor en pacientes femeninas menores de 45 años

d) Es mejor si afecta los dos lóbulos

e) Es mejor en presencia de ganglios metastásicos cervicales

CC24. Las metástasis del cáncer papilar de tiroides son más frecuentes en:

a) Hueso

b) Ganglios cervicales

c) Cerebro
d) Pulmón

e) Hígado

CC25. Estirpe histológica de peor pronóstico en el cáncer de tiroides:

a) Folicular

b) Papilar

c) Anaplásico

d) Medular

e) Linfoma

CC26. El cáncer de tiroides es más frecuente en presencia de:

a) Nódulo único

b) Bocio coloide

d) Múltiples nódulos tiroideos

e) Tiroiditis

f) Bocio difuso tóxico

CC27. Señale el cáncer de tiroides que se origina en células secretoras de calcitonina:

a) Papilar

b) Medular

c) Anaplásico

d) Folicular

e) Hürthle

CC28. Cáncer de tiroides que presenta predisposición familiar:

a) Papilar

b) Anaplásico

c) Medular
d) Folicular

e) Linfoma

CC29. El linfoma de la glándula tiroides, con respecto a todos los linfomas extranodales, representa el:

a) 30%

b) 20%

c) 10%

d) 2%

e) Menos del 1%

CC30. Según estadísticas mexicanas la frecuencia de neoplasias malignas de la glándula tiroides representa,
con respecto a los cánceres en general, el:

a) 1-2%

b) 5 %

c) 7%

d) 10%

e) 15%

CC31. Gran parte de las neoplasias malignas de tiroides son tumores sólidos, pero ante la presencia de un
"quiste tiroideo" ¿en qué porciento se trata de un cáncer de tiroides?

a) < del 3

b) Entre el 3 y 25

c) Entre 30 y 40

d) Entre 40 y 60

e) Más del 60

CC32. ¿En Ca de tiroides cuál de los siguientes no es un factor de mal pronóstico?

a) La variedad de células altas

b) La presencia de metástasis ganglionares locales al momento del diagnóstico en mujeres


mayores de 45 años

c) La extensión tumoral extratiroidea


d) La multicentricidad demostrada

e) La presencia de metástasis ganglionares locales después del tratamiento inicial adecuado en


pacientes jovenes

CC33. Con respecto al estudio del DNA en el cáncer de tiroides, la aneuploidia, determinada en citometría de
flujo tiene significancia pronóstica adversa en:

a) Carcinoma papilar

b) Estudio estadístico multivariado

c) Carcinoma medular

d) Carcinoma papilar, medular y folicular

e) No tiene utilidad en el cáncer de tiroides

CC34. El carcinoma medular de tiroides representa el 5-10% de todos los cánceres de tiroides y en lo que
respecta a su forma esporádica o familiar:

a) Predomina la esporádica en un 90%

b) Se presentan en una proporción de 50% para cada una

c) Predomina la esporádica en un 75%

d) Predomina la esporádica solo en relación con el que forma parte de los síndromes endócrinos
múltiples

e) Predomina la esporádica solo en relación con el que forma parte de la familiar sin síndrome
endocrino múltiple

CC35. La edad más frecuente en que se presenta el carcinoma papilar de tiroides es:

a) En la década de los cuarenta

b) Después de los 50 años

c) Antes de los 30 años

d) En mujeres a los 30 años

e) En los hombres después de los 50 años

CC36. Con respecto a la frecuencia del carcinoma de tiroides, en México, se sabe que esta neoplasia en el
sexo masculino ocupa el sitio número:

a) Tercero

b) Décimo
c) Depende de la zona, ya no que hay diferencias marcadas

d) Vigésimo

e) Después del vigésimo quinto

CC37. Las neoplasias malignas de tiroides son frecuentes en la mujer y ocupan, por tanto, en su frecuencia,
el sitio número:

a) Segundo

b) Cuarto

c) Octavo

d) Décimo quinto

e) Después del décimo sexto

CC38. Con respecto al carcinoma folicular de tiroides, son características importantes las descritas a
continuación, excepto:

a) Su frecuencia es menor que la del carcinoma papilar (17%)

b) Presenta una menor frecuencia de metástasis ganglionares cervicales

c) Presenta una mayor frecuencia de metástasis a distancia

d) El pronóstico es peor que el del papilar

e) La captación del I-131 es semejante al papilar

CC39. En que porciento, los pacientes con cáncer medular del tiroides con pN(+), tienen riesgo de
desarrollar metástasis a distancia durante su seguimiento a 20 años:

a) Menor al 10

b) El 20

c) El 40

d) El 55

e) Mayor al 70

CC40. Son los factores pronósticos más importantes para el cáncer de tiroides (según la clínica Lahey-
AMES), excepto:
a) Edad

b) Metástasis a distancia

c) Tamaño tumoral

d) Extensión extratiroidea

e) Grado de diferenciación

CC41. La frecuencia de cáncer en los nódulos tiroideos "fríos" en el gamagrama, es aproximadamente del:

a) 1-2%

b) 3-5%

c) 6-10%

d) 11-15%

e) 15-30%

CC42. La frecuencia de cáncer en los nódulos tiroideos "calientes" en el gamagrama, es del orden:

a) Del 10%

b) Del 30%

c) Del 4%

d) Del 20%

e) Menor del 1%

CC43. Sitio donde el cartílago tiroides presenta deficiencia en su pericondrio:

a) Espacio paraglótico

b) Espacio cricotiroideo

c) Comisura anterior

d) Espacio de Reinke

e) Línea arcuata

CC44. El ganglio prelaringeo o delfiano tiene relación con el drenaje de:


a) Epiglotis suprahioidea

b) Zona marginal

c) Epiglotis infrahioidea

d) Subglótico

e) Espacio pre-epiglótico

CC45. Las estructuras supraglóticas tienen su drenaje linfático en los ganglios:

a) Linfáticos cervicales superiores profundos

b) Yugulodigástricos

c) Linfáticos cervicales inferiores profundos

d) Linfáticos pretraqueales

e) Linfáticos supraesternales

CC46. El triángulo carotídeo superior está limitado por:

a) Borde anterior del músculo esternocleidomastoideo, el vientre posterior del digástrico, el


músculo omohioideo

b) Borde anterior del esternocleidomastoideo, vientre anterior del digástrico, músculo omohioideo

c) Borde anterior de esternocleidomastoideo, vientre posterior del digástrico, músculo


esternotiroideo

d) Borde posterior del esternocleidomastoideo, vientre posterior del digástrico, músculo


omohioideo

e) Músculo esternotiroideo, músculo omohioideo y borde anterior del esternocleidomastoideo

CC47. El nervio hipogloso se localiza, en relación a la bifurcación de la arteria carótida:

a) A 1 cm por debajo de la bifurcación en dirección posteroanterior

b) En una dirección posteroanterior a 1.5-2 cm por encima de la bifurcación de la arteria carótida

c) Por dentro de la arteria carótida interna y externa

d) A nivel de la arteria carótida primitiva

e) A nivel del hueso hioides


CC48. El nivel VI de ganglios linfáticos se refiere a:

a) Los ganglios paratraqueales

b) Los ganglios retrofaríngeos y retrotraqueales

c) Los ganglios comprendidos entre el hueso hioides, la escotadura supraesternal y las arterias
carótidas comunes

d) Los ganglios delfianos y suprahioideos

e) Los ganglios mediastinales

CC49. Dentro de la anatomía quirúrgica del cuello, en lo que compete a la cirugía de tiroides, es de capital
importancia el conocimiento de las glándulas paratiroides, a este respecto sabemos que generalmente son
cuatro, en un porciento de:

a) 98%

b) 84%

c) 65%

d) Menos del 50%

e) No hay un estudio preciso, por lo que son muy variables

CC50. En la anatomía quirúrgica de la glándula tiroides es importante el conocimiento del riego sanguíneo de
las glándulas paratiroides, el cual depende principalmente de:

a) Ramas directas de la carótida externa

b) Ramas de la tiroidea inferior

c) Ramas esofágicas

d) Ramas de la tiroidea superior

e) Ramas tirocervicales

CC51. Las ramas principales del nervio facial (VII par craneal) son las siguientes:

a) Temporocigomática y cérvicofacial

b) Buccinadora y bucal

c) Mandibular y buccionadora
d) Cérvicofacial y buccionadora

e) Auricular mayor y temporocigomática

CC52. Los ganglios linfáticos metastásicos son relativamente frecuentes en pacientes con carcinoma
escamoso del conducto auditivo externo, presentándose en una frecuencia aproximada referida en la
literatura universal de:

a) 0-5%

b) 6-20%

c) 25-30%

d) 35-40%

e) Ninguno de los anteriores

CC53. ¿Cuál de los factores siguientes no afecta la sobrevida en cáncer de oído?

a) Afección del nervio facial

b) Infiltración del oído medio

c) Ganglios parotídeos

d) Infiltración de la dura madre

e) Diferenciación y tumor mayor de 3 cm

CC54. Son factores pronósticos adversos en el cáncer de oído los siguientes, excepto:

a) Invasión a canal auditivo cartilaginoso

b) Invasión a glándula parotida

c) Invasión a articulación tempomandibular

d) Afección a pabellon auricular

e) Invasión a membrana del tímpano y oído medio

CC55. De los tumores del espacio parafaríngeo, la lesión maligna más frecuente es:

a) Tumor de cuerpo carotídeo

b) Linfoma

c) Sarcoma neurogénico
d) Meningioma maligno

e) Metástasis de cáncer de tiroides

CC56. ¿Cuál de las estructuras anatómicas siguientes no es parte al espacio parafaríngeo?

a) Músculos pterigoideos

b) Músculos tensores del velo del paladar

c) Arteria maxilar interna

d) VII par craneal

e) XII par craneal

CC57. En cuál de las neoplasias siguientes se encuentra más establecido el rol del virus de Epstein Barr:

a) Carcinoma nasofaríngeo no queratinizante

b) Enfermedad de Hodking

c) Sarcoma de Kaposi

d) Linfoma de Burkitt

e) Leiomiosarcoma

CC58. El I-131 no es útil en el tratamiento de pacientes que tienen cáncer de tiroides del tipo:

a) Papilar

b) Folicular

c) Medular

d) Mixto papilar y folicular

e) El I-131 es útil en todos los tipos de cáncer en tiroides

CC59. Indicación absoluta de radioterapia posoperatoria en tumores de glándulas salivales:

a) Tumor en cola parotidea

b) Tumor de 2 cm

c) Tumor bien diferenciado


d) Invasión perineural mayor

e) Adenoma en lóbulo superficial

CC60. Porciento de recurrencia después de la enucleación simple sin radioterapia para adenomas de la
parótida:

a) Menos del 5

b) 25

c) 40-50

d) 75

e) 100

CC61. Indicación de radioterapia posoperatoria en tumores de glándulas salivales menores del paladar duro:

a) Tumor submucoso del paladar

b) Involucro del nervio palatino mayor

c) Adenoma pleomorfo de 1 cm

d) Involucro nervioso mínimo

e) Bajo grado de malignidad

CC62. Tratamiento más efectivo en tumores de alto grado de malignidad en glándulas salivales:

a) QTP neoadyuvante

b) RTP y cirugía

c) QTP y RTP

d) Cirugía y RTP

e) Sólo Cirugía

CC63. En el acceso transmandibular paramediano es necesario realizar la osteosíntesis con:

a) Una miniplaca cerca del reborde alveolar

b) Una miniplaca al borde basal mandibular


c) Dos placas paralelas, una al borde alveolar y otra al borde basal

d) Una placa en cara lingual

CC64. ¿Qué porciento ocupa el carcinoma de lengua de todos los cánceres de cavidad bucal?

a) 5

b) 30-35

c) 10-25

d) 50-70

e) Más de 70

CC65. El tratamiento del cáncer avanzado de cavidad bucal es primordialmente:

a) Quirúrgico

b) Radioterapia

c) Quimioterapia

d) Multidisciplinario

e) Inmunoterapia

CC66. El tratamiento tradicional del cáncer localmente avanzado de laringe es:

a) Cirugía radical

b) Cirugía + RTP

c) QTP + cirugía

d) RTP + cirugía + RTP

e) QTP + cirugía + QTP

CC67. Resultado obtenido con la quimioterapia neoadyuvante o simultánea con radioterapia en el


tratamiento del cáncer epidermoide avanzado de cabeza y cuello:

a) Mejorar la calidad de vida

b) Conservar órganos
c) Aumentar el período libre de enfermedad

d) Evitar recurrencias

e) Facilitar la rehabilitación

CC68. Es el esquema de quimioterapia que ofrece mejor resultado en el cáncer epidermoide en el área de
cabeza y cuello:

a) Carboplatino

b) Adriamicina

c) Cisplatino + 5FU

d) Fluorouracilo

e) Bleomicina

CC69. La utilidad de la quimioterapia de inducción seguida de radioterapia, con la finalidad de la


conservación de la laringe en cánceres avanzados, se fundamenta en que:

a) La quimioterapia evita las micrometástasis sistémicas por lo que mejora la sobrevida

b) La quimioterapia "sensibiliza" principalmente las células neoplásicas a la radioterapia

c) Todos los pacientes tratados con este método, tienen la misma sobrevida que los tratados en
forma convencional

d) Los pacientes que responden adecuadamente a la quimioterapia responden en igual forma a la


radioterapia y su sobrevida es igual a la que se obtiene con el tratamiento convencional

e) La poliquimioterapia es la única que actúa sobre las células neoplásicas y mejora el resultado
de la radioterapia

CC70. ¿Qué tratamiento NO prescribirá usted en un paciente masculino de 65 años de edad con cáncer de
hipofaringe; clasificado como T3 N1 MO etapa clínica III?

a) QTP+ RT simultánea con rescate quirúrgico

b) Cirugía radical

c) RTP y cirugía radical

d) Cirugía radical + RTP

e) QTP + cirugía radical + RTP


CC71. Con el uso de quimioterapia neoadyuvante o simultánea con radioterapia en tumores malignos
epidermoides avanzados en el área de cabeza y cuello se observa una respuesta global (completa y parcial)
del orden del:

a) 10-20%

b) 30-40%

c) 40-60%

d) 60-80%

e) 100%

CC72. El tratamiento aceptado por la mayoría de los autores para las metástasis cervicales clínicas de cáncer
epidermoide en cabeza y cuello es:

a) RTP radical

b) QTP neoadyuvante

c) Cirugía radical modificada

d) Cirugía radical

e) Cirugía radical RTP

CC73. La disección supraomohioidea “extendida” se refiere a la escisión de los grupos ganglionares


siguientes:

a) Nivel I a IV

b) Nivel II a IV

c) Nivel I a III

d) Nivel IV y V

e) Nivel VI

CC74. La disección radical de cuello modificada tipo 1 se refiere a:

a) Resección de niveles I-III de ganglios linfáticos

b) Resección de niveles I, III y IV de ganglios linfáticos y yugular interna

c) Resección de niveles I-VI y vena yugular interna

d) Resección de los cinco niveles ganglionares del cuello y yugular interna


e) Resección de los cinco niveles ganglionares del cuello, resección del músculo
esternocleidomastoideo, y resección de la vena yugular interna

CC75. Es la cirugía mínima en los tumores tiroideos:

a) Enucleación

b) Lobectomía

c) Istmectomía

d) Tiroidectomía total

e) Tiroidectomía subtotal

CC76. El tratamiento de las metástasis a cuello de los carcinomas bien diferenciados de tiroides, es
actualmente la disección radical modificada (DRM) o selectivas, ya que se obtienen resultados semejantes
que con la disección radical clásica (DRC), la disección rutinaria del nivel ganglionar I no se justifica debido
a:

a) La disección radical clásica deja muchas secuelas funcionales motoras

b) Debido a que hay metástasis en el 1% de los casos solamente

c) La disección radical clásica no modifica el pronóstico

d) Causa artificios en los rastreos con I-131 por cambios posquirúrgicos

e) Por la presencia de xerostomía de la cavidad bucal

CC77. La tiroidectomía total en el carcinoma diferenciado de tiroides esta indicada, excepto en:

a) Presencia de metástasis a distancia

b) Cáncer folicular con invasión vascular y capsular extensa y tumor mayor de 4 cm.

c) Presencia de enfermedad ganglionar uni o bilateral

d) Tumores menores de 3 cm, sin extensión extracapsular ni metástasis a distancia

e) Presencia de extensión tumoral extratiroidea

CC78. La lesión del nervio laríngeo superior (en su rama externa) provoca como secuela la dificultad para
que el paciente dé tonos altos con su voz y además de que la voz "se apaga" en el transcurso del día al hablar
continuamente. Esto es debido a:
a) La alteración de la movilidad de la cuerda vocal correspondiente, por las anastomosis del
nervio mencionado con el nervio laríngeo inferior, que tiene esta función

b) Las alteraciones sensitivas endolaríngeas que se provocan al lesionar esta estructura nerviosa

c) Por la parálisis del músculo constrictor inferior de la faringe correspondiente

d) Por la parálisis del músculo cricotiroideo correspondiente

e) Por la parálisis del músculo cricoaritenoideo correspondiente

CC79. Vía de acceso en el tratamiento quirúrgico del nasoangiofibroma más adecuada en la etapa I:

a) Sublabial

b) Transpalatina

c) Rinotomía lateral

d) Weber Ferguson

e) Transnasal

CC80. Tratamiento de elección en los tumores de cuerpo carotideo:

a) RTP

b) QTP

c) Cirugía

d) Observación

e) Multidisciplinario

CC81. Los pares craneales con mayor riesgo de lesión durante la resección de un paraganglioma carotídeo
son:

a) X y XII

b) X, XI y XII

c) VII y X

d) XII y VII

e) X, XII y VII
CC82. La vía de acceso quirúrgica del espacio parafaríngeo usada con mayor frecuencia es:

a) Transparotídea

b) Transmandibular

c) Disección de la fosa infratemporal/base de cráneo

d) Transcervical

e) Transcérvico submandibular

CC83. Complicación más severa derivada de la biopsia incisional en tumores parotideos:

a) Formación de hematoma

b) Escaso tejido para su interpretación

c) Parálisis facial y fístula salival

d) Cicatriz inadecuada

e) Lesión del auriculotemporal

CC84. Masculino de 50 años de edad con carcinoma epidermoide 3 cm de diámetro en el trígono retromolar,
su tratamiento de elección es:

a) Cirugía

b) Radioterapia externa

c) Cirugía más radioterapia

d) Braquiterapia

e) Quimioterapia inductiva más radioterapia

CC85. Carcinoma de labio T3 se define:

a) Tumor mayor de 2 cm

b) Tumor menor de 3 cm

c) Tumor mayor de 4 cm en dimensión

d) Tumor que invade tejido óseo

e) Tumor que invade partes blandas y piel del mentón


CC86. Paciente con tumor de 3 cm de diámetro localizado en labio inferior con cuello negativo, se etapifica
según el sistema internacional TNM como:

a) Etapa IV

b) Etapa Ia

c) Etapa II

d) Etapa III

e) Etapa Ib

CC87. ¿Qué porciento de pacientes con diagnóstico de cáncer de lengua presentan ganglios cervicales
clínicamente positivos?

a) Menos del 10

b) Del 10 al 20

c) 30

d) 60

e) 80

CC88. ¿Cuál es el sitio principal de metástasis ganglionares en el carcinoma de mucosa bucal?

a) Nivel I, II y III

b) Nivel II, III y IV

c) Nivel V

d) Nivel I, II, III y IV

CC89. Masculino de 65 años al que se estudia por disfonia de seis y medio meses de evolución, se detecta
un tumor en la cuerda vocal izquierda, no invade otras estructuras pero provoca parálisis cordal total. No se
detectan ganglios positivos en cuello. El resto de los estudios de extensión son negativos, con estos datos la
clasificación de la AJCC sería:

a) Tl b, NO, MO

b) T2, NO, MO

c) T3, NO, MO

d) Transglótico

e) Faltan datos para poder clasificar adecuadamente


CC90. El sitio de la laringe que más se encuentra afectado por cáncer según la literatura extranjera es:

a) Supraglotis

b) Glotis

c) Subglotis

d) a y b son correctas

e) a y c son correctas

CC91. Qué porciento ocupa el cáncer de laringe en la frecuencia por topografia del total de neoplasias
malignas de cabeza y cuello en México:

a) 6.5%

b) 12%

c) 4.5%

d) 1.5%

e) 7.5%

CC92. Un ganglio linfático metastásico con histología de cáncer epidermoide a nivel yugulodigástrico (grupo
II) tiene como probable sitio del primario en:

a) Senos paranasales

b) Glándula submaxilar

c) Tiroides

d) Nasofaringe y laringe

e) Nasofaringe, base de lengua y amígdala

CC93. En las neoplasias de cabeza y cuello el N2b se refiere a:

a) Ganglios linfáticos múltiples ipsilaterales de 3 y 6 cm de diámetro

b) Ganglios linfáticos unilaterales de 2 cm de diámetro

c) Ganglios linfáticos de más de 6 cm de diámetro

d) Ganglios linfáticos bilaterales de menos de 3 cm de diámetro

e) Ganglios linfáticos bilaterales de menos de 6 cm de diámetro


CC94. Paciente de 60 años de edad con cáncer epidermoide de seno paranasal maxilar derecho que infiltra
piso de órbita, etmoides, pared posterior del seno y piel de mejilla con adenopatía submaxilar homolateral de
2 cm única ¿cómo lo clasifica acorde a TNM?

a) T2 N1 M0

b) T4 N1 M0

c) T3 N1 M0

d) T4 N0 M0

e) T4 N0 M0

CC95. Paciente femenino de 26 años de edad con nódulo tiroideo único. Durante su estudio la BAAF reportó un
pequeño fragmento de tejido con amiloide en su estroma. Con esto el diagnóstico a descartar como más probable
es:

a) Tiroiditis de Hashimoto

b) Linfoma no Hodgkin primario de tiroides

c) Carcinoma anaplásico de tiroides

d) Carcinoma diferenciado de tiroides

e) Carcinoma medular de tiroides

CC96. Estudios paraclínicos indispensables para evaluación adecuada de nódulo tiroideo:

a) USG y BAAF

b) Gamagrama

c) TAC

d) Resonancia magnética

e) Perfil tiroideo

CC97. Estirpe histológica del cáncer de tiroides cuyas metástasis capta el I-131:

a) Diferenciado

b) Anaplásico

c) Medular

d) Sarcoma

e) Hürthle
CC98. Una de las diferencias principales entre el carcinoma de células de Hürthle con el folicular, es la
evidencia de que el primero capta yodo en forma considerablemente menor.¿Que finalidad persigue la
administración de I-131, en pacientes operados de tiroidectomía por carcinoma de Hürthle:

a) Debe de utilizarse porque su efectividad es limitada

b) Debe de utilizarse en dosis curativas

c) Debe de usarse como dosis ablativa de tejido residual captante para usar la tiroglobulina como
marcador tumoral

d) Debe de usarse solo con la presencia de tiroglobulina elevada sustancialmente con suspensión
de el tratamiento frenador de TSH

e) Debe de usarse en pacientes con factores pronósticos adversos

CC99. El gen p53 a través de una proteína inhibe en forma normal la transcripción del DNA y al haber
mutaciones se inhibe esta función y se desarrolla cáncer. En el caso del tiroides esta situación se representa
clínicamente como:

a) No se ha demostrado efecto en el cáncer de tiroides

b) Sólo su expresión se relaciona con el carcinoma anaplásico de tiroides

c) Su significado pronóstico está por determinarse, pues se manifiesta en varias circunstancias de


manera inconstante

d) Es exclusiva de los poco diferenciados (carcinoma medular)

e) No se manifiesta en los bien diferenciados

CC100. Los estudios preoperatorios indispensables en un paciente con tumores de cuerpo carotídeo de alto
riesgo (tamaño de más de 5 cm, cercanía a la base del cráneo y cirugía previa) son:

a) Arteriografía carotídea, con prueba de oclusión con balón y TAC con xenón

b) Resonancia magnética con fase angiográfica y TAC

c) Arteriografía carotídea, con prueba de oclusión con balón

d) TAC y arteriografía carotídea

e) Ultrasonido Doppler y TAC

CC101. Con respecto a los tumores del espacio parafaríngeo, la manifestación clínica que con mayor
frecuencia se presenta es:

a) Tumor faríngeo

b) Tumor en cuello

c) Disfagia

d) Sensación de cuerpo extraño


e) Trismus

CC102. Una persona que desarrolló un cáncer laringeo el cual fue completamente resecado, podrá tener con
mayor frecuencia en el futuro, además de metástasis de este primario, cuál de los siguientes tipos de cáncer:

a) Adenocarcinoma de pulmón

b) Cáncer de vejiga

c) Cáncer de esófago

d) Cáncer epidermoide de pulmón

e) Cáncer gástrico

CC103. La frecuencia de involucro del nervio facial en tumores parotideos es de:

a) 2 a 5%

b) 10 a 20%

c) 20 a 50%

d) 50 a 75%

e) más del 80%

CC104. Es indicación para realizar disección de cuello electivo suprahioidea en tumores de glándulas
salivales mayores:

a) Adenoma del lóbulo profundo

b) Oncocitoma

c) Tumor de Whartin en cola parotidea

d) Adenocarcinoma de 4 cm

e) Tumor de células acinares en lóbulo superficial

CC105. ¿Cuál de las medidas siguientes no es útil cuando se presenta una fístula quilosa posoperatoria por
disección radical de cuello?

a) Dieta a base de triglicéridos de cadena media

b) Dieta a base de triglicéridos de cadena pesada

c) Nutrición parenteral total


d) Aspiración y elevación de la cabeza

e) Compresión externa

CC106. La cuantificación de la tiroglobulina, en pacientes con carcinoma de tiroides, no se considera como


un marcador tumoral propiamente dicho, por la razón siguiente:

a) Porque se secreta sin la estimulación de la tirotrofina

b) Porque se puede elevar en padecimientos no neoplásicos de la tiroides

c) Porque se negativiza en presencia de anticuerpos antitiroglobulina

d) Porque su cantidad es variable en lesiones papilares y foliculares

e) Porque sus niveles son distintos con tratamiento supresivo con levotiroxina

CC107. De los grupos de análisis de factores de riesgo, el único (de los enlistados) que toma en cuenta el
carcinoma medular es:

a) SAG

b) DAMES

c) AMES

d) EORTC

e) MACIS

CC108. Del sistema de clasificación pronóstica de MACIS (Clínica Mayo) es una modificación de la AMES
y sus principales diferencias son:

a) El aumento en las dimensiones del tamaño y el grado de diferenciación

b) La presencia o no de invasión extratiroidea

c) El sistema de clasificar la edad del paciente

d) Extensión extracapsular del tumor y su extirpación quirúrgica completa

e) El grado de diferenciación

CC109. Con respecto al carcinoma medular de tiroides es sabido que ante un primario de 2 cm o mayor,
tiene metástasis ganglionares en los grupos ganglionares yugulares en un porciento de:

a) Menos del 10

b) Del 20

c) Del 40
d) Del 60

e) Mayor del 70 según la serie

CC110. En el estudio de los nódulos tiroideos, en lo referente al la presencia de un tumor maligno, el


gamagrama tiroideo y las pruebas funcionales no tienen mucho valor, pero en una lesión nodular
asintomática sin estudios funcionales, se corre el riesgo de que posterior a su estudio morfológico con
BAAF, se desencadene hipertiroidismo al dar tratamiento supresor en:

a) Los nódulos con tiroiditis de Hashimoto

b) Los nódulos con tiroiditis subaguda

c) Los nódulos con tiroiditis de Riedel

d) Los nódulos funcionales autónomos

e) Los portadores de carcinoma folicular muy bien diferenciado

CC111. Valorando el papel de la BAAF en tiroides se acepta que más que su sensibilidad para diagnosticar
malignidad (alrededor de 90%) es la especificidad, esto es, que los nódulos estudiados que son reportados
como negativos lo sean en la realidad, este porciento debe de ser de:

a) 50%

b) 70%

c) 85%

d) Superior a la media de la sensibilidad

e) Superior al 90%

CC112. Son aseveraciones sobre el carcinoma diferenciado de tiroides las siguientes, exepto:

a) Su presencia durante el embarazo no es un factor pronóstico adverso

b) La histología más común es el carcinoma papilar

c) El estudio clínico determinante es la punción con aguja fina

d) El tratamiento con I-131 y teleterapia no está indicado durante el embarazo

e) Por la presencia de las gonadotrofinas coriónicas, su pronóstico es discretamente peor

CC113. Variantes histológicas de cáncer papilar del tiroides de mal pronóstico, excepto:

a) Nodular bien diferenciado


b) Variedad esclerosante

c) De células altas

d) De células columnares

e) Variedad difusa

CC114. Característica(s) histológica(s) que define(n) al carcinoma folicular de tiroides:

a) Grado de diferenciación

b) Subtipo histológico

c) Presencia de invasión capsular y vascular

d) Multicentricidad

e) a y b

CC115. Tipo de tumor tiroideo que la OMS divide en mínimamente invasivo y ampliamente invasivo:

a) Cáncer papilar

b) Cáncer medular

c) Cáncer de células de Hurthle

d) Cáncer folicular

e) Cáncer anaplásico

CC116. Con base en el sistema de AMES en cáncer de tiroides bien diferenciado, son pacientes de bajo
riesgo los siguientes, excepto:

a) Todos los pacientes varones menores de 51 años

b) Todas las mujeres menores de 51 años

c) Sin metástasis a distancia

d) Cáncer intratiroideo menor de 5 cm

e) Sólo a y b

CC117. A un paciente de 40 años de edad con tumor intratiroideo bien diferenciado de 6 cm y un ganglio de
4 cm homolateral, se le clasifica:
a) T3 N2a M0

b) T3 N2b M0

c) T3 N1b M0

d) T2 N2a M0

e) T3 N1a M0

CC118. A un paciente de 65 años de edad con tumor intratiroideo bien diferenciado de 3 cm y ganglios
bilaterales, se le clasifica:

a) T2 N1b M0

b) T2 N2c M0

c) T3 N2c M0

d) T2 N1a M0

e) T2 N2b M0

CC119. Son subsitios de la región supraglótica, excepto:

a) Epiglotis

b) Comisura anterior

c) Aritenoides

d) Pliegues aritenoepiglóticos

e) Vestíbulo

CC120. A un paciente de 70 años con un tumor supraglótico que invade la pared medial del seno piriforme,
con un ganglio de 2 cm de diámetro homolateral y sin metástasis a distancia, se le clasifica:

a) T3 N1 M0

b) T4 N2a M0

c) T3 N2a M0

d) T4 N2b M0

e) T4 N1 M0
CC121. A un paciente con tumor glótico con extensión a la supraglotis y ganglios bilaterales menores de 3
cm, sin fijar la cuerda vocal y sin metástasis a distancia, se le clasifica:

a) T4 N2c M0

b) T3 N2c M0

c) T2 N2c M0

d) T4 N2b M0

e) T3 N2b M0

CC122. Tumor en nasofaringe, en un paciente de 60 años de edad, que invade senos paranasales, sin
adenomegalias cervicales y sin metástasis a distancia, se clasifica:

a) T2 N0 M0

b) T3 N0 M0

c) T4 N0 M0

d) T1 N0 M0

e) T2b N0 M0

CC123. En pacientes con cáncer de tiroides diferenciados, con residual microscópico en el gamagrama
tiroideo posoperatorio, la dosis ablativa de Iodo131 por administrar es de:

a) 50 milicuries

b) 150 milicuries

c) 400 milicuries

d) 450 milicuries

e) 600 milicuries

CC124. Fue el primero en describir la técnica clásica de la disección radical de cuello:

a) Hayes Martin

b) George Crile

c) Rovieres

d) Umberto Veronesi
e) Halsted

CC125. El grupo ganglionar V en cuello corresponde al grupo de ganglios:

a) Yugular superior

b) Yugular inferior

c) Triángulo posterior

d) Compartimento central

e) Yugular medio

CC126. En la disección selectiva supraomohioidea de cuello se resecan los siguientes niveles ganglionales,
excepto:

a) I

b) II

c) III

d) IV

e) Ninguna de las anteriores

CC127. En la clasificación TNM de tumores de piel de cabeza y cuello, el T3 corresponde a:

a) Tumor de diámetro máximo o menor o igual a 2 cm

b) Tumor de diámetro máximo o menor o igual a 3 cm

c) Tumor que invade cartílago, músculo o hueso

d) Tumor de diámetro máximo mayor de 5 cm

e) Ninguna de las anteriores

CC128. En tumores de cavidad bucal se considera el factor pronóstico más importante:

a) Edad menor de 40 años

b) Metástasis ganglionares cervicales

c) Tumor con espesor mayor de 4 mm

d) Infiltración tumoral perineural y vascular


e) Tumor residual en bordes o lecho quirúrgico

CC129. Es el tumor maligno más frecuente en la glándula parótida:

a) Carcinoma exadenoma pleomórfico

b) Carcinoma mucoepidermoide

c) Carcinoma adenoideoquístico

d) Carcinoma de células acinares

e) Linfomas

CC130. Los siguientes tumores dan metástasis en el 60% de los casos a los ganglios prelaríngeos,
pretraqueales y paratraqueales:

a) Subglóticos

b) Supraglóticos

c) Glóticos

d) Sólo a y b

e) Ninguno de los anteriores

CC131. Los tumores de las glándulas salivales con mayor riesgo de dar metástasis ganglionares son:

a) Indiferenciados

b) Mucoepidermoide

c) Adenoideoquístico

d) Carcinomas papilares

e) Carcinomas acinares

CC132. En los campos de radiación a cuello se incluyen las áreas ganglionares siguientes, excepto:

a) Submandíbulares

b) Yugulocarotideos

c) Espinales

d) Supraclaviculares
e) Occipitales

CC133. Los obturadores palatino se dividen en los tipos siguientes:

a) Quirúrgicos

b) Intermedios y transicionales

c) Definitivos

d) Sólo a y b

e) Todos los anteriores

CC134. Actualmente se estima que los sujetos podrán desarrollar voz esofágica satisfactoria en
poslaringectomía en el porciento siguiente:

a) 10

b) 30

c) 70

d) 90

e) Ninguna de las anteriores

CC135. Subsitio anatómico de la cavidad bucal que carece de glándulas salivales menores:

a) Paladar blando

b) Piso de la boca

c) Mucosa del carrillo

d) Región anterior del paladar duro

e) Región lateral del paladar duro

CC136. Glándula salival mayormente afectada por neoplasias:

a) Submaxilar

b) Sublingual

c) Parótida
d) Accesorio de paladar

e) Accesorio de faringe

CC137. Es con mucho el tumor maligno de nasofaringe más frecuente:

a) Linfoma

b) Nasoangiofibroma

c) Cáncer epidermoide

d) Melanoma

e) Linfoepitelioma

CC138. Ganglios de primer relevo en el drenaje linfático de la nasofaringe:

a) Nivel I

b) Nivel II

c) Nivel III

d) Retrofaringeos

e) Nivel IV

CC139. El sarcoma de cabeza y cuello más frecuente en la infancia es el:

a) Retinoblastoma

b) Rabdomiosarcoma

c) Sarcoma Ewing

d) Osteosarcoma

e) Angiosarcoma

CC140. Los niveles ganglionares que se afectan con mayor frecuencia en el carcinoma de laringe son:

a) Niveles I, II, y III

b) Niveles II y III únicamente

c) Niveles II, III y IV


d) Niveles II, III, IV y V

e) Niveles III, IV y V

CC141. Son características del carcinoma medular de tiroides, excepto:

a) No produce tiroglobulina

b) Da metástasis con frecuencia a ganglios linfáticos cervicales

c) No responde al manejo con I-131

d) Es una neoplasia maligna poco frecuente

e) Su diagnóstico citológico es dificil

CC142. Cáncer de tiroides que frecuentemente es bilateral, multicentrico y produce metástasis cervicales en
forma temprana es:

a) Cáncer anaplásico

b) Cáncer medular

c) Cáncer folicular

d) Cáncer papilar

e) Linfoma

CC143. El carcinoma de glándula tiroides anaplásico de células pequeñas no es considerado como tal
(anaplásico) por la siguiente razón:

a) Porque existe confusión con la tiroiditis de Hashimoto

b) Porque su evolución clínica es inconstante

c) Porque con esa morfología, en su mayoría son carcinomas medulares o linfomas

d) Porque aun con inmunohistoquímica no se puede diferenciar del carcinoma folicular agresivo

e) Porque se han reportado evoluciones clínicas distintas, compatibles con padecimientos


malignos pero bien diferenciados

CC144. Con respecto al cáncer de tiroides durante la infancia, en términos generales, sus características son
semejantes a las de adulto, excepto:

a) La gran mayoría son papilares

b) El porciento de multicentricidad es igual que en el adulto


c) En general, el pronóstico es peor que en el adulto

d) Las metástasis pulmonares se presentan en el 20% de los casos

e) Se utiliza la dosis ablativa de I-131 igual que en el adulto

CC145. El tratamiento supresivo de TSH en los carcinomas diferenciados de glándula tiroides es importante
no hacerlo hasta niveles indetectables de la hormona (TSH) por la razón siguiente:

a) Porque se ha demostrado que las determinaciones de TSH no detectables, dan el mismo


resultado que las de menor dosis de levotiroxina

b) Porque en la mayoría de los pacientes son dosis elevadas de tiroxina se necesitan utilizar
medicamentos beta bloqueadores, con sus efectos secundarios correspondientes

c) Porque no se han demostrado efectos positivos en el pronóstico (sobrevida total) de los


pacientes sometidos a suspresión de TSH con levotiroxina

d) Por los efectos secundarios gastrointestinales de las dosis elevadas de levotiroxina

e) Por la osteoporosis que se provoca con las dosis elevadas de levotiroxina

CC146. ¿Con cuál de los síndromes congénitos siguientes NO se asocia el cáncer medular?

a) Síndrome NEM I (neoplasias endócrinas múltiples)

b) Síndrome NEM II

c) Síndrome de COWDEN

d) Síndrome de Di GEORGE

e) Síndrome de GARDNER

CC147. El tratamiento con I-131 del carcinoma de tiroides bien diferenciado se usa con mucha frecuencia.
Debido a lo anterior debemos conocer que la posibilidad de desarrollar leucemia se presenta después de
utilizar una dosis acumulada de:

a) Alrededor de 2000 mCi

b) Alrededor de 1500 mCi

c) Alrededor de 800 mCi

d) Alrededor de 500 mCi

e) No se ha determinado la dosis con claridad


CC148. ¿Cuál de los factores siguientes no tiene significación epidemiológica en la presencia de un tumor
de cuerpo carotídeo?

a) Sexo

b) Edad

c) Altura sobre el nivel del mar

d) Alimentación

CC149. En cuál de las neoplasias siguientes se encuentra más establecido el rol del virus de Epstein Barr:

a) Carcinoma nasofaríngeo no queratinizante

b) Enfermedad de Hodking

c) Sarcoma de Kaposi

d) Linfoma de Burkitt

e) Leiomiosarcoma

CC150. La rama mandibular del nervio facial se localiza:

a) A 3 cm por debajo del ángulo de la mandíbula

b) Por detrás de los vasos faciales

c) Por delante de los vasos faciales, por debajo del reborde inferior de la mandíbula

d) Entre las fibras del músculo masetero

CC151. ¿Qué dosis de RTP se considera curativa en tumores de cabeza y cuello?

a) 4000 5000 rads

b) 5000 7000 rads

c) 7000 9000 rads

d) 3500 4000 rads

e) Más de 9000 rads

CC152. El sitio más frecuente de tumores en cabeza y cuello es:


a) Laringe

b) Cavidad bucal

c) Glándulas salivales

d) Labio

e) Hipofaringe

CC153. En cáncer de tiroides bien diferenciado en grupo de pacientes de alto riesgo, está indicado en el
posoperatorio de tiroidectomía total:

a) Rastreo gamagráfico con Iodo131 a las 3-6 semanas, sin ingerir hormonas

b) Rastreo gamagráfico con tecnecio a las 3 semanas. Sin ingerir hormonas

c) Rastreo gamagráfico con Iodo131 a las 3-6 semanas, ingiriendo hormonas tiroideas

d) Sólo a y b

e) Ninguna es correcta

CC154. La sustitución de hormonas tiroideas y dosis supresiva de TSH se hace con levotiroxina o
tetrayodotiroxina, la dosis requerida suele variar de:

a) 75-100 mg/día

b) 50-75 mg/día

c) 100-200 mg/día

d) 200-400 mg/día

e) Más de 400 mg/día

CC155. Más del 90% de todos los cánceres de cabeza y cuello corresponden a la variedad del:

a) Adenocarcinoma

b) Carcinoma de células basales

c) Melanoma

d) Carcinoma epidermoide

e) Sarcomas
CC156. En qué porciento de los varones jóvenes con cáncer de tiroides de tipo papilar se presentan
metástasis ganglionares subclínicas:

a) 20

b) 40

c) 5

d) 75

e) 100

CC157. En general, los tumores de las glándulas salivales tienden a invadir siguiendo la vía:

a) Hematógena

b) Local–extensión directa

c) Linfática

d) Neural

e) Ninguna de las anteriores

CC158. La resección segmentaria de mandíbula se puede dividir en los tipos siguientes, según el sitio de
defecto óseo, excepto:

a) Sinfisiaria

b) Lateral

c) Condilar

d) De la rama ascendente

e) Ninguna de las anteriores

CC159. La radioepitelitis aguda por radioterapia (25 Gy) generalmente se manifiesta en la semana:

a) 1ª

b) 2ª

c) 3ª

d) 4ª

e) 5ª
CC160. Fármaco que se utiliza como radiosensibilizador en el tratamiento de las neoplasias de cabeza y
cuello:

a) Adriamicina

b) Metrotexate

c) Gemcitabicina

d) 5 fluoracilo

e) Bleomicina

CC161. ¿Con qué frecuencia el carcinoma folicular de tiroides, presenta metástasis a distancia al momento
del diagnóstico?

a) Menos de 5%

b) 10 a 15%

c) 20 a 30%

d) 50%

e) 60%

CC162. Un carcinoma medular de tiroides de 4 cm de diámetro mayor y N0, ¿el tratamiento de elección es?

a) Hemitiroidectomía

b) Tiroidectomía total

c) Tiroidectomía total y disección central de cuello

d) Tiroidectomía total, disección central de cuello DRM bilateral

e) Tiroidectomía total, disección radical de cuello y DRC bilateral

CC163. De todos los cánceres de tiroides, ¿qué porciento representa al carcinoma medular?

a) 1

b) 5

c) 15

d) 20
e) 30

CC164. De los tumores malignos de la hipofaringe al carcinoma epidermoide le corresponde el:

a) 15%

b) 35%

c) 50%

d) 75%

e) 95%

CC165. ¿Por qué razón el carcinoma de tiroides bien diferenciado del adulto, empeora el pronóstico
directamente proporcional a la edad de los pacientes?

a) Por factores inmunológicos

b) Por cambios degenerativos en el tejido glandular tiroideo

c) Por combinación de factores hereditarios y ambientales

d) Por causas desconocidas

e) Por la acumulación de proteínas iodadas dentro de la tiroides

CC166. En el carcinoma bien diferenciado de tiroides en su variedad papilar son variantes más agresivas
biológicamente las abajo descritas, excepto:

a) Células altas

b) Columnar

c) Esclerosis difusa

d) Oxifílica

e) Encapsulada

CC167. En referencia al carcinoma de tiroides, en su variante medular, sabemos que la variedad esporádica
representa el porciento siguiente, en cuanto a su frecuencia:

a) 10

b) 30
c) 0

d) 75

e) 90

CC168. En el carcinoma medular de tiroides, están presentes las metástasis ganglionares al momento del
diagnóstico en:

a) 20%

b) 30%

c) 45%

d) 65-70%

e) 80%

CC169. En el carcinoma anaplásico de tiroides, en fecha reciente se dejó de reconocer como variedad
histológica, por presentar errores diagnósticos frecuentes, la variedad siguiente:

a) Sarcomatoide

b) De células fusiformes

c) De células gigantes

d) De células pequeñas

e) Escamoide

CC170. Tratando el melanoma y de acuerdo a su histogénesis, las lesiones cutáneas se dividen en las formas
descritas a continuación, excepto:

a) Melanoma nodular

b) Amelánico

c) Léntigo maligno

d) Acral lentiginoso

e) El de diseminación superficial

CC171. Hablando del melanoma cutáneo, cuál es el factor pronóstico más importante relacionado con el
primario:
a) Ulceración

b) Índice mitótico

c) Grosor de la lesión

d) Tipo tumoral

e) Infiltración tumoral linfocítica

CC172. Los siguientes son subsitios anatómicos de la cavidad bucal, excepto:

a) Labios

b) Paladar blando

c) Mucosa bucal

d) Trígono retromolar

e) Lengua

CC173. Lo siguiente es correcto con respecto al cáncer de lengua, excepto:

a) El cáncer epidermoide constituye el 90%

b) Es el subsitio más frecuente

c) Recurre localmente

d) Presencia de metástasis ganglionares subclínicas elevadas

e) Niveles ganglionares involucrados II-V

CC174. Los siguientes son factores de riesgo relacionados con el cáncer de cavidad bucal, excepto:

a) Deficiencia de zinc

b) Deficiencia de vitamina A y C

c) Tabaquismo

d) Alcohol

e) Infección por Treponema pallidum

CC175. Los niveles ganglionares que se afectan en el cáncer de trígono retromolar son:
a) I-V

b) I, II y III

c) II, III, IV

d) VI

e) I, II, III, IV, V

CC176. El subsitio anatómico más frecuente de cáncer en la cavidad bucal es:

a) Lengua

b) Piso de la boca

c) Mucosa bucal

d) Trígono retromolar

e) Labio

CC177. ¿Cuál es el porciento de metástasis cervicales en cáncer de labio inferior?

a) < 10

b) 15-20

c) 50

d) >50

e) 30

CC178. Porciento de metástasis cervicales en tumores de la comisura labial:

a) <10

b) 20

c) < 5

d) 50

e) 80

CC179. ¿En qué subsitio anatómico de la cavidad bucal son más frecuentes las metástasis ganglionares?
a) Comisura labial

b) Encía

c) Piso de la boca

d) Lengua

e) Trígono retromolar

CC180. Las aseveraciones siguientes son verdaderas con respecto tumores del piso de la boca, excepto:

a) T1 y T2 pueden ser tratados con radioterapia

b) Sobrevida a 5 años del 85% etapas

c) De los pacientes tienen metástasis ganglionares

d) La incidencia de metástasis a distancia es 30%

e) Manejo multidisciplinario en etapas avanzadas

CC181. El tratamiento del cáncer de cavidad bucal localmente avanzado es:

a) Quirúrgico

b) Inmunoterapia

c) Braquiterapia

d) Multidisciplinario

e) Radioterapia

CC182. En los tumores de piso de la boca la disección electiva de cuello está indicada porque:

a) Mejora el control locorregional de la enfermedad

b) Alta incidencia de metástasis ocultas

c) Indicada en lesiones tempranas

d) Mejora la sobrevida

e) Debe realizarse en lesiones que tiene una profundidad > 4 mm

CC183. El subtipo histológico más frecuente de las glándulas salivales menores es:
a) Mucoepidermoide

b) Carcinoma adenoideo quístico

c) Epidermoide

d) Adenocarcinoma

e) Carcinoma de células acinares

CC184. El subtipo histológico más frecuente de las glándulas salivales mayores es:

a) Tumor maligno mixto

b) Adenocarcinoma

c) Epidermoide

d) Indiferenciado

e) Mucoepidermoide

CC185. Las aseveraciones siguientes son correctas con respecto al tumor maligno mixto, excepto:

a) Representa 3-12%

b) Ocurre más frecuente en glándulas salivales menores

c) Está asociado a recurrencias de tumores benignos mixtos

d) Su diagnóstico es difícil

e) Tienen un pronóstico pobre

CC186. Los niveles ganglionares afectados en el cáncer de parótida son:

a) I – II – III – IV - V

b) I y II

c) II,III y IV

d) I,II y III

e) I

CC187. Son factores de riesgo de metástasis ganglionares para cáncer de glándulas salivales, excepto:
a) Tamaño tumoral

b) Grado histológico

c) Subtipo histológico

d) Grado de diferenciación

e) Invasión perineural

CC188. La radioterapia posoperatoria está indicada en cáncer de glándulas salivales, excepto:

a) Invasión a partes blandas

b) Bordes positivos

c) Ganglios positivos

d) Tumor de alto grado

e) Invasión al espacio parafaríngeo

CC189. Los siguientes son factores de mal pronóstico en el cáncer de glándula salival, excepto:

a) T3-4

b) Grado histológico

c) Estado ganglionar

d) Invasión al lóbulo profundo

e) Invasión neural

CC190. Las siguientes aseveraciones son verdaderas en el cáncer de glándulas salivales, excepto:

a) La supervivencia no depende del estadio clínico:

b) El riesgo de recurrencia es mayor en caso de carcinoma de glándulas salivales accesorias

c) El grado histológico afecta la supervivencia

d) El subtipo mucoepidermoide es más frecuente en las lesiones de glándulas mayores

e) La radioterapia como tratamiento primario, está indicado cuando hay contraindicación


quirúrgica
CC191. Métodos auxiliares de diagnóstico indispensables para evaluación en tumores avanzados de
glándulas salivales:

a) TAC

b) Gamagrama

c) TAC y BAAF

d) Resonancia magnética

e) US Y BAAF

CC192. ¿Cuál en el tratamiento del carcinoma adenoideo quístico de glándulas parótidas de 4 cm?

a) Parotidectomía total

b) Parotidectomía total más radioterapia

c) Parotidectomía superficial

d) Radioterapia

e) Quimioterapia

CC193. Son subtipos histológicos del tumor maligno mixto los siguientes, excepto:

a) Carcinoma ex adenoma pleomórfico

b) Carcinosarcoma

c) Tumor mixto metastásico

d) Carcinoma no-invasivo

e) Cribiforme

CC194. Son patrones histológicos del carcinoma adenoideoquístico:

a) Tubular, sólido, cribiforme

b) Cribiforme, sólido y carcinosarcoma

c) Bajo grado, grado intermedio y alto grado

d) Papilar, mucinoso, trabecular

e) Trabecular, cribiforme, sólido


CC195. Mencione el factor con mayor riesgo del cáncer folicular de tiroides, para desarrollo de metástasis:

a) Tamaño del primario

b) Invasión capsular

c) Invasión vascular

d) Invasión extratiroidea

e) Invasión neural

CC196. ¿Qué condición patológica previa tendrá mayor riesgo de desarrollar malignidad?

a) Bocio multinodular no tóxico

b) Nódulo no tóxico

c) Bocio multinodular tóxico

d) Nódulo tóxico

e) Ninguna de las anteriores

Relaciona las columnas siguientes, clasificación de grupos de riesgo y factores pronósticos a cáncer de
tiroides:

CC197. AMES (a) a) LAHEY

CC198. MACIS (d) b) MAZAFERRI

CC199. OHIO STATE (b) c) DEGROOT

CC200. CHICAGO ( c ) d) MAYO

CC201. ¿Qué tipo de disección de cuello estaría indicada en cáncer diferenciado de tiroides?

a) Electiva

b) Terapéutica

c) Radical clásica

d) Supraomohioidea

e) Radical modificada

CC202. De los tipos de cáncer medular de tiroides, ¿Cuál es el de peor pronóstico?

a) Esporádico
b) NEM Ia

c) NEM IIb

d) Familiar

e) Ninguno

CC203. Marcadores tumorales para el cáncer medular de tiroides:

a) Calcitonina y antígeno carcinoembrionario

b) Antígeno carcinoembrionario y alfafetoproteína

c) Calcitonina y alfafetoproteína

d) Gonadotropina

e) RET

CC204. Subsitio de la laringe con mayor riesgo de metástasis ganglionares:

a) Supraglotis

b) Glotis

c) Subglotis

d) Epiglótico

e) Espacio glótico

CC205. Lesión premaligna con mayor riesgo de cáncer de laringe:

a) Eritroplasia

b) Leucoplasia

c) Displasia

d) Hiperplasia

e) Metaplasia

CC206. Paciente con diagnótico de cáncer de laringe, y adenomegalias metastásicos múltiples homolaterales
menores de 6 cm:

a) N1
b) N2a

c) N2b

d) N2c

e) N3

CC207. ¿A qué tipo corresponde la disección modificada de cuello, en que se preserva nervio espinal y vena
yugular interna?

a) I

b) II

c) III

d) IV

e) V

CC208. La disección selectiva de cuello posterolateral incluye los grupos ganglionares siguientes:

a) II y III

b) III y IV

c) III, IV y V

d) II, III, IV y V

e) II, III, IV, V y VI

CC209. En los carcinomas de laringe ¿cuáles niveles ganglionares se encuentran más frecuentemente
afectados?

a) II y III

b) III y IV

c) III, IV y V

d) II, III, IV y V

e) II, III, IV, V y VI


CC210. A la dosis por fracción, menor que la convencional, dada dos veces al día para obtener una dosis
mayor en el tiempo total del tratamiento estándar, se le llama:

a) Fraccionamiento acelerado

b) Hiperfraccionamiento

c) Fraccionamiento convencional

d) Hiperfraccionada continua

e) Radioterapia acelerada

CC211. Dos fracciones por día, dosis mayor por fracción, llegando a una dosis total igual que al tratamiento
estándar:

a) Fraccionamiento acelerado

b) Hiperfraccionamiento

c) Fraccionamiento convencional

d) Hiperfraccionada continua

e) Radioterapia acelerada

CC212. Paciente con cáncer glótico, alterada la movilidad de la cuerda vocal y adenomegalia ipsilateral
menor de 3 cm ¿a qué etapa corresponde?

a) II

b) III

c) IV

d) IV a

e) IV b

CC213. Dentro de las lesiones premalignas ¿cuál presenta mayor riesgo de transformación maligna en
cáncer de cavidad bucal?

a) Leucoplasia

b) Eritroplasia

c) Displasia

d) Hiperplasia verrucosa
e) Metaplasia

CC214. De las aseveraciones siguientes en cáncer de cavidad bucal, en relación a las metástasis es falsa:

a) Factor más importante

b) Disminuye curación en el 50%

c) Metástasis ocultas en el 50%

d) Ganglios palpables

e) Niveles ganglionares afectados I, II y III

CC215. Paciente con tumor en lengua de 1.5 cm de diámetro y adenomegalia ipsilateral de 4 cm, que etapa
clínica es:

a) I

b) II

c) III

d) IIIA

e) IV A

CC216. Paciente con tumor de 3 cm en piso de boca y adenomegalia menor de 3 cm, ¿A qué etapa
corresponde?

a) I

b) II

c) III

d) IVa

e) IVb

CC217. Indicaciones de disección electivo ganglionar de cuello, en cáncer de lengua bucal, excepto:

a) Tumor > 2 cm

b) Tumor < 1 cm

c) Infiltración a piso de boca


d) Tumor < 2 cm mal diferenciados

e) Espesor > 2 mm

CC218. Son los tres tipos de cáncer más comunes del pabellón auricular:

a) Carcinoma basocelular, sarcoma, carcinoma epidermoide

b) Carcinoma epidermoide, micosis fingoide, adenocarcinoma

c) Carcinoma epidermoide, melanoma, carcinoma basocelular

d) Melanoma, carcinoma basocelular, carcinoma mucoepidermoide

e) Carcinoma basocelular, micosis fungoide, carcinoma epidermoide

CC219. Respecto al papiloma nasal invertido todo es cierto, excepto:

a) Usualmente es bilateral y causa obstrucción nasal

b) Histológicamente es una lesión benigna

c) Afecta primariamente la pared lateral

d) El 13% tiene una transformación de carcinoma epidermoide

e) El seno etmoidal es uno de los lugares en que más frecuentemente se presentan

Relacione cada seno paranasal con el lugar a donde drena:

CC220. Seno frontal ( ) a) Infundíbulo etmoideo del meato medio


CC221. Seno esfenoidal ( ) b) Dentro del meato superior
CC222. Seno maxilar ( ) c) Porción anterior del meato medio
CC223. Celdas etmoidales posteriores ( ) d) Receso arriba de la concha superior

CC224. Respecto al mucocele todo es cierto, excepto:

a) No son peligrosos ya que no invaden estructuras adyacentes

b) Ocurre como una complicación de la sinusitis crónica

c) Se pueden clasificar en primarios y secundarios

d) Pueden romper hacia el cráneo con resultados catastróficos

e) Es una masa expansiva de epitelio cuboidal mucoide


CC225. El abordaje para una etmoidectomía se realiza por vía:

a) Transfacial, transesfenoidal, transpalatina

b) Transantral, transesfenoidal, transfacial

c) Transnasal, transmandibular, transantral

d) Transfacial, transpalatina, transnasal

e) Transantral, transnasal, transfacial

CC226. ¿Qué proporción de lesiones blancas de cavidad oral son malignas?

a) Menos de 0.1%

b) Entre 1 y 2%

c) 10%

d) Más de 40%

e) 80%

Correlacione las ramas extratemporales del VII par craneal con la acción que realizan:

CC227. Elevar la ceja ( ) a) Cervical


CC228. Silvar ( ) b) Mandibular
CC229. Contracción de músculos del cuello ( c) Bucal
)
CC230. Sonreir ( ) d) Cigomático
CC231. Apretar los ojos ( ) e) Temporal

CC232. El hiperparatiroidismo causado por cáncer de la paratiroides representa el:

a) Menos del 5%

b) 10%

c) 15%

d) 20%

e) 40%

Acerca de la glándula submandibular escriba falso o verdadero según corresponda:


CC233. El nervio lingual es superficial a la glándula ( )

CC234. La bulimia puede causar crecimiento de la glándula ( )

CC235. El nervio hipogloso es profundo e inferior a la glándula ( )

CC236. En el alcoholismo nunca puede crecer la glándula ( )

CC237. Una estenosis del conducto o un calculo pueden hacer que la glándula aumente de
tamaño ( )

CC238. Durante una parotidectomía las referencias para localizar el nervio facial son todas las siguientes,
excepto:

a) Justo posterior al canal auditivo externo

b) En el agujero estiolomastoideo

c) A 6 mm medial a la sutura timpanomastoidea

d) Profundo respecto a la vena facial posterior

e) Las rama mandibular cruza sobre la arteria facial en el borde anterior del macetero

CC239. La cirugía adecuada para un tumor mixto benigno de parótida es:

a) Biopsia incisional para confirmar el diagnóstico

b) Enucleación

c) Parotidectomía superficial

d) Parotidectomía total

e) Parotidectomía radical

CC240. Las indicaciones de radioterapia en glándula parótida son todas, excepto:

a) Extensión extraparotidea

b) Después de biopsia incisional que demuestra un carcinoma

c) Tumores de alto grado

d) Afección linfática documentada

e) Tumores que afectan el nervio facial


CC241. En los tumores malignos de glándulas salivales mayores ¿en qué proporción existe metástasis no
palpables a los ganglios del cuello?

a) Menos del 15%

b) 20%

c) 30%

d) 40%

e) 45%

CC242. Respecto a la sialometaplasia necrotizante todo es cierto, excepto:

a) Es una necrosis coagulativa

b) Afecta glándulas salivales menores

c) Se encuentra metaplasia escamosa prominente en los acinos y en los conductos

d) Metastatiza rápidamente

e) Presenta hiperplasia seudoepiteliomatosa

Escriba cierto o falso según corresponda

Subsitio anatómico Porciento de metástasis


ganglionar ocultas en cuello

CC243. Lengua móvil ( ) 34%


CC244. Piso de boca ( ) 30%
CC245. Mucosa bucal ( ) 9%
CC246. Seno piriforme ( ) 38%
CC247. Faringe posterior ( ) 30%

CC248. El origen del fibrosarcoma ameloblástico es:

a) Odontogénico

b) No odontogénico

c) Inflamatorio

d) Infeccioso

e) Desconocido
CC249. Respecto al ameloblastoma maligno es cierto que:

a) Es igual que el carcinoma ameloblástico

b) Las células que metastatizan a pulmón, son citológicamente malignas

c) Su pronóstico es bueno

d) Las células del tumor primario son citológicamente malignas

e) Las células que metastatizan a ganglios linfáticos son citológicamente benignas

CC250. Del fibroma osificante todo es cierto, excepto:

a) Presenta un crecimiento lento

b) Histológicamente hay estroma de colágeno y depósito cementoide

c) Es más frecuente en la segunda década de la vida

d) Radiológicamente puede ser radiopaco o radiolúcido

e) Es un tumor no odontogénico

Relacione el tipo de cirugía con el tumor:

CC251. Fibroma ameloblástico ( a) Resección compuesta


)
CC252. Ameloblastoma b) Resección marginal
( )
CC253. Fibrosarcoma ameloblástico ( c) Enucleación
)
CC254. Mixoma odontogénico (
)
CC255. Fibroodontoma
( )

CC256 Qué subtipos de papiloma virus en pólipos laríngeos y nasales se asocian con degeneración maligna:

a) 6, 11

b) 16, 18

c) 6, 16

d) 11, 18

e) 6, 11, 16, 18
CC257. El carcinoma adenoideo quístico se presenta en senos paranasales en:

a) 0%

b) 10%

c) 25%

d) 50%

e) 100%

CC258. Los tumores de senos paranasales en general dan metástasis a distancia en:

a) 10%

b) 25%

c) 50%

d) 75%

e) 100%

Relacione el subsitio del tumor laríngeo con las características

CC259. Representa el 67% del total de tumores de esta región a)


( ) Supraglótico
CC260. Representa el 31 % del total de tumores de esta región b) Glótico
( )
CC261. Tiene poco drenaje linfático ( ) c) Subglótico
CC262. Se extiende a través del cartílago cricoides
( )
CC263. Se detecta de manera temprana por el cambio del tono de
voz ( )

CC264. Cuando el nervio laríngeo recurrente es lesionado:

a) Existe pérdida motora de los músculos extrínsecos de la laringe ipsilateral

b) La cuerda local se posiciona lateralmente

c) El músculo cricotiroides aduce la cuerda vocal

d) Hay hipertonía del cricotiroideo (único músculo extrínseco de la laringe)

e) Existe pérdida de la función sensitiva de la supraglotis

CC265. La sobrevida del cáncer laríngeo por subsitio y estadio T es correcta para todos, excepto:
a) T1 supraglótico 75% con radioterpia sola

b) T3 supraglótico 52% con radioterapia sola

c) T1 glótico 95% con cualquier modalidad de tratamiento (radioterapia o cirugía)

d) T3 glótico 70% con cirugía sola

e) T1 subglótico 70% con terapia combinada

CC266. Un paciente con cáncer laríngeo desarrollará un carcinoma pulmonar en un:

a) 0 a 1%

b) 5 a 10%

c) 20 a 25%

d) 50 a 60%

e) 100%

CC267. Todas son contraindicaciones para realizar una laringeotomía horizontal supraglótica, excepto:

a) Lesiones mayores de 3 cm

b) Movilidad cordal afectada

c) Extensión a fascia prevertebral

d) Ganglio cervical único palpable

e) Afección del aritenoides uni o bilateral

CC268. Un carcinoma de células escamosas de la tráquea tratado con resección y radioterapia posoperatoria,
tiene una sobrevida a 5 años del:

a) 5%

b) 25%

c) 50%

d) 80%

e) 95%
CC269. Respecto al adenocarcinoma folicular todo es cierto, excepto

a) Disemina por vía linfática, preferentemente

b) Microscópicamente parece invación a la cápsula y vasos

c) Representa el 10% de los cánceres tiroideos

d) Tiene un pronóstico peor que el adenocarcinoma papilar

e) Las metástasis pueden ser diagnosticadas por yodo radiactivo

CC270. Respecto al carcinoma de células de Hurthle todo es cierto, excepto:

a) Es un subtipo de adenocarcinoma folicular

b) Se presenta más frecuentemente en jóvenes

c) Tiene un curso más agresivo

d) No se pueden identificar metástasis por yodo radiactivo

e) Las células tienen citoplasma eosinófilo

CC271. Respecto al carcinoma medular de tiroides todo es cierto, excepto:

a) Representa el 2 a 5% de todos los cánceres tiroideos

b) Nace de las células foliculares del toroides que secretan calcitonina

c) La calcitonina se puede utilizar para identificar este tipo de cáncer

d) Presentan frecuencia elevada de metástasis ganglionares en cuello

e) El antígeno carcinoembrionario y la calcitonina sérica pueden ser útiles en el diagnóstico y


seguimiento del paciente

CC272. Todo es cierto respecto al higroma quístico, excepto:

a) Es conocido como linfangioma quístico

b) Se presenta más frecuentemente entre los 15 a 20 años

c) En su forma común es suave a la palpación

d) Con resecciones parciales existe 50% de recurrencia

e) El único tratamiento exitoso se da con la resección completa


CC273. Todo es cierto acerca del tumor de cuerpo carotídeo, excepto que:

a) 10% son malignos

b) 10% secretan catecolaminas

c) 1% son multicéntricos

d) Nacen de nervios parasimpáticos

e) Se conocen como chemodectomas

CC274. En el síndrome nevobasocelular o de Gorlin’s todo es cierto excepto:

a) Es una condición genética autosómica recesiva

b) Ocasiona quistes en la mandíbula

c) Presentan múltiples carcinomas basocelulares entre la pubertad y los 30 años

d) El número de lesiones puede ser de cientos

e) El examen completo del cuerpo debe realizarse cada 3 a 6 meses

Correlacione el subtipo de melanoma con las características clínicas

CC275. Se presenta predominantemente en a) Melanoma de diseminación superficial


ancianos y areas expuestas al sol ( )
CC276. Es más frecuente en el dorso del tronco y b) Melanoma nodular
mide más de 1 cm ( )
CC277. Nódulo negro azulado con piel normal c) Lentigio maligno
alrededor ( )
CC278. Ocurre más frecuentemente en los pies d) Acral lentiginoso
(d)
CC279. Inicialmente se aprecia como mácula que
incrementa su pigmentación ( )

Conteste falso o verdadero respecto a la radioterapia en cabeza y cuello:

CC280. La radioterapia puede ser un tratamiento definitivo único en tumores en estadio temprano de
glándulas salivales mayores.

a) Verdadero b) Falso

CC281. La presencia de extensión extraganglionar del tumor es una indicación de radioterapia.

a) Verdadero b) Falso

CC282. Los pacientes con enfermedad de la colágena no tienen contraindicación para recibir radioterapia.
a) Verdadero b) Falso

CC283. La xerostomía se presenta con dosis de 250 a 500 cGy a las glándulas salivales

a) Verdadero b) Falso

CC284. La incidencia de osteonecrosis puede ser grandemente disminuida por un meticuloso cuidado
dental, antes durante y después de la radioterapia.

a) Verdadero b) Falso

CC285. La unión de capilares entre injerto y lecho receptor se presenta a los:

a) 0 a 2 días

b) 3 a 5 días

c) 8 a 10 días

d) 14 a 16 días

e) 19 a 21 días

CC286. En pacientes que requieren de apoyo ventilatorio se debe realizar traqueostomía cuando se espere un
período de apoyo mayor a:

a) 5 días

b) 15 días

c) 30 días

d) 60 días

e) 90 días

CC287. Cuando hablamos de la imagen de resonancia magnética en cabeza y cuello todo es cierto, excepto:

a) Es forzoso usar gadolinio para poder demostrar arterias y venas

b) Está contraindicada en pacientes con marcapasos

c) No delinea la cortical del hueso con tan buen detalle como la tomografía axial computada

d) No se administra radiación ionizante con el estadio


e) No delinea el hueso trabecular tan detalladamente como la tomografia axial computada

CC288. ¿Qué estructura no se encuentra en el espacio parafaríngeo en forma normal?

a) Grasa

b) Lóbulo profundo de la parótida

c) Rama mandibular del nervio facial

d) Plexo venoso pterigoideo

e) Ramas de la arteria faríngea ascendente

CC289. Cuando se infla un globo intratraqueal en una intubación ¿cuál es la presión ideal a que debe estar
para no afectar la perfusión capilar de la mucosa y evitar isquemia y estenosis futuras?

a) Menos de 10 mmHg

b) Menos de 25 mmHg

c) Menos de 50 mmHg

d) Menos de 70 mmHg

e) Menos de 100 mmHg

CC290. En un paciente sin historia de enfermedad pulmonar obstructiva crónica se indica apoyo ventilatorio
cuando la pCO2 es mayor de:

a) 25 mmHg

b) 50 mmHg

c) 75 mmHg

d) 100 mmHg

e) 150 mmHg

CC291. El cáncer de cavidad bucal se presenta con mayor frecuencia en:

a) Paladar blando

b) Mucosa bucal

c) Piso de boca

d) Encía
e) Lengua

CC292. La evaluación más precisa de invasión mandibular en caso de Ca de piso de boca es:

a) Clínicoquirúrgica

b) Clínicoquirúrgica más ortopantomografía más TAC con Dentascan

c) Con ortopantomografía

d) Con TAC con dentascan

e) Con gamagrafía ósea

CC293. El trismus en el cáncer de piso de boca indica:

a) Invasión al cuello

b) Invasión a mandíbula

c) Gran volumen tumoral

d) Invasión a base de lengua

e) Invasión al músculo pterigoideo interno

CC294. Las lesiones que con mayor frecuencia se asocian a carcinoma en cavidad bucal son:

a) Herpes simple

b) Leucoplasia verrugosa

c) Eritroplasia

d) Papilomas

e) Leucoplaquia plana

CC295. Las zonas de cavidad bucal que se asocian con más frecuencia a eritroplaquia en alcoholismo
intenso son:

a) Paladar blando

b) Piso de boca y cara ventral de lengua

c) Encías y carrillo

d) Mucosa bucal y labios

e) Lengua
CC296. De los factores siguientes ¿cuál es el que con mayor frecuencia se correlaciona con metástasis
ganglionares?

a) Diferenciación

b) Tamaño tumoral

c) Ulceración

d) Profundidad de invasión

e) Invasión perineural

CC297. De los siguientes subsitios de cáncer de cavidad bucal ¿cuál es el que tiene menor frecuencia de
metástasis?

a) Piso de boca

b) Mucosa bucal

c) Encía

d) Paladar duro

e) Trígono retromolar

CC298. La invasión mandibular en cáncer de piso de boca no radiado puede ser por vía:

a) Linfática y vascular

b) Neural o a través del alveolo dentario

c) A través del periostio y del alveolo dentario

d) Neural y periostial

e) Linfática

CC299. Según la frecuencia de metástasis ganglionares la disección selectiva del cuello está indicada en
carcinomas T1 y T2 de:

a) Paladar y trígono retromolar

b) Encía y mucosa bucal

c) Labio y lengua

d) Piso de boca y lengua


e) Piso de boca

CC300. La disección selectiva del cuello para manejo de cáncer de la lengua bebe incluir los niveles:

a) I, II

b) I, II, III, IV

c) I

d) III, IV, V

e) I, II, III

CC301. El cáncer de piso de boca se maneja en tumores T1 y T2 preferentemente con cirugía sobre
radioterapia por su:

a) Alta frecuencia de metástasis ganglionares

b) Mayor rapidez del tratamiento

c) Menor morbilidad de cirugía en pacientes físicamente aptos

d) Menor índice terapéutico de radioterapia

e) Menor costo de cirugía

CC302. El cáncer avanzado de piso de boca con invasión mandibular, irradiado, debe ser manejado
idealmente con:

a) Resección tumoral y mandibulectomía segmentaria

b) Resección tumoral y hemimandibulectomía

c) Resección tumoral y mandibulectomía marginal

d) Resección tumoral y reirradiación

e) Resección tumoral y mandibulectomía total

CC303. El tratamiento del cáncer avanzado de (T3–T4) cavidad bucal es:

a) Radioterapia, preferentemente

b) Cirugía preferentemente

c) Cirugía y radioterapia posoperatoria


d) Radioterapia seguida de cirugía

e) Quimioterapia y radioterapia

CC304. En la disección selectiva del cuello es válido el concepto siguiente:

a) Tiene un valor terapéutico definido

b) Debe utilizarse aun en presencia de metástasis linfáticas clínicas

c) Debe utilizarse como procedimiento pronóstico y con estudio transoperatorio

d) Debe utilizarse en pacientes sin metástasis linfáticas y tiene un valor terapéutico definido

e) Es un procedimiento pronóstico y siempre debe ser seguido de radioterapia posoperatoria

CC305. La reconstrucción ideal de piso de boca se realiza con:

a) Colgajos nasolabiales

b) Colgajo microvascular de radial

c) Colgajo del pectoral mayor

d) Colgajo del trapecio

e) Colgajo de la lengua

CC306. La mandibulectomía marginal que ofrece mayor resistencia a las fracturas es:

a) Sagital

b) Oblicua

c) Horizontal

d) Vertical

e) Inferior

CC307. El carcinoma de piso de boca que invade base de lengua se reseca de manera óptima con:

a) Mandibulotomía lateral

b) Mandibulotomía media
c) Glosomandibulotomía

d) Vía transbucal

e) Colgajo de mejilla

CC308 Está indicada la radioterapia posterior a disección selectiva como parte del manejo de cáncer de
lengua en:

a) Caso de metástasis ganglionar determinada por estudio patológico

b) Sólo en caso de >3 metástasis ganglionares patológicas

c) Sólo en caso de existir ruptura ganglionar

d) En caso de tumor primario ulcerado

e) En caso de tumor primario exofítico

CC309. La variedad histológica más frecuente de cáncer de glándulas salivales menores en cavidad bucal es:

a) Adenoma pleomorfo

b) Carcinoma ex adenoma pleomorfo

c) Carcinoma adenoideo quístico

d) Adenocarcinoma ductal

e) Carcinoma mucoepidermoide

CC310. Los márgenes histológicos más aceptados en cáncer de lengua y piso de boca son:

a) 5 mm

b) 10 mm con palpación digital durante resección para determinación de extensión tumoral

c) 20 mm en sentido superficial

d) 3 mm

e) 15 mm

CC311. La leucoplaquia bucal se asocia con carcinoma en:

a) Más de 90% de los casos


b) Alrededor de 70% de los casos

c) En aproximadamente 10% de los casos

d) En menos del 1% de los casos

e) No se asocia con carcinoma

CC312. El subsitio que se asocia con más frecuencia a metástasis cervicales en cavidad bucal es:

a) Paladar

b) Piso de boca

c) Lengua

d) Carrillo

e) Labio

CC313. Sitio más frecuente de presentación de melanoma de cavidad bucal:

a) Lengua

b) Piso de boca

c) Paladar

d) Encía

e) Labio

CC314. La(s) vía(s) de diseminación más frecuente(s) en cáncer adenoideo quístico de piso de boca son:

a) Linfática

b) Vascular

c) Perineural

d) Linfática y perineural

e) Infiltración local y perineural

CC315. La disección selectiva del cuello se debe realizar en cáncer de piso de boca incluyendo los niveles:

a) I
b) I, II, III

c) I, II, III, IV

d) I, II

e) II, III, IV, V

CC316. Epitelio de recubrimiento interno del vestíbulo nasal:

a) Respiratorio: columnar ciliado

b) Estratificado escamoso con anexos

c) Plano estratificado queratinizante

d) Olfatorio: alto, seudoestratificado y columnar con cilios especializados

e) Transicional

CC317. Los factores etiológicos más frecuentemente implicados en el desarrollo de carcinomas sinunasales
son:

a) Factores ambientales como radiación ultravioleta y gas radón

b) Factores ocupacionales como la exposición a níquel, cromo, aceite isopropílico, hidrocarburos


volátiles

c) Tabaquismo y alcoholismo

d) Virus del papiloma humano 16 y 18

e) Factores dietéticos como alimentos salados y deficiencia de vegetales

CC318. Tumor maligno más frecuente en las fosas nasales y senos paranasales:

a) Carcinoma adenoideoquístico

b) Linfoma

c) Estesioneuroblastoma

d) Melanoma

e) Carcinoma epidermoide

CC319. El sitio más frecuentemente involucrado por los tumores malignos de los senos paranasales y las
fosas nasales es:
a) Seno frontal

b) Seno esfenoidal

c) Seno etmoidal

d) Seno maxilar

e) Cavidad nasal

CC320. La manera de diseminación predominante del carcinoma epidermoide del antro maxilar es:

a) Extensión directa

b) Diseminación linfática

c) Vía hematógena

d) Diseminación perineural

e) Ninguna de las anteriores

CC321. La vía de diseminación típica de un tumor en la fosa nasal hacia la fosa craneal anterior es a través
de:

a) La lámina papirácea

b) El forámen esfeno palatino

c) La lámina cribosa

d) La hendidura esfenoidal superior

e) La hendidura esfenoidal inferior

CC322. El estudio de imagen inicial que proporciona más información cuando se sospecha un cáncer de los
senos paranasales es:

a) Las placas simples de Caldwell y Watters

b) La tomografía computada

c) La imagen por resonancia magnética

d) El gammagrama óseo

e) La tomografía por emisión de positrones


CC323. El tratamiento de elección de los tumores de senos paranasales en etapas III y IV resecables es:

a) Radioterapia radical

b) Quimioterapia de inducción seguido de radioterapia

c) Quimioterapia y radioterapia concomitantes

d) Cirugía exclusivamente

e) Cirugía radical y radioterapia adyuvante

CC324. Es indicador de irresecabilidad, en el caso de los carcinomas sinunasales:

a) Invasión extensa de la base del cráneo

b) Invasión del parénquima cerebral

c) Invasión del seno cavernoso o ambas carótidas

d) Invasión de ambas órbitas

e) Todos los anteriores

CC325. Un carcinoma del antro maxilar que invade el contenido de la órbita y más allá del piso de ésta o de
su pared medial, se clasifica como:

a) T1

b) T2

c) T3

d) T4

e) Ninguno de los anteriores

CC326. La porción posestiloidea del espacio parafaríngeo contiene los elementos anatómicos siguientes:

a) Tejido conectivo laxo y adiposo

b) El plexo venoso pterigoideo

c) La porción profunda de la glándula parótida

d) El contenido de la vaina carotídea y pares craneales IX al XII

e) Los músculos prevertebrales y la fascia prevertebral


CC327. La asimetría estática de los labios debido a parálisis o paresia de los músculos depresores del labio
inferior se debe a lesión de la rama del nervio facial siguiente:

a) Temporal

b) Cigomática

c) Bucal

d) Marginal o mandibular

e) Cervical

CC328. Estudio más importante en el diseño del tratamiento subsecuente de las neoplasias de la glándula
parótida:

a) Biopsia por aspiración con aguja delgada

b) Tomografía computada

c) Imagen por resonancia magnética

d) Sialografía

e) Estudio histopatológico transoperatorio del producto de una parotidectomía

CC329. El tumor más frecuente de las glándulas salivales es el:

a) Oncocitoma

b) Adenoma pleomorfo

c) Tumor de Warthin (cistadenoma papilar linfomatoso)

d) Carcinoma mucoepidermoide

e) Carcinoma adenoideoquístico

CC330. El diagnóstico diferencial de las neoplasias parotídeas incluye al síndrome de Sjogren, el cual
consiste en:

a) Enfermedad de Mickulicz, queratoconjuntivitis, xerostomía, artritis reumatoide e


hipergamaglobulinemia

b) Lesión linfoepitelial benigna asociada a SIDA

c) Múltiples adenocarcinomas: en útero, mama y glándulas salivales entre otras


d) Poliposis colónica y otros adenocarcinomas extracolónicos

e) Tumores parotídeos y sarcomas

CC331. Los siguientes son considerados factores etiológicos en el desarrollo de neoplasias de glándulas
salivales, excepto:

a) Hábitos positivos a tabaco y alcohol

b) Bajas dosis de radioterapia o exposición acumulada a la irradiación

c) Exposición al aserrín

d) Exposición a radiaciones ultravioleta tipo B

e) Exposición al asbesto

CC332. El tumor maligno de glándulas salivales que invade al nervio facial corresponde a la siguiente
categoría de acuerdo al sistema de estadificación propuesto por la IUCC en 1997:

a) T1

b) T2

c) T3

d) T4

e) No está contemplada en la estadificación

CC333. El carcinoma adenoideoquístico tiene un comportamineto biológico que lo distingue de otros


tumores de glándulas salivales, este comportamiento se caracteriza por:

a) Rápido crecimiento, alta frecuencia de metástasis ganglionares y frecuente diseminación


perineural

b) Baja frecuencia de metástasis ganglionares y metástasis a sistema nervioso central

c) Diseminación hematógena predominante y baja recaída local

d) Diseminación perineural y muy alta capacidad de diseminación linfática

e) Crecimiento insidioso, diseminación perineural y capacidad para producir metástasis distantes

CC334. Las variedades histológicas más frecuentes involucradas en los tumores metastásicos a glándulas
salivales son:
a) Adenocarcinomas de mama y pulmón

b) Carcinoma escamoso de la piel y melanoma

c) Osteosarcoma y sarcoma de Ewing

d) Cáncer renal (células claras) y carcinomas foliculares del tiroides

e) Carcinomas indiferenciados de primario desconocido

CC335. La resección del nervio facial durante una parotidectomía en tumores malignos tiene la indicación
siguiente:

a) En todos los casos

b) En el caso de carcinomas adenoideoquísticos o invasión perineural

c) En todos los tumores de alto grado

d) En los tumores mayores de 2 cm

e) Sólo cuando está directamente involucrado por el tumor

CC336. La radioterapia posoperatoria para el tratamiento de los tumores malignos de glándulas salivales está
indicada en los casos siguientes, excepto:

a) Todos los casos

b) Todos los casos, excepto tumor de bajo grado y menor de 2 cm completamente resecado

c) Sólo en tumores de alto grado e irresecables

d) Sólo en los tumores recurrentes

e) En ningún caso

CC337. La quimioterapia en el manejo de los tumores malignos de glándulas salivales es útil:

a) Como adyuvante en todos los casos de malignidad

b) Como adyuvante en los tumores de alto grado o recurrentes

c) Como adyuvante en el carcinoma adenoideoquístico

d) En el manejo de los tumores sintomáticos irresecables

e) No está indicada en ningún caso


CC338. Una de las aseveraciones siguientes en relación con el carcinoma mucoepidermoide es correcta:

a) Es una neoplasia de alto grado, invariablemente

b) Se comporta como una neoplasia de bajo grado, invariablemente

c) Es el tumor maligno más frecuente de la glándula parótida

d) Ocurre preferentemente en el lóbulo profundo de la parótida

e) Es un tumor muy radiosensible y radiocurable

CC339. El concepto de papiloma Schneideriano incluye:

a) Papiloma fungiforme

b) Papiloma invertido

c) Papiloma de células cilíndricas

d) Papiloma escamoso

e) Todos los anteriores

CC340. La disección ganglionar de cuello debe realizarse en presencia de:

a) Tumores radioquimiosensibles

b) Primario infraclavicular

c) Primario controlado

d) Metástasis a distancia

e) Metástasis irresecables

TUMORES DE TEJIDOS BLANDOS Y ÓSEOS

TB1. Variedad de condrosarcoma sensible de QT y RT perteneciente a la serie de sarcoma de células


redondas:

a) Condrosarcoma clásico

b) Condrosarcoma de partes blandas

c) Condrosarcoma diferenciado

d) Condrosarcoma mesenquimatoso
e) Condrosarcoma de células claras

TB2. Son ramas de la arteria femoral las arterias siguientes, excepto:

a) Epigástrica superficial

b) Pudenda externa superficial

c) Epigástrica inferior

d) Femoral profunda

e) Circunfleja ilíaca superficial

Responda con falso o verdadero a las aseveraciones siguientes:

TB3. Los ganglios linfáticos inguinales son superficiales y profundos. Los superficiales son 5-6 del grupo
superior situados por debajo del ligamento inguinal y 4-5 del grupo inferior situados por arriba del cayado
de la vena safena interna. Los profundos son 1-3 ganglios situados medialmente a la vena femoral.

a) Verdadero b) Falso

TB4. El triángulo femoral es también conocido por el nombre de canal aductor y sus límites son lateralmente
el músculo sartorio, mediante aductor largo y superiormente el ligamento inguinal.

a) Verdadero b) Falso

TB5. El anillo está limitado por delante por el ligamento inguinal, por detrás por el músculo pectíneo y su
facia, medialmente por el borde del ligamento lacunar y lateralmente por la arteria femoral.

a) Verdadero b) Falso

TB6. La apariencia radiológica de "piel de cebolla" es característica del:

a) Condrosarcoma

b) Osteosarcoma telangiectásico

c) Sarcoma de Ewing

d) Osteosarcoma fibroblástico

e) Osteoclastoma
TB7. Factores pronósticos más importantes en sarcomas de partes blandas:

a) Etapa clínica y metástasis óseas

b) Grado mitótico, tamaño tumoral >5 cm, profundidad de la lesión, presencia de metástasis
linfáticas, proximidad de la lesión

c) Invasión paquete neurovascular, mala respuesta a quimioterapia-radioterapia

d) Localización retroperitoneal T < de 5 cm, grado intermedio de diferenciación

e) Biopsia previa T< 5 cm de localización superficial

TB8. Los siguientes sarcomas de partes blandas frecuentemente dan metástasis linfáticos:

a) Epitelioide, sinovial, rabdomiosarcoma

b) Schwanoma maligno, linfangiosarcoma

c) Angiosarcoma, sarcoma osteogénico

d) Rabdomiosarcoma de la órbita, léiomiosarcoma uterino

e) Mesenquimoma maligno y sarcoma no clasificado

TB9. Factor pronóstico más importante como indicador de sobrevida en pacientes con sarcomas de partes
blandas:

a) Tamaño tumoral

b) Localización anatómica

c) Grado histológico

d) Edad del paciente

e) Infiltración ósea

TB10. Sitio más común de metástasis en pacientes con sarcomas de partes blandas y hueso:

a) Pulmón

b) Cerebro

c) Hueso

d) Hígado

e) Bazo
TB11. Sitio más común de metástasis de los tumores retroperitoneales:

a) Hueso

b) Hígado

c) Pulmón

d) Cerebro

e) Mesenterio

TB12. Factores más importantes para sobrevida en pacientes con metástasis pulmonares por sarcoma:

a) Tamaño del tumor primario y número de metástasis

b) Grado de diferenciación del tumor primario

c) Número de metástasis y el intervalo libre de enfermedad

d) Grado de diferenciación tumoral y tamaño de las metástasis

e) Respuesta a la quimioterapia

TB13. Se considera lesión precancerosa que se acompaña de desarrollo de sarcomas osteogénicos


sincrónicos en varias áreas:

a) Osteocondromatosis

b) Enfermedad de Paget

c) Encondramatosis

d) Tumor de células grandes

e) Tumores pardos

TB14. El sarcoma de partes blandas más común en personas menores de 21 años de edad es el:

a) Sarcoma sinovial con el 56%

b) Rabdomiosarcoma con el 45%

c) Sarcoma indiferenciado con el 75%

d) Rabdomiosarcoma con el 71%


e) Sarcoma indiferenciado con el 56%

TB15. Porciento de pacientes con sarcomas de tejidos blandos (metástasis a pulmón) que pueden curarse con
resección pulmonar:

a) 10–20

b) 20–25

c) 30–50

d) 40–50

e) 50–60

TB16. Órganos que con mayor incidencia se encuentran afectados por metástasis en sarcomas osteogénicos
(en autopsias):

a) Pulmón, hueso, riñón

b) Pulmón, hígado, cerebro

c) Hígado, ganglios linfáticos, cerebro

d) Hígado, riñón, peritoneo

e) Pulmón, corazón, ganglios linfáticos

TB17. ¿Cuál es la incidencia de extensión medular en sarcomas osteogénicos?

a) 10%

b) 20%

c) 30%

d) 40%

e) 50%

TB18. ¿Cuál es la incidencia de metástasis ganglionares de los sarcomas de partes blandas?

a) 2–39%

b) 40–50%

c) 51–60%
d) 61–70%

e) > del 70%

TB19. ¿Cuál es la incidencia de metástasis linfáticas en tumores óseos y su importancia pronóstica?

a) 0–3%, se asocian a mal pronóstico

b) 5–15%, se asocian a mal pronóstico

c) 3–10%, se asocian a mal pronóstico

d) 2–8%, no tienen ninguna implicación pronóstica

e) 15–20%, no tienen significado pronóstico

TB20. ¿Cuál de los siguientes no es un mecanismo implicado en el desarrollo de metástasis articulares en


sarcomas óseos?

a) Diseminación pericapsular

b) Extensión directa a través de estructuras intraarticulares

c) Extensión a través del cartílago articular

d) Contaminación por fracturas óseas

e) Diseminación linfática

TB21. Lo siguiente se ha asociado al desarrollo de sarcoma de partes blandas, excepto:

a) Mutación del gen P53

b) Infección por el virus ROUS sarcoma

c) Exposición al clohidrato del polivino

d) Radiación ionizante

e) Síndrome de Stewart-Treves

TB22. Los factores siguientes han sido asociados con una mayor incidencia de sarcomas en tejidos blandos,
excepto:

a) El síndrome de Li-Fraumeni

b) El retinoblastoma
c) El síndrome de Stewart-Treves

d) La poliposis familiar

e) La neoplasia endocrina múltiple tipo II

TB23. El histiocitoma fibroso maligno de partes blandas se caracteriza por:

a) Afectar principalmente retroperitoneo

b) Su mayor localización es en las extremidades inferiores

c) Su pico de incidencia entre los 15-20 años de edad

d) Que afecta invariablemente las vainas nerviosas

e) Una enfermedad básicamente de la infancia

TB24. Factor pronóstico más importante relacionado con metástasis a distancia y sobrevida, en paciente con
sarcoma de partes blandas de extremidades:

a) Gen Erb B-2

b) Gen P53

c) Grado de diferenciación de la lesión

d) Localización de la lesión

e) Edad de paciente

TB25. El factor más importante para predecir la sobrevida en las metástasis pulmonares potencialmente
resecables por osteosarcoma:

a) Localización

b) Número

c) Tipo histológico

d) Grado del tumor

e) Intervalo libre

TB26. ¿En que tipo de tumor se tiene mejor respuesta con quimioterapia a base de vincristina, actinomicina
D y ciclofosfanida?
a) Leiomiosarcoma gastrointestinal

b) Mesotelioma difuso

c) Condrosarcomas extraesquelético

d) Angiosarcoma

e) Sarcoma sinovial, fibrohistiocitoma maligno

TB27. Marcador más importante para determinar la diferenciación epitelial en el sarcoma sinovial:

a) Antígeno carcinoembrionario

b) Vimentinina

c) Citoqueratina

d) Desmina

e) Antígeno común leucocitario

TB28. Tratamiento de elección por sarcomas que se localizan en el tejido celular subcutáneo de bajo grado de
malignidad como el dermatofibrosarcoma:

a) Resección compartamental

b) Escisión local con margen de 2 cm más radioterapia

c) Resección con mínimo de 5 cm de margen

d) Quimioterapia, radioterapia y cirugía

e) Quimioterapia neoadyuvante, cirugía, y Rt

TB29. Requisitos para someter a un paciente, con metástasis pulmonares por sarcoma, a una resección de
metástasis:

a) Control del tumor primario, sin enfermedad extrapulmonar

b) Resecabilidad total de las lesiones que permita además una reserva pulmonar adecuada

c) Metástasis única y edad menor de 45 años

d) Son correctas a y b

e) Son correctas a y c

TB30. El procedimiento de Tikhoff-Linberg esta indicado en los siguientes casos de sarcoma:

a) De la cadera sin afección neurovascular


b) Del hombro sin afección neurovascular

c) De la pelvis

d) De la rodilla sin afección neurovascular

e) Ninguno de los enunciados

TB31. El procedimiento de Tikhoff-Linberg consiste en resección:

a) De tercio distal del fémur y tercio próximal de la rodilla y rotula

b) Del tercio distal de la clavícula, tercio próximal del humero, parte o toda la escápula y músculos
adyacentes

c) De tercio próximal del fémur, pubis e iliaco y músculos adyacentes

d) Del tercio medio de la mandíbula

e) De ninguna de las estructuras anatómicas antes enunciadas

TB32. Son contraindicaciones para cirugía conservadora de extremidades en sarcomas:

a) Implicación neurovascular o muscular

b) Fracturas patológicas y/o infección

c) Incisión por biopsia previa o manipulación previa inadecuada

d) Solo a y c

e) Todas

TB33. Paciente masculino de 46 años de edad que se le diagnostica por tomografía una masa de 6 cm
retroperitoneal cuya biopsia sugirió el diagnóstico de liposarcoma de bajo grado el tratamiento más
adecuado es:

a) QT preoperatoria seguida de resección y RT externa posoperatoria

b) Quimioterapia preoperatoria seguida de resección

c) Resección quirúrgica completa si es posible

d) Quimioterapia preoperatoria basada en la quimiosensibilidad de la lesión primaria, resección


quirúrgica completa, seguida de radioterapia posoperatoria

e) Resección parcial para paliación de los síntomas seguida de RT posoperatoria

TB34. Para realizar cirugía conservadora en paciente con sarcoma osteogénico se requiere que:
a) La lesión sea de bajo grado, intracompartamental, sin respuesta a QT

b) La lesión sea de alto grado etapa clínica IIb sin respuesta a QT

c) El sarcoma osteogénico de células claras esté avanzado localmente con moderada respuesta a
QT

d) El sarcoma osteogénico tenga extensión al paquete neurovascular

e) El sarcoma presente invasión a tejidos blandos pero con buena respuesta a QT

TB35. Las metástasis pulmonares en pacientes con sarcomas actualmente se tratan con:

a) Quimioterapia

b) Radioterapia

c) Quimioterapia y radioterapia

d) Quimioterapia + resección cuando técnicamente es posible

e) Únicamente resección cuando técnicamente es posible

TB36. Conducta más apropiada a seguir en un paciente con sarcoma de partes blandas de muslo de 7 cm, de
bajo grado sometido a escisión amplia de la lesión y con margen circunferencial de 2 cm:

a) Observación

b) Braquiterapia externa adyuvante

c) Radioterapia externa adyuvante

d) Quimioterapia más braquiterapia adyuvantes

e) Quimioterapia intraarterial adyuvante

TB37. ¿Qué porciento de los sarcomas de tejidos blandos se presentan en las extremidades?

a) 40

b) 60

c) 30

d) 90

e) 15

TB38. Tipo histológico más frecuente de sarcoma de tejidos blandos en el adulto:

a) Rabdomiosarcoma
b) Sarcoma neurogénico

c) Histiocitoma fibroso maligno

d) Liposarcoma

e) Sarcoma sinovial

TB39. ¿Qué posibilidad existe de que un paciente con neurofibromatosis desarrolle transformación de un
neurofibroma a sarcoma neurogénico a lo largo de su vida?

a) 13%

b) 23%

c) 35%

d) Menor a 1%

e) 50%

TB40. Son los dos factores pronósticos más importantes para sarcomas de tejidos blandos:

a) Localización y grado

b) Tamaño y localización

c) Localización y edad del paciente

d) Tamaño y grado

e) Grado y edad del paciente

TB41. ¿Qué porciento de los pacientes con un sarcoma de extremidad tiene recurrencia local después de una
resección compartamental bien realizada y bien indicada?

a) 15

b) 10

c) 2

d) 25

e) 32
TB42. De los siguientes sarcomas de tejidos blandos ¿Cuál tiene mayor tendencia a diseminarse por vía
linfática?

a) Histiocitoma fibroso maligno

b) Sarcoma neurogénico

c) Leiomiosarcoma

d) Sarcoma epiteloide

e) Sarcoma alveolar de partes blandas

TB43. Porciento de pacientes con sarcoma de tejidos blandos que presentan metástasis pulmonares en el
momento del diagnóstico:

a) 33

b) 45

c) menos de 5

d) 15

e) 1

TB44. ¿Qué porciento de pacientes con sarcoma de extremidades, tiene recurrencia local después de una
resección marginal?

a) 40 a 50

b) 10 a 40

c) 60 a 90

d) menos de 10

e) 50 a 60

TB45. ¿Cuál es la recurrencia local de un sarcoma de tejidos blandos de extremidad cuando se efectúa
resección amplia sin radioterapia adyuvante?

a) 10%

b) 50%

c) 25%

d) 80%
e) 100%

TB46. Un factor importante en el retraso del diagnóstico de los sarcomas retroperitoneales es su


localización. ¿Cuál es el tamaño promedio de los sarcomas retroperitoneales en el momento del diagnóstico?

a) 3 cm

b) 60 cm

c) 10 cm

d) 25 cm

e) 50 cm

TB47. ¿Cuál es la sobrevida global a cinco años en pacientes con sarcomas retroperitoneal,
independientemente del tratamiento instituido?

a) 16 a 35%

b) 40 a 50%

c) 10 a 15%

d) 50 a 65%

e) Menos de 10%

TB48. La radioterapia preoperatoria, intraoperatoria o posoperatoria en sarcoma retroperitoneales:

a) Mejora la sobrevida

b) Mejora el control local

c) No ha demostrado ningún beneficio

d) Incrementa el índice de curación

e) Disminuye la recurrencia local

TB49. Los sarcomas tienen una mayor frecuencia en:

a) Jóvenes, adultos

b) Viejos

c) Niños
d) Niños con mongolismo

e) Personas con ingesta de alimentos contaminados con aspergilus flavus

TB50. Un sarcoma de alto grado de malignidad es:

a) De crecimiento lento

b) Siempre metastatiza al hígado

c) Responde muy bien a la quimioterapia

d) Pobremente diferenciado, hipercelular, con extensa necrosis e hipervascularidad

e) Con gran diversidad morfológica

TB51. La variedad histológica sarcomatosa de partes blandas más frecuente es:

a) Histiocitoma fibroso maligno

b) Liposarcoma

c) Rabdomiosarcoma

d) Sarcoma

e) Sarcoma epiteliode

TB52. Para etapificar un sarcoma de partes blandas se toma en cuenta (AJCC):

a) Origen histológico y cantidad de mitosis

b) Tamaño mayor de 5 cm, localización y respuesta a la quimioterapia

c) Tamaño, grado histológico, metástasis, localización y profundidad de invasión

d) Respuesta a radioterapia, localización e histología

e) Tumor menos a 5 cm, grado histológico alto,y presencia de vimentina

TB53. Un paciente con sarcoma menos de 5 cm de alto grado histológico sin enfermedad metastásica se
clasifica como etapa clínica (AJCC):

a) I

b) IIB

c) IV

d) IIIA
e) IIIB

TB54. Paciente con tumor grado I, de 5 cm, con metástasis ganglionar se clasifica como:

a) EC III

b) EC IIB

c) EC IIIB

d) EC IIA

e) EC IV

TB55. Cuando quirúrgicamente se realiza una resección amplia en un sarcoma se entiende que:

a) Se deja la seudocápsula

b) Se resecó todo el tumor macroscópico

c) Se reseca la seudocápsula

d) Se reseca un margen de tejido normal en su contorno

e) Se remueven los tejidos de todo el compartimiento anatómico

TB56. Cuando se realiza toma de biopsia incisional de un sarcoma equivale a:

a) Escisión local amplia

b) Resección compartamental

c) Excisión intracapsular

d) Excisión extracapsular

e) Excisión local muy amplia

TB57. Tratamiento de elección en los sarcomas retroperitoneales:

a) Resección quirúrgica completa

b) Radioterapia preoperatoria + cirugía radical

c) Quimioterapia + cirugía radical

d) Resección con rayo láser

e) Quimioterapia + radioterapia + cirugía


TB58. La mejor indicación para emplear radioterapia como tratamiento combinado en sarcomas es:

a) Cuando se realiza excisión local

b) Resección extracapsular

c) Resección radical

d) Resección local amplia

e) En al área donde se realiza disección ganglionar

TB59. Justificación para emplear radioterapia en la cirugía conservadora de sarcomas:

a) Es capaz de esterilizar extensiones microscópicas

b) No presenta morbilidad

c) Tiene acción directa esterilizando el tumor primario

d) Mejora el control local cuando se ha dejado tumor microscópico

e) Puede controlar la enfermedad metastásica en tránsito

TB60. En que caso indicaría usted radioterapia posoperatoria:

a) Sarcoma grado intermedio de 7 cm en compartimiento anterior del muslo tratado con resección
local amplia

b) Mismo tumor anterior tratado con resección compartimiento anterior

c) Sarcoma 7 cm G2 tratado con quimioterapia resección intracapsular

d) Sarcoma G2 de 7 cm tratado con excisión local

e) Sarcoma G3 con 7 cm de diámetro con metástasis pulmonares

TB61. La disección ganglionar en caso de sarcoma está indicada en:

a) Sarcoma de alto grado cercano a la axila

b) Sarcomas retroperitoneales tipo leiomiosarcoma

c) Caso de angiopericitomas mayores de 5 cm

d) Caso de sospecha clínica de enfermedad metastásica microscópica

e) Presencia de metástasis ganglionares clínicas

TB62. La enfermedad metastásica en los sarcomas de alto riesgo es:


a) Proporcional al número de mitosis por campo

b) Proporcional al tamaño del tumor primario

c) Regularmente requiere menos de tres meses en aparecer clínicamente

d) Generalemnte no se presenta en tumores retroperitoneales

e) Destruida por un buen esquema de quimioterapia

TB63. Cuál es el sarcoma osteogénico de pronóstico menos favorable:

a) Intraóseo

b) Paraostal

c) Condroblástico

d) Osteoblástico

e) Teleangiectásico

TB64. Subtipo histológico más frecuente del rabdomiosarcoma en la infancia:

a) R. botrioide 30%

b) R. alveolar 55%

c) R. embrionario 57%

d) R. alveolar 85%

e) R. embrionario 87%

TB65. El sarcoma osteogénico multifocal esclerosante:

a) Se presente en niños menores de 10 años es muy raro y afecta varios sitios del esqueleto

b) Se presenta en adultos mayores de 70 años y acompaña a la enfermedad de Paget

c) Tiene su mayor incidencia entre 15 y 18 años y ataca un solo hueso largo

d) Es exclusivo de huesos planos y redondos

e) Es metacrónico

TB66. La apariencia radiológica en "pompas de jabón" es característica del:


a) Condrosarcoma

b) Osteosarcoma telangiectásico

c) Sarcoma de Ewing

d) Osteosarcoma fibroblástico

e) Osteoclastoma

TB67. Patrones radiológicos más comunes en el osteosarcoma:

a) Esclerótico, osteolítico, mixto

b) Osteoblástico, osteolítico, fibroblástico

c) Osteoblástico, condroblástico, fibroblástico

d) Osteolítico, fibroblástico, mixto

e) Ninguno

TB68. Tratamiento de elección en un paciente con sarcoma diseminado:

a) Cirugía radical

b) Analgésicos y quimioterapia

c) Cirugía radical y radioterapia

d) Radioterapia externa a hemicuerpo

e) Resección compartamental

TB69. Factor de buen pronóstico en sarcoma de Ewing:

a) Tumor atípico pélvico sin respuesta a Qt

b) Buena respuesta a quimioterapia sin diseminación metastásica

c) Sarcoma de Ewing típico, con gamagrama óseo positivo

d) Tumor resecado con cirugía radical más radioterapia

e) Etapa clínica IIa sin respuesta a Qt. de localización pélvica

TB70. ¿Cuál es la incidencia de metástasis saltonas en sarcomas óseos y su importancia?

a) La incidencia es del 6–25% y se asocian con la presencia de recurrencias locales y a distancia,


sin afectar el pronóstico de sobrevida
b) La incidencia es del 10–20% y se asocian con la presencia de recurrencias locales
exclusivamente

c) La incidencia es del 20–35% y se asocian con la presencia de recurrencias locales, a distancia


y menor sobrevida global

d) La incidencia es del 2–15% y se asocian con recurrencias óseas, a tejidos blandos y a


distancia, sin afectar el pronóstico de sobrevida

e) La incidencia es del 30% y se asocian con recurrencias locales, a distancia y menor sobrevida

B71. Incidencia de sarcomas originados por radiación:

a) 5%

b) 10%

c) 15%

d) 20%

e) 30%

TB72. Cuándo hablamos de etapa clínica IIb en sarcomas óseos nos referimos a un tumor:

a) De bajo grado intracompartamental sin metástasis

b) Extracompartamental con metástasis

c) Intracompartamental con metástasis ganglionares

d) De alto grado intracompartamental

e) De alto grado extracompartamental sin metástasis

TB73. Los sarcomas de partes blandas muestran un elevado porciento de metástasis a pesar de un adecuado
control local con cirugía y radioterapia. La quimioterapia con ciclofosfamida, vincristina, adriamicina y
dacarbazina (CYVADIC) de manera adyuvante:

a) No mejora el control local y no mejora la sobrevida global

b) Mejora el control local y mejora la sobrevida global

c) No mejora el control local pero mejora el período libre de recurrencia

d) Mejora el control local y el período libre de recurrencia pero no mejora la sobrevida global

e) No mejora el periodo libre de recurrencia pero mejora la sobrevida global

TB74. Sobrevida global del rabdomiosarcoma embrionario en niños:


a) 69% a 3 años

b) 73% a 5 años

c) 25% a 3 años

d) 39% a 5 años

e) 85% a 5 años

TB75. Sarcoma de partes blandas que con mayor frecuencia de metástasis a los ganglios linfáticos:

a) Schwannoma

b) Sarcoma de células claras

c) Rabdomiosarcoma

d) Sarcoma sinoval

e) Leiomiosarcoma

TB76. Condiciones para cirugía conservadora curativa en esqueleto en casos de sarcoma osteogénico:

a) Tumor localizado con afección a tejidos periarticulares sin respuesta a Qt

b) Tumor que no invade paquete neurovascular pero con extensión masiva de partes blandas

c) Tumor menor de 5 cm, pero con fractura en terreno patológico y hematoma disecante

d) Sarcoma osteogénico IIb con buena respuesta a Qt. sin compromiso neurovascular y con
posibilidades de obtener prótesis

e) Paciente sometido a biopsia por tru-cut con posibilidades para comprar prótesis de titanium

TB77. Un masculino de 35 años de edad con sarcoma osteogénico, esta recibiendo altas dosis de metotrexate
. las siguientes son estrategias para reducir la toxicidad, excepto:

a) Administración de bicarbonato de sodio intravenoso

b) Administración de leucovorín

c) Hidratación intravenosa

d) Administración de mesna

e) Monitoreo de los niveles de metotrexate


TB78. A un joven de 20 años de edad se le realiza biopsia de un tumor en la tibia que resulto ser sarcoma
osteogénico. Los estudios de extensión fueron negativos. El tratamiento más apropiado en ese momento es:

a) Quimioterapia

b) Quimioterapia seguida de resección quirúrgica de la lesión

c) Quimioterapia seguida de amputación

d) Amputación de la tibia proximal

e) Resección de la lesión seguida de radioterapia

TB79. Paciente femenino de 15 años de edad con dolor e inflamación alrededor de la porción distal del
fémur el estudio de imagenología que debe realizarse inicialmente es:

a) Resonancia magnética

b) Tomografía computada

c) Rastreo óseo

d) Radiografías simples

e) Ultrasonido

TB80. Cuándo en cirugía oncológica se hable de escisión local amplia debe entenderse como:

a) Resecar el tumor y su seudocápsula

b) Resecar el comportamiento muscular

c) Realizar biopsia escisional

d) Resecar con el tumor un margen de tejido normal

e) Resecar tejido normal extracompartamental

TB81. Entre las neoplasias malignas que se diagnostican cada año en los Estados Unidos ¿Qué porciento
representan los sarcomas de tejidos blandos?

a) 10

b) 15

c) 5

d) 90

e) 1
TB82. En aproximadamente 75% de los casos, para extirpar un sarcoma retroperitoneal se tiene que efectuar
resección en bloque con órganos adyacentes. ¿Qué órganos son los más frecuentemente resecados?

a) Bazo y riñón

b) Colon y Bazo

c) Colon e hígado

d) Colon y riñón

e) Hígado y riñón

TB83. Fármaco más activo y de primera línea en el tratamiento sistémico de los sarcomas de tejidos
blandos:

a) DTIC

b) Ifosfamida

c) Adriamicina

d) Cisplatino

e) Metotrexate

TB84. Sarcoma más frecuente en el área de cabeza y cuello, en niños:

a) Sarcoma de Ewing

b) Sarcoma sinovial

c) Histiocitoma fibroso maligno

d) Sarcoma osteogénico

e) Rabdomiosarcoma

TB85. Sarcoma óseo que se presenta en la 2ª década de la vida, prefiere diáfisis de los huesos largos:

a) Sarcoma de células gigantes

b) Sarcoma de Ewing

c) Condrosarcoma mesequimatoso

d) Sarcoma osteogénico

e) Fibrosarcoma

TB86. Marcadores tumorales que tienen valor pronóstico en sarcomas de personas adultas:
a) Número de mitosis

b) Contenido de citokeratina

c) Contenido de DNA

d) Contenido de RNA

e) Contenido de proteína 2

TB87. Sarcoma primario de hueso más frecuente:

a) Sarcoma Ewing

b) Condrosarcoma

c) Sarcoma de células gigantes

d) Fibrosarcoma

e) Sarcoma osteogénico

TB88. ¿Cuál es el porciento aproximado de sarcomas dentro de las neoplasias malignas del adulto en
México?

a) 5 a 10

b) 30

c) 10

d) 1 a 3

e) Más del 30%

TB89. ¿Cuál es la localización más frecuente en los sarcomas de partes blandas?

a) Extremidades superiores

b) Cabeza y cuello

c) Área ginecológica

d) Región mamaria

e) Extremidades inferiores

TB90. EL grado histológico sirve para predecir el comportamiento biológico de los sarcomas de partes
blandas, son criterios generales para gradar un tumor, los siguientes, excepto?

a) Índice mitótico
b) Morfología nuclear

c) Celularidad

d) Necrosis

e) Vascularización

TB91. Un tumor intracompartamental (T1) de alto grado sin metástasis es etapificado como:

a) IB

b) IIA

c) IIB

d) III

e) Ninguna de las anteriores

TB92. Son indicaciones para ofrecer cirugía conservadora en sarcomas de partes blandas, excepto:

a) Diámetro tumoral máximo de 10 cm

b) Alto grado histológico

c) Tumor subcutáneo o intracapsular

d) Paquete vasculonervioso respetado

e) Localización en tercio medio del segmento de la extremidad

TB93. El tipo III de la clasificación de resección de la pared torácica, se considera como:

a) Toracovertebral con resección del cuerpo vertebral

b) Resección toracoabdominal

c) Resección de la pared torácica (costillas)

d) Resección toracoesternal

e) Desarticulación interescapulotorácica

TB94. Es la manifestación clínica inicial de los sarcomas retroperitoneales:

a) Constipación
b) Ictericia

c) Dolor abdominal

d) Edema de miembros pélvicos

e) Tumoración abdominal

TB95. Los sitios principales de metástasis a distancia en los sarcomas retroperitoneales son en orden de
frecuencia:

a) Pulmón, hígado, sistema nervioso central

b) Hígado, pulmón, sistema nervioso central

c) Hígado, pulmón, hueso

d) Hueso, hígado, pulmón

e) Sólo a y d son ciertas

TB96. La frecuencia de afección ganglionar en los sarcomas retroperitoneales es del:

a) 1%

b) 5%

c) 10–15%

d) 30%

e) 7%

TB97. La supervivencia a 5 años en los pacientes con sarcomas de partes blandas de las extremidades tipo
osteogénico es en general de:

a) 15–30%

b) 20–30%

c) 75–0%

d) 40%

e) Ninguno de los anteriores


TB98. Una tumoración sospechosa de ser sarcoma de tejidos blandos con diámetro de 5 cm requiere manejo
inicial con:

a) BAAF

b) Biopsia con tru cut

c) BAAF más Biopsia escisional

d) Biopsia escisional

e) Biopsia incisional

TB99. El linfedema crónico después de la disección de los ganglios axilares, asociado con sarcoma de
tejidos blandos se conoce como:

a) Síndrome de Li Fraumeni

b) Síndrome de Steward Treves

c) Síndrome de Gardner

d) Ninguna de la anteriores

e) A y b son falsas

TB100. ¿Cuál es el sarcoma óseo más frecuente después de la maduracion del esqueleto?

a) Condrosarcoma

b) Osteosarcoma

c) Sarcoma de Ewing

d) Fibrohistiocitoma

e) Dermatofibosarcoma

TB101. Es el hueso más frecuentemente afectado por los sarcomas óseos:

a) Tibia

b) Húmero

c) Fémur

d) Peroné

e) Pelvis
TB102. Es el tipo más común de sarcoma del aparato genitourinario del adulto:

a) Angiosarcoma

b) Sarcoma retroperitoneal

c) Liposarcoma

d) Leiomiosarcoma

e) Histiocitoma fibroso maligno

TB103. Es el tipo histológico más frecuente en sarcomas mamarios secundarios a radioterapia?

a) Fibrosarcoma

b) Leiomiosarcoma

c) Liposarcoma

d) Angiosarcoma

e) Ninguno de los anteriores

TB104. De los siguientes ¿cuál no se considera como tumor óseo benigno?

a) Osteocondroma múltiple

b) Encondromas

c) Osteoblastoma

d) Osteoma osteoide

e) Adamantimoma

TB105. En pacientes con sarcomas de alto grado, la quimioterapia basada en ciclofosfamida más
doxorrubicina más MTX.

a) Disminuye las recurrencias locales

b) Mejora el intervalo libre de enfermedad

c) No mejora la sobrevida

d) Sólo tratamiento paliativo


e) A y b son ciertas

TB106. Es la edad de presentación típica de los osteosarcomas:

a) Adolescencia (12–18 años)

b) Infancia (< 12 años)

c) Adultos (18–45 años)

d) Mayores de 60 años

e) A y b son ciertas

TB107. Es el único marcador de laboratorio que se eleva en los osteosarcomas:

a) DHL

b) Fosfatasa alcalina

c) Fosfatasa ácida

d) Antígeno carcinoembrionario

e) Calcio sérico

TB108. La prevalencia de fracturas patológicas en sarcomas óseos es de:

a) 20–30%

b) 1–3%

c) 10%

d) 10–15%

e) 20%

TB109. Los sarcoma de cabeza y cuello suelen presentarse con las características siguientes, excepto:

a) Gran agresividad

b) Diseminación nerviosa y muscular

c) Metástasis pulmonares

d) Diseminación linfática
TB110. Una lesión sarcomatosa intracompartamental (T1) de cualquier grado (G1 o G2) con metástasis se
cataloga como:

a) III B

b) III A

c) II B

d) II A

e) A y b son correctas

TB111. Según la Nueva Estatificación de Sarcomas (1997) las metástasis ganglionares se estadifican como:

a) EC II

b) EC IIIa

c) EC IIIb

d) EC IV

e) EC IVa

TB112. Los agentes quimioterapéuticos más efectivos usados en perfusión aislada de extremidades por
sarcoma de tejidos blandos son:

a) Actinomicina D y Adriamicina

b) Melfalan y actinomicina D

c) Adriamicina y factor de necrosis tumoral

d) Melfalan y factor de necrosis tumoral

e) Factor de necrosis tumoral y actinomicina D

TB113. ¿A qué órgano metastatizan más frecuentemente los sarcomas retroperitonales?

a) Pulmón

b) Hueso

c) hígado

d) Ganglios retroperitoneales

e) Intestino delgado
TB114. Con respecto a la radioterapia adyuvante en pacientes con sarcoma de alto grado en extremidades
tratado en forma conservadora se puede decir:

a) Tiene igual recurrencia que la amputación

b) Aumenta la sobrevida global

c) Aumenta la sobrevida libre de enfermedad

d) Aumenta el control local comparado con la cirugía sola

e) Tiene igual control local comparado con la cirugía sola

TB115. ¿Cuál es el sitio más frecuente de sarcomas de tejidos blandos?

a) Retroperitoneo

b) Cabeza y cuello

c) Visceral

d) Extremidad superior

e) Extremidad Inferior

TB116. De los síndromes siguientes ¿cuál no aumenta la predisposición genética a los sarcomas de tejidos
blandos?

a) Neurofibromatosis tipo I

b) Síndrome de L1 Fraumeni

c) Síndrome de Gardner

d) Síndrome de Carney

e) Síndrome de Von Hippel-Lindau.

TB117. ¿Cuál es el único sarcoma originado de un epitelio?

a) Sarcoma sinovial

b) Sarcoma epiteloide

c) Hemangiopericitoma maligno

d) Angiosarcoma
e) Mesotelioma

TB118. ¿Además del pulmón, cuál es el sitio más frecuente de metástasis en sarcomas de extremidades?

a) Hueso

b) Ganglios linfáticos

c) Hígado

d) Cerebro

e) Intestino delgado

TB119. De los siguientes sarcomas de tejidos blandos, ¿cuál metastatiza con más frecuencia a cerebro?

a) Sarcoma sinovial

b) Sarcoma epiteloide

c) Liposarcoma

d) Leiomiosarcoma

e) Sarcoma alveolar

TB120. Con respecto a la biopsia incisional en sarcoma de extremidades es incorrecto:

a) Seguir el eje de la extremidad

b) Realizar hemostasia cuidadosa

c) Tallar colgajos

d) No tallar colgajos

e) No colocar drenajes

TB121. La biopsia por aspiración con aguja fina en sarcoma de tejidos blandos es usualmente utilizada para:

a) Realizar inmunohistoquímica

b) Diagnóstico de subtipo histológico

c) Diagnóstico primario

d) Confirmación de recurrencia local


e) Diagnóstico del grado

TB122. ¿Es el límite superior de una sacrectomía?

a) S3

b) L5

c) S1

d) S2

e) L4

TB123. Con respecto a la radioterapia preoperatoria, mencione el enunciado falso:

a) Aumenta las complicaciones locales

b) Posee menor campo de radiación

c) Tiene mayor sobrevida

d) Tiene igual control local

e) Usualmente no se añade un boost

TB124. ¿Cuál es considerada la piedra angular en el tratamiento de los sarcomas de tejidos blandos?

a) RT preoperatoria

b) QT neoadyuvante

c) Cirugía con márgenes negativos

d) Tumorectomía

e) Rt posoperatoria

TB125. Con respecto a la RT posoperatoria en cirugía conservadora de extremidades, mencione el


enunciado incorrecto.

a) Disminuye la recurrencia local y aumenta sobrevida global

b) No impacta en la recurrencia local y sobrevida global

c) Aumenta la sobrevida local y la sobrevida global


d) Disminuye la recurrencia local y la sobrevida global

e) Disminuye la recurrencia local y no impacta en la sobrevida global

TB126. Con respecto a las amputaciones, en el tratamiento de sarcoma de tejidos blandos de extremidades
mencione el enunciado correcto:

a) Es el tratamiento de elección

b) Aumenta el control local y la sobrevida global

c) Disminuye el control local y la sobrevida gloral

d) Aumenta el control local y no impacta en sobrevida

e) No impacta en control local y en la sobrevida

TB127. La cirugía conservadora con márgenes negativos y RT posquirúrgica comparado con amputaciones:

a) aumenta el control local y la sobrevida global

b) disminuye el control local y la sobrevida global

c) No impacta el control local y la sobrevida global

d) Menor control local y sobrevida global igual

e) Mayor control local y sobrevida global igual

TB128. ¿Cuál es el sarcoma más frecuente en retroperitoneo?

a) Leiomiosarcoma

b) Histiocitoma fibroso maligno

c) Liposarcoma

d) Angiosarcoma

e) Sarcoma Sinovial

TB129. Indicación de disección ganglionar electiva en sarcomas:

a) Sarcoma alveolar

b) Sarcoma sinovial

c) Sarcoma epiteloide
d) Angiosarcoma

e) Ninguno de los anteriores

TB130. Indicación de disección ganglionar terapéutica en sarcoma.

a) Metástasis ganglionares

b) Adenomegalias

c) Sarcoma de cabeza y cuello

d) Sarcoma de extremidades

e) Sarcoma de retroperitoneo

TB131. ¿Cuál es el tratamiento de elección en sarcoma granulocítico de 10 x 10 cm en cara anterior de


muslo?

a) Tumorectomía

b) Excisión amplia

c) Compartamentectomía

d) Amputación

e) Quimioterapia

TB132. ¿Cuál es el órgano más frecuentemente resecado en sarcoma de retroperitoneo?

a) Colon

b) Bazo

c) Intestino delgado

d) Riñón

e) Hígado

TB133. De los sarcomas siguientes ¿cuál tiene más probabilidad de responder a QT neoadyuvante?

a) Liposarcoma mixoide

b) Dermatofibrosarcoma protuberans
c) Leiomiosarcoma epiteloide

d) Rabdomiosarcoma embrionario

e) Rabdomiosarcoma alveolar

TB134. En los sarcomas siguientes EC IV ¿en cuál se recomienda realizar una tomografía cerebral electiva?

a) Sarcoma sinovial

b) Sarcoma alveolar

c) Sarcoma de células claras

d) Tumor granular maligno

e) Osteosarcoma extraesquelético

TB135. En general, ¿Cuál es el porciento de metástasis ganglionares en sarcomas de tejido blandos?

a) Menos del 5

b) 15

c) 25

d) 33

e) 50

TB136. ¿De los siguientes sarcomas, cuál no se origina del mesodermo?

a) Osteosarcoma

b) Mesotelioma

c) Liposarcoma

d) Leiomiosarcoma

e) Schwannoma maligno

TB137. ¿Cuál sería, cómo principio quirúrgico, el margen profundo de resección en un tumor de pared
abdominal anterior de 8x5 cm?

a) Tejido celular subcutáneo


b) Aponeurosis

c) Oblicuo menor

d) Músculo transverso

e) Peritoneo

TB138. La única indicación para resecar un sarcoma de retroperitoneo dejando tumor macroscópico es:

a) Cuando invade colon

b) Cuando invade la vena cava inferior

c) Nunca debe dejarse tumor macroscópico

d) Cuando se pretende aliviar algún síntoma

e) Cuando invade el estómago

TB139. ¿Cuál es el límite externo de resección en la disección ganglionar pélvica?

a) Nervio obturador

b) Nervio crural

c) Arteria ilíaca externa

d) Nervio genitocrural

e) Borde externo del psoas

TB140. Límite anatómico que marca la indicación entre desarticulación coxofemoral y hemipelvectomía
externa en sarcoma de tejido blando.

a) Trocánter mayor

b) Articulación coxofemoral

c) Trocánter menor

d) Espina ilíaca anterosuperior

e) Articulación sacroilíaca

TB141 En la hemipelvectomía ampliada, el margen proximal de resección ósea es:


a) Articulación coxofemoral

b) Espina ilíaca posterior

c) Tercio posterior de cresta ilíaca

d) Articulación sacroilíaca

e) Agujeros sacros

TB142. ¿El sarcoma granulocítico es?

a) Tipo de liposarcoma

b) Tipo de leiomiosarcoma

c) Leucocitos neoplásicos en tejidos blandos

d) Sarcoma de origen desconocido

e) Sarcoma de origen fibrohistiocítico

TUMORES MIXTOS

Correlacione el síndrome paraneoplásico del cáncer pulmonar con manifestación clínica:

MX1. Endocrino ( )

MX2. Neurológico ( )

MX3. Hematológico ( )

MX4. Cutáneo ( )

MX5. Esquelético ( )

a) Eosinofilia

b) Osteoartropatia hipertrofica pulmonar

c) Acantosis nigricans

d) Síndrome carcinoide

e) Polimiositis

MX6. Un carcinoma pulmonar con presencia de ganglios supraclaviculares ipsilaterales se clasifica como:
a) NX

b) N2

c) N3

d) MX

e) M1

Correlacione las siguientes columnas según la topografía y frecuencia de presentación en mediastino:

MX07. Neoplasias del timo ( )

MX08. Tumor neurogénico ( )

MX09. Sarcoidosis ( )

MX10. Hernia Morgagni ( )

MX11. Tumor esofágico ( )

a) Mediastino medio

b) Mediastino posterior

c) Mediastino anterosuperior

Los tumores del mediastino se clasifican en. Correlacione las columnas:

MX12. Neuroblastoma ( )

MX13. Chemodectoma ( )

MX14. Meningocele ( )

MX15. Linfangioma ( )

MX16. Xantogranuloma ( )

a) Tumor mesenquimatoso

b) Originario de ganglios simpáticos

c) Quiste

d) Originado en tejido paraganglionar

Correlacione el tumor mediastinal con el síndrome sistémico asociado:


MX17. Aplasia eritrocitaria ( )

MX18. Dolor inducido por alcohol ( )

MX19. Osteoartritis ( )

MX20. Neoplasia endocrina múltiple ( )

MX21. Úlcera péptica ( )

a) Neurilemoma

b) Timoma

c) Neurofibroma

d) Carcinoide tÍmico

e) Enfermedad de Hodgkin

MX22. Según la estadificación de Masaoka, un timoma con invasión macroscópica al pericardio


corresponde a un estadio:

a) I

b) II

c) III

d) IVa

e) IVb

MX23. En el timoma, la aplasia eritrocitaria pura se considera un desorden autoinmune y se encuentra en los
pacientes en una frecuencia de:

a) 0%

b) 5%

c) 15%

d) 30%

e) 50%

MX24. La indicación de radioterapia en un timoma con resección completa es a partir del estadio:

a) I
b) II

c) III

d) IV

e) No tiene indicación

MX25. La edad de mayor incidencia en osteosarcoma es:

a) De los 20 a los 35 años

b) De los 15 a los 30 años

c) De los 40 a los 60 años

d) Mayores de 80 años

e) Menores de 30 años

MX26. La frecuencia de condrosarcomas secundarios es de:

a) 10%

b) 25%

c) 50%

d) 75%

e) 90%

MX27. Según su estirpe, los tumores óseos más frecuentes son:

a) Hematopóyeticos

b) Condrogénicos

c) Osteogénicos

d) Fibrogénicos

e) Vasculares

MX28. Es la principal vía de diseminación de los tumores óseos:

a) Hematógena
b) Linfática

c) Transcortical

d) Transmedular

e) Extensión directa

MX29. Un sarcoma óseo en estadio clínico IA se refiere a:

a) Tumor de alto grado extracompartamental

b) Tumor de bajo grado extracompartamental

c) Tumor de alto grado intracompartamental

d) Tumor de bajo grado intracompartamental

e) Tumor benigno

MX30. Un tumor con ganglios positivos en sarcomas óseos extracompartamentales corresponde a un


estadio:

a) II A

b) II B

c) III B

d) IV

e) No se incluyen los ganglios en la estadificación

MX31. El osteosarcoma parosteal:

a) Es de origen centromedular

b) Es la variedad de osteosarcoma más frecuente

c) Siempre es de bajo grado

d) Tiene mejor pronóstico que el osteosarcoma clásico

e) Los osteosarcomas parosteales de bajo grado requieren quimioterapia adyuvante

MX32. El osteosarcoma parosteal:


a) Es de origen centromedular

b) Es la variedad de osteosarcoma más frecuente

c) La localización más frecuente es tibia

d) Siempre es de bajo grado

e) El tratamiento es, exclusivamente, la resección marginal

MX33. Del Sarcoma de Paget se puede decir que:

a) El 20% de los pacientes con enfermedad de Paget desarrollan sarcoma de Paget

b) Se presenta en pacientes adolescentes

c) Es la variedad de osteosarcoma más frecuente

d) Es de buen pronóstico

e) Los sitios más frecuentes son pelvis, fémur y húmero

MX34. El osteosarcoma de superficie de alto grado:

a) Es de mejor pronóstico que los parosteales

b) Se localiza con mayor frecuencia en el fémur

c) No requiere de quimioterapia en ninguna de sus modalidades

d) Es en pacientes ancianos

e) Es una lesión epifisiaria

MX35. En la comparación del tratamiento local de los sarcomas óseos, cirugía conservadora versus
amputación, los resultados en gran parte de los estudios indican:

a) Una ventaja para la cirugía radical (amputación) en términos de control local y supervivencia

b) Una ventaja para la cirugía conservadora en términos de control local y supervivencia

c) No hay diferencia entre ambos procedimientos en términos de recurrencia local ni


supervivencia

d) La cirugía conservadora sólo está indicada en caso de paliación

e) La cirugía radical sólo está reservada para los pacientes mayores de 60 años
MX36. Es el tumor maligno más frecuente en hueso:

a) Metastásico

b) Fibrosarcoma

c) Histiocitoma fibroso maligno

d) Ewing

e) Osteosarcoma

MX37. Es la variedad de condrosarcoma más frecuente:

a) Periférico

b) Central

c) Mesenquimal

d) Diferenciado

e) Células claras

MX38. El condrosarcoma:

a) Es más frecuente en adolescentes

b) Su localización más frecuente es el fémur distal

c) Es el tumor más frecuente del esternón y la escápula

d) Su localización más frecuente es la columna lumbar

e) S epresenta con mayor frecuencia en pacientes mayores de 70 años

MX39. La tasa de supervivencia global a 5 años para los pacientes con condrosarcoma es aproximadamente
del:

a) 5%

b) 20%

c) 50%

d) 80%

e) 99%
MX40. Del condrosarcoma se puede decir que:

a) Gran parte de los condrosarcomas secundarios son de bajo grado

b) Gran parte de los condrosarcomas secundarios son de alto grado

c) La supervivencia en los pacientes con tumores periféricos de bajo grado apenas alcanza un 5%
a 5 años

d) La supervivencia en los pacientes con tumores centrales de alto grado apenas alcanza un 95%
a 5 años

e) La tasa de enfermedad metastásica al diagnóstico de los pacientes con condrosarcomas


secundarios es del 80%

MX41. El tratamiento de los pacientes con condrosarcoma de grados bajo e intermedio

a) Incluye la resección de la lesión y quimioterapia adyuvante

b) Incluye quimioterapia neoadyuvante y la resección de la lesión

c) Incluye radioterapia y quimioterapia concomitante

d) Incluye sólo la resección de la lesión

e) Únicamente se les administra quimioterapia

MX42. La resección segmentaria de la pelvis tipo III, se refiere a:

a) Resección de la cresta iliaca

b) Resección del sacro

c) Resección del acetábulo

d) Resección de la rama púbica

e) Resección del agujero obturador

MX43. El condrosarcoma de células claras:

a) Requiere de tratamiento sistémico con quimioterapia

b) Es la variedad más frecuente de condrosarcomas

c) Es una neoplasia de bajo potencial maligno


d) El 90% de los pacientes presentan enfermedad metastásica al diagnóstico

e) Ocurre en niños

MX44. Es el tratamiento de los condrosarcomas irresecables:

a) Quimioterapia

b) Quimiorradioterapia concomitante

c) Inmunoterapia

d) Resección parcial en etapas

e) Radioterapia

MX45. Es la variedad que se caracteriza por la presencia de un patrón histológico bifásico, agresivo y
predominio en huesos planos:

a) Condrosarcoma periférico

b) Condrosarcoma central

c) Condrosarcoma de células claras

d) Condroblastoma

e) Condrosarcoma mesenquimal

MX46. La elevación de fosfatasa alcalina en los pacientes con osteosarcoma:

a) Conlleva a un peor pronóstico

b) No tiene significancia pronóstica

c) Conlleva a un mejor pronóstico

d) No tiene utilidad en el osteosarcoma

e) Les confiere una mejor respuesta a quimioterapia

MX47. Sitio más frecuente de recurrencia en los pacientes con osteosarcoma:

a) Hígado

b) Sitio quirúrgico
c) Piel

d) Encéfalo

e) Pulmón

MX48. Sitio más frecuente de metástasis del osteosarcoma:

a) Hígado

b) Sitio quirúrgico

c) Piel

d) Encéfalo

e) Pulmón

MX49. ¿Cuál de los factores siguientes le confiere mejor pronóstico a los enfermos con sarcoma sinovial?

a) Edad joven

b) Patrón monofásico

c) Aneuploidia

d) Localización proximal

e) Tumores mayores de 5 cm

MX50. Hallazgo cromosómico clásico en el sarcoma sinovial:

a) Translocación 11-22

b) Cromosomas gigantes

c) Trisomía 2-8

d) Translocación 21-22

e) Translocación 18-11

MX51. La translocación cromosómica característica (t18-11.2) del sarcoma sinovial se presenta en el


porciento siguiente:

a) 10

b) Raro

c) 30
d) 90

e) 50

MX52. El diagnóstico diferencial del sarcoma sinovial monofásico fusocelular incluye, principalmente:

a) Sarcoma osteogénico

b) Fibrosarcoma

c) Liposarcoma

d) Melanoma

e) Leiomiosarcoma

MX53. Para realizar el diagnóstico diferencial preciso entre sarcoma sinovial monofásico y carcinosarcoma,
melanoma y fibrosarcoma se recurre a:

a) Microscopía electrónica

b) Microscopía de luz

c) Rearreglo cromosómico

d) Inmunohistoquímica

e) No es posible diferenciarlos

MX54. Porciento de enfermedad ganglionar asociada con sarcoma sinovial:

a) Menos de 5

b) 10-15

c) 40-50

d) 80

e) 90

MX55. Hallazgo radiológico hasta en 20% de los pacientes con sarcoma sinovial:

a) Destrucción ósea

b) Calcificación focal o múltiple


c) Invasión muscular

d) Reacción subperióstica

e) Triángulo de Codman

MX56. ¿Cuál de las condiciones siguientes son propuestas para realizar disección profiláctica ganglionar en
sarcoma sinovial?

a) Tumor distal de 2 cm

b) Tumor de más de 5 cm y patrón pleomórfico independiente del sitio

c) Tumor de 3 cm proximal

d) Tumor de 2 cm en tronco

e) Tumor de 3 cm en el área de cabeza y cuello

MX57. Porciento de P53 espresado en sarcoma sinovial:

a) 5

b) 10

c) 30

d) 50

e) 100%

MX58. Grupo de pacientes con sarcoma sinovial y pronóstico más favorable:

a) Mujer menor de 40 años y tumor distal

b) Hombre mayor de 50 años y tumor en cabeza y cuello

c) Mujer de 60 años con tumor aneuploide en abdomen

d) Hombre joven con tumor de 10 cm en tórax

e) Mujer de 25 años con tumor de 20 cm retroperitoneal

MX59. El hallazgo microscópico más común en el sarcoma sinovial bifásico corresponde a:

a) Áreas papilares aplanadas


b) Áreas vasculares y papilares

c) Áreas seroglandulares y colágena

d) Células epiteliales en cordones

e) Componente fusocelular y epitelio columnar alto

MX60 La sobrevida actual con el manejo multimodal a 10 y 5 años del sarcoma sinovial es,
respectivamente:

a) 5 y 10%

b) 20 y 30%

c) 40 y 50%

d) 70 y 80%

e) 100%

MX61. De los factores siguientes uno de ellos le confiere peor pronóstico a los enfermos con sarcoma
sinovial:

a) Localización distal

b) Edad joven

c) Patrón bifásico

d) Sobreexpresión de p53

e) Marcada calcificacón

MX62. Subvariedad de mejor pronóstico del liposarcoma bien diferenciado:

a) Mixoide

b) De células redondas

c) Tipo esclerosante

d) Desdiferenciado

e) Pleomórfico

MX63. El sitio de origen más común de los liposarcomas retroperitoneales es:


a) Paracaval

b) Pélvico

c) Perirrenal

d) Paravertebral

e) Paraaórtico

MX64. La traslocación característica del liposarcoma mixoide t(12:16) se presenta en:

a) 5%

b) 10%

c) 40%

d) 75%

e) 100%

MX65. El porciento de sobrevida general a 5 años para los liposarcomas con manejo multimodal es:

a) 10

b) 20

c) 50

d) 75

e) 100

MX66. La variedad histologíca con mejor respuesta a la quimioterapia adyuvante dentro de los liposarcomas
es:

a) Esclerosante

b) Mixoide

c) Inflamatorio

d) Lipoma like

e) Pleomórfico o células redondas


MX67. El pronóstico actual de los liposarcomas ha mejorado debido a:

a) Resecciones más agresivas

b) Radioterapia preoperatoria yo postoperatoria

c) Disección ganglionar profiláctica

d) Quimioterapia adyuvante

e) a y b son correctas

MX68. El subtipo histológico más común de los liposarcomas (40 a 50%) corresponde a:

a) Lipoblástico

b) Pleomórfico

c) Mixoide

d) Desdiferenciado

e) Células redondas

MX69. ¿Cuál de los factores pronósticos siguientes se ha asociado con mejor pronóstico en los
liposarcomas?

a) Edad joven

b) Liposarcoma pleomórfico

c) Sobreexpresión de p 53

d) Ulceración

e) Tumor mayor a 5 cm

MX70. Variante histológica de peor pronóstico en el liposarcoma:

a) Esclerosante

b) Lipoma like

c) Inflamatorio

d) Células redondas

e) Mixoide
MX71. Porciento de recurrencia local con manejo exclusivamente quirúrgico (excisión amplia radical) para
liposarcomas desdiferenciados mayores a 5 cm.

a) 5

b) 20

c) 45

d) 60

e) 100

TUMORES DE TÓRAX

TO1. Patología benigna relacionada más frecuentemente con carcinoma bronquioalveolar de pulmón:

a) EPOC

b) Esclerodermia

c) Tuberculosis

d) Histoplasmosis

e) Asma

TO2. Variedad histológica de cáncer del pulmón más frecuente en mujeres jóvenes, especialmente no
fumadoras:

a) Carcinoma epidermoide

b) Adenocarcinoma

c) Carcinoma de células pequeñas

d) Carcinoma bronquioalveolar

e) Carcinoma de células grandes

TO3. Los factores pronósticos de sobrevida más importantes en los pacientes con cáncer del pulmón de
células pequeñas etapas III y IV son:

a) Contenido de DNA, fracción de células en fase S y mutaciones de P53

b) Tamaño tumoral, variedad histológica y estado ganglionar

c) Localización del tumor, presencia o ausencia de neumopatía asociada y edad del paciente
d) Etapa clínica, estado funcional y pérdida ponderal significativa

e) Estado ganglionar, tamaño tumoral y localización central o periférica del tumor

TO4. La probabilidad de histología mixta en el cáncer del pulmón es de:

a) 10%

b) 20%

c) 30%

d) 40%

e) 50%

TO5. Estirpe histológica más frecuente en cáncer de pulmón:

a) Carcinoma de células pequeñas

b) Carcinoma epidermoide

c) Adenocarcinoma

d) Carcinoma de células grandes

e) b y c

TO6. Causa más frecuente de síndrome de vena cava superior:

a) Cáncer de testículo

b) Cáncer broncogénico

c) Linfoma

d) Cáncer renal

e) Cáncer de próstata

TO7. El SVCS es más frecuente en la población adulta que presenta:

a) Linfoma maligno

b) Coriocarcinoma

c) Melanoma maligno

d) Cáncer broncogénico

e) Cáncer tiroideo intratorácico


TO8. El tabaco es uno de los factores de riesgo con más frecuencia asociado con cáncer broncogénico
incluyendo a los fumadores pasivos, en quienes el principal carcinogenético encontrado en el humo inhalado
es:

a) N-nitrosaminas

b) Cadmium

c) Niquel

d) Benzopirenos

e) Nicotina

TO9. En la enfermedad metastásica por cáncer broncogénico se considera que el 20% de los pacientes en
que se reseca la metástasis única, pueden sobrevivir hasta cinco años; cuando la metástasis se localiza en:

a) Cerebro

b) Hígado

c) Glándulas suprarrenales

d) Hueso

e) Piel

TO10. Síndrome frecuentemente asociado con tumores germinales extragonadales en mediastino:

a) Down

b) Acatosis nigricans

c) Paterson-Kelly

d) Klinefelter

e) Tietze

TO11. Los tumores malignos primarios que se originan en el corazón son raros, el de mayor frecuencia es el:

a) Liposoma

b) Sarcoma neurogénico

c) Angiosarcoma

d) Linfoma

e) Mesotelioma

TO12. Los tumores de la parrilla costal son raros, el tumor de mayor incidencia en el adulto es el:
a) Condrosarcoma

b) De Ewing

c) Fibrosarcoma

d) Osteosarcoma

e) Mieloma múltiple

TO13. Un nódulo pulmonar solitario es probablemente benigno cuando:

a) Es mayor de 3 cm

b) Se mantiene estable en su tamaño por dos años o más

c) La calcificación excéntrica está presente

d) Existen márgenes irregulares especulados

e) Se asocia con síntomas endocrinos

TO14. Además de la exposición a asbestos, se han relacionado con la etiología del mesotelioma los factores
siguientes, excepto:

a) Exposición a fibras de zeolite

b) Historia previa de radioterapia

c) Tabaquismo

d) Historia familiar de mesotelioma

TO15. Porciento de incremento en la incidencia de cáncer pulmonar cuando un no fumador vive con un
fumador comparado con un no fumador que vive con otro no fumador:

a) 3

b) 15

c) 30

d) 75

e) 100

TO16. Porciento de pacientes en Estados Unidos con cáncer de pulmón de células no pequeñas candidatos a
cirugía potencialmente curativa al momento del diagnóstico:
a) 10 b) 30 c) 50 d) 70 e) 90

TO17. No es contraindicación para cirugía de pulmón:

a) SVCS

b) Parálisis de nervio laríngeo recurrente

c) Síndrome de Horner

d) Histología de células pequeñas

e) Metástasis de ganglios subcarinales mediastínicos ipsilaterales

TO18. Tratamiento estándar del carcinoma de células pequeñas del pulmón con enfermedad limitada:

a) QT, exclusivamente

b) QT de inducción y cirugía

c) Cirugía para resección, QT y RT adyuvantes

d) QT y RT

e) RT, exclusivamente

TO19. El tratamiento estándar del timoma no invasor, completamente resecado (etapa I de Masaoka),
consiste en cirugía:

a) Más quimioterapia y radioterapia adyuvantes

b) Exclusivamente

c) Radioterapia adyuvante

d) Más quimioterapia y radioterapia concomitantes

e) Quimioterapia adyuvante

TO20. La cirugía es el principal tratamiento en el cáncer pulmonar, ante la presencia de lesión resecable la
cirugía mínima aceptada con mayor factor de control local es:

a) Cuña

b) Segmentectomía

c) Lobectomía
d) Bilobectomía

e) Neumonectomía

TO21. El tratamiento sistémico del cáncer broncogénico juega un papel importante en la actualidad, la droga
considerada con mayor posibilidad de respuesta como único agente es:

a) Ciclofosfamida

b) Metotrexate

c) Etopósido

d) Motomicina C

e) Cisplatino

TO22. En relación con la irradiación profiláctica del encéfalo (IPE) en cáncer broncogénico de células
pequeñas, el enunciado correcto es:

a) La IPE aumenta la sobrevida

b) La IPE normalmente comprende dosis de 5000 rads

c) La IPE comprende fracciones diarias de 200 a 300 rads/2 a 3 semanas

d) La IPE puede darse simultáneamente con quimioterapia

e) La IPE beneficia incluso a pacientes con respuesta parcial del sitio primario

TO23. La cirugía de la parrilla costal es un procedimiento quirúrgico complejo que requiere abordaje
multidisciplinario con morbilidad mayormente asociada a:

a) Sangrado

b) Pérdida del colgajo

c) Infección

d) Insuficiencia respiratoria

e) Rechazo del material protésico

TO24. El mejor acceso para el tratamiento de las metástasis pulmonares en las lesiones cercanas al hilio
pulmonar es:

a) Esternotomía
b) Toracoscopía

c) Mediastinostomía anterior

d) Toracotomía posterolateral

e) Toracotomía anterior

TO25. Paciente de 60 años de edad con carcinoma de células pequeñas en pulmón, cursa con enzimas
hepáticas elevadas al doble del límite superior y creatinina de 1.7 mg/L. Usted plantea utilizar un esquema
que incluye etopósido ¿Cuál de las características siguientes debe tomar en cuenta?

a) El paciente no puede ser tratado con etopósido

b) El paciente definitivamente debe tener una reducción de la dosis debido a la disfunción


hepática

c) El paciente seguramente tendrá mayor riesgo de experimentar neuropatía periférica

d) El paciente tendrá un incremento en la mielotoxicidad debido a disfunciones hepáticas y


renales

e) La dosis de etopósido debe de ser reducida en un 50% debido a la insuficiencia renal

TO26. Las lesiones malignas se presentan, en general, con los patrones radiográficos siguientes, excepto:

a) Crecimiento rápido

b) Superficie ulcerada

c) Unidades Hounsfield menores de 175

d) Ausencia de calcio

e) Unidades Hounsfield mayores de 175

TO27. Estirpe histológica que constituye entre el 33-64% de todos los tumores malignos de pulmón.
Frecuentemente se presenta en zona central, invadiendo el cartílago bronquial y produciendo obstrucción.
Las lesiones periféricas pueden evitarse y simular abscesos pulmonares; microscópicamente se presentan
como lesiones con abundante queratina:

a) Carcinoma de células escamosas

b) Adenocarcinoma

c) Carcinoma de células grandes

d) Carcinoma de células pequeñas

e) Carcinoma bronquioalveolar
TO28. Procedimiento específico para determinar la naturaleza benigna o maligna de las adenomegalias
mediastinales en el cáncer de pulmón:

a) Tomografía computada con ventana para mediastino con medio de contraste

b) Imagen por resonancia magnética

c) Tomografía con emisión de positrones

d) Biopsia dirigida con aguja fina

e) Mediastinoscopía

TO29. Los pacientes con cáncer de pulmón T4 N0 M0 que se presentan con derrame pleural y citología
positiva para malignidad, en ausencia de metástasis a distancia, se estadifican en la etapa clínica:

a) Ib

b) IIa

c) IIb

d) IIIa

e) IIIb

TO30. En pacientes con mesotelioma maligno el derrame pleural maligno es la principal manifestación de
enfermedad, sin embargo, el signo-síntoma más relevante al manifestarse la enfermedad es:

a) Disnea

b) Tos

c) Taquicardia

d) Estertores bronquiales

e) Dolor torácico

TO31. La sintomatología en el cáncer broncogénico se relaciona con su localización, en los tumores centrales
su principal manifestación clínica inicial es:

a) Disnea

b) Dolor torácico

c) Absceso pulmonar

d) Tos

c) Hemoptisis
TO32. Estudio de mayor valor en la detección de cáncer pulmonar:

a) Broncoscopía

b) Citología en expectoración

c) Telerradiografía de tórax postero-anterior

d) TAC de tórax

e) Gammagrafía pulmonar

TO33. La clasificación de Masaoka de los timomas malignos es la más aceptada universalmente, y a los
pacientes con implantes intrapericárdicos los clasifica como etapa:

a) I

b) IIb

c) III

d) IVa

e) IVb

TO34. La medicina nuclear es un recurso para el diagnóstico presuncional de masas mediastinales. Ante la
sospecha de un factor feocromocitoma del mediastino el estudio a solicitar para su localización es:

a) Galio 67

b) Tecnecio 99

c) Yodo 123

d) Metaidobenzilguanidina-Yodo 131

e) Xenon 133

TO35. La exposición a asbestos puede producir cualesquiera de las manifestaciones pleurales siguientes,
excepto:

a) Calcificaciones

b) Placas

c) Derrame

d) Mesotelioma maligno
e) Mesotelioma benigno

TO36. En la nueva etapificación internacional para cáncer de pulmón de células no pequeñas de 1997, la
presencia de otro nódulo distinto al del primario se clasifica como:

a) T1

b) T2

c) T3

d) T4

e) M1

TO37. Factor genético que no se considera de importancia en el cáncer de pulmón, principalmente en el de


células pequeñas:

a) Factor de crecimiento insulínico

b) Factor de crecimiento transferrínico

c) Factor de crecimiento pulmonar fetal (bombesina)

d) Gen supresor ErbAb

e) Ninguno de los enunciados

TO38. El carcinoma de células pequeñas del pulmón se origina de:

a) Las células de clara

b) Los neumocitos tipo I

c) Los neumocitos tipo II

d) Las células de Kultchinsky

e) Las células mucinosas

TO39. Compartimiento(s) del mediastino donde se ubican con más frecuencia los tumores neurogénicos:

a) Anterior

b) Medio

c) Posterior
d) Anterior y medio

e) Anterior y superior

TO40. Tumores más frecuentes en el mediastino anterior:

a) Linfomas

b) Tímicos

c) Neurogénicos

d) De células germinales

e) Tiroideos

TUMORES DE PIEL, ANEXOS Y MELANOMA

PA1. Alteraciones genéticas encontradas en melanoma maligno:

a) 17p, 18q, 21

b) 7, 9, 11

c) Ip, 6q, 7 y 9

d) 5, 12, 17 y 18p

PA2. Marcador más específico para melanoma maligno:

a) Vimentina

b) Proteína S-100

c) HMB-45

d) Ninguno de los anteriores

PA3. Según su patrón de crecimiento, el tipo de melanoma que se presenta con mayor frecuencia:

a) Extensión superficial

b) Acral lentiginoso

c) Nodular

d) Léntigo maligno

e) Ulcerado
PA4. Factor pronóstico independiente más importante en pacientes con melanoma etapa clínica I y II de la
clasificación de la AJCC:

a) Edad

b) Sexo

c) Nivel de invasión (Clark)

d) Grosor de la lesión (Breslow)

e) Presencia de aneuploidia

PA5. Es factor de buen pronóstico en melanoma de cabeza y cuello:

a) Afección a mucosas

b) Metástasis ganglionares

c) Profundidad mayor de 3 mm

d) Ulceración

e) Melanoma léntigo maligno

PA6. ¿Qué porciento aproximado en los melanomas de cabeza y cuello se encuentran en el pabellón
auricular?

a) 0-5

b) 7-15

c) 17-25

d) 26-36

e) Más del 40

PA7. De todos los melanomas de la mujer, el de los genitales ocupa el:

a) 1%

b) 2%

c) 3%

d) 4%

e) 5%
PA8. De acuerdo a su patrón de crecimiento ¿Cuál es el tipo de melanoma que se presenta con mayor
frecuencia?

a) Extensión superficial

b) Acral lentiginoso

c) Nodular

d) Léntigo maligno

e) Ulcerado

PA9. ¿A partir de qué nivel de Clark los diferentes autores indican la DR ganglionar electiva de primera
intención?

a) I

b) II

c) III-IV

d) IV

e) V

PA10. Según los resultados del ECOG estaría indicado administrar tratamiento adyuvante con altas dosis de
interferón alfa 2b sistémico a pacientes con melanoma que tengan una lesión:

a) Localizada <0.75 mm de profundidad

b) Localizada de 0.76-1.5 mm de profundidad

c) Localizada de 1.6-4 mm de profundidad

d) Localizada a más 4 mm de profundidad

e) De cualquier profundidad con metástasis a ganglios linfáticos

PA11. En el melanoma de grosor intermedio (1 a 4 mm) la disección ganglionar linfática electiva beneficia
la sobrevida:

a) De todos los pacientes

b) De pacientes con micrometástasis

c) De los pacientes que tienen múltiples ganglios afectados

d) De pacientes que tienen invasión extraganglionar


e) De ningún paciente

PA12. Mencione cuál es el factor etiólogico más importante en el carcinoma de piel.

a) Tener piel blanca

b) Expenerse a radiaciones ionizantes

c) Exponerse a radiaciones ultravioleta

d) Presentar irritación cutánea crónica

e) VPH

PA13. Los siguientes son lesiones premalignas para cáncer de piel, excepto.

a) Queratosis actínica

b) Cuerno cutáneo

c) Queratosis seborreica

d) Dermatitis por radiación

e) Enfermedad de Bowen

PA14. ¿Qué porciento de carcinoma de células basales de piel produce metástasis?

a) 5

b) 10

c) 1

d) 0.1

e) 8%

PA15. Son tumores benignos de la piel, excepto:

a) Nevus epidérmicos

b) Queratosis seborreica

c) Acantoma de células claras

d) Queratosis actínica
e) Sólo a y b

PA16. La leucoplasia en que porciento presenta carcinoma de células escamosas.

a) 15

b) 20

c) 3-6

d) 10

e) 1

PA17. Porciento global de metástasis en carcinoma epidermoide de piel:

a) 0

b) 2

c) 10

d) 40

e) 70

PA18. Son subtipos de carcinoma de células básales de piel, excepto:

a) Infiltrativo

b) Pigmentario

c) Esclerosante

d) Superficial

e) Nódulo ulcerativo

PA19. Variedad de carcinoma de células basales de piel más agresivo.

a) Fibroepitelial

b) Superficial

c) Esclerosante

d) Pigmentado
e) Sólo a y b

PA20. En un paciente con carcinoma epidermoide de piel de 5 cm ¿cuál es el margen mínimo adecuado en
su escisión quirúrgica?

a) 5 mm

b) 2 cm

c) 1 cm

d) 3 cm

e) 4 cm

PA21. Un carcinoma epidermoide de piel T3, N1, M0 corresponde al estadio clínico siguiente:

a) III

b) IV

c) II

d) I

PA22. En el melanoma cutáneo de Breslow II, los márgenes requeridos para evitar recurrencia son de:

a) 1 cm

b) 2 cm

c) 3 cm

d) 4 cm

e) 5 cm

PA23. Son indicaciones de la disección ganglionar electiva en el melanoma maligno

a) Melanomas con grosor de 1 mm

b) Melanomas con grosor mayor de 1-2 mm

c) Melanomas con grosor mayor de 4 mm

d) Melanomas con grosor mayor de 2 mm


e) Sólo b y c

PA24. Márgenes quirúrgicos necesarios para lesiones mayores de 4 mm en profundidad en melanoma


maligno:

a) 1 cm

b) 2 cm

c) 3 cm

d) 4 cm

e) 5 cm

PA25. El porciento de metástasis regionales en pacientes con melanoma de 1 a 4 mm de grosor es:

a) 20

b) 40

c) 60

d) 80

PA26. Son factores de riesgo de recurrencia del melanoma, excepto.

a) Melanoma lentiginoso

b) Espesor de 4 mm

c) Ulceración

d) Localización en cara

e) Localización en piel cabelluda

PA27. En un melanoma cutáneo de cara anterior de brazo izquierdo con Breslow de 0.75 mm sin evidencia
de metástasis ganglionares regionales y a distancia, el tratamiento de elección es:

a) Escisión con margen de 1 cm, únicamente

b) Escisión con margen de 2 cm, únicamente

c) Escisión con margen de 3 cm, únicamente

d) Escisión con margen de 1 cm y disección radical ganglionar regional


e) Escisión con margen de 2 cm y disección radical ganglionar regional

PA28. En un melanoma cutáneo de cara anterior de brazo izquierdo con Breslow de 2 mm, sin evidencia de
metástasis ganglionares regionales y a distancia, el tratamiento de elección es:

a) Escisión con margen de 1 cm, únicamente

b) Escisión con margen de 2 cm, únicamente

c) Escisión con margen de 3 cm, únicamente

d) Escisión con margen de 1 cm y disección radical ganglionar regional

e) Escisión con margen de 2 cm y disección radical ganglionar regional

PA29. En un melanoma cutáneo de la región dorsal, en una lesión Breslow entre 1 y 2 mm, el margen de
sección adecuado es:

a) 0.5 cm

b) 1 cm

c) 2 cm

d) 3 cm

e) 5 cm

PA30. Es el factor más importante en el carcinoma basocelular relacionado con mal pronóstico:

a) Márgenes positivos

b) Lesiones recurrentes

c) Histología basoescamosa

d) Lesiones grandes y ulceradas

e) Lesiones recurrentes

PA31. De los enunciados siguientes relacionados a la radiación ultravioleta, elija el incorrecto:

a) Es carcinogénica

b) La luz UVa no causa directamente cáncer pero sí interactúa con otras sustancias

c) La luz UVb Tiene una longitud de onda entre 280 y 320


d) Incrementa con la proximidad al Ecuador

e) Se triplica con cada 10 grados de reducción de la latitud

PA32. ¿Cuál de los factores siguientes no aumenta el riesgo de desarrollar carcinoma de células escamosas?

a) Radiación ultravioleta

b) Psoralenos más radiación tipo UVa

c) Virus del papiloma humano 6 y 12

d) Radiación ionizante

e) Hidrocarburos

PA33. De los factores intrínsecos siguientes no es factor para cáncer de células escamosas:

a) Xeroderma pigmentoso

b) Albinismo

c) Inmunosupresión

d) Dermatosis inflamatorias

e) Pecas de Hutchinson

PA34. Pacientes con xeroderma pigmentoso pueden desarrollar lesión neoplásica de piel desde:

a) La edad pediátrica

b) Los 20 a los 30 años

c) Después de los 40 años

d) Nunca presentan lesiones malignas

e) Después de los 70 años

PA35. Los rayos ultravioleta b producen:

a) Activación de la proteína P53

b) Inhibe la acción de las ciclinas

c) Induce la formación de dímeros del DNA


d) Induce la formación de dímeros del RNA

e) Aumenta el número de células de Langerhans en la piel

PA36. Los siguientes son efectos de la luz ultravioleta, excepto:

a) Induce la formación de dímeros de DNA

b) Hay disminución de la inmunidad celular

c) Disminuye el número de células de Langerhans en la piel

d) Altera la acción de las células Natural Killer

e) Produce aumento de la producción de anticuerpos

PA37. De los cánceres de piel, el de células escamosas representa el:

a) 10%

b) 20%

c) 30%

d) 40%

e) 50%

PA38. Gran parte de los cánceres de células escamosas se localizan en:

a) Tronco

b) Extremidades superiores

c) Cabeza y cuello

d) Extremidades inferiores

e) Igual porciento en todas las áreas

PA39. Del cáncer de piel se puede decir que:

a) Es la primera causa de cáncer en los Estados Unidos

b) Su incidencia sólo es superada por el cáncer de pulmón

c) En el resto del mundo constituye la segunda causa de cáncer después del de mama
d) Tiene el mismo comportamiento que el melanoma en las áreas tropicales

e) Su comportamiento es tan lento que no provoca mortalidad

PA40. Señale el enunciado correcto con respecto a carcinoma epidermoide de piel:

a) Hasta el 42% de los pacientes con transplante renal desarrollaran cáncer de piel

b) Es más frecuente el carcinoma basocelular que el epidermoide en pacientes inmunosuprimidos

c) Los pacientes cursan con los linfocitos T auxiliares aumentados y linfocitos T supresores
disminuidos

d) La cantidad de linfocitos es mucho mayor alrededor del tumor mientras más grandes son estos

e) El antígeno de CCS determinado en el tumor es el mejor predictor de metástasis

PA41. ¿Cuál es la incidencia de metástasis ganglionar reportada en general a ganglios linfáticos de


carcinoma epidermoide?

a) 0.5 a 3.7%

b) 4 al 8%

c) 9 al 13%

d) 15 al 20%

e) Más del 30%

PA42. Cuando el carcinoma de células escamosas invade hasta las estructuras nerviosas ¿cuál es su potencial
metastático?

a) 1 al 10%

b) 10 al 20%

c) 30 al 40%

d) 40 al 50%

e) Más del 50%

PA43. Cuál de los datos siguientes es el factor pronóstico más importante en un paciente con CCS.

a) Cambio de coloración de la lesión tumoral


b) Tumor moderadamente diferenciado

c) Diámetro tumoral de 1.5 cm

d) Profundidad de invasión mayor de 6 mm

e) Ulceración y sangrado

PA44. La sobrevida a 5 años de pacientes con carcinoma epidermoide de piel con metástasis a ganglios
linfáticos es del:

a) 90% o más

b) 60 a 80%

c) 40 al 60%

d) 20 al 40%

e) Menos del 20%

PA45. Al paciente con carcinoma epidermoide de piel con metástasis a ganglios linfáticos debe tratarse con:

a) Radioterapia, únicamente

b) Excisión amplia más disección ganglionar

c) Cirugía más radioterapia

d) Quimioterapia neoadyuvante

e) Radioterapia más quimioterapia adyuvante

PA46. El 50% de las recurrencias por cáncer epidermoide de piel se presentan durante:

a) El primer año

b) Los 2 primeros años

c) Los primeros 5 años

d) Los 5 a 10 años

e) Después de los 10 años

PA47. ¿Cuál es el margen quirúrgico que se debe dar como mínimo para realizar una buena excisión de un
carcinoma epidermoide de piel?
a) 1 cm

b) 2 cm

c) 3 cm

d) 4 cm

e) No es necesario dar márgenes por que no recurren

PA48. La queratosis actínica se caracteriza por:

a) Displasia celular únicamente en la capa superficial de la piel

b) Displasia que abarca hasta la capa de células granulosas

c) Displasia que abarca hasta la capa de células espinosas

d) Displasia en cualquiera de las capas por encima de la membrana basal

e) Displasia que alcanza la dermis superficial

PA49. Son características de la papulosis Bowenoide, excepto que:

a) Es una variedad de carcinoma epidermoide infiltrante

b) Es una lesión potencialmente maligna, considerada por algunos como cáncer in situ

c) Se presenta como múltiples pápulas violáceas

d) Se ubica en el área de los genitales

e) Es causada por el VPH 16

PA50. El carcinoma verrucoso:

a) Es un carcinoma de células escamosas de bajo grado

b) Es un tumor de crecimiento lento

c) Puede invadir hasta hueso y aun así no presentar metástasis

d) Su etiología se relaciona con el VPH 6 y 11

e) Todas lo anterior es correcto

PA51. El carcinoma de células basales:


a) Representa el 70 a 80% de las lesiones neoplásicas de la piel

b) Se ubica con mayor frecuencia en extremidades

c) Da frecuentemente metástasis ganglionares

d) Se desarrollan principalmente en áreas cubiertas

e) B y d son correctas

PA52. Frecuencia de asociación entre presencia de un cuerno cutáneo y una lesión subyacente:

a) Menos del 10%

b) del 10 al 20%

c) del 30 al 40%

d) del 50% o más

e) Nunca se asocia con una neoplasia

PA53. ¿Cuál de las siguientes es la variedad más frecuente de carcinoma basocelular?

a) Nodular

b) Superficial

c) Pigmentado

d) Esclerosante o morfea

e) Quístico basal

PA54. ¿Cuál es el margen quirúrgico que debería darse como mínimo al resecar un cáncer basocelular?

a) 5 mm

b) 10 mm

c) 15 mm

d) 20 mm

e) No es necesario dar margen

PA55. El carcinoma basoescamoso:

a) Es sinónimo del tipo Morphea


b) Tiene el mismo grado de agresividad que un cáncer basocelular

c) Está formado por un cáncer basocelular con un componente epidermoide benigno

d) Posee componentes epidermoide maligno y basocelular benigno

e) Posee componentes epidermoide y basocelular malignos

PA56. ¿Cuál de los siguientes no es tratamiento óptimo para un cáncer basocelular?

a) Electrodisecación

b) Cirugía de Mohs

c) Aplicación de Podofilina

d) Escisión con láser

e) Radioterapia

PA57. La utilización de interferón intralesional en carcinoma de células basales es una técnica:

a) Actualmente aceptada como estándar

b) No útil

c) Está en protocolos de investigación

d) Que nunca se ha utilizado

e) Que tiene el mismo efecto que los retinoides

PA58. La edad de mayor incidencia de melanoma en México es:

a) De los 20 a los 35 años

b) De los 15 a los 25 años

c) De los 40 a los 60 años

d) Mayores de 80 años

e) Menores de 30 años

PA59. Son factores asociados a una frecuencia mayor de melanoma, excepto:

a) Xeroderma pigmentoso
b) Exposición solar

c) Tipos de piel V y VI de Fitzpatrick

d) Antecedente familiar

e) Antecedente de melanoma previo

PA60. El riesgo de malignización del nevo melanocítico gigante es aproximadamente del:

a) 3 a 15%

b) 20 a 30%

c) 28 a 45%

d) 41 a 60%

e) 85 a 95%

PA61. El riesgo de melanoma en los pacientes con síndrome de nevo atípico o síndrome B-K, en la
clasificación D2 de Kraemer, es de:

a) 10-20%

b) 20-30%

c) 30-40%

d) 50-70%

e) 90-100%

PA62. Es el patrón de crecimiento clínico-patológico más común del melanoma en la población anglosajona:

a) Diseminación superficial

b) Léntigo maligno

c) Acral lentiginoso

d) Nodular

e) Desmoplásico

PA63. Es el patrón de crecimiento clínico-patológico más común en México:


a) Diseminación superficial

b) Lentigo maligno

c) Acral lentiginoso

d) Nodular

e) Desmoplásico

PA64. Son características clínicas compatibles con el diagnóstico de melanoma en una lesión pigmentada:

a) Diámetro mayor de 6 mm

b) Color uniforme

c) Bordes bien definidos

d) Simetría de la lesión

e) Elevación de la lesión

PA65. Además de los estudios básicos preoperatorios (biometría hemática, tiempos de coagulación y de
valoración cardiovascular en los casos requeridos), son estudios de extensión rutinarios y necesarios para los
pacientes con melanoma con Breslow menor de 1 mm y sin datos de pobre pronóstico:

a) TAC del área afectada y de cráneo

b) Telerradiografía del tórax y pruebas de función hepática

c) Ultrasonografía hepática

d) Centellografía ósea

e) TAC de tórax

PA66. Los factores pronósticos principales del melanoma, en ausencia de ganglios afectados, son:

a) Niveles de Breslow y Clark

b) Nivel de Breslow y presencia de ulceración

c) Nivel de Clark y presencia de ulceración

d) Nivel de Breslow, edad y localización anatómica

e) Nivel de Clark, edad y localización anatómica


PA67. Es el margen adecuado para las lesiones primarias con reporte de melanoma in situ:

a) 0.5 cm

b) 1 cm

c) 2 cm

d) 4 cm

e) 5 cm

PA68. Es el beneficio de la linfadenectomia terapéutica en melanoma:

a) 10%

b) 30%

c) 50%

d) 70%

e) 80%

PA69. Son los pacientes beneficiados de una linfadenectomía electiva

a) Mayores de 60 años con lesiones con Breslow menor de 1 mm y ulcerados

b) Menores de 60 años con lesiones con Breslow entre 3 y 4 mm y no ulcerados

c) Menores de 60 años con lesiones con Breslow entre 0.5 y 1 mm no ulcerados

d) Menores de 60 años con lesiones con Breslow entre 1 y 2 mm no ulcerados

e) Mayores de 60 años con lesiones con Breslow entre 1 y 2 mm ulcerados

PA70. Son indicaciones de determinación de ganglio centinela en melanoma:

a) Lesiones con Breslow menor de 1 mm, no ulcerados, en fase de crecimiento horizontal sin
datos de regresión

b) Lesiones con Breslow mayor de 1 mm y ganglios clínicamente negativos

c) Lesiones con Breslow mayor de 1 mm y ganglios clínicamente sospechosos

d) Lesiones con Breslow mayor de 4 mm y ganglios clínicamente negativos con lesiones en


tránsito

e) Lesiones con Breslow menor de 4 mm con ganglios clínicamente sospechosos


PA71. Es indicación absoluta de linfocentellografía preoperatoria en las lesiones localizadas en:

a) Planta del pie

b) Subungueal

c) Pierna

d) Tórax

e) Antebrazo

PA72. En relación con la terapia adyuvante con interferón de altas dosis la aseveración correcta es:

a) Es una dosis bien tolerada

b) Se recomienda utilizar durante tres meses

c) Se aplica mensualmente

d) La dosis es de 10’000,000 U tres veces a la semana

e) Mejora la supervivencia en un 50%

PA73. Son los componenetes del esquema Darmouth, ampliamente utilizado en Melanoma:

a) Platino, BCNU, DTIC, tamoxifén

b) Platino, ciclofosfamida, etopósido

c) Bleomicina. tamoxifén y taxanos

d) Platino y taxol

e) Bleomicina, etopósido y platino

PA74. Son las drogas con mayor tasa de respuesta en perfusión aislada de extremidades:

a) Factor de necrosis tumoral y platino

b) Factor de necrosis tumoral y taxanos

c) Melfalán y platino

d) Melfalán y taxanos

e) Melfalán y factor de necrosis tumoral


PA75. Son indicaciones de radioterapia adyuvante después de disección ganglionar:

a) Extensión extracapsular y más de tres ganglios linfáticos afectados

b) Más de tres ganglios linfáticos afectados y ulceración de la lesión primaria

c) Un ganglio afectado y permeación vascular y linfática

d) Ganglio centinela positivo

e) Todos los pacientes deberán recibir radioterapia posoperatoria

PA76. Es el tratamiento de elección para las metástasis únicas encefálicas resecables por melanoma:

a) Cirugía exclusivamente

b) Cirugía, radioterapia y tomozolamida

c) Radioterapia y tomozolamida

d) Cirugía y tomozolamida

e) Tomozolamida exclusivamente

PA77. Son los factores pronósticos principales en enfermedad metastásica por melanoma:

a) Sitio metastásico y profundidad de invasión

b) Sitio metastásico y número de ganglios afectados

c) Sitio metastásico y edad

d) DHL y Breslow

e) DHL y sitio metastásico

CIRUGÍA RECONSTRUCTIVA ONCOLÓGICA

CR1. El principal aporte sanguíneo del colgajo frontal proviene de los vasos:

a) Músculo temporal

b) Arteria supratroclear

c) Arteria supraorbitaria

d) Arteria temporal superficial


e) Arteria maxilar interna

CR2. Los objetivos en la reconstrucción orofaríngea incluyen los siguientes, excepto:

a) Mantener la continencia oral

b) Proveer un gran reservorio que permita acomodar el bolo alimenticio

c) Evitar la aspiración

d) Preservar el habla

e) Proteger estructuras vitales

CR3. El colgajo nasolabial es efectivo para reconstruir los defectos siguientes excepto:

a) Alveolo superior

b) Punta nasal

c) Trígono retromolar

d) Piso anterior de la boca

e) Labio superior

CR4. El eje vascular del músculo pectoral mayor corre:

a) Paralelo a la dirección de sus fibras

b) Paralelo al esternón

c) Siguiendo una línea de la punta del hombro al xifoides

d) Siguiendo una línea del punto medioclavicular al xifoides

e) Siguiendo una línea de la orquilla esternal al complejo areola-pezón

CR5. Cuál de los siguientes enunciados es cierto con respecto al colgajo miocutáneo de pectoral mayor:

a) La isla de piel debe posicionarse completamente sobre el músculo para mantener la


vascularidad a través de las perforantes

b) El colgajo debe pasarse por debajo de la clavícula para alcanzar la cabeza

c) Generalmente debe cerrarse el área donadora con un injerto de piel


d) El colgajo puede incluir una o más islas de piel

e) El aporte vascular principal es a través de la arteria torácica lateral

CR6. Cuál de los siguientes colgajos osteocutaneos libres (microquirúrgicos) es el más utilizado para
reconstrucción mandibular:

a) Cresta iliaca

b) Peroné

c) Radio

d) Escapular

e) Dorsal pedio

CR7. Los siguientes son colgajos fasciocutáneos o musculares que pueden incorporar un segmento óseo, que
recibe irrigación a través del mismo pedículo vascular, excepto:

a) Serrato anterior

b) Recto abdominal

c) Lateral de brazo

d) Deltopectoral

e) Esternocleidomastoideo

CR8. Cuál de los siguientes enunciados es FALSO con respecto a la osteoradionecrosis de cabeza y cuello:

a) El principal síntoma es dolor

b) La fisiopatología es debida a hipoxia, hipovascularidad e hipocelularidad

c) El hueso maxilar es más vulnerable que la mandíbula

d) Se asocia más frecuentemente a extracciones dentarias

e) La falta de respuesta a tratamiento conservador requiere de resección amplia de los tejidos


afectados y reconstrucción inmediata

CR9. Los siguientes colgajos tubulizados han sido descritos para reconstrucción de hipofaringe y esófago
cervical, excepto:

a) Radial de antebrazo
b) Recto abdominal

c) Deltopectoral

d) Un segmento de yeyuno

e) Pectoral mayor

CR10. ¿Cuál es el máximo diámetro de un defecto en el dorso nasal, que puede ser reconstruido con un
colgajo local de transposición (tipo Limberg) o de rotación del dorso nasal?

a) 1 cm

b) 2 cm

c) 3 cm

d) 4 cm

e) 5 cm

CR11. Las siguientes son posibles complicaciones de un colgajo de rotación-avance de mejilla tipo
Mustardé para reconstrucción de párpado inferior, excepto:

a) Pérdida de pestañas

b) Ptósis y retracción del colgajo de mejilla

c) Necrosis del colgajo secundaria a isquemia

d) Sinblefarón lateral

e) Un canto lateral redondo

CR12. Las siguientes arteria y vena son vasos receptores útiles para anastomosis de colgajos libres
(microvasculares) en reconstrucción de mama y tórax, excepto:

a) Toracodorsal

b) Subclavia

c) Mamaria interna

d) Torácica lateral

e) Tiroidea superior y yugular externa


CR13. El colgajo más utilizado para reconstrucción mamaria con tejido autólogo es:

a) Dorsal ancho

b) Recto abdominal

c) Glúteo superior

d) Glúteo inferior

e) Toracoepigástrico

CR14. Cuál de los colgajos siguientes es el más utilizado para reconstrucción total del pene:

a) Libre osteocutaneo de peroné

b) Inguinal tubulizado

c) Libre osteocutaneo dorsal pedio

d) Libre lateral de brazo

e) Libre radial de antebrazo

CR15. El tiempo de isquemia caliente (desconectado de su aporte sanguíneo a temperatura ambiente) que
puede tolerar un colgajo muscular o miocutáneo, antes de sufrir daño por isquemia-reperfusión es de:

a) 4 horas

b) 5 horas

c) 6 horas

d) 7 horas

e) 8 horas

CR16. Los colgajos libres (microvasculares) siguientes pueden ser sensitivos si se incluye el nervio que
provee inervación al segmento cutáneo y se anastomosa a un nervio sensitivo en el sitio receptor, excepto:

a) Osteocutáneo de peroné

b) Radial de antebrazo

c) Dorsal pedio

d) Anterior de muslo

e) Lateral de brazo
CR17. ¿Cuál es la mejor opción para reconstruir un defecto de huesos largos mayor de 5.0 cm, después de
proveer una adecuada cubierta cutánea?

a) Un injerto de hueso no-vascularizado

b) Sinostosis tibioperoneal

c) Un injerto vascularizado de peroné ipsilateral

d) Un injerto de peroné vascularizado contralateral

e) Un injerto vascularizado de cresta ilíaca

TUMORES INFANTILES

IN1. ¿Qué patrón histológico de tumor renal da más frecuentemente metástasis a cerebro?

a) Epitelial

b) Sarcoma de células claras

c) Mixto

d) Rabdoide

e) Mesenquimatoso

IN2. El riesgo de desarrollar leucemia en un niño con síndrome de Down es:

a) 1 de cada 150

b) 1 de cada 2880

c) 1 de cada 95

d) 1 de cada 10

e) 1 de cada 3500

IN3. El diagnóstico definitivo de leucemia se establece con:

a) Estudio de líquido cefalorraquídeo

b) Análisis del frotis de sangre periférica

c) Biopsia de ganglio linfático

d) Estudio de médula ósea por aspiración

e) Marcadores inmunológicos
IN4. La causa más importante de mortalidad en niños con leucemia aguda es:

a) Insuficiencia cardíaca congestiva secundaria a infiltración del miocardio

b) Infiltración al sistema nervioso central

c) Hemorragia secundaria a trombocitopenia

d) Infección

e) Insuficiencia renal aguda secundaria a hiperuricemia

IN5. La incidencia mayor de leucemia aguda linfoblástica es:

a) Entre 1 y 2 años de edad

b) Entre 3 y 5 años de edad

c) Entre 7 y 10 años de edad

d) Entre 10 y 15 años de edad

e) Antes del año de edad

IN6. La presentación clínica más frecuente de leucemia meníngea es:

a) Fiebre, anorexia y alteraciones visuales

b) Cefalea, vómito y letargia

c) Irritabilidad, convulsiones y coma

d) Ataxia y parálisis de los nervios craneales

e) Parálisis del nervio facial

IN7. ¿Cuál de las siguientes es la complicación más frecuente de la leucemia aguda no linfoblástica?

a) Infiltración meníngea

b) Hiperuricemia

c) Hemorragia intracerebral

d) Trombocitopenia

e) Visceromegalia

IN8. ¿Cuál es el tumor sólido más frecuente en pediatría en la República Mexicana?


a) Tumor de Wilms

b) Neuroblastoma

c) Enfermedad de Hodgkin

d) Tumores del sistema nervioso central

e) Retinoblastoma

IN9. Factores pronósticos más significativos en el tumor de Wilms:

a) Edad y etapa

b) Etapa y sexo

c) Histología y edad

d) Histología y etapa

e) Sexo y edad

IN10. Las entidades siguientes se asocian con hemihipertrofia, excepto:

a) Tumor de Wilms

b) Hepatoblastoma

c) Adenocarcinoma suprarrenal

d) Síndrome de Beckwith-Wiedemann

e) Rabdomiosarcoma de vías biliares

IN11. La sobrevida a dos años en tumores de Wilms con histología desfavorable según el Estudio Nacional
de Wilms:

a) 45%

b) 72%

c) 39%

d) 53%

e) 10%

IN12. El sitio menos frecuente de metástasis del neuroblastoma es:

a) Hueso
b) Hígado

c) Bazo

d) Médula ósea

e) Ganglio linfático

IN13. ¿En cuál de las estructuras siguientes se origina el carcinoma de células renales?

a) Túbulos contorneados proximales

b) Túbulos contorneados distales

c) Pelvis renal

d) Glomérulo

e) Asa de Henle

IN14. ¿Cuál de los huesos siguientes es el que con mayor frecuencia se afecta en el sarcoma de Ewing?

a) Húmero

b) Fémur

c) Tibia

d) Escapula

e) Cúbito

IN15. Con cuál de las entidades siguientes se debe de diferenciar el fibrosarcoma:

a) Fibromatosis juvenil

b) Fascitis nodular

c) Dermatofibrosarcoma protuberans

d) a y b

e) Todas las anteriores

IN16. La etapificación de Chang es indispensable en el tumor:

a) Astrocitoma pilocítoco

b) Ependimoma

c) Tumor primario germinal del sistema nervioso


d) Glioma del puente

e) Méduloblastoma

IN17. ¿Cuál es la neoplasia maligna más frecuente en pediatría a nivel mundial?

a) Tumor de Wilms

b) Tumores del sistema nervioso central

c) Leucemia aguda

d) Enfermedad de Hodgkin

e) Linfoma no Hodgkin

IN18. El tratamiento del neuroblastoma etapa I es:

a) Cirugía

b) Cirugía más radioterapia

c) Cirugía, radioterapia y quimioterapia

d) Cirugía más quimioterapia

e) Radioterapia

IN19. La radioterapia para méduloblastoma debe incluir:

a) Radiación a todo sistema nervioso

b) Radiación a campos comprometidos

c) Radiación a columna

d) No está indicada la radiación

e) Radiación al sistema nervioso central y columna vertebral

IN20. La fracción beta de gonadotrofina coriónica:

a) Tiene una vida media de 18 a 48 horas

b) Es producida por el sincitiotrofoblasto

c) Se eleva en coriocarcinoma
d) Se eleva en algunos seminomas

e) Se da en todos los anteriores

IN21. La acronimia WARG incluye las características siguientes:

a) Deleción del 11p15.5

b) Macrosomia

c) Tumor de Wilms

d) Supresión de WT 2

e) Todas las anteriores

IN22. Con respecto al Tumor de Wilms los factores pronósticos en recaída son:

a) El sitio de recaída

b) El tratamiento utilizado previamente

c) El tiempo de la recaída

d) Todos los anteriores

e) Ninguno de los anteriores

Correlacione las columnas entre tipo de cáncer y ocupación paterna:

IN23. TSNC ( ) a) Campos electromagnéticos

IN24. Tumor de Wilms ( ) b) Solventes en aviación

IN25. Leucemia ( ) c) Industria del plomo

IN26. Hepatoblastoma ( ) d) Soldadura

IN27. Neuroblastoma ( ) e) Gasolinera

IN28. De las citoquinas siguientes cuál estimula la proliferación del linfoma y es expresada por la célula de
Reed-Sternberg:

a) IL-1

b) IL-13
c) IL-9

d) IL-5

e) Ninguna de las anteriores

IN29. En el neuroblastoma, en qué fase del ciclo celular arrestan células los retinoides:

a) S

b) M

c) G1

d) G2

e) G0

IN30. De los siguientes factores predisponentes para osteosarcoma, ¿Cuál se asocia a un mal pronóstico?

a) Radioterapia

b) Enfermedad de Ollier

c) Enfermedad ósea de Paget

d) Retinoblastoma

e) Todos los anteriores

IN31. La indicación de resecar completamente el teratoma diferenciado por quimioterapia:

a) Mantiene la alteración cromosómica y puede indiferenciarse nuevamente

b) Realmente no existe indicación y se puede dejar sin resecar

c) Es resistente a la quimioterapia y radioterapia

d) Sólo a y c

e) Ninguna de las anteriores

IN32. En relación con el rabdomiosarcoma (RMS), cuál de las aseveraciones siguientes es correcta:

a) El RMS embrionario ha sido mapeado en el llp15.5

b) En etapas III y IV la histología pierde valor pronóstico


c) El RMS de oído medio es parameníngeo y puede ser botrioide

d) El RMS de órbita sin metástasis tiene buen pronóstico

e) Todas las anteriores

IN33. Con respecto a la hidroxidoxorrubicina:

a) Antraciclico no fase específico

b) Se debe administrar en infusión de 24 horas para disminuir la cardiotoxicidad

c) La dosis tope debe ser disminuida si se radia el mediastino

d) Se degrada con la luz

e) Todas las anteriores

En relación con el tumor de Wilms, relacione las preguntas siguientes:

IN34. Blastema ( ) a) Permanece después de quimioterapia

IN35. Epitelio ( ) b) Resistente a la quimioterapia

IN36. Estroma ( ) c) Gran capacidad de invasión

IN37. Anaplásico ( ) d) Buen pronóstico con quimioterapia

IN38. Con respecto a RMS alveolar, cuál de las aseveraciones siguientes es correcta:

a) Frecuente localización en extremidades.

b) La alteración cromosómica asociada es t(2 ;13) (q35 ;q14)

c) El SNC es sitio frecuente de metástasis

d) La adolescencia es la etapa de la vida más afectada

e) Todas las anteriores

IN39. Basados en lo subclínico del hepatoblastoma comparado con el hepatocarcinoma, la ictericia es más
frecuente en el primero:

a) Verdadero b) Falso
IN40. ¿Cuál de las enfermedades siguientes no es factor predisponente para leucemia aguda?

a) Síndrome de Bloom

b) Síndrome de Down

c) Neurofibromatosis

d) Ataxia telangiectasia

e) Enfermedad de Hirschprung

IN41. Con respecto a trisomía 21 y leucemia linfoblástica aguda:

a) Se asocia frecuentemente a mortalidad por toxicidad al tratamiento

b) Su buen pronóstico es debido a que generalmente se presenta el inmunofenotipo pre B

c) Su buen pronóstico es porque las alteraciones cromosomicas de alto riesgo no están presentes
en ellos

d) a y b

e) a y c

IN42. Las leucemias agudas de inmunofenotipo T son más frecuentes en:

a) Adolescentes

b) Países altamente desarrollados

c) Síndrome leucemia linfoma

d) Todas las anteriores

e) Ninguna de las anteriores

Relacione las columnas:

IN43. Síndrome de Klinefelter ( ) a) Hepatocarcinoma


IN44. Enfermedad de Ollier ( ) b) Tumores de SNC
c) Osteosarcoma
d) Hepatoblastoma
e) Leucemias

IN45. Son genes recesivos los siguientes, excepto:


a) RB1

b) p 53

c) NF1

d) MYC

e) VHL

Relacione las columnas siguientes:

IN46. S. Soto ( ) a) WT1

IN47. WAGR ( ) b) WT2

IN48. Del. 11p13 ( )

IN49. Restos metanéfricos ( )

IN50. S. Denys Drash ( )

IN51. S. Beckwith Wiedemann (b)

IN52. Los siguientes son factores pronósticos independientes en leucemia linfoblástica aguda, excepto:

a) Edad

b) Citomorfología

c) Cuenta de leucocitos mayor a 50,000

d) Cromosoma Philadelphia

e) Inmunofenotipo T

Relacione las columnas siguientes

IN53. CD 1 ( b ) a) My10/HPCA-1

IN54. CD 34 ( a ) b) T6

IN55. CD 10 ( c ) c) CALLA-J5

IN56. CD 24 ( e ) d) B4

IN57. CD 19 ( d ) e) BA-1
Relacione las columnas siguientes por orden de frecuencia en las complicaciones, la enfermedad injerto
contra huésped:

IN58. Exantema 25 al 50% M2sc ( ) a) x

IN59. Eritroderma generalizado ( ) b) x x

IN60. Bilirrubina 3 a 6 mg- Dl ( ) c) x x x

IN61. Dolor abdominal o íleo ( ) d) x x x x

IN62. Es un efecto secundario al uso conjunto de radioterapia abdominal y actinomicina D:

a) Perforación intestinal

b) Disminución de la secreción de colecistoquinina

c) Enfermedad veno-oclusiva hepática

d) Incremento del riesgo de hepatoblastoma en l5 veces

e) Aumento de la secreción tubular renal de potasio

Respecto a enfermedad de Hodgkin relacione las columnas siguientes:

IN63. Afecta del 10-15% de los pacientes y su presentación es a) Depleción linfocitaria


localizada ( )
b) Celularidad mixta
IN64. Se presenta en pacientes HIV(+) ( ) c) Predominio linfocitario
IN65. Se caracteriza por la variedad lacunar de la célula de Reed- d) Esclerosis nodular
Sternberg ( )
IN66. Presenta abundantes células de Reed-Sternberg ( )

IN67. Son fenómenos inmunológicos asociados a enfermedad de Hodgkin, excepto:

a) Púrpura trombocitopénica idiopática

b) Anemia hemolítica

c) Enfermedad de Kawasaky

d) Síndrome nefrótico

IN68. Son características del tumor de Ewing, excepto:

a) Tumor de células pequeñas redondas y azules

b) Se asocia a t(11,22)
c) Su origen es mesenquimatoso

d) Se presenta en las diáfisis óseas

Relacione las dosis de radioterapia con las neoplasias siguientes:

IN69. 2400cGy (d) a) Control local de Tumor de Wilms

IN70. 4500-6000cGy ( b ) b) Tumor primario SNC

IN71. 1200cGy (c) c) Leucemia testicular

IN72. 1000cGy (a) d) Leucemia SNC

IN73. Característica de la histiocitosis de células de Langerhans:

a) Produce síndrome hemofagocítico

b) Pertenece a clase I de histiocitosis

c) Los gránulos de Birbeck en médula ósea es patognomónico

d) Es de buen pronóstico en menores de l año

IN74. Son características citoquímicas de la leucemia linfoblástica aguda:

a) PAS positiva

b) Fosfatasa alcalina leucocitaria positiva

c) Sudán negro negativo

d) Mieloperoxidasa negativa

e) Todas las anteriores

Relacione las siguientes columnas:

IN75. Sitio común mandíbula-órbita ( ) a) Endémico

IN76. Incidencia 0.2/100 mil ( ) b) Esporádico

IN77. Secreción de IgM ( )

IN78. Son características de la leucemia mieloide aguda relacionada al tratamiento:


a) La médula ósea puede presentar fibrosis reticular extensa

b) Presencia de trisomía 8 y monosomía 7

c) El uso de VP-16 a altas dosis se relaciona con alteración en Cr11q23

d) Pueden ser secundarias al uso de alquilantes y radioterapia

e) Todas las anteriores

Relacione las columnas:

IN79. Sx de Kerner.Morrison ( ) a) Aumento del péptido


intestinal vasoactivo
IN80. Sx de Pepper ( ) b) Opsoclonus, mioclonus
IN81. Sx de Kisbourne ( ) c) Hepatomegalia masiva

IN82. Variedad de osteosarcoma que se origina en la corteza:

a) Osteoblástico

b) Condroblástico

c) Paraosteal

d) Telangiectásico

e) Células redondas

IN83. En la terapia quirúrgica del osteosarcoma:

a) Todo paciente con el diagnóstico de osteosarcoma debe ser sometido a cirugía definitiva antes
de iniciar quimioterapia

b) La cirugía conservadora esta contraindicada ante cualquier fractura patológica

c) El cirujano debe estar totalmente seguro de dejar los bordes libres de tumor

d) La toma de biopsia abierta está totalmente contraindicada

e) El principal objetivo de la quimioterapia neoadyuvante es el preservar la extremidad

IN84. La cistitis hemorrágica como efecto secundario a la administración de Ciclofosfamida se debe al


metabolito siguiente:

a) Acroleina

b) Aldofosfamida
c) 4- Hidroxiciclofosfamida

d) 4-Ketoxiciclofosfamifa

e) Carboxiciclofosfamida

IN85. El neuroblastoma con diploidia es resistente a la quimioterapia siguiente:

a) Ciclofosfamida

b) Metotrexate

c) Doxorrubicina

d) Cisplatino

e) a y c

IN86. Vía de diseminación extraocular del retinoblastoma:

a) Invasión de nervio óptico vía hematógena y linfática

b) Vía hematógena, linfática, invasión de nervio óptico y extensión directa por los vasos esclerales

c) Vía hematógena, invasión de nervio óptico y LCR

d) Vía linfática, invasión de nervio óptico y siembras en humor vitreo

e) Siembras en humor vítreo, líquido cefalorraquídeo y vía hematógena

IN87. Las rosetas de Flexner-Winsternsteiner son grupos de células alrededor del neuropilo

a) Verdadero b) Falso

IN88. Mecanismo de producción para retinoblastoma trilateral:

a) Diseminación a través del nervio óptico en retinoblastoma bilateral sincrónico

b) Aparición de retinoblastoma intracraneal en células de origen fotoreceptor en la glándula pineal

c) Diseminación através del LCR de retinoblastoma bilateral

d) Diseminación por vía hematógena de retinoblastoma bilateral

e) Extensión de retinoblastoma bilateral a través del espacio subaracnoideo hacia la glándula


pineal
IN89. En la histiocitosis de células de Langerhans, cuál de las secuelas siguientes es siempre irreversible:

a) Exoftalmos

b) Lesión lítica

c) Diabetes insípida

d) Dermatitis seborreica

e) Inmunodeficiencia

IN90. Las neoplasias malignas germinales primarias de mediastino se pueden asociar a:

a) Síndrome de coagulación intravascular diseminada

b) Pubertad precoz

c) Leucemia aguda no linfoblástica M3

d) Todas las anteriores

e) Ninguna de las anteriores

Relacione las columnas siguientes con respecto a leucemias agudas no linfoblásticas:

IN91. M0 ( ) a) Se relaciona con CID


IN92. M1 ( ) b) Frecuentemente da cloromas
IN93. M2 ( ) c) El diagnóstico se hace por
inmunofenotipo
IN94. M3 ( ) d) Es el subtipo más frecuente
IN95. M4 ( ) e) Presenta > 90% de blastos y menos de
10% de granulocitos o monocitos en
maduración

IN96. En la leucemia aguda megacarioblastica (M7), la peroxidasa plaquetaria se comprueba por


microscopia electrónica.

a) Verdadero b) Falso

IN97. Son considerados factores adversos en leucemia aguda no linfoblástica (LANL), los siguientes,
excepto:

a) Cuenta de leucocitos de más de 100 000

b) LANL secundaria
c) Monosomía 7

d) LANL subtipo M1 con cuerpos de Auer

e) LANL precedida por síndrome mielodisplásico

IN98. En leucemia aguda no linfoblástica con el fin de bloquear la glicoproteína-p se ha usado el fármaco
siguiente:

a) Factor estimulante de colonias de granulocitos y fenitoina

b) Verapamil y ciclosporina

c) Interferón a y ATRA

d) Ninguno de los anteriores

IN99. El porciento de leucemias agudas no linfoblásticas que no entran en remisión es de:

a) 1 a 2

b) 5 a 10

c) 15 a 20

d) 30 a 40

e) Ninguna de las anteriores

IN100. Son factores de riesgo para el desarrollo de la enfermedad de Hodgkin los siguientes, excepto:

a) Ataxia-telangiectasia

b) Tirosinemia

c) Infección por virus Eptein-Barr

d) Infección por citomegalovirus

e) Inmunodeficiencia iatrogénica

IN101. Variedad histológica de la enfermedad de Hodgkin que se relaciona en un 96% de positividad con la
proteína de membrana latente del virus Epstein-Barr:

a) Esclerosis nodular

b) Predominio linfocitario
c) Depleción linfocitaria

d) Celularidad mixta

IN102. Causa(s) de la anemia en enfermedad de Hodgkin:

a) Alteración en la movilización de los depósitos de hierro

b) Hemólisis

c) Infiltración a médula ósea

d) Todas los anteriores

e) Sólo b y c son correctos

IN103. La laparotomía estadificadora en enfermedad de Hodgkin sólo debe realizarse cuando ésta modifica
el tratamiento final:

a) Verdadero b) Falso

IN104. En los niños el linfoma no-Hodgkin habitualmente es extralinfático.

a) Verdadero b) Falso

Relacione las columnas con respecto a las variedades histológicas de linfoma no-Hodgkin:

IN105. Células B ( ) a) Linfoblástico


IN106. Primario mediastinal ( ) b) Burkitt
IN107. Expresa CD 30 ( ) c) Células
grandes
IN108. Presenta la traslocación t(2;5) ( )
IN109. Indistinguible de leucemia
aguda linfoblástica ( )

IN110. La expresión de TRK A se relaciona inversamente con la expresión de MYCN.

a) Verdadero b) Falso

IN111. El sitio primario más afectado por neuroblastoma es:

a) Mediastino
b) Cuello

c) Abdomen

d) Suprarrenal

e) Desconocido

IN112. Todos los síndromes siguientes son paraneoplásicos en neuroblastoma, excepto:

a) Opsoclonos-mioclonus

b) Homer-Wright

c) Horner

d) Kerner-Morrison

e) Hutchinson

IN113. La deleción o pérdida del alelo del cromosoma 1 se asocia con mal pronóstico en neuroblastoma,
independientemente de la etapa.

a) Verdadero b) Falso

IN114. El tratamiento del neuroblastoma se basa en la etapa y edad del paciente. Los pacientes considerados
como de bajo riesgo son:

a) Pacientes mayores de un año con etapa 1 y 2A y menores de un año con etapas 2B, 3 y 4S

b) Pacientes menores de un año independientemente de la etapa

c) Pacientes mayores de un año con etapas 1, 2A, 2B y 3

d) Pacientes con etapa 1

IN115. El síndrome de Denys-Drash se compone de:

a) Hemihipetrofia, hipoglicemia y tumor de Wilms

b) Aniridia, retardo mental, alteraciones genitourinarias

c) Tumor de Wilms, hemihipertofia, seudohermafroditismo

d) Seudohermafroditismo, falla renal progresiva y tumor de Wilms


IN116. Una de las indicaciones en el tumor de Wilms bilateral es la quimioterapia neoadyuvante:

a) Verdadero b) Falso

IN117. El gen p53:

a) Es un oncogén localizado en el cromosoma 17q13

b) Se encuentra mutado en sólo el 50% de los sarcomas

c) Es una mutación del gen p53 es exclusivamente somática

d) La vida media del p53 mutado es menor

e) Todas las anteriores

IN118. En el osteosarcoma la inactivación funcional de RB1 y p53 son requeridas para su iniciación:

a) Otros eventos están ligados con la pérdida de estos dos genes como la ampliación de c- myc

b) La sobreexpresión de c-fos se reporta en el 61% de los osteosarcomas

c) Existe pérdida de heterocigosidad del cromosoma 13 q y 17p

d)Todas las anteriores son ciertas

e) Ninguna de las anteriores es cierta

IN119. Los osteosarcomas pueden ser subdivididos en intramedular, yuxtacortical y secundarios. El convencional
se presenta en el 86% de los casos y puede ser:

a) Osteoblástico

b) Fibroblástico

c) Condroblástico

d) Todas las anteriores

e) Ninguna de las anteriores

IN120. El osteosarcoma yuxtacortical es siempre grado 1.

a) Verdadero b) Falso
IN121. En la graduación histopatológica del condrosarcoma es indispensable para definir su
comportamiento:

a) La escala es diferente a la del osteosarcoma (va de 1 a 3)

b) La mayoría de los condrosarcomas son grado 1 y 2

c) La medición de la actividad mitótica no es útil

d) Todas las anteriores son ciertas

e) Ninguna de las anteriores es cierta

IN122. En la evaluación de la respuesta a la quimioterapia neoadyuvante del osteosarcoma, la modificación


en el tamaño del tumor es valorada adecuadamente por la:

a) Radiografía simple

b) Tomografía computada

c) Resonancia magnética

d) Gamagrafía ósea con tecnecio 99

e) Angiografía

IN123. De los pacientes con osteosarcoma y metástasis pulmonares sometidos a tratamiento paliativo:

a) El 80 % de los pacientes con OS al momento del diagnóstico tienen metástasis pulmonares

b) La presencia de lesión metastásica pulmonar un año después de terminar el tratamiento tiene


mejor pronóstico

c) Todas las lesiones metastásicas deben ser resecadas quirúrgicamente para hablar de curación

d) No todas las lesiones identificadas por tomografía computada de tórax son metástasis en OS

e) Todas las anteriores

IN124. Los niveles elevados de la fosfoproteína celular MDM2 pueden inactivar la actividad supresora de
p53.MDM2 que se localiza en el cromosoma:

a) 17 p13.1

b) 12 q 13-14

c) 9; 22 (q34; q11)

d) t (11;22)

e) t (2;13) (q35;q14)
IN125. La fosfatasa alcalina placentaria se presenta como marcador de diferenciación en los tumores de
células germinales, excepto en el:

a) Teratoma maduro

b) Coriocarcinoma

c) Seminoma

d) Tumor de senos endodérmicos

e) Germinoma primario de SNC

IN126. Los tumores germinales extragonadales se presentan frecuentemente en la línea media por alteración
en los patrones migratorios de las gónadas embrionarias. En la población pediátrica el sitio más frecuente de
presentación es:

a) Mediastinal

b) Intracraneal

c) Sacrococcígeo

d) Retroperitoneal

e) Pericárdico

IN127. Los tumores germinales de localización mediastinal:

a) Son exclusivos de mediastino anterior

b) Se pueden asociar a neoplasias hematológicas

c) Son más frecuentes en el sexo femenino

d) En la adolescencia son muy sintomáticos

e) Se asocian frecuentemente con Síndrome de Bloom

IN128. Los tumores gonadales que más frecuentemente se asocian a pubertad precoz son:

a) Cordones sexuales

b) Senos endodérmicos

c) Seminoma

d) Disgerminoma

e) Teratoma inmaduro
IN129. Los niveles séricos de alfafetoproteína en recien nacidos pueden llegar a encontrarse normalmente
hasta 12,600 ng/mL.

a) Verdadero b) Falso

IN130. Es factor predisponente para el origen del seminoma mencionado como antecedente de
criptorquidea:

a) El tumor se presenta siempre en el testículo no descendido

b) La orquidopexia temprana evita el desarrollo del seminoma

c) El riesgo de cáncer testicular no se incrementa

d) El carcinoma embrionario también se puede presentar como resultado de la criptorquidea

e) El testículo no descendido no debe ser sometido a biopsia, ya que esto incrementa el riesgo de
seminoma

IN131. Con respecto a las neoplasias primarias de ovario en pediatría:

a) Las pacientes con gonadoblastoma deben ser sometidas a ooforectomía bilateral

b) Las pacientes con disgerminoma deben ser sometidas a ooforectomía bilateral

c) Las pacientes con tumor de senos endodérmicos deben ser sometidas a ooforectomía bilateral

d) Las pacientes con tumor germinal mixto y metástasis a distancia deben ser sometidas a
ooforectomía bilateral

e) Las pacientes con tumor de senos endodérmicos y alfafetoproteína superior a 10,000 ng/mL
deben ser sometidos a ooforectomía bilateral.

IN132. Es innegable la importancia de la determinación sérica de alfafetoproteína (AFP) y fracción beta de


gonadotropina coriónica humana (BGCH), no sólo como confirmación diagnóstica sino como marcadores
importantes en la respuesta al tratamiento, la vida media de ambas es de:

a) AFP 20 a 30 días BGC 7 días

b) AFP 24 a 36 horas BCG 5 a 7 días

c) AFP 5 a 7 meses BGC 24 a 36 meses

d) AFP 5 a 7 días BGC 24 a 36 horas

e) AFP y BGC 24 a 36 horas


IN133. La mayor radiosensibilidad es en la:

a) Fase M

b) Fase S tardía

c) Fase G 0

d) Fase G 2

e) No involucra al ciclo celular

IN134. El riesgo de leucemia en gemelos monocigóticos se estima hasta en un 25%, el riesgo del gemelo no
afectado iguala al de la población general a la edad de:

a) 2 años

b) 5 años

c) 7 años

d) 10 años

e) El riesgo nunca es igual al de la población general

IN135. Es el autor de la teoría que postula que dos mutaciones secuenciales espontáneas ocurren en genes
regulatorios en la población de células linfoides que tienen estrés de proliferación significativo para el desarrollo
de LLA.

a) Greaves

b) Knudson

c) Inmortalización de linfocitos citotóxicos

d) Burkitt

e) Pinkel

IN136. Es una proteína antiapoptótica que inmortaliza las células y sus niveles elevados han sido relacionados
con pobre pronóstico en leucemia aguda:

a) BAX

b) p53

c) cdk4
d) bcl-2

e) TCR

IN137. Son características citomorfológicas de los blastos L1 según la FAB las:

a) Células predominantemente pequeñas, cromatina homogénea, forma nuclear regular,


nucleolos no visibles, basofilia citoplásmica leve a moderada

b) Células grandes homogéneas, cromatina nuclear homogénea, nucleolos no aparentes,


citoplasma abundante

c) Células grandes heterogéneas, cromatina nuclear heterogénea, indentaciones nucleares


comunes, nucleolos visibles, basofilia citoplásmica variable

d) Células pequeñas heterogéneas, cromatina nuclear homogénea y fina, núcleo oval o redondo
regular, nucleolo prominente, vacuolas prominentes

e) Células grandes homogéneas, cromatina nuclear heterogénea, nucleolos aparentes, núcleo


irregular indentado, vacuolas prominentes

IN138. Son marcadores de inmunofenotipo específicos para leucemia linfoblástica aguda de células T:

a) CD 3, CD5, CD13, CD 19

b) CD 2, CD 3, CD 5, CD 7

c) CD 2, CD 5, CD 10, CD 19.

d) CD 2, CD 3; HLA DR, CD 33

e) CD 10, CD 19, CD 22, CD 20

IN139. Trisomías que pueden estar presentes en pacientes con LLA y se han asociado a bajo riesgo de
fracaso al tratamiento:

a) Trisomía 18 y 21

b) Trisomía 12 y 16

c) Trisomía 4 y 12

d) Trisomía 4 y 10

e) Trisomía 17 y 21

IN140. Se considera como mejor pronóstico a los pacientes con leucemia linfoblástica aguda portadores de:

a) Diploidia
b) Hiperdiploidia >50 cromosomas

c) Hipodiploidia <40 cromosomas

d) Tetraploidia

e) Hipodiploidia 41-45 cromosomas

IN141. Alteración estructural cromosómica más frecuente en pacientes con LLA:

a) t (9;22)(q34;q11)

b) t (7;10)(q35;q24)

c) t (12;21)(p12;q22)

d) t (1;19)(q23;p13)

e) t (8;22)(q24;q11)

IN142. Su administración nocturna ha mostrado una mayor eficacia en pacientes con leucemia linfoblástica
aguda

a) 6 Mercaptopurina

b) Vincristina

c) Ciclofosfamida

d) Quimioterapia intratecal

e) Adriamicina

IN143. Los pacientes con leucemia linfoblástica aguda con recaída dentro de los seis primeros meses de
diagnóstico tienen una sobrevida de:

a) 40%

b) 60%

c) 35%

d) 50%

e) 10%

IN144. Dosis adecuada de radioterapia para enfermedad testicular en pacientes con LLA:

a) 4000 cGy

b) 1200 cGy

c) 1800 cGy
d) 2400 cGy

e) 6000 cGy

IN145. Porciento de casos en los que se estima que el retinoblastoma tiene presentación bilateral hereditaria:

a) 60

b) 15

c) 25

d) 70

e) 50

IN146. Los pacientes con retinoblastoma hereditario tienen alta predisposición a segundas neoplasias, lo que
está determinado genéticamente por una:

a) Amplificación de c-myc

b) Deleción de fos

c) Mutación de p53

d) Mutación de RB1

e) Afección de 11q23

IN147. La actividad de la proteína Rb como supresora se lleva a cabo durante qué fase del ciclo celular:

a) G0

b) G1

c) S

d) G2

e) M

IN148. La proteína RB tiene como función regular elementos promotores como:

a) c-myc

b) n-myc

c) E2F

d) Bcl-2

e) C-fos
IN149. De las características histopatológicas del retinoblastoma podemos encontrar las rosetas de:

a) Homer-Wright

b) Wilson

c) Kerner Morrison

d) Horner

e) Flexner-Winstersteiner

IN150. Los patrones de diseminación intraocular de retinoblastoma son:

a) Exofítico y endofítico

b) Monomórfico y Polimórfico

c) Hematógeno y Linfático

d) Vítreo y Retiniano

e) Ninguno

IN151. La manifestación clínica más común de retinoblastoma es la leucocoria seguida en frecuencia de:

a) Inflamación orbitaria

b) Pupila fija

c) Heterocromía del iris

d) Estrabismo

e) Ceguera

IN152. La enucleación bilateral en retinoblastoma se justifica cuando:

a) El paciente es menor a tres años

b) No hay visión salvable en ninguno de los dos ojos

c) Existe deleción 13q14

d) Resistencia a ciclofosfamida

e) Madre con antecedente de retinoblastoma


IN153. La crioterapia es efectiva en el tratamiento de retinoblastoma en tumores:

a) Menores a 5 mm de diámetro y 4 mm de grosor

b) Con glaucoma neurovascular

c) Con siembras vítreas grandes

d) Menores a 3 mm de diámetro y 2 mm de grosor

e) Con desprendimiento de retina

IN154. La fotocoagulación en retinoblastoma se prefiere para tumores pequeños localizados en:

a) Vítreo

b) Cámara anterior

c) Porción posterior de la retina

d) Adyacentes al iris

e) Porción anterior de retina

IN155. El objetivo de la radioterapia en retinoblastoma es:

a) Favorecer recuperación retiniana

b) Control local de la enfermedad con preservación de la visión

c) Potencializar efecto de antracíclicos

d) Provocar necrosis del nervio óptico

IN156. Cuando existe diseminación a través de la esclera, el porciento de riesgo de metástasis es de:

a) 20

b) 30

c) 60

d) 90

e) 100
IN157. Se asocia comúnmente a retinoblastoma bilateral como secuela tardía:

a) Osteosarcoma

b) Linfoma de Burkitt

c) Leucemia granulocítica crónica

d) Histiocitosis

e) Enfermedad de Hodgkin

IN158. La mayor parte de los casos de rabdomiosarcoma se presenta en:

a) Adolescentes

b) Lactantes menores

c) Menores de seis años

d) La segunda década de la vida

e) Recién nacidos

IN159. La traslocación característica del rabdomiosarcoma alveolar es:

a) t (11;22)

b) t (4;11)

c) t (12;21)

d) t (2;8)

e) t (2;13)

IN160. El rabdomiosarcoma embrionario presenta la alteración genética siguiente:

a) Pérdida de heterocigocidad de 11p15

b) t (11;22)

c) Deleción 13q14

d) Isocromosoma 16

e) Pérdida del 12p


IN161. El sitio más frecuente de metátasis en rabdomiosarcoma es:

a) Hígado

b) Bazo

c) Médula ósea

d) Hueso

e) Pulmón

IN162. Es el principal factor pronóstico en pacientes con rabomiosarcoma:

a) Edad

b) Histología

c) Ploidia

d) Respuesta al tratamiento

e) Afección linfática

IN163. Son sitios primarios de pronóstico favorable en el rabdomiosarcoma, excepto:

a) Órbita

b) Vagina

c) Vejiga

d) Extremidad

e) Párpado

IN164. Complicación de la linfadenectomía retroperitoneal en rabdomiosarcoma paratesticular:

a) Eyaculación retrógrada

b) Diseminación a pulmón

c) Aumento del estadio

d) Contaminación peritoneal

IN165. El síndrome de ácido retinoico se trata con:


a) Líquidos intravenosos

b) Albúmina al 25%

c) Dexametasona

d) Antiinflamatorios no esteroideos

e) Bloqueadores de los canales de calcio

IN166. ¿Cuál de los siguientes enunciados es correcto con respecto a la vincristina?

a) La principal vía de metabolismo y excreción es hepática

b) Un porciento muy pequeño se encuentra unido a proteínas plasmáticas

c) Puede administrarse por vía intramuscular

d) Atraviesa la barrera hematoencefálica de manera significativa

e) En insuficiencia renal la dosis debe ajustarse

IN167. ¿Cuál de los siguientes agentes se asocia a toxicidad pulmonar?

a) Mitoxantrona

b) Doxorrubicina

c) Idarubicina

d) Bleomicina

e) Actinomicina D

IN168. Con respecto a la carmustina ¿cuál de los enunciados siguientes es correcto?

a) El nadir en las cuentas de leucocitos ocurre 14 días después de su aplicación

b) Tiene una vida media de aproximadamente 36 horas

c) Atraviesa la barrera hematoencefálica de manera significativa

d) Su uso no se ha asociado a la aparición de segundas neoplasias

e) Las reacciones alérgicas son frecuentes

IN169. ¿Cuál de los siguientes efectos colaterales del arabinósido de citosina se observa particularmente
cuando el agente se administra en dosis altas?
a) Mielosupresión

b) Disfunción cerebelosa

c) Hidradenitis

d) Pancreatitis

e) Náusea y vómito

IN170. ¿Cuál de los agentes siguientes es una prodroga?

a) Carboplatino

b) L-asparaginasa

c) Procarbazina

d) Metotrexate

e) Doxorrubicina

IN171. El uso de alopurinol durante el tratamiento de leucemias y linfomas puede causar nefropatía. ¿Cuál
de los siguientes mecanismos es causante?

a) Necrosis de las células tubulares proximales

b) Lesión glomerular directa

c) Precipitación de cristales de xantina

d) Inhibición de la reabsorción de sodio en el túbulo distal

e) Todos los anteriores

IN172. ¿Cuál de los siguientes medicamentos incrementa la toxicidad de la 6 mercaptopurina?

a) Vincristina

b) Prednisona

c) Aspirina

d) Alopurinol

e) Furosemide
IN173. ¿Cuál de los siguientes enunciados con respecto a la leucemia granulocítica crónica es correcto?

a) La translocación 9;22 se encuentra presente en el 45-50% de los pacientes

b) La translocación 9;22 produce arresto en la maduración en la serie mieloide

c) La presencia de 15% de blastos o más en la circulación define a la fase acelerada

d) El interferón produce remisión citogenética en el 65-70% de los pacientes

e) El busulfán es el agente de elección para citorreducción

IN174. El linfoma mediastinal tiene las características siguientes, excepto:

a) Su origen es de células T

b) La localización es mediastino anterior

c) Desarrolla síndrome vena cava superior

d) El tratamiento es quirúrgico

IN175. El linfoma abdominal predominantemente es de células B.

a) Verdadero b) Falso

IN176. En el linfoma no Hodgkin abdominal con infiltración retroperitoneal la conducta es:

a) Biopsia + linfadenectomía

b) Resección y anastomosis

c) Biopsia tumoral

d) Aspirado de médula ósea

IN177. En caso de enfermedad de Hodgkin con ganglios mediastinales y periesplénicos la conducta


quirúrgica es:

a) Esplenectomía

b) Hemiesplenectomía

c) Biopsia de ganglios

d) Sólo b y c
IN178. En paciente con masa mediastinal con síndrome de vena cava superior, se debe tomar la biopsia con:

a) Anestesia general

b) Sedación y anestesia local

c) Anestesia local y posición semifowler

d) Todas las anteriores son correctas

IN179. En caso de tumor de Wilms bilateral la conducta es:

a) Biopsia y estadificación individual

b) Nefrectomía bilateral

c) Estadificación tomográfica

d) Quimioterapia neoadyuvante

IN180. Se considera estadio III de tumor de Wilms a los siguientes, excepto:

a) Infiltración capsular

b) Derrame tumoral intrabdominal

c) Implantes peritoneales

d) Resección incompleta

IN181. Se considera a la nefroblastomatosis contralateral estadio V:

a) Verdadero b) Falso

IN182. Conducta en la extensión de tumor de Wilms a vena cava y/o atrial:

a) Resección tumoral y aspiración de trombo

b) Quimioterapia + cirugía

c) Cirugía + quimioterapia adyuvante

d) Quimioterapia neoadyuvante + cirugía

IN183. Son factores pronósticos en el NTWS 5 la edad y el peso del tumor:

a) Verdadero b) Falso
IN184. Ante la presencia de teratoma maduro bilateral la conducta es:

a) Salpingooforectomía bilateral

b) Biopsia bilateral + quimioterapia

c) Salpingooforectomia unilateral y biopsia contralateral

d) Quimioterapia

IN185. En disgerminoma de ovario unilateral, la conducta es:

a) Salpingooforectomia bilateral

b) Salpingooforectomia + biopsia contralateral

c) Biopsia tumoral + quimioterapia

d) Quimioterapia neoadyuvante

IN186. Posterior a la resección de un tumor de senos endodérmicos, la conducta con marcadores tumorales
posterior a la cirugía es:

a) Vigilancia cada mes

b) Toma de control a los 25 días

c) Toma cada dos meses

d) Alta de la paciente

IN187. Con respecto a laparatomía, es correcto en pacientes con tumor de ovario lo siguiente, excepto:

a) Citología de líquido peritoneal

b) Biopsia ganglios sospechosos

c) Biopsia de peritoneo

d) Biopsia lesión tumoral sin resección

IN188. La conducta quirúrgica del quiste ovárico de 4 cm asintomático es:

a) Salpingooforectomia bilateral

b) Aspiración del líquido por ultrasonido


c) Vigilancia

d) Laparatomía consistente en salpingooforectomía

IN189. Porciento en que se presentan las neoplasias malignas en la población de niños:

a) 10

b) 5

c) 2

d) 6

e) Ninguna de las anteriores

IN190. En orden decreciente, los tumores sólidos malignos más frecuentes en los Estados Unidos son:

a) Leucemia, linfoma, tumores SNC, neuroblastoma

b) Tumores SNC, linfomas, tumor de Wilms, neuroblastoma

c) Tumores SNC, linfomas, neuroblastoma, sarcoma de tejidos blandos

d) Ninguna de las anteriores

IN191. Tumores descritos originalmente asociados a síndrome de Li Fraumeni:

a) Tumores cerebrales, carcinoma adrenocortical, osteosarcoma, sarcomas de tejidos blandos y


cáncer de mama.

b) Cáncer de colon, tumores cerebrales, carcinoma adrenal y tumor de Wilms

c) Neuroblastoma, leucemias agudas, enfermedad de Hodgkin e histiocitosis

d) Sarcoma de células claras de riñón, meduloblastoma y sarcoma de Ewing

e) Ninguna de las anteriores

IN192. ¿Cuál es el tiempo de aplicación ideal de la quimioterapia combinada en tumores sólidos malignos?

a) 10 días

b) 14 días

c) 7 días
d) 21 días

e) 28 días

IN193. Porciento de pacientes con tumores sólidos malignos metastásicos que logran permanecer vivos sin
enfermedad:

a) 10

b) 20

c) 30

d) 40

e) 60

IN194. Los tejidos con adecuada oxigenación son más sensibles a la radioterapia:

a) Verdadero b) Falso

IN195. Tasa de complicación relacionada con BAAD:

a) 0.01 a 0.3%

b) 1 a 3%

c) 10 a 30%

d) Ninguna de las anteriores

IN196. Los siguientes se consideran en el grupo de tumores de células pequeñas redondas y azules:
Hepatoblastoma, sarcoma de Ewing, rabdomiosarcoma, neuroblastoma y linfoma.

a) Verdadero b) Falso

IN197. Dosis de pentobarbital recomendada para la sedación de pacientes:

a) 3 a 10 mg/kg

b) 1 a 3 mg/kg

c) 3 a 6 mg/kg

d) 6 a 10 mg/kg
IN198. Son ventajas de la biopsia vídeo asistida por laparoscopía a nivel hepático, excepto:

a) Visualización de todo el hígado

b) Biopsia visualización dirigida

c) Resección de tumores

d) Aplicación embolización

e) Hemostasia

IN199. En los pacientes con tumores hepáticos, neuroblastoma y linfoma No-Hodgkin es útil la laparoscopía
para evaluar su respuesta a la quimioterapia.

a) Verdadero b) Falso

IN200. Contraindicación absoluta para realizar toracoscopía:

a) Derrame pleural

b) Empiema pulmonar

c) Adhesión pleural por sinequias

d) Ninguna de las anteriores

IN201. Son algunas de las ventajas de la toracoscopia: biopsia dirigida para muestra, mínimas
complicaciones y visualización de todo el tórax:

a) Verdadero b) Falso

IN202. Tumor(es) que produce(n) metástasis pulmonares:

a) Tumor de Wilms

b) Osteosarcoma

c) Sarcoma de tejidos blandos

d)Todos los anteriores

e) Sólo b y c

De las aseveraciones siguientes, cuales son falsas y cuales verdaderas.

IN203. Son las tres técnicas útiles para diagnóstico de lesiones pulmonares: BAAD, toracoscopía y biopsia
abierta
a) Verdadero b) Falso

IN204. 60% de las masas mediastinales son malignas

a) Verdadero b) Falso

IN205. BAAF puede obtener 50 a 70% de material diagnóstico

a) Verdadero b) Falso

IN206. La complicación más importante asociada al BAAF mediastinal es neumotórax

a) Verdadero b) Falso

IN207. La mediastinoscopía es poco útil en niños

a) Verdadero b) Falso

IN208. Cuál es el tumor más frecuente en cabeza y cuello según la serie informada por Jaffe:

a) Rabdomiosarcomas

b) Angiofibroma juvenil de nasofaringe

c) Linfomas

d) Neuroblastoma

e) Tumores germinales

IN209. ¿En los pacientes con leucemia aguda está indicada la BAAF por vía transescrotal para determinar
recaída?

a) Verdadero b) Falso

IN210. ¿Cuándo hizo la presentación oficial del neuroblastoma el Dr. Wilms?

a) 1900

b) 1899

c) 1890

d) 1910
IN211. De las neoplasias malignas sólidas en la etapa neonatal, cuál es la más frecuente:

a) Leucemia linfoblástica aguda

b) Nefroblastoma

c) Neuroblastoma

d) Rabdomiosarcoma

e) Fibrosarcoma infantil

IN212. Porciento de mortalidad en la etapa neonatal que ocurre en la primera semana en pacientes con
cáncer:

a) 10

b) 20

c) 30

d) 60

e) 50

IN213. En la etapa neonatal ¿de qué depende la conducta quirúrgica con respecto a los quistes de ovario?

a) Evolución clínica > 4 semanas

b) Tamaño de los quistes > 4 cm

c) Las características macroscópicas

d) Observación clínica

IN214. ¿Cuál es el procedimiento quirúrgico en un RMS parameningeo con imagen de infiltración a través
de la fosa pterigoidea hacia el cerebro?

a) Resección parcial del tumor + Linfadenectomía cervical

b) Resección completa del tumor + disección radical de cuello

c) Biopsia del tumor

d) Quimioterapia adyuvante

e) Radioterapia adyuvante
IN215. ¿Cuál es la conducta quirúrgica correcta en caso de un RMS paratesticular con infiltración del
músculo del Dartos?

a) Orquiectomía con disección de ganglios femorales

b) Orquiectomía con ligadura alta del cordón y hemiescrotectomía con linfadenectomía regional

c) Orquiectomía radical con disección radical del piso pélvico

d) Orquiectomía radical con disección del músculo del Dartos

e) Ninguna de las anteriores

IN216. ¿En que sitios, en los pacientes con RMS, está indicado aplicar la estrategia de la reexcisión
primaria?

a) Cuello y tórax

b) Orbita y vejiga

c) Extremidades y tronco

d) Útero y vesícula biliar

e) Retroperitoneal y extremidades

IN217. Factores pronósticos en general para RMS establecidos por el IRS al momento del diagnóstico:

a) Edad, estadio clínico, localización

b) Edad, grupo clínico, localización

c) Grupo clínico, edad, estadio clínico

d) Localización, grupo y estadio clínico

e) Ninguna de las anteriores

IN218. ¿Cuál es la conducta actual en RMS de extremidades en estadio II?

a) Desarticulación sin linfadenectomía

b) Resección del tumor sin linfadenectomía

c) Resección del tumor con linfadenectomía regional

d) Ninguna de las anteriores


IN219. ¿Qué porciento de los tumores testiculares tiene hidrocele al diagnóstico?

a) 2

b) 0.2

c) 20

d) 0.02

e) 40

IN220. ¿Cuál es el tumor maligno más frecuente de la glándula parótida en niños?

a) Adenocarcinoma

b) Carcinoma basaloide

c) Sialoblastoma

d) Rabdomiosarcoma

e) Carcinoma mucoepidermoide

IN221. ¿Cuál es la conducta quirúrgica en un tumor de la glándula salival mayor?

a) Biopsia abierta y disección ganglionar

b) Biopsia por aspiración con aguja delgada

c) Resección parcial con disección nervio

d) Resección total con disección radical de cuello

e) Ninguna de las anteriores

IN222. ¿Cuál es la conducta quirúrgica inicial con un tumor sólido maligno del fémur distal?

a) Resección completa

b) Resección parcial

c) Desarticulación

d) Biopsia del tumor

e) Amputación
IN223. ¿Qué procedimiento quirúrgico definitivo se sugiere a un paciente de 8 años con OS convencional que
presente buena respuesta a la quimioterapia?

a) Desarticulación

b) Amputación

c) Resección en bloque + autoinjerto de peroné vascularizado

d) Colocación de una endoprótesis convencional + resección en bloque

e) Colocación de una endoprótesis no convencional + resección en bloque

IN224. ¿Cuál es el sitio más frecuente de OS en adolescentes?

a) Húmero próximal

b) Húmero distal

c) Fémur distal

d) Fémur próximal

e) Tibia distal

IN225. Son datos radiológicos de agresividad de un tumor óseo, excepto:

a) Lisis o destrucción ósea, neoformación ósea tumoral

b) Bordes mal definidos

c) Reacción perióstica discontinua

d) Bandas de radiopacidad

e) Destrucción de la cortical e invasión a tejidos blandos

IN226. El neuroblastoma (NB) es un tumor sólido que se origina de las células de la cresta neural y por lo
tanto puede desarrollarse en cualquier sitio del sistema nervioso simpático. De acuerdo al sitio primario de
origen, señale ¿cuál presenta un mejor pronóstico para la sobrevida?

a) Suprarrenal

b) Ganglios paraórticos

c) Mediastino posterior

d) Cerebro

e) Sólo a y d
IN227. Al diagnóstico, una estadificación precisa es esencial para planear un tratamiento adecuado. En
relación al neuroblastoma, ¿en la actualidad, qué sistema permite una mejor definición de la extensión y
resecabilidad de la enfermedad?

a) Sistema Internacional de Estadificación del Neuroblastoma (INSS)

b) Estadificación de Evans (CCG)

c) Estadificación de St. Jude (POG)

d) Estadificación tumor-nódulo-metástasis (TNM)

e) Estadifiación de Shimada

IN228. De acuerdo a la estadificación actual más aceptada para NB señale: ¿ Qué estadio presenta un
paciente con tumor abdominal en la línea media con afección ganglionar bilateral?

a) Estadio III

b) Estadio 3

c) Estadio D

d) Estadio 2b

e) Estadio 4

IN229. El protooncogén MYCN se encuentra localizado en el brazo corto del cromosoma 2 y es un


marcador bilogico importante para evaluar la evolución del NB, la ampliación múltiple de este gen se
relaciona con:

a) Mayor tumorogenesis

b) Estadio avanzado de la enfermedad

c) progresión rápida de la enfermedad

d) Pobre pronóstico

e) Todas las anteriores

IN230. La quimioterapia, la cirugía, y la radioterapia son las modalidades de tratamiento para NB. El
tratamiento actual depende del estadio de la enfermedad, de acuerdo con lo anterior, señale: ¿cuáles son las
metas del tratamiento quirúrgico?

a) Resección completa durante la primera intervención (cirugía radical)


b) Resección completa si es posible, estadificación y/o biopsia

c) Únicamente biopsia

d) Únicamente estadificación

e) Todas la anteriores

IN231. En pacientes con NB en etapa avanzada donde el procedimiento quirúrgico inicial permite realizar el
diagnóstico y determinar la extensión del tumor, señale: ¿en qué momento de la quimioterapia neoadyuvante
se debe realizar un segundo procedimiento quirúrgico con el objeto de obtener la resección completa del
tumor?

a) Entre la 3ª y 8ª semanas

b) Entre la 6ª y 11ª semanas

c) Entre la 13ª y 18ª semanas

d) Al término de la quimioterapia

e) Después de la 18ª semana

IN232. ¿Qué histología de la enfermedad de Hodgkin compromete con mayor frecuencia el mediastino?

a) Predominancia linfocítica

b) Esclerosis nodular

c) Celularidad mixta

d) Depleción linfocítica

e) No clasificable

IN233. La mortalidad por cáncer en población de 0 a 14 años ocupa en los Estados Unidos la:

a) Primera causa

b) Quinta causa

c) Segunda causa

d) Tercera causa

e) Cuarta causa
IN234. La mortalidad por cáncer en población pediátrica de 4 a 15 años ocupa en la República Mexicana la:

a) Primera causa

b) Quinta causa

c) Segunda causa

d) Tercera causa

e) Cuarta causa

IN235. En términos generales el cáncer en la población infantil afecta más:

a) Sexo femenino

b) Abajo del año de edad

c) Arriba de los 11 años

d) Sexo masculino

e) Igual al sexo masculino y femenino

IN236. En la mayor parte de las neoplasias malignas de la población infantil la etiología es:

a) Viral

b) Bacteriana

c) Agentes químicos

d) Alteración genética

e) Desconocida

IN237. ¿Cuál es la neoplasia maligna más frecuente en pediatría en la República Mexicana?

a) Tumor de Wilms

b) Tumores del sistema nervioso central

c) Leucemia aguda

d) Enfermedad de Hodgkin

e) Linfoma no Hodgkin

IN238. La histiocitosis de Langerhans corresponde a:


a) Granuloma reactivo

b) Clase 1

c) Clase 2

d) Clase 3

e) Histiocitosis maligna

IN239. El tumor intraocular más frecuente en la infancia es:

a) Neuroblastoma

b) Retinoblastoma

c) Glioma

d) Melanoma

e) Tumores metastásicos

IN240. La alteración citogenética de retinoblastoma es:

a) Deleción 15

b) t(9;22)

c) Deleción 13q14

d) t(1;19)

e) Deleción del cromosoma 1

IN241. ¿Cuál de los estudios radiológicos siguientes es de más utilidad como estudio inicial para evaluar
enfermedad de Hodgkin infradiafragmática?

a) Gamagrafía con galio

b) Serie gastroduodenal

c) Colon por enema

d) Linfografía pedia bilateral con tomografía computada

e) Angiografía selectiva

IN242. Todos los estudios de laboratorio siguientes son útiles para evaluar la enfermedad de Hodgkin,
excepto:

a) Cobre sérico
b) Eritrosedimentación

c) Hierro sérico

d) Deshidrogenasa láctica

e) Fosfoquinasa

IN243. Desde el punto de vista radiológico las siguientes son imágenes del osteosarcoma, excepto:

a) Estriaciones en puesta de sol

b) Respuesta perióstica irregular con hueso de neoformación

c) Cambios osteoblásticos

d) Tumor que más frecuentemente compromete la diáfisis

e) Triángulo de Codman

IN244. En leucemia aguda linfoblástica, en general, los pacientes con terapia previa responden menos a
tratamientos subsecuentes debido a los factores siguientes, excepto:

a) Menos reserva medular

b) Pobre función orgánica

c) Frecuencia elevada de infecciones

d) Resistencia de las células leucémicas

e) Reacciones alérgicas frecuentes

IN245. Síntomas más comunes en leucemia aguda linfoblástica son:

a) Dolor abdominal y fatigabilidad

b) Fiebre, palidez, astenia y lesiones purpúricas

c) Hematemesis y melena

d) Dolor articular y óseo, con pérdida de peso

e) Pérdida de peso, palidez y fatigabilidad

IN246. La causa más común de insuficiencia renal en niños con leucemia aguda linfoblástica es:

a) Infiltración leucémica al riñón

b) Deshidratación severa secundaria al vómito excesivo

c) Hiperuricemia con nefropatía por ácido úrico


d) Pielonefritis

e) Acidosis láctica

IN247. ¿Cuál es la complicación pulmonar más frecuente en leucemia aguda en niños?

a) Infiltrados leucémicos

b) Hemorragia pulmonar

c) Infección

d) Fibrosis pulmonar

e) Derrame pleural

IN248. La combinación más efectiva para producir inducción a la remisión en leucemia aguda linfoblástica
de riesgo habitual es:

a) Ciclofosfamida y prednisona

b) Prednisona y 6 mercaptopurina

c) Prednisona y L asparaginasa

d) Vincristina, prednisona y L asparaginasa

e) Vincristina y L asparaginasa

IN249. Los síndromes siguientes se asocian con un aumento en la frecuencia de linfoma no Hodgkin,
excepto:

a) Síndrome de Chediak-Higashi

b) Ataxia telangiectasia

c) Síndrome de Wiskott Aldrich

d) Anomalía de Pelger-Huet

e) Síndrome del paciente trasplantado

IN250. El linfoma de peor pronóstico en los niños es:

a) Pobremente indiferenciado no Burkitt

b) Pobremente indiferenciado Burkitt

c) Linfoma linfoblástico
d) Mixto

e) Linfoma anaplásico K1+

IN251. Las neoplasias siguientes tienen una asociación viral, excepto:

a) Burkitt

b) Carcinoma de nasofaringe

c) Sarcoma de Kaposi

d) Granuloma letal de la línea media

e) Linfoma primario del sistema nervioso central

IN252. El sitio más común del méduloblastoma es:

a) Columna vertebral

b) Cerebelo

c) Lóbulo occipital

d) Lóbulo frontal

e) Región supraselar

IN253. Los factores pronósticos más importantes en la histiocitosis de Langerhans son:

a) Edad y compromiso a piel

b) Edad y disfunción de médula ósea, hígado y/o pulmón

c) Compromiso a pulmón e hígado

d) Edad, compromiso a piel, gánglios, encías y hueso

e) Histología y hallazgos en microscopía electrónica

IN254. Según el Estudio Nacional de Wilms ¿Cuál es la indicación de quimioterapia preoperatoria en el


tumor de Wilms?

a) Hipertensión arterial, metástasis pulmonares, tamaño del tumor

b) Tumor de más de 20 cm de diámetro

c) Tumor que rebasa la línea media

d) a y b
e) Ninguna

IN255. La sobrevida a dos años en tumores de Wilms con histología favorable según el Estudio Nacional de
Wilms es:

a) 92%

b) 97%

c) 78%

d) 89%

e) 70%

IN256. La diarrea en el neuroblastoma es secundaria a la activación de:

a) 5 hidroxitriptamina

b) Noradrenalina

c) Ácido vanililmandélico

d) Polipéptido vasoactivo intestinal

e) Ácido homovanílico

IN257. El neuroblastoma con sitio primario de peor pronóstico es:

a) Suprarrenal

b) Cadena pélvica profunda

c) Mediastino posterior

d) Retroorbicular

e) Tracto olfatorio

IN258. ¿Con qué entidad se debe hacer el diagnóstico diferencial de retinoblastoma?

a) Larva migratoria visceral

b) Enfermedad de Coats

c) Desprendimiento de retina

d) Toxoplasmosis
e) Todas las anteriores

IN259. El factor pronóstico más importante en el retinoblastoma es:

a) Edad

b) Extensión de la enfermedad

c) Grado de diferenciación histológica

d) Sexo

e) Ninguna de las anteriores

IN260. El tumor óseo primario maligno más común en pediatría es:

a) Linfoma de hueso

b) Sarcoma de Ewing

c) Condrosarcoma

d) Sarcoma de las células reticulares

e) Osteosarcoma

IN261. Las situaciones siguientes predisponen la formación de sarcoma osteogénico, excepto:

a) Traumatismo

b) Displasia fibrosa

c) Enfermedad de Paget ósea

d) Radiación previa a hueso

e) Presencia del cromosoma 13q 14

IN262. ¿Cuál de las entidades siguientes se pueden confundir con el sarcoma de Ewing desde el punto de
vista radiológico?

a) Neuroblastoma

b) Sarcoma de las células reticulares

c) Osteomielitis

d) Ninguna de las anteriores

e) Todas las anteriores


IN263. El sitio más frecuente del rabdomiosarcoma embrionario es:

a) Pared torácica

b) Cabeza y cuello

c) Extremidades

d) Intraabdominal

e) Tracto genitourinario

IN264. El sarcoma botroides se puede localizar en los sitios siguientes, excepto:

a) Genitales

b) Vías biliares

c) Oído

d) Vejiga

e) Ojo

IN265. El factor pronóstico más importante en el rabdomiosarcoma es:

a) Edad del paciente

b) Tipo histológico

c) Sitio del tumor

d) Extensión de la enfermedad e histología

e) Histología y edad

IN266. ¿Cuál de los tumores siguientes se considera funcionante?

a) Carcinoma de las células embrionarias

b) Disgerminoma

c) Tumores de la granulosa

d) Teratoma maligno

e) Tumor de senos endodérmicos

IN267. En la histiocitosis de Langerhans el anticuerpo monoclonal diagnóstico es:


a) CD 17

b) CD 22

c) CD 4

d) CD8

e) CD1a

IN268. El factor pronóstico en la histiocitosis de Langerhans más importante es:

a) Disfunción pulmonar, hepática y de médula ósea

b) Disfunción pulmonar y hepática

c) Disfunción de médula ósea hepática, pulmonar y edad menor de dos años

d) Disfunción de médula ósea

e) Trombocitopenia

IN269. El glioblastoma multiforme constituye un tumor de la glía clasificado como:

a) Astrocitoma I

b) Astrocitoma IV

c) Astrocitoma III

d) Astrocitoma II

e) Astrocitoma diferenciado

IN270. El retinoblastoma bilateral puede condicionar una segunda neoplasia descrita como:

a) Leucemia aguda linfoblástica

b) Tumor de Wilms

c) Osteosarcoma

d) Neuroblastoma

e) Meduloblastoma

IN271. La manifestación clínica inicial más frecuente de retinoblastoma es:

a) Estrabismo

b) Nistagmus
c) Exoftalmos

d) Leucocoria

e) Proptosis

IN272. El tratamiento inicial de elección del linfoma de Burkitt es:

a) Radioterapia

b) Radioterapia y quimioterapia

c) Cirugía y radioterapia

d) Cirugía, radioterapia y quimioterapia

e) Cirugía y quimioterapia

IN273. La terapia ideal del méduloblastoma avanzado (etapas T3 y T4) es:

a) Cirugía

b) Cirugía y radioterapia

c) Cirugía, radioterapia a cráneo y al axis

d) Radioterapia a cráneo y neuroaxis

e) Cirugía, radioterapia a cráneo y neuroaxis más quimioterapia

IN274. ¿Cuál de las siguientes son complicaciones quirúrgicas en la nefrectomía por tumor de Wilms?

a) Ruptura del tumor intraabdominal

b) Vólvulo posoperatorio

c) Obstrucción intestinal

d) Desprendimiento de trombo tumoral

e) a y d

IN275. El manejo del rabdomiosarcoma va a depender de los factores siguientes, excepto:

a) Histología del tumor

b) Localización
c) Extensión de la enfermedad

d) Presencia de metástasis

e) Edad del paciente

IN276. El abordaje quirúrgico en el glioma del puente es:

a) Con craneotomía y resección

b) Cirugía estereotáxica y biopsia

c) Craneotomía subfrontal y resección

d) Craneotomía de fosa posterior y resección

e) La cirugía esta contraindicada

IN277. El tratamiento ideal para el retinoblastoma en etapa I debe ser:

a) Exenteración

b) Enucleación

c) Radioterapia únicamente

d) Quimioterapia únicamente

e) Crioterapia o fotocoagulación

Correlacione las dos columnas:

IN278. Metotrexate ( )

IN279. Citosin arabinósido ( )

IN280. Cis diamino dicloro platino ( )

IN281. 5-fluorouracilo ( )

IN282. Reserpina ( )

a) Inhibe la resistencia a las drogas

b) bloquea síntesis de DNA

c) Inhibe la enzima dihidrofolato reductasa

d) Inhibe síntesis DNA y RNA


e) Bloquea la reparación de DNA

Correlacione las columnas siguientes:

IN283. Seminoma (b)

IN284. Senos endodérmicos (a)

IN285. Coriocarcinoma germinal (d)

IN286. Tumor de cordones sexuales (e)

IN287. T. germinal primario mediastino ( c )

a) Eleva la alfafetoproteína

b) Criptorquidea asociada

c) Células de Sertoli

d) LAM M3

e) Enzima en orina

IN288. El síndrome de Beckwith Wiedemann:

a) Puede confundirse con el síndrome de Denys-Drash ya que ambos presentan microsomía

b) Entre más dismorfias presenta más posibilidad de tumor de Wilms

c) Se puede asociar a tumor de Wilms, adenocarcinoma y hepatoblastoma

d) Cursan con hiperglicemia severa por afección de factor semejante al de crecimiento de insulina

e) Todas las anteriores

IN289. Los siguientes agentes quimioterapéuticos son mielosupresores, excepto:

a) Actinomicina D

b) Vinblastina

c) Bleomicina

d) Daunorubicina

e) Carboplatino
IN290. En relación con el retinoblastoma, cuál de las aseveraciones es correcta:

a) Es indicación de enucleación cuando no se observa la retina después de tratamiento

b) Es indicación absoluta de enucleación cuando no hay visión

c) Es indicación de enucleación ante glaucoma neovascular

d) Todas las anteriores

e) Ninguna de las anteriores

IN291. El hepatoblastoma tumor que afecta principalmente a menores de un año:

a) Predomina su presentación multifocal

b) Puede marcar para fracción beta de gonadotrophina coriónica

c) Se asocia frecuentemente con virus de hepatitis B

d) Todas las anteriores

e) Ninguna de las anteriores

IN292. La etapa 4S según el INSS comprende, además de las metástasis a piel, lo siguiente:

a) Metástasis a hígado y médula ósea > 10% de células neoplásicas

b) Metástasis a hueso y médula ósea < 10% de células neoplásicas

c) Metástasis a hígado y médula ósea < 10% de células neoplásicas

d) Metástasis a hígado y myc amplificado

e) Metástasis a hígado, hueso y en menores de 365 días de edad

Relacione las columnas:

IN293. Poliposis adenomatosa familiar (d)

IN294. Síndrome de Turcot (b)

IN295. Deficiencia de alfa l antitripsina (a)

a) Hepatocarcinoma

b) Tumores de SNC
c) Osteosarcoma

d) Hepatoblastoma

e) Leucemias

IN296. En leucemias agudas el reporte de ploidia tiene relevancia pronóstica, mencione cuál de los
enunciados siguientes es correcto:

a) La ploidia se debe determinar en fase G1 del ciclo celular.

b) Hiperdiploidia en leucemia linfoblástica aguda es indicativo de mal pronóstico.

c) Hipodiploidia es más frecuente en leucemia aguda no linfoblástica.

d) La determinación de la ploidia tiene mayor valor que la definición de alteraciones estructurales


en los cromosomas.

e) Hiperparadiploidia es factor de buen pronóstico en leucemia linfoblástica aguda.

IN297. Mecanismo(s) de resistencia a los fármacos de quimioterapia:

a) Disminución del transporte hacia el interior de la célula

b) Reparación de DNA aumentada

c) Uso de vías alternas para obtener metabolitos

d) Amplificación genética de enzimas blanco de la acción del fármaco

e) Todas las anteriores

IN298. El mecanismo potenciador de la quimioterapia de la aminofilina:

a) Favorece el influjo del fármaco al alterar la permeabilidad de membrana

b) Reemplaza el rol molecular del oxígeno como radical libre

c) Facilita la entrada a la apoptosis de las células con p53 deficiente

d) Acelera la mitosis para disminuir la reparación del DNA

e) Favorece la entrega de fármaco al tumor

IN299. Tumor primario de SNC en que la toma del líquido cefalorraquídeo en busca de células tumorales es
necesaria:

a) Astrocitoma pilocítico
b) Meduloblastoma

c) Astrocitoma anaplásico

d) Tumores germinales

e) Glioma de la vía visual

IN300. Son radiosensibilizadores, excepto:

a) Oxígeno

b) Amifostine

c) Cisplatino

d) Nitroimidazoles

e) Análogos de la timidina

IN301. Son características morfológicas de los blastos L2, excepto:

a) Tamaño heterogéneo

b) Nucleolos no visibles

c) Cromatina nuclear variable

d) Citoplasma moderadamente abundante

e) Indentación nuclear común

IN302. Con respecto a la leucemia granulocítica crónica lo siguiente es verdadero, excepto:

a) En 5% de los pacientes no se detecta el Cromosoma Ph(+)

b) El aumento de eosinófilos en médula osea es marcador de fase acelerada

c) La proteína bcr-abl tiene un peso 210 kd

d) En la fase blástica es más común la transformación mieloide

e) En la fase crónica el diagnóstico se realiza en sangre periférica

IN303. Cuál de las opciones siguientes es incorrecta:

a) La frecuencia de osteosarcoma como segunda neoplasia es 2000 veces más que en el resto
de la poblacion en cualquier presentación de Retinoblastoma como primera neoplasia

b) Los niveles elevados deMDM2 (fosfoproteína celular) activan la actividad supresora de p53
c) El oncogén p53 se encuentra localizado en el cromosoma 12q 13-14

d) Todas las anteriores

e) Ninguna de las anteriores

IN304. Medicamento utilizado en el osteosarcoma que puede producir segunda neoplasia, específicamente
leucemia:

a) Cis diamino dicloro platino

b) Ifosfamida

c) Etopósido

d) Todas las anteriores

e) Ninguna de las anteriores

IN305. En enfermedad de Hodgkin la tríada sintomática fiebre, pérdida de peso y sudación es reconocida
como factor pronóstico, cuál de las interleucinas siguientes es la causa de fiebre por actividad tumoral:

a) IL 5

b) IL 1

c) IL 9

d) IL 2

e) a y c

f) b y d

IN306. En el diagnóstico diferencial de retinoblastoma está incluida la enfermedad de Coat:

a) Verdadero b) Falso

IN307. Cuál de los agentes quimioterapéuticos siguientes puede provocar amaurosis:

a) Ciclofosfamida

b) Etopósido

c) Vincristina

d) Adriamicina
e) L-Asparaginasa

IN308. De los agentes quimioterapéuticos siguientes, cuál no necesita metabolizarse para ser activo:

a) Mostaza nitrogenada

b) Ciclofosfamida

c) Ifosfamida

d) 5-fluoracilo

e) Melfalán

IN309. Alteración cromosómica asociada con buen pronóstico en leucemia aguda no linfoblástica:

a) t(8;21); t(15;17) e inv(16)

b) t(12;21); t(11;14)

c) m7, m5, del 5, del 7

d) t(6;11), t(9;22),

e) Ninguna de las anteriores

IN310. En la etapa IV de la estadificación de Ann Arbor ocurre:

a) Infiltración difusa o diseminada de uno o más organos o tejidos extralinfáticos con o sin
afección a ganglios linfáticos

b) Infiltración a médula ósea o SNC

c) Infiltración difusa o diseminada de más de un tejido extralinfatico

d) Afección de una o más regiones ganglionares a ambos lados del diafragma e infiltración a
médula ósea

IN311. Qué proteína del retinoblastoma (pRb) proporciona a las células la habilidad de detener su
replicación:

a) La fosforilación suprime esta habilidad en la fase G1/S del ciclo celular.

b) La fosforilación aumenta la habilidad de pRb e inicia en la fase G2 del ciclo celular.

c) La fosforilación disminuye al final de la de la fase M y reentrando a G-1


d) La proteína pRb se mantiene siempre forforilizada durante el ciclo celular.

e) Son ciertas a y c

IN312. Estadificación histopatológica de los sarcomas primarios de hueso:

a) El osteosarcoma convencional es grado 4

b) El osteosarcoma de mandíbula siempre es grado 4

c) El osteosarcoma telangiectásico y de células pequeñas es grado 4

d) Todas las anteriores son ciertas

e) Sólo a y c son ciertas

IN313. Los pacientes con tumores de células germinales han alcanzado una sobrevida de cinco años superior
al 90%. El grupo de la Universidad de Indiana marca como factores pronósticos independientes a la:

a) Metástasis hepáticas, a SNC y óseas

b) Masa mediastinal independientemente del tamaño

c) Fracción beta de gonadotrofina coriónica mayor de 10,000 mL U/mL

d) Alfafetoproteína mayor de 1000 ng/mL

e) Fosfatasa alcalina placentaria positiva

IN314. Los tumores germinales gonadales tienen las siguientes caractrísticas:

a) En el testículo se originan las células que NO han entrado en meiosis

b) En el ovario se originan en las células que NO han entrado en meiosis

c) Sólo en los tumores testiculares se encuentra i(12p)

d) Únicamente los disgerminomas expresan c-kit

e) La fosfoproteína MDM2 siempre es positiva en seminomas avanzados

IN315. La radioterapia como parte del tratamiento del cáncer se ha reportado desde el descubrimiento de
Roetgën hace más de un siglo. El sistema internacional de unidades (unidades SI) identifica al Grey (Gy)
como unidad de dosis absorbida y equivale a:

a) 1000 RADS

b) 10,000 RADS
c) 1 RADS

d) 100,000 RADS

e) 100 RADS

IN316. El patrón citoquímico de la leucemia linfoblástica aguda es la:

a) Mieloperoxidasa positiva, sudán negro negativo, PAS positivo

b) Mieloperoxidasa negativa, sudán negro positivo, PAS negativo

c) Mieloperoxidasa negativa, sudán negro negativo, PAS positivo

d) Mieloperoxidasa positiva, sudán negro negativo, PAS negativo

e) Mieloperoxidasa positiva, sudán negro positivo, PAS negativo

IN317. Traslocación cromosómica más común de la leucemia linfoblástica aguda L3:

a) t (9;22)

b) t (1;19)

c) t (17;19)

d) t (4;11)

e) t (8;14)

IN318. El retinoblastoma bilateral esporádico se relaciona con:

a) Edad paterna avanzada

b) Exposición a DFH durante el embarazo

c) Síndrome de feto-alcohol

d) Síndrome de Bloom

e) Ocupación paterna relacionada con el manejo de hidrocarburos

IN319. Cuando el retinoblastoma se encuentra limitado al globo ocular, la decisión terapéutica se basa en:

a) Potencial de visión del ojo afectado

b) Edad del paciente

c) Experiencia del oftalmólogo


d) Sensibilidad a la radioterapia

e) Disponibilidad de quimioterapia

IN320. Son indicaciones de enucleación en retinoblastoma:

a) Glaucoma neovascular y pérdida de la visión del ojo afectado

b) Neovascularización retinal y tumores pequeños

c) Recurrencias pequeñas después de radioterapia y tumores en parte posterior de retina

d) Tumores pequeños en porción anterior de la retina

e) Edad menor a dos años y neovascularización retinal

IN321. Son sitios parameníngeos del rabdomiosarcoma los siguientes, excepto:

a) Nasofaríngeo

b) Fosa pterigoidea

c) Orofaríngeo

d) Paranasal

e) Oído medio

IN322. Efecto indeseable de la L-asparginasa:

a) Alopecia

b) Mucositis

c) Nefropatía

d) Comportamiento inapropiado y alucinaciones

e) Cardiomiopatía

IN323. Con respecto a la ciclofosfamida ¿cuál de los enunciados siguientes es correcto?

a) Es un vesicante potente

b) Puede producir crisis convulsivas secundarias a hiponatremia

c) En un 10 a 15% de los pacientes produce insuficiencia renal reversible


d) El uso crónico se asocia a degeneración macular

e) Todas las anteriores

IN324. ¿Cuál de las afirmaciones siguientes con respecto al transplante de médula ósea en leucemia
granulocítica crónica es correcta?

a) Los mejores resultados se observan en pacientes transplantados dentro del primer año del
diagnóstico

b) El riesgo de recaída después del transplante es del 5 a 7%

c) El rechazo es una de las principales razones de fracaso

d) Los resultados del transplante en fase crónica son similares a los que se obtienen en fase
acelerada

e) La depleción de células T del injerto reduce el riesgo de recaída

IN325. La aspiración con aguja fina para biopsia renal está indicada en todo tumor renal con infiltración a la
vena cava inferior:

a) Verdadero b) Falso

IN326. Los tumores epiteliales tienen mayor riesgo de metástasis peritoneal que los germinales:

a) Verdadero b) Falso

IN327. De acuerdo a la hipótesis de Goldie-Coldman, qué factores provocan que el tumor desarrolle
resistencia:

a) Sensibilidad del tumor

b) Tamaño del tumor

c) Tasa de mutación del tumor

d) a y b son correctas

e) b y c son correctas

IN328. ¿En qué pacientes pediátricos con cáncer está indicada la laparoscopía diagnóstica?

a) En aquellos pacientes que se haya agotado los estudios radiográficos tales como TAC o RM
b) En pacientes con sospecha de linfoma

c) En aquellos que exista la posibilidad de establecer estadificación

d) a y c son correctas

e) Ninguna de las anteriores

RADIOBIOLOGÍA ONCOLÓGICA

RO1. La radioterapia es una modalidad terapéutica con fin curativo en tumores:

a) Indiferenciados

b) Muy vascularizados

c) En etapas tempranas, sobre todo los epidermoides

d) En esquemas de hipofraccionamiento

e) Cuando se relaciona todo lo anterior

RO2. En radioterapia, la unidad del Sistema Internacional que se define como la cantidad de energía
absorbida por unidad de masa, es conocida como:

a) Sievert

b) Becquerel

c) Gray

d) Roentgen

e) Curie

RO3. Ejemplo de radiación ionizante de tipo corpuscular:

a) Electrones

b) Rayos X

c) Rayos gamma

d) Neutrones

e) a y d son correctas

RO4. Qué tipo de irradiación actúa mediante el fenómeno fotoeléctrico:


a) Electrones

b) Ortovoltaje

c) Rayos gamma como los del cobalto 60

d) Neutrones

e) Otras partículas pesadas

RO5. Los isótopos radiactivos más empleados en radioterapia son:

a) Cobalto 60

b) Iridio 192

c) Yodo 125

d) Radium

e) a y b son correctas

RO6. En seguridad radiológica la dosis absorbida por el personal ocupacionalmente expuesto, se expresa en:

a) Roentgen

b) Becquerel/tiempo

c) Rads

d) Gray/tiempo

e) Sievert/tiempo

RO7. El uso de electrones en radioterapia se indica en:

a) Lesiones profundas

b) Lesiones de piel, únicamente

c) Lesiones relativamente supérficiales

d) Lesiones donde está involucrado el hueso

e) Todas las anteriores

RO8. El empleo de condensadores tiene como objetivo:


a) Generar una distribución de dosis más homogénea

b) Permitir una mayor penetración del haz de radiación

c) Protección de órganos

d) Debe utilizarse siempre en superficies irregulares

e) Facilitar la colocación del paciente

RO9. Radiosensibilidad se refiere a:

a) Curabilidad de los tumores

b) Disminución rápida del volumen tumoral

c) Menor probabilidad de recurrencia

d) Todo lo anterior

e) Nada de lo anterior

RO10. La planeación de una rehabilitación protésica del maxilar ante una resección quirúrgica de esta área,
depende de:

a) La variedad histológica del tumor

b) La etapa clínica de la neoplasia

c) De la posibilidad de manejo adyuvante con quimioterapia

d) La extensión tumoral y el abordaje quirúrgico

e) Del tipo de material empleado en la elaboración de la prótesis

RO11. Una de las ventajas del acelerador sobre las bombas de cobalto es:

a) Manejar diferentes tipos de energía

b) Producir más penumbra de los haces de radiación

c) Ser un equipo más barato y, por tanto, más accesible

d) El costo de mantenimiento es menor

e) Todo lo anterior

RO12. Cuando se realiza un implante intersticial:


a) Se requiere de una adecuada dosimetría

b) Puede ser realizado y supervisado por un cirujano oncólogo

c) Se hace con agujas de radium

d) Es importante la tasa de dosis de alrededor de 2 Gy, por hora, para mantener bajas las
posibilidades de lesiones tardías

RO13. De los siguientes mencione el tejido más radiosensible:

a) Vagina

b) Recto

c) Vejiga

d) Íleon

e) Cérvix

RO14. El combinar quimioterapia y radioterapia de manera concomitante logra:

a) Aumento en el control local

b) Aumento de la toxicidad aguda

c) Aumento en la sobrevida

d) Sólo a y b

e) A y c

RO15. Son tumores radiocurables los siguientes, excepto:

a) Melanomas

b) Rabdomiosarcoma embrionario

c) Linfomas

d) Cáncer de cérvix

e) Seminomas

RO16. La fase del ciclo celular donde hay mayor sensibilidad a la radiación es:
a) G1

b) S

c) G2

d) G2-M

e) M

RO17. Son tumores radioresistentes los siguientes, excepto:

a) Glioblastoma multiforme

b) Leucemia mieloblástica

c) Melanoma

d) Cáncer renal de células claras

e) Condrosarcoma

RO18. Son modalidades de braquiterapia, excepto:

a) Intracavitaria

b) Intersticial

c) De contacto

d) Electrones a cuerpo total

e) Colocación de semillas radiactivas

RO19. Al administrar radioterapia segmentaria al hemicuerpo superior la morbilidad más importante se


produce en:

a) SNC

b) Pulmón

c) Hígado

d) Médula ósea

e) Piel y tejidos blandas

RO20. Las neoplasias poco diferenciadas son más radiosensibles por:


a) Mejor oxigenación

b) Fracción de crecimiento alta

c) Fracción de crecimiento baja

d) Más hipoxia celular

e) Ninguna de las anteriores

RO21. El signo de Lhermitte se presenta como morbilidad tardía de la radiaterapia sobre:

a) Médula espinal torácica

b) Tallo cerebral

c) Médula espinal cervical

d) Plexo branquial

e) Sólo se presenta cuando se da radiorterapia sobre toda la médula espinal

RO22. En el Ca Cu el empleo de la braquiterapia:

a) Es un tratamiento de radioterapia externa

b) Requiere emplear siempre Cs 137

c) Disminuye con ello la dosis que recibe el recto y la vejiga

d) Permite administrar una dosis mayor al sitio primario

e) Los resultados son iguales si se emplea o no la braquiterapia

RO23. El paciente con seminoma etapa I, y afectación del cordón espermático, el tratamiento con
radiaciones más frecuente es:

a) Hemi Y invertida

b) Barra lumboaórtica

c) Y invertida

d) Sólo irradiación de a cadenas ganglionares pélvicas y el escroto.

e) Incluir inicialmente el abdomen, y luego la Y invertida

RO24. En la enfermedad de Hodgkin la radioterapia está indicada en las etapas siguientes:

a) Ib

b) Ia, IIa, sin factores de mal pronóstico


c) En cualquier etapa

d) Sólo en etapas Ia

e) Sólo como adyuvante a la quimioterapia

RO25. De las siguientes urgencias oncológicas, cuál es la que se trata con radioterapia:

a) Síndrome VCS

b) Compresión medular

c) Obstrucción laríngea

d) Todas las anteriores

e) Sólo a y b

RO26. El tratamiento con radioterapia está indicado aún en ausencia de reporte de patología en pacientes
con:

a) Síndrome VCS

b) Lesión cerebral a nivel del tálamo

c) Compresión medular con síntomas neurológicos leves

d) Todo lo anterior

e) Sólo b y c

RO27. Uno de los medicamento siguienes es un radiosensibizador:

a) Ciclofosfamida

b) Mitomicina

c) VP 16

d) CDDP y Melfalán

RO28. Alternativas de tratamiento para dolor producido por metástasis óseas diseminadas:

a) Radiación segmentaria

b) Estroncio 89

c) Yodo 125

d) Sólo b y c

e) A y b
RO29. Medicamento que sinergiza el efecto de la radioterapia en el tratamiento de tumores epidermoides
avanzados del área de cabeza y cuello:

a) Bleomicina

b) Adriamicina

c) Metotrexate

d) VP-16

e) Platino

RO30. La lesión celular originada por la radiación y que casi siempre lleva a la muerte celular es:

a) Lesión de la membrana celular

b) Lesión de los organelos citoplasmáticos

c) Lesión del RNA

d) Doble ruptura de la cadena del DNA

e) Lesiones múltiples mitocondriales

RO31. Dos capas hemirreductores disminuyen la transisión de la radiación en un:

a) 10%

b) 25%

c) 50%

d) 75%

e) 100%

RO32. El objetivo de la radioterapia en el manejo oncológico es:

a) Tratamiento sistémico de la enfermedad

b) Control locorregional del cáncer

c) Sólo manejo con fin paliativo

d) Sólo empleada como adyuvante a otras terapias


e) Ninguna de las anteriores

RO33. De los enunciados siguientes, selñale el que corrersaponde a la radioterapia segmentaria


hemicorporal:

a) Se da en una sola sesión con cobalto 60

b) Es útil en el sarcoma de Kaposi y micosis fungoides

c) Indicada sobretodo para paliación del dolor óseo

d) Se ha abandonado por la morbilidad atardía pulnmonar

e) Se emplea sólo en neoplasia superficiales

RO34. Hay una mayor morbilidad pulmonar cuando se combina la radioterapia y uno de los siguientes
medicamentios:

a) Metotrexate

b) Procarbacina

c) VP-16

d) Ciclofosfamida

e) Bleomicina

ONCOLOGÍA MÉDICA

OM1. El sulfato de vincristina pertenece al grupo de:

a) Antibióticos

b) Alcaloides

c) Antracíclicos

d) Alquilantes

e) Antimetabolitos

OM2. Pertenece al grupo de alquilantes:

a) Metotrexate

b) Prednisona

c) Mostaza nitrogenada
d) Epirrubicina

e) Vindesina

OM3. La ciclofosfamida pertenece al grupo de:

a) Antracíclicos

b) Alcaloides

c) Antibióticos

d) Alquilantes

e) Antimetabolitos

OM4. La vinblastina pertenece al grupo de:

a) Antracíclicos

b) Alcaloides

c) Antibióticos

d) Alquilantes

e) Antimetabolitos

OM5. El etopósido pertenece al grupo de:

a) Antracíclicos

b) Alcaloides

c) Antibióticos

d) Alquilantes

e) Antimetabolitos

OM6. El cisplatino pertenece al grupo de:

a) Antracíclicos

b) Alcaloides

c) Antibióticos
d) Alquilantes

e) Antimetabolitos

OM7. Las nitrosoureas pertenecen al grupo de:

a) Alquilantes

b) Miscelaneos

c) Antimetabolitos

d) Antibióticos

e) Alcaloides

OM8. El arabinosido de citosína pertenece al grupo de

a) Alquilantes

b) Miscelaneos

c) Antimetabolitos

d) Antibióticos

e) Alcaloides

OM9. El 5-fluoruracilo pertenece al grupo de:

a) Alquilantes

b) Misceláneos

c) Antimetabolitos

d) Antibióticos

e) Alcaloides

OM10. El melfalán pertenece al grupo de:

a) Alquilantes

b) Miscelaneos

c) Antimetabolitos
d) Antibióticos

e) Alcaloides

OM11. Los siguientes fármacos son fase específicos, excepto:

a) Los antimetabolitos

b) Los antibióticos

c) Los alquilantes

d) Los alcaloides de la vinca

e) Los taxoides

OM12. El 5FU es un:

a) Alquilante

b) Antimetabolito

c) Antibiótico

d) Alcaloide

e) Hormonal

OM13. La mitomicina se clasifica como:

a) Antimetabolito

b) Antibiótico

c) Alquilante

d) Alcaloide

OM14. La ifosfamida pertenece al grupo de:

a) Agentes alquilantes

b) Antimetabolitos

c) Antibióticos antitumorales

d) Plantas alcaloides
c) Hormonales

OM15. Las Antraciclinas pertenecen al grupo de:

a) Agentes alquilantes

b) Antimetabolitos

c) Antibióticos antitumorales

d) Alcaloides

e) Hormonales

OM16. La Bleomicina pertenece al grupo de:

a) Agentes alquilantes

b) Antimetabolitos

c) Antibióticos antitumorales

d) Alcaloides

e) Hormonales

OM17. La Actinomicina D pertenece al grupo de:

a) Agentes alquilantes

b) Antimetabolitos

c) Antibióticos antitumorales

d) Alcaloides

e) Hormonales

OM18. Fármaco antineoplásico más nefrotóxico:

a) Cisplatino

b) Bleomicina

c) Adriamicina

d) Vinblastina

e) Ciclofosfamida
OM19. Medicamento que mejora el apetito en pacientes con cáncer:

a) Ciproeptadine

b) Sulfato de hidracine

c) Acetato de megestrol

d) Cimetidina

e) Ranitidina

OM20. Antineoplásico que ejerce su acción inhibiendo la enzima dehidrofolatorreductasa:

a) Vinblastina

b) Etopósido

c) 6 Mercaptopurina

d) 5 Fluouracilo

e) Metotrexate

OM21. En una investigación clínica hecha recientemente en los Estados Unidos se encontró que el efecto
del cartílago de tiburón en pacientes con neoplasias malignas avanzadas produce:

a) Remisión parcial en el 10%

b) Mejoría sintomática en el 40%

c) Enfermedad estable en el 17%

d) Remisión completa en el 3%

e) Progresión de la enfermedad

OM22. Agente quimioterapéutico que incrementa su efectividad antitumoral a temperaturas elevadas:

a) Bleomicina

b) Fluorouracilo

c) Metotrexate

d) Inhibidores de la topoisomerasas

e) Vincristina

OM23. Las siguientes son drogas cuya potencial toxicidad pulmonar puede incrementarse con la
administración de oxígeno, excepto:
a) Mitomicina C

b) Carboplatino

c) Bleomicina

d) Ciclofosfamida

e) BCNU

OM24. La cardiotoxicidad a la ciclofosfamida es potencializada con la exposición previa a:

a) Metotrexate

b) Esteroides

c) Antraciclinas

d) Vincristina

e) Citosina arabinósido

OM25. La toxicidad selectiva de mayor importancia de los inhibidores del huso mitótico es la:

a) Cardiotoxicidad

b) Nefrotoxicidad

c) Neurotoxicidad

d) Toxicidad pulmonar

e) Toxicidad hepática

Correlacione la toxicidad con el agente terapéutico que la provoca anotando en el paréntesis la letra
correspondiente:

AGENTE TOXICIDAD

OM26. Adriamicina ( ) a) Neurotoxicidad

OM27. Platino ( ) b) Cistitis

OM28. Vinblastina ( ) c) Toxicidad renal

OM29. Ciclofosfamida ( ) d) Cardiotoxicidad

OM30. Metotrexate ( ) e) Hepática


OM31. Efecto no deseable de la Bleomicina más importante:

a) Neuropatía periférica

b) Hematológica

c) Insuficiencia renal

d) Fibrosis pulmonar

e) Digestivo

OM32. La anemia hemolítica y la disfunción renal pueden estar presentes en pacientes tratados con
Mitomicina C, cuya dosis acumulativa excede 60 mg en aproximadamente:

a) 4-15%

b) 50-70%

c) 0-1%

d) 20-30%

OM33. Efecto adverso de la L Asparaginasa que pocas veces se presenta:

a) Anormalidad en las pruebas de función hepática

b) Azotemia

c) Disfución renal

d) Pancreatitis

e) Mielosupresión

OM34. En la actualidad el régimen de condicionamiento previo a un transplante heterológo en pacientes con


neoplasias malignas esta conformada por:

a) Fármacos citóxicos antineoplásicos a dosis convencionales

b) El empleo de radiación ionizante

c) Altas dosis de quimioterapia y/o radioterapia

d) Altas dosis de quimioterapia

e) Altas dosis de quimioterapia e inmunosupresores


OM35. ¿Qué efectos se persiguen al producir una ablación de medula ósea?

a) Formación de espacio medular y supresión inmune

b) Supresión inmune y erradiación de enfermedad

c) Erradiación de enfermedad y formación de espacio medular

d) Supresión del fenómeno injerto contra enfermedad y supresión inmune

e) Formación de espacio, supresión inmune y erradicación de enfermedad

OM36. Fármaco que debe suspenderse o evitarse cuando existe diarrea secundaria a la quimioterapia:

a) Metoclopramida

b) Codeína

c) Caolín-pectina

d) Paracetamol

e) Aspirina

OM37. Principal mecanismo de acción del 5-Fluorouracilo:

a) Inhibición de la enzima timidilato sintetasa

b) Inhibir la enzima dehidrofolatorreductasa

c) Interacción directa con la topoisomerasa II

d) Inhibir la ribonucleótido reductasa

Correlacione las columnas siguientes anotando en el paréntesis el tratamiento correspondiente a la neoplasia:

Neoplasias Tratamiento

OM38. Coriocarcinoma (d) a) 5FU+Leucovorin

OM39. Cáncer de colon (a) b) Platino+Ciclofosfamida

OM40. Melanoma (c) c) Interferón

d) Metotrexate

e) FAC

OM41. A un paciente con carcinoide ampliamente metastásico intratable, se le puede realizar lo siguiente,
excepto:
a) Destumorización

b) Quimioterapia

c) Bloqueadores del canal de calcio

d) Análogos de la somatostatina

e) Quimioterapia intraperitoneal

OM42. El mejor analgésico para el dolor crónico de los pacientes con cáncer es:

a) El dextropoxifeno

b) El demerol

c) El nalbufina

d) El metamizol

e) La morfina

OM43. Los microabscesos de Pautrier son característicos de:

a) Histoplasmosis pulmonar relacionada a VIH

b) Lesiones diseminadas en piel por infiltracion por Linfoma Cutaneo

c) Alteraciones histopatológicas de una micosis fungoide

d) Microabscesos hepáticos en pacientes inmunosuprimidos

e) Imagen radiológica por múltiples metástasis hepáticas

OM44. Los siguientes esquemas para la terapia de los linfomas pertenecen a los llamados de tercera
generación, excepto:

a) VACOP–B

b) MACOP–B

c) m-BACOD

d) Pro-MACE-CytaBOM

OM45. La Hipótesis de Goldie–Coldman está relacionada con:

a) La inducción de apoptosis por quimioterapia


b) La inducción de apoptosis por radioterapia

c) La inducción de diferenciacion celular

d) La resistencia a drogas en relación a tamaño tumoral

e) La supresión del gen P 53 para aumentar la respuesta a citostáticos

OM46. Oncogén fuertemente relacionado con angiogénesis tumoral:

a) BRCA 2

b) Rb1

c) N-MYC

d) RAS

e) BCL 2

Relacione las columnas siguientes:

OM47. Eritrodisestesia palmoplantar ( )


OM48. Síndrome urémico hemolítico ( )
OM49. Leucemia no linfoblástica aguda ( )
OM50. Infiltrado pulmonar intersticial ( )
OM51. Secreción inapropiada de hormona antidiurética ( )

a) Mitomicina C

b) Etopósido

c) 5-fluoracilo

d) Vincristina

e) Metotrexate

OM52. La sobreexpresión del oncogén c-erb B 2 en pacientes con Ca de mama indica:

a) Su presencia nos indica resistencia a antraciclinas

b) Mayores probabilidades de beneficio con tamoxifén adyuvante

c) Sólo es importante como factor de riesgo para cáncer de mama familiar

d) Es el factor pronóstico más importante

e) Menor beneficio con terapia adyuvante a base de tamoxifén y CMF


OM53. La quimioterapia adyuvante en disgerminoma de ovario está indicada en:

a) Todas las pacientes

b) A partir de etapa I a con tumor mayor de 7 cm

c) Nunca está indicada

d) Sólo simultánea a RT

e) A partir de etapa I b

OM54. La clasificación por riesgos de acuerdo a el Grupo Colaborador Internacional de los tumores de
células germinales, los pacientes con tumores seminomatosos de mal pronóstico son:

a) No lo incluye

b) Extragonadal en mediastino

c) Metástasis viscerales extrapulmonares

d) Extragonadal en retroperitoneo

Correlacionar columnas:

OM55. P53 ( a ) a) Gen supresor

OM56. N M YC (b) b) Oncogén

OM57. Rb (a)

OM58. BRCA 2 (a)

OM59. Marcador con mayor valor predictivo en relación a cáncer de mama:

a) Catepsina D

b) Ca 15.3

c) Receptores hormonales

d) P 53

OM60. Una de las características de los derivados del platino es:


a) El oxaliplatino se une a las proteínas hasta en un 90%

b) El daño renal por oxaliplatino es principalmente en el glomérulo

c) El carboplatino prodece más neurotoxicidad que el cisplatino

d) La principal toxicidad del oxaliplatin es mielosupresión

e) El mecanismo de acción es formando uniones covalentes en la base del ADN

OM61. En relación al etopósido es cierto lo siguiente:

a) La toxicidad hematológica es acumultaiva

b) Es un inhibidor de topoisomerasa II

c) Es un inhibidor de topoisomerasa I

d) La penetración al líquido pleural es alta

e) La toxicidad tardía más importante es esterilidad

OM62. De los siguientes enunciados referentes a gemcitabina, seleccione el verdadero:

a) Es un análogo del 5 fluoracilo

b) Es un agente fase específico con acción en metafase

c) Es un análogo de la citarabina

d) La dosis recomedada es de 1000–1250 mg/m 2 c/21 días

e) Su vía de elimación es primordialmente riñón

OM63. En relación a taxanos:

a) La premedicación con esteroides es para disminuir emesis importante

b) La trombocitopenia es la principal toxicidad hematológica

c) En comparación con paclitaxel produce mayor frecuencia de edema

d) Son fase específicos, actúan en fase S

e) Requiere ajuste de dosis en caso de depuración de creatinina de 80 o menos


OM64. Uno de los tratamientos hormonales para cáncer de próstata son los análogos de LHRH, la siguiente
aseveración es correcta:

a) Su mecanismo de acción es produciendo incremento de las gonadotropinas a nivel periférico

b) Una precaución inicial es la posible exacerbación inicial de la sintomatología

c) Las respuestas oscilan entre el 10–20%

d) Siempre debe administrarse con un antiandrógeno no esteroideo

e) Está indicado a partir de la etapa II

OM65. Entre los eventos adversos de la terapia hormonal en cáncer de próstata es cierto:

a) El acetato de megestrol es la terapia de elección de segunda línea

b) La ginecomastia secundaria a flutamida se observa sólo en el 2%

c) La toxicidad hepática producida por análogos de LHRH es irreversible

d) La única toxicidad aguda grave por ketoconazol es necrosis hepatocelular

e) La toxicidad más frecuente del acetato de megestrol es náusea y diarrea

OM66. El virus de Epstein Barr se ha asociado con las neoplasias siguientes, excepto:

a) Sarcoma de Kaposi

b) Enfermedad de Hodgkin

c) Carcinoma nasofaríngeo

d) Linfoma de Burkitt

e) Linfoma de células T

OM67. Una mujer de 42 años de edad se realiza su Pap anual y le reportan células escamosas atípicas de
significancia indeterminada. Con lo anterior usted sugiere:

a) Que continue realizandose su Pap anual rutinariamente

b) Realizarse frecuentemente sus Pap (cada 4 o 6 meses)

c) Realizarse colposcopía

d) Realizarse biopsia por conización

e) Realizarse histerectomía simple


OM68. ¿Para qué tipo de carcinoma tiroideo no se utiliza el Iodo131?

a) Papilar

b) Folicular

c) Medular

d) Mixto papilar-folicular

e) El Iodo131 es utilizado para todos los tipos de carcinoma tiroideo

OM69.Cuál de los antineoplásicos siguientes no está asociado con el mecanismo de resistencia que se
encuentra a su lado

ANTINEOPLÁSICO MECANISMO DE RESISTENCIA

a) Ciclofosfamida Aldehido deshidrogenasa

b) Ara C Resistencia citoquinética

c) Etopósido Elevación de niveles de topoisomerasa 1

d)Metotrexate Amplificación genética

e) Radioterapia Hipoxia en el tumor

f) Doxorrubicina Multirresistencia a drogas

OM70. Los antineoplásicos siguientes son, de alguna manera, cardiotóxicos, excepto:

a) Herceptin (trastuzamab)

b) Cisplatino

c) Taxol

d) 5-Fluorouracilo

e) Metotrexate

OM71. Los alcaloides de la vinca producen un bloqueo mitótico por que:

a) Inhiben las DNA polimerasas

b) Estabilizan los microtúbulos

c) Despolimerizan filamentos intermedios


d) Despolimerizan microtúbulos

e) Inhiben la liberación de la glicoproteína-p

OM72. La hipercalcemia por cáncer de mama está causada predominantemente por:

a) Liberación por las metástasis óseas

b) Producción tumoral de Vitamina D

c) Producción tumoral de PTH like

d) Producción tumoral de PTH

e) Producción tumoral de factores activadores de los osteoclastos

OM73. ¿Con qué tratamiento se logra una paliación rápida y duradera de la disfagia en los pacientes con
cáncer de esófago irresecable?

a) Braquiterapia intraluminal

b) Terapia fotodinámica

c) Terapia con láser

d) Stent (endoprótesis)

e) Dilatación con balón

OM74. Cuál es la terapia estándar en el cáncer esofágico localizado en pacientes en buen estado:

a) Resección con terapia adyuvante

b) Resección con radioterapia posoperatoria

c) Resección con quimioterapia adyuvante

d) Quimioterapia neoadyuvante y resección

e) Quimioradioterapia con braquiterapia adyuvante

OM75. Todos son predisponentes para cáncer gástrico, excepto:

a) Pólipos adenomatosos gástricos

b) Irradiación gástrica previa

c) Cirugía gástrica previa


d) Presencia de H. pylori

e) Gastritis atrófica

OM76. Una mujer de 60 años de edad es vista por el internista por pérdida de peso, anorexia, ictericia y
distención abdominal. El primer paso para llegar al diagnóstico es:

a) Resonancia magnética nuclear sin contraste de abdomen superior

b) Ultrasonido de abdomen superior seguido de TAC

c) Colangiografía transhepática

d) Colangiopancreatografía retrógrada endoscópica seguida de exploración abdominal

e) Tomografía por emisión de positrones (PET) de todo el cuerpo

OM77. Todos son considerados pólipos colorrectales precancerosos, excepto:

a) Adenoma velloso

b) Adenoma tubular

c) Pólipos hiperplásicos

d) Adenomas tubulovellosos

e) Adenomas de 4 cm

OM78. Los siguientes son factores de riesgo para cáncer de vejiga, excepto:

a) Tabaquismo (cigarro)

b) Schistosoma haemattobium

c) Altos niveles de testosterona

d) Aminas aromáticas

e) Fenotipo acetilador

OM79. En cáncer vesical, cuál paciente representa un candidato razonable para preservar la vejiga con
radiación y quimioterapia?

a) Tumor T3a (células transicionales)

b) T4 con hidronefrosis
c) T2 con carcinoma in situ asociado

d) Adenocarcinoma T3a

OM80. Cuál de los siguientes enunciados es correcto en referncia al uso de quimioterapia en pacientes con
carcinoma de células escamosas de cervix?

a) La quimioterapia combinada con tres fármacos es mejor que con un farmaco en pacientes que
tienen metástasis a distancia, ya que se mejora el periodo libre de enfermedad y la sobrevida
global en tres veces más al utilizar 3 fármacos.

b) La combinación de cisplatino y 5 fluorouracilo es el tratamiento de elección después de la


histerectomía radical con linfadenectomía que reporta ganglios positivos.

c) El Carboplatino como monofármaco induce un 30% de respuestas clínicas completas en


pacientes con carcinoma escamoso metastásico versus 10-12% de respuestas completas con
cisplatino sólo.

d) La quimioterapia neoadyuvante se recominda para pacientes con carcinoma de cervix de 5 cm


cuando la histerectomía radical y la linfadenectomía pélvica son parte del plan terapéutico en ese
paciente.

e) Ninguna de las anteriores

OM81. El tratamiento estándar con quimioterapia para pacientes con cáncer epitelial de ovario con cirugía
óptima es:

a) Cisplatino

b) Carboplatino

c) Cisplatino + Ciclofosfamida

d) Carboplatino + Ciclofosfamida

e) Carboplatino + Paclitaxel

OM82. Cuál es el sitio más común de metástasis para el tumor trofoblástico gestacional?

a) Vagina

b) Pulmón

c) Ovarios

d) Cerebro

e) Hígado
OM83. Cuál es el fármaco más activo para el carcinosarcoma uterino?

a) Ifosfamida

b) Cisplatino

c) Doxorrubicina

d) Etopósido

OM84. Un hombre de 45 años de edad es valorado por un nevo negro de 8 mm. en la pared abdominal. La
biopsia excisional reporta un melanoma nodular con Clark III y 1.5 mm de espesor. El paso siguiente debe
ser:

a) Interferón alfa adyuvante por un año

b) Los márgenes varían según el sitio de presentación, en este caso 4 cm de margen es lo


apropiado

c) Una excisión de 2 cm de margen en el primario y considerarse linfadenectomía por mapeo y


ganglio centinela

d) No es necesario terapia adicional

OM85. Un hombre de 35 años de edad con sarcoma osteogénico está recibiendo metotrexate a altas dosis.
Todos los enunciados son correctos para reducir el riesgo de toxicidad, excepto:

a) Administración de bicarbonato de sodio intravenoso

b) Administración de leucovorin

c) Hidratación intravenosa

d) Administración de mesna

e) Monitoreo de los niveles séricos de metotrexate

OM86. Cuál de los factores de riesgo siguientes tiene la mayor significancia estadística epidemiológica
como agente etiológico en la neoplásia del cervix:

a) Virus herpes siple tipo 2

b) Múltiples parejas sexuales

c) Virus de papiloma humano (VPH)

d) Tabaquismo
e) Inicio de vida sexual a edad temprana

OM87. Son factores de riesgo asociados a cáncer endometrial, excepto:

a) Nuliparidad

b) Obesidad

c) Diabetes

d) Tumor ovárico de células de la teca-granulosa

e) Anticonceptivos orales

OM88. Los tumores trofobláticos gestacionales no metastásicos se tratan usualmente con:

a) Quimioterapia combinada

b) Histerectomía

c) Histerectomía y quimioterapia adyuvante con monofármaco

d) Quimioterapia con monofármaco

e) Dilatación y curetage

OM89. Los tumores trofoblásticos gestacionales metastásicos de bajo riesgo se tratan usualmente con:

a) Quimioterapia con monofármaco con metotrexate o actinomicina D

b) Quimioterapia combinada con metotrexate, actinomicina D y etopósido

c) Cirugía con quimioterapia adyuvante

d) Quimioterapia con monofármaco con etopósido

e) Quimioterapia combinada con cisplatino y paclitaxel

OM90. Los tumores trofoblásticos gestacionales metastásicos de alto riesgo se tratan usualmente con:

a) Quimioterapia con monofármaco con metotrexate o actinomicina D

b) Quimioterapia combinada con metotrexate, actinomicina D y etopósido

c) Cirugía con quimioterapia adyuvante

d) Quimioterapia con monofármaco con etopósido

e) Quimioterapia combinada con cisplatino y paclitaxel


OM91. Una quimioterapia tópica efectiva para micosis fungoides es:

a) Metotrexate

b) Ara C

c) Taxol

d) Ciclofosfamida

e) Mostasa nitrogenada

OM92. Signos y síntomas que constituyen los síntomas B:

a) Fiebre, pérdida de peso, sudoración nocturna

b) Fiebre, prurito y sudores nocturnos

c) Fiebre, anorexia y pérdida de peso

d) Pérdida de peso, sudoración nocturna y prurito

e) Fiebre, fatiga y pérdida de peso

OM93. Tropisetrón, granisetrón y ondansetrón han demostrado efectividad y seguridad en cuál de los
síndromes de vómito siguientes:

a) Emesis tardía

b) Enfermedad por movimiento

c) Emesis anticipatoria

d) Emesis por obstrucción intestinal

e) Emesis inducida por radiación

OM94. Con cuál de los siguientes agentes la neuropatía periférica progresa después de suspender el
fármaco:

a) Vincristina

b) Taxol

c) Cisplatino

d) Suramina
OM95. Paciente femenino de 38 años de edad con carcinoma ductal infiltrante poco diferenciado de 0.5 cm
y con 18/18 ganglios negativos para metástasis, su tratamiento posterior a la mastectomía radical modificada
y disección radical de axila es:

a) 6 ciclos de FAC

b) 6 ciclos de CMF

c) Antraciclinas + Taxanes

d) Tamoxifén

e) Observación

OM96. Paciente femenino de 52 años de edad con diagnóstico de cáncer de mama IIIB, recibió cuatro ciclos
de FAC presentando estatismo por tres ciclos y posteriormente recibiendo radioterapia, logrando remisión
parcial. Se sometió a cirugía radical que mostró tumor de 3 x 4 cm y 5 ganglios positivos, el siguiente
tratamiento es:

a) FAC por cuatro ciclos

b) CMF + tamoxifén

c) Taxanes

d) 5FU / cisplatino

e) Sólo tamoxifén

OM97. El mejor tratamiento de quimioterapia neoadyuvante en cáncer de pulmón de células no pequeñas es:

a) Etopósido / Cisplatino

b) Carboplatino / Taxanes

c) Cisplatino / Vinorelbine

d) Cisplatino / Gemcitabine

e) Aún no está determinado

OM98. Son antineoplásicos de segunda línea en cáncer colorrectal, excepto:

a) Irinotecán

b) Oxaliplatino

c) Levamisol
d) Interferón

e) Cisplatino

OM99. La terapia adyuvante en cáncer gástrico ha mostrado los siguientes resultados:

a) Sólo aumento en el periodo libre de enfermedad

b) Aumento de la sobrevida global

c) a y b son ciertas

d) Ninguna de las anteriores

OM100. El alopurinol, inhibidor de la xantino-oxidasa, es capaz de potenciar el efecto de los siguientes


fármacos antitumorales, excepto:

a) 6-mercaptopurina

b) Ciclofosfamida

c) 5-fluouracilo

d) Azatioprina

OM101. El cisplatino, análogo del platino, es capaz de inducir nefrotoxicidad al afectar:

a) Segmento S3 de los túbulos contorneados proximales

b) El túbulo colector

c) Segmento S1 de los túbulos contorneados proximales

d) La rama ascendente del asa de Henle

OM102. La fisiopatología del desarrollo de cardiotoxicidad por adriamicina involucra:

a) Fijación del complejo NADH-adriamicina

b) Generación de radicales superóxido

c) Reducción de las concentraciones de glutatión

d) Disfunción mitocondrial

e) Todas las anteriores


OM103. En la antibioticoterapia empírica e inicial del paciente con neutropenia y fiebre, de bajo riesgo,
puede emplearse:

a) Amikacina + vancomicina

b) Cefalosporina de segunda generación + vancomicina + amkacina

c) Fluconazol + vancomicina + imipenem

d) Quinolonas

e) Cefalosporina con acción antipseudomona + amikacina

OM104. El efecto tóxico pulmonar de la bleomicina comunmente es observado cuando se emplean dosis
mayores a:

a) 300 unidades

b) 450 unidades

c) 550 unidades

d) 150 unidades

OM105. El esquema terapéutico más utilizado en las neoplasias epidermoides del cáncer de cabeza y cuello
incluye:

a) Pacitaxel y carboplatino

b) Carboplatino y 5FU

c) 5FU y metotrexate

d) 5FU y cisplatino

e) Todos

OM106. El medicamento con mejor actividad antitumoral contra los sarcomas de partes blandas es:

a) Dactinomicina

b) Dacarbizina

c) Paclitaxel

d) Ifosfamida

e) Ninguna
OM107. El fármaco con mejor actividad, utilizado como agente único, contra el melanoma maligno es:

a) Dacarbazina

b) Cisplatino

c) Vinblastina

d) Tamoxifén

e) Ninguno

OM108. ¿Cuál de los siguientes medicamentos antineoplásicos puede producir el síndrome hemolítico-
urémico?

a) Ifosfamida

b) Adriamicina

c) Mitomicina C

d) Mitoxantrone

e) Todos

OM109. Es el factor pronóstico más importante en el Sarcoma osteogénico que norma la conducta
terapéutica médica:

a) Necrosis tumoral

b) Grado de diferenciación

c) Tamaño tumoral

d) Invasión a tejidos blandos

e) Todos

OM110. La dosis en mg/m2 de los medicamentos que integran el esquema FAC en la terapia adyuvante del
cáncer mamario son:

a) 500-40-500 mg/m2

b) 500-50-50 mg/m2

c) 600-40-600 mg/m2

d) 600-50-500 mg/m2
e) Ninguno

OM111. El esquema quimioterapéutico más comúnmente empleado en la terapia adyuvante del cáncer de
colon es:

a) 5FU + leucovorin

b) Gemcitabina + 5FU

c) Tomudex

d) Oxaliplatino

OM112. Bajo condiciones ideales, es el esquema de quimioterapia a utilizar de primera elección en


pacientes con cáncer de ovario y citorreducción subóptima:

a) Cisplatino + ciclofosfamida

b) Gemcitabina + cisplatino

c) Paclitaxel + carboplatino

d) Docetaxel + vinorelbina

OM113. La dosis de carboplatino debe ser calculada según:

a) La filtración glomerular

b) La fracción exctretada de sodio

c) La fórmula de Calvert

d) la depuración de creatinina

OM114. La descripción del modelo de crecimiento tumoral se le atribuye a:

a) Goldie y Collman

b) Halsted

c) Calvert

d) Gompertz

OM115. La teoría de generación de resistencia tumoral fue aplicada en 1984 por:


a) Goldie y Collman

b) Halsted

c) Calvert

d) Gompertz

OM116. Se les considera radiopotencializadores a los fármacos siguientes, excepto:

a) 5FU

b) Gemcitabina

c) Paclitaxel

d) Adriamicina

OM117. Son fármacos alquilantes los siguientes, excepto:

a) Ifosfamida, 6-mercaptopurina, tiotepa y mecloretamina

b) Cisplatino, paclitaxel, adriamicina y bleomicina

c) Etoposido, vinorelbina, tamoxifén y metotretaxe

d) 5FU, gemcitabina, vincristina y tioguanina

OM118. El principal antagonísta del metotrexate es:

a) Ácido araquidónico

b) Ácido fólico

c) Ácido folínico

d) Ácido acético

OM119. Es el metabolismo de la ifosfamida que induce el efecto neurotóxico:

a) Acroleina

b) Cetoaldehído

c) Oxazofosforina

d) Dehidrofosforina
OM120. Los siguientes compuestos pertenecen al grupo de antibióticos, excepto:

a) Adriblastina

b) Daunomicina

c) Actinomicina D

d) Bleomicina

e) Epipodofilotoxina

OM121. La acción farmacológica de los alcaloides de la vinca es sobre la:

a) Mitosis

b) Fase S

c) Fase G1

d) Fase G2

e) Meiosis

OM122. La imidazol carboxamida pertenece al grupo de:

a) Antracíclicos

b) Alcaloides

c) Antibióticos

d) Alquilantes

e) Antimetabolitos

OM123. El Metotrexate pertenece al grupo de:

a) Antracíclicos

b) Alcaloides

c) Antibióticos

d) Alquilantes

e) Antimetabolitos
OM124. La L-asparaginasa pertenece al grupo de:

a) Alquilantes

b) Misceláneos

c) Antimetabolitos

d) Antibióticos

e) Alcaloides

OM125. Inhibe la dihidrofolato reductasa:

a) Cisplatino

b) Nitrosourea

c) Vindesina

d) Teneposido

e) Metotrexate

OM126. El alopurinol inhibe la síntesis de la:

a) Deamino nucleosidasa

b) Hipoxantina

c) Xantino-oxidasa

d) Transpeptidasa

e) Glucoronil transferasa

OM127. Gen productor de resistencia a las drogas antineoplásicas:

a) RTB

b) WT

c) mdr

d) myc

e) p53
OM128. Los siguientes son efectos tóxicos reconocidos de la mitomicina C, excepto:

a) Insuficiencia renal asociada a anemia hemolitica microangiopática

b) Radiodermitis húmeda con el uso de radioterapia simultánea

c) Neumonitis

d) Síndrome de Lermitte

e) La difusión hepática incremento el riesgo de desarrollo de nefrotoxicidad

OM129. Mecanismo básico de acción de los taxanes:

a) Inhibición de la síntesis de purinas y pirimidinas

b) Alquilación

c) Intercalación

d) Inhibición de la despolimerización de los microtúbulos

e) Inhibición de la polimerización de los microtúbulos

Correlacione el marcador con el tumor identificado, anotando en el paréntesis la letra correspondiente:

Tumor Marcador

OM130. Colon (d) a) CA 15:3

OM131. Testículo (b) b) Alfafetoproteína

OM132. Ovario (e) c) CA 19-9

OM133. Linfoma (f) d) Antígeno carcinoembrionario

OM134. Mama (a) e) CA 125

OM135. Páncreas ( c ) f) Beta 2 microglobulina

OM136. El Letrozol pertenece al grupo de:

a) Agentes alquilantes

b) Antimetabolitos

c) Antibióticos antitumorales
d) Alcaloides

e) Antiaromatasas

OM137. El mecanismo de acción del tamoxifén como agente quimopreventivo es debido principalmente a:

a) Tiene un intenso efecto introductor estrogénico en el tejido mamario humano

b) No tiene efecto agonista antiestrogénico o estrogénico en el tejido mamario

c) No juega un rol en el tratamiento de los carcinomas con receptores estrogénicos negativos

d) Suprime el factor de crecimiento 1 tipo insulina en cáncer mamario

e) No induce la síntesis de la transformación del factor de crecimiento beta

OM138. En la mujer posmenopáusica la mayor fuente de estrogenos plasmáticos:

a) Son los estrógenos adrenales

b) Son las carnes rojas y las grasas en la dieta

c) Es la conversión enzimática de los andrógenos adrenales

d) Los fitoestrógenos

e) Es el tejido estromal ovárico

OM139. Localización anatómica de centro del vómito:

a) Corteza cerebral

b) Cerebelo

c) Esfínter esofágico inferior

d) Región postrema de la médula oblongada

e) Ganglios basales

OM140. Fármaco relacionado con el desarrollo del síndrome urémico hemolítico:

a) Vincristina

b) Metotrexate

c) Mitomicina

d) Docetaxel
e) Bleomicina

OM141. La expresión del oncogén HER-2/neu se asocia a:

a) Mayor frecuencia de metástasis cerebrales

b) Una pobre respuesta a los taxanes

c) La aparición de metástasis tardías, generalmente después de 36 meses de la cirugía inicial

d) La necesidad de administrar radioterapia poscirugía

e) Predice pobre respuesta al tamoxifén

OM142. ¿Cuál de las siguientes es la menos predecible de las miopatías cardiacas por antracíclicos?

a) Ritmo de galope

b) Ecocardiograma

c) Intervalos sistólicos

d) Estudio de gamagrafía cardíaca

e) Electrocardiograma

OM143. Características del donador compatible no relacionado:

a) Parentesco con el paciente receptor y similitud antigénica

b) Parentesco con el receptor y similitud parcial antigénica

c) Sin relación familiar y con similitud antigénica

d) Sin relación familiar y similitud parcial antigénica

e) Sin relación familiar ni similitud antigénica

OM144. Son drogas cardiotóxicas las siguientes, excepto la:

a) Adriamicina

b) Daunomicina

c) Rubidazone

d) Ciclofosfamida

e) Actinomicina D
OM145. La patología de la cardiotoxicidad a la ciclofosfamida se caracteriza por:

a) Fibrosis miocárdica

b) Fibrosis pericárdica

c) Fibroelastosis endocárdica

d) Degeneración mitocondrial

e) Pancarditis hemorrágica

OM146. Disritmia que con mayor frecuencia se relaciona a la infusión de Placlitaxel:

a) Taquicardia supraventricular

b) Extrasistoles ventriculares

c) Bloqueo auriculo-ventricular

d) Taquicardia helicoidal

e) Bradicardia sinusal

OM147. Variedad de neuropatia más frecuente relacionada a Placlitaxel:

a) Motora

b) Sensitiva

c) Central

d) Autonómica

e) Mixta

OM148. Las afirmaciones siguientes son ciertas en relación con la toxicidad pulmonar de la Bleomicina,
excepto:

a) El riesgo es muy alto a partir de dosis acumulativas>300UI

b) La insuficiencia renal incrementa el riesgo de toxicidad pulmonar

c) La aplicación intramuscular reduce el riesgo en comparación con la aplicación intravenosa

d) La mielosupresión es importante

e) Produce neurotoxicidad motora


OM149. Uno de los mecanismos de resistencia tumoral a los alquilantes es:

a) La intercalación de las moléculas al DNA

b) Inactivación para la conjugación con glutation

c) La disminución de los compuestos azufrados

d) El menor nivel de enzima transportadora intracelular

e) La modificación genética

OM150. El cisplatino puede ocasionar a las células ciliadas del órgano de corti lesiones, causando:

a) Tinitus

b) Pérdida de la audición y tinitus

c) Vértigo

d) Convulsiones y vértigo

e) Desequilibrio

OM151. ¿Cuál de los siguientes radionucleotidos se usa comúnmente en el tratamiento de las metástasis
óseas sintomáticas?

a) Rhenium 186

b) Estroncio 89

c) Yodo 123

d) Fósforo 33

OM152. Las siguientes son propiedades de la doxorrubicina y de otras antraciclinas, excepto que:

a) La interposición de la droga ocurre entre las bases adyacentes de DNA alterando la topografía
del mismo

b) La habilidad para intercalarse entre las bases del DNA se correlaciona con el efecto tumoral

c) Estos agentes aumentan el daño en el DNA mediado por las topoisomerasas II

d) Estos agentes se metabolizan como radicales libres

e) El daño mediado por los radicales libres es responsable de la cardiotoxicidad


OM153. ¿Cuál de los siguientes síndromes paraneoplásicos es el resultado de la producción de anticuerpos
por el tumor?

a) Hipercalcemia

b) Degeneración cerebral subaguda

c) Hiponatremia

d) Hipoglicemia

e) Ninguno de los anteriores

OM154. Limita la dosis del carboplatin:

a) Nefrotoxicidad

b) Neurotoxicidad

c) Leucopenia

d) Trombocitopenia

e) Ototoxicidad

OM155. La excreción renal del carboplatin es del:

a) 100%

b) 90%

c) 80%

d) 70%

e) 50%

OM156. La dosis máxima total acumulable por metro cuadrado de superficie corporal de la epirubicina es de
mg:

a) 500

b) 700

c) 900

d) 1100

e) 1500
OM157. Efecto tóxico más importante de la amifostina:

a) Alopecia

b) Neurotoxicidad

c) Estomatitis

d) Hipotensión

e) Flebitis

Correlacione las columnas siguientes anotando en el paréntesis el tratamiento correspondiente a la neoplasia:

Neoplasias Tratamiento

OM158. Coriocarcinoma (d) a) 5FU+Leucovorin

OM159. Cáncer de mama (e) b) Platino+Ciclofosfamida

OM160. Cáncer de testículo (f) c) Interferón

OM161. Cáncer de ovario (b) d) Metotrexate

OM162. Cáncer de colon (a) e) Fac

OM163. Melanoma (c) f) Vp16+Platino+Bleomicina

OM164. En relación con la terapia adyuvante para el cáncer:

a) Actualmente existe un tratamiento adyuvante posoperatorio efectivo en el melanoma etapa III

b) La terapia de combinación para melanoma con agentes que incluyan cisplatino y tamoxifén
esta limitada a pacientes posmenopáusicas

c) La quimioterapia adyuvante se basa en la destrucción total de las células tumorales

d) La quimioterapia neoadyuvante se basa en la destrucción total de las células tumorales

e) La quimioterapia adyuvante mejora la sobrevida de los pacientes operados de cáncer mamario,


de ovario y de colon

OM165. Los estudios clínicos randomizados que utilizan los factores estimulantes de colonias como
profilaxis para apoyar una quimioterapia, han documentado lo siguiente excepto que:

a) Acortan la duración de la neutropenia severa en cerca de un 50%

b) Disminuyen la incidencia de fiebre neutropénica

c) Prolonga la sobrevida

d) Disminuyen la utilización de antibióticos parenterales


e) Acortan la hospitalización

OM166. En el tratamiento de las enfermedades malignas de la infancia la quimioterapia combinada ha


permitido una reducción en las dosis de radioterapia ¿En cuál de los casos siguientes?

a) Enfermedad de Hodking

b) Tumor de Wilms

c) Tumores de sistema nervioso central

d) Rabdiomiosarcoma

e) a y b

OM167. Fármaco de elección inicial para el tratamiento de la diarrea secundaria a quimioterapia:

a) Codeínea

b) Caolín-pectina

c) Loperamida

d) Difenoxilato con atropina

e) Bismuto

OM168. En caso de diarrea grave o severa secundaria a la QT, que no cede a los agentes más comunes, el
tratamiento de elección, es:

a) Tintura de opio

b) Atropina

c) Ocreotibo

d) Anticolinérgicos

e) Ninguno

OM169. Fármacos de elección para el tratamiento de la compresión de médula espinal por metástasis son:

a) Predisona-betametasona

b) Metilprednisolona-prednisona

c) Metilprednisolona-dexametasona
d) Betametasona-dexametasona

e) Hidrocortisona-betametasona

OM170. El anticuerpo monoclonal anti-CD 20 está indicado:

a) Terapia en linfomas indolentes

b) Para el diagnóstico diferencial de linfomas de Células T o NK

c) Terapia de primera línea en linfomas cutáneos de células T

d) Como condicionamiento para pacientes que recibirán trasplante de médula ósea

e) Nada de lo anterior

OM171. Linfomas frecuentemente relacionados a VIH:

a) Linfoma de células T periféricas y linfoma de células pequeñas nodular

b) Linfoma de células pequeñas núcleo no hendido y linfoma inmunoblástico

c) Micosis fungoide y linfoma –leucemia de Células T

d) Enfermedad de Hodgkin y linfoma MALT

OM172. En la prevención de síndrome de lisis tumoral se incluye lo siguiente, excepto:

a) Soluciones alcalinas

b) Alopurinol

c) Soluciones sin potasio

d) Soluciones fisiológicas más potasio

e) Diuréticos

OM173. Estudio clínico fase III es aquel que:

a) Se realiza en animales para investigar toxicidad y farmacocinética de la droga

b) Comparar un esquema estandar con un nuevo tratamiento

c) Su mayor objetivo es demostrar la toxicidad de una droga

d) Sólo requiere pocos pacientes para demostrar la actividad de la droga


OM174. En relación a la toxicidad ocular del citosín arabinósido:

a) Es independiente de la dosis

b) Provoca retinopatía por regeneración axonal

c) Produce conjuntivitis por inhibición de la síntesis de ADN en el epitelio corneal

d) Es de mecanismo desconocido

e) Sólo se produce con dosis acumulativas de 10 g x m 2

OM175. La supervivencia total a cinco años en los pacientes con tumores germinales no seminomatosos de mal
pronóstico tratados adecuadamente es de:

a) 70%

b) 80%

c) 90%

d) 48%

OM176. En el Ca Cu locorregionalmente avanzado, la RT simultánea con QT que incluyan Cis platino ha


logrado:

a) Ninguna mejoría en cuanto a supervivencia libre de enfermedad

b) En estudio de metaanálisis reducción altamente significativa en el riesgo de recurrencia

c) En estudio de metaanálisis ninguna mejoría en el riesgo de recurrencia

d) 80-90 porciento de respuestas completas

OM177. Con respecto al manejo hormonal de pacientes con cáncer de próstata, ¿Cuál de los enunciados
siguientes es verdadero?

a) La flutamida y acetato de ciproterona son antiandrógenos no esteroideos

b) El ketoconazol a dosis de 1200 mg diarios disminuye los niveles de progesterona e incrementa


los de androstenediona y testosterona

c) Flutamida bloquea los receptores androgénicos por competencia con los andrógenos

d) Se recomienda la aplicación de análogos de LHRH contra orquiectomia debido a mayor


eficacia
OM178. Grupo de antineoplásicos que está asociado con el efecto vesicante más frecuente y potente:

a) Agentes alquilantes

b) Antimeabolitos

c) Antibióticos

d) Alcaloides de la Vinca

e) Enzimas

OM179. ¿Cuál de las siguientes fases del ciclo celular es la más radiosensible?

a) Go

b) G 1

c) M

d) S

e) Todas las fases son igualmente radiosensibles

OM180. La administración intraarterial del 5-Fluorouracilo desoxirribonucleósido (FUDR), ha sugerido en


diferentes estudios aleatorizados que puede ser beneficioso en cuál de las condiciones siguientes:

a) Cáncer de cólon metastásico y confinado al hígado

b) Cáncer gástrico metastásico al pulmón

c) Cáncer de páncreas metastásico al peritoneo

d) Cáncer de ovario metastásico a ganglios paraórticos

e) Mesotelioma pleural

OM181. ¿Cuál de los antineoplásicos siguientes puede causar más neuritis?

a) Vincristina

b) 5-Fluorouracilo

c) Doxorrubicina

d) Interleucina-2

e) Ciclofosfamida
OM182. La resistencia a los alcaloides de la vinca y los taxanes se asocia a la sobreexpresión de:

a) DNA topoisomerasa II

b) DNA polimerasa Delta

c) Proteínas de filamentos intermedios

d) Citoquinas

e) Glicoproteína-p

OM183. El interferón es igual o más efectivo que la quimioterapia en las siguientes neoplasias, excepto:

a) Linfoma folicular

b) Carcinoma renal

c) Melanoma

d) Carcinoma colorrectal

e) Leucemia mielógena crónica

OM184. En un paciente con cáncer de páncreas con metástasis hepáticas, existen una variedad de opciones
de tratamiento: Asumiendo que todos los pacientes reciben un manejo óptimo en el control del dolor y la
náusea, las opciones podrían ser: (1) 5 Fluorouracilo, (2) gemcitabine, (3)medicamentos en investigación
fase I o II, (4) radioterapia o (5) tratamiento de soporte. Cuál de los siguientes es más apropiado:

a) 1 o 2

b) 5

c) 1, 2 o 3

d) 1 y 4

e) 2, 3 o 5

OM185. Una mujer de 67 años de edad previamente sana es vista por una masa rectal, al ser operada de
resección quirúrgica de la misma, se reporta un adenocarcinoma rectal que invade hasta la serosa, 2/10
ganglios positivos para metástasis. Cuál de los siguientes, representa el mejor tratamiento para esta paciente:

a) Administración de 5FU en combinación con leucovorin

b) Administración de 5FU sólo

c) Administración de 5FU, lemustina (metil-CCNU) y vincristina en combinación con radioterapia


d) 5FU en combinación con radioterapia

e) Radioterapia sola

OM186. Mujer de 48 años de edad con cáncer de mama, etapa II (ganglios positivos), tratado con
lumpectomía y radioterapia, seguida de quimioterapia adyuvante con FAC. El tumor fue marcadamente
positivo para RE y RP. Después de la quimioterapia inició tratamiento con tamoxifén. Durante la
quimioterapia dejó de menstruar y después de iniciar el tamoxifén ha presentado bochornos intensos que
incluso la debilitan ¿Qué terapia sugiere?

a) Terapia de remplazo estrogénico

b) Un barbitúrico

c) Un agente progestacional

d) Reiniciar tamoxifén

e) Sintroid

OM187. Una paciente con historia de cáncer de mama la cual fue tratada con lumpectmía y disección radical
de axila, seguida de radioterapia posoperatoria, cinco años más tarde presenta recurrencia en la mama
ipsilateral. Con esto, la paciente debe ser tratada inicialmente con:

a) Lumpectomía con radioterapia posoperatoria

b) Quimioterapia sistémica

c) Mastectomía total

d) Tamoxifén sólo

e) Incremento con radioterapia al sitio de la recurrencia

OM188. El alopurinol, inhibidor de la xantino-oxidasa, es capaz de potenciar el efecto de los fármacos


antitumorales siguientes, excepto:

a) 6-mercaptopurina

b) Ciclofosfamida

c) 5-fluoururacilo

d) Azatioprina

OM189. El cisplatino, análogo del platino, es capaz de inducir daño nefrotóxico al afectar:
a) El segmento S3 de los túbulos contorneados proximales

b) El túbulo colector

c) El segmento S1 de los túbulos contorneados proximales

d) La rama ascendente del asa de Henle

OM190. La fisiopatología del desarrollo de cardiotoxicidad por adriamicina involucra:

a) Fijación del complejo NADH-adriamicina

b) Generación de radicales superóxido

c) Reducción de las concentraciones de glutatión

d) Disfunción mitocondrial

e) Todas las anteriores

OM191. El medicamento con mejor actividad antitumoral contra los sarcomas de partes blandas es:

a) Dactinomicina

b) Dacarbazina

c) Paclitaxel

d) Ifosfamida

e) Ninguna

Relacione las columnas siguientes, neoplasia correspondiente con marcador tumoral:

OM192. Carcinoma epitelial de ovario ( )

OM193. Carcinoma medular de tiroides ( )

OM194. Feocromocitoma ( )

OM195. Carcinoma germinal de testículo ( )

OM196. Enfermedad neoplásica gestacional del trofoblasto ( )

OM197. Adenocarcinoma de páncreas ( )

OM198. Enfermedad ósea metastásica ( )

OM199. Adenocarcinoma de colon ( )


OM200. Carcinoma epidermoide de pulmón ( )

OM201. Tumor carcinoide ( )

a) 5 HAA

b) CK-BB

c) CA-125

d) ACE

e) Catecolaminas

f) Calcitonina

g) AFP

h) HGC (Fracción B)

i) CA 19-9

j) Fosfatasa alcalina

OM202. La dosis de carboplatino debe calcularse según la:

a) Filtración glomerular

b) Fracción excretada de sodio

c) Fórmula de Calvert

d) Depuración de creatinina

OM203. La descripción del modelo de crecimiento tumoral se le atribuye a:

a) Goldie y Coldman

b) Halsted

c) Calvert

d) Gompertz

OM204. La teoría de generación de resistencia tumoral fue aplicada en 1984 por:

a) Goldie y Coldman

b) Halsted

c) Calvert
d) Gompertz

OM205. Es el metabolito de la ifosfamida que induce el efecto neurotóxico:

a) Acroleina

b) Cetoaldehído

c) Oxazofosforina

d) Dehidrofosforina

OM206. En relación con el tratamiento adyuvante en cáncer de recto etapa III, sólo es cierto que:

a) El tratamiento adyuvante es igual que en cáncer de colon

b) Debe recibir radioterapia como única modalidad adyuvante

c) La combinación de quimioterapia–radioterapia es la indicada dado que el 90% de los pacientes


presentan recurrencia local

d) La radioterapia adyuvante es necesaria por el 50% de recurrencia local

e) Más del 70% recurren a distancia, por lo que es indispensable la quimioterapia adyuvante

OM207. En relación con la combinación quimioterapia-radioterapia en cáncer de recto, una de las


aseveracionesa siguientes es correcta:

a) Han presentado mayor morbimortalidad en el transoperatorio cuando se aplican antes de la


cirugía

b) Los resultados son iguales en supervivencia y periodo libre de enfermedad comparativamente


contra radioterapia sola

c) Se puede utilizar indistintamente mitomicina C, carmustina o 5FU concomitantemente con


radioterapia

d) En pacientes con invasión a órganos vecinos, la combinación preoperatoria puede incrementar


la resecabilidad del 60 al 80%

e) Se incrementa la toxicidad hematológica, principalmente

OM208. El beneficio del tratamiento adyuvante en cáncer de colon etapa Dukes C es:

a) Incrementar la supervivencia hasta en 50%

b) Reducir la recurrencia en aproximadamente 40% y la mortalidad en 30%

c) Mínimo, pues sólo mejora discretamente el periodo libre de enfermedad

d) Reducir las recurrencias locales


e) Ninguno

OM209. El medicamento que más se ha utilizado en la terapia adyuvante para cáncer colorrectal es el 5FU,
de lo siguiente sólo es cierto que:

a) Está indicado por 12 meses en combinación con levamisol, como la mejor opción

b) Se utiliza leucovorín antes del 5FU para reducir su toxicidad

c) Debe aplicarse sólo intravenosamente y en bolo, pues con ello se redu ce la toxicidad
hematológica

d) La aplicación en infusión continua ha ofrecido discretamente mejores porcientos de respuesta


comparativamente con aplicaciones en bolo, pero con mayor toxicidad hematológica

e) La combinación 5FU-leucovorín ofrece los mismos resultados que el uso de 5FU-levamisol


pero con menor toxicidad.

OM210. En el tratamiento de pacientes con cáncer colorrectal metastásico una de las siguientes
aseveraciones es correcta:

a) Con la combinación 5FU-leucovorín se obtienen porcientos de respuesta hasta del 60% y


supervivencia media de hasta 24 meses

b) La mejor combinación es con 5FU-levamisol, con mínima toxicidad

c) Se puede utilizar indistintamente en primera línea 5FU, UFT o raltitrexed

d) La combinación 5FU-leucovorín incrementa la supervivencia media a 12-18 meses,


comparativamente con el mejor soporte médico que es no mayor de 6 meses

e) Las respuestas clínicas completas con 5FU-leucovorín son hasta del 20%

OM211. En el tratamiento del cáncer colorrectal metastásico se ha tratado de efectuar modulación


bioquímica del 5FU. Elija la opción correcta:

a) El ácido folínico disminuye la síntesis del ADN por la formación del complejo FdUMP, timidilato
sintetasa y un cofactor, incrementando la eficacia del 5FU

b) El leucovorín incrementa los efectos terapéuticos del 5FU y reduce sus efectos tóxicos

c) El único mecanismo sinérgico del interferón con el 5FU es que inhibe la timidilato sintetasa

d) El trimetrexate un análogo de la purina, incrementa la citotoxicidad del 5FU pero con toxicidad
hematológica grado 3-4 en más del 75% de los pacientes

e) En general el uso de 5FU ofrece los mismos resultados de respuesta que con la combinación
5FU-leucovorín
OM212. En los últimos años se han incorporado nuevas drogas en el tratamiento del cáncer colorrectal metastásico
o recurrente, elija el enunciado correcto:

a) El UFT está compuesto de dos moléculas, el tegafur y el uracil. Esta última inhibe la enzima DPD
(primera enzima degradadora del 5FU) incrementado teóricamente su eficacia

b) El capecitabine es metabolizado a nivel hepático por la timidin fosforilasa, convirtiendose a 5FU y


de esa manera actuar en las células neoplásicas

c) El raltritexed, un potente inhibidor de la enzima dehidrofolato reductasa, incrementa la eficacia


del 5FU

d) El CPT11, un análogo de la enzima topoisomerasa I, llega a presentar hipersensibilidad al frío


hasta en el 50% de los pacientes

e) El uso de oxaliplatino sólo está recomentado en pacientes resistentes a 5FU

OM213. Con la quimioterapia irinotecán-5FU-leucovorín aplicada en el cáncer colorrectal metastásico se


debe considerar lo siguiente:

a) Es un tratamiento de segunda o tercera línea

b) Los principales eventos adversos son neutropenia, alopecia y diarrea

c) La dosis del irinotecán que se aconsejan son 200 mg/m 2 semanal por 4 semanas con 2
semanas de descanso

d) También se puede aplicar irinotecán 500 mg/m 2 cada 3 semanas

e) En primera línea se han obtenido iguales porcientos de respuesta y supervivencia en


comparación con 5FU-leucovorín

OM214. En el cáncer colorrectal metastásico se ha utilizado la combinación oxaliplatino-5FU y leucovorín,


de lo siguiente sólo es cierto:

a) Las más graves y frecuentes complicaciones de la combinación son trombocitopenia,


hipersensibilidad al frío y diarrea

b) El oxaliplatino, al igual que el cisplatino, requiere hiperhidratación y mismo manejo antiemético

c) Ha demostrado mejores porcientos de respuesta y discreta mejoría en supervivencia en primera


línea, en comparación con 5FU-leucovorín

d) Se debe considerar esta combinación en primera línea más que la combinación irinotecán-5FU-
leucovorín, pues ofrece mejores resultados

e) Se utiliza sólo en pacientes refractarios a 5FU


OM215. El pronóstico en cáncer gástrico avanzado (etapa III-IV) es malo, se consideran todos estos casos
incurables. Entonces, ¿Existe algún beneficio de la quimioterapia en este grupo de pacientes?

a) No deben recibir quimioterapia, pues aún así todos morirán entre 3 a 5 meses

b) Existe alta toxicidad a la quimioterapia, con nula ganancia en supervivencia

c) Con quimioterapia se puede incrementar la supervivencia media hasta en 10 meses, siendo


superior al mejor soporte médico de 3 a 5 meses

d) Es cuestionable su uso, pues se considera un tumor poco sensible a quimioterapia

e) Es un tumor sensible a quimioterapia (respuestas hasta de 50%) que suelen durar hasta 15
meses

OM216. En la mayoría de los pacientes con cáncer gástrico el tratamiento con quimioterapia se considera
paliativo. Elija la aseveración correcta:

a) La combinación cisplatino y 5FU es considerada la estándar

b) La incorporación del irinotecán al cisplatino ha ofrecido respuestas mayores del 70%, lo que
podrá ocupar pronto la primera línea

c) Existe un gran número de combinaciones sin tenerse hasta ahora la combinación ideal

d) Es un tumor altamente sensible a quimioterapia teniéndose respuestas del 50 al 80%

e) Debe ser aplicada concomitante con radioterapia para obtener algún beneficio

OM217. En relación con el tratamiento adyuvante en cáncer gástrico elija la respuesta correcta:

a) La supervivencia a 5 años en pacientes con ganglios positivos N1 es del 70% y con N2 del
60%, lo que no justifica el uso de tratamiento adyuvante

b) Hay beneficio sólo en pacientes con ganglios negativos

c) Recientes publicaciones han demostrado que la combinación QT-RT puede beneficiar a


pacientes con metástasis a ganglios linfáticos, por lo que deberá considerarse el tratamiento
adyuvante

d) Aún con la combinación QT-RT, ningún estudio ha demostrado que la adyuvancia beneficie a
estos pacientes

e) Mejora el periodo libre de enfermedad y supervivencia en pacientes que recibieron algún


esquema que tuviese platino

OM218. ¿Cuál sería la ventaja el utilizar QT neoadyuvante en cáncer gástrico?

a) Ninguno
b) Incrementa la supervivencia y período libre de enfermedad a todos los pacientes

c) Debe aplicarse con radioterapia para lograr algún beneficio

d) Disminuye las metástasis a ganglios linfáticos

e) Mejora la sintomatología y puede incrementar la posibilidad de resecabilidad

OM219. De la QT neoadyuvante en cáncer gástrico, elija el enunciado correcto:

a) Se debe utilizar en pacientes considerados potencialmente resecables, a pesar de tener


porcientos de resecabilidad de 30 a 50

b) La supervivencia media en pacientes sometidos a QT neoadyuvante oscila entre 15 y 18 meses

c) Existe alta toxicidad en el transoperatorio por lo que no se recomienda la QT neoadyuvante

d) Se debe aplicar sólo a pacientes con etapas IA a II

e) Cerca del 50% de los pacientes pueden progresar durante la QT neoadyuvante

OM220. En relación al tratamiento paliativo con QT en cáncer de páncreas, elija la aseveración correcta:

a) Lo más importante en la QT son los porcientos de respuesta para poder predecir su


supervivencia media, a mayor porciento de respuesta mayor supervivencia

b) Los agentes más activos son el etopósido, la carmustina y mitomicina C

c) La gemcitabine ha ofrecido mejor calidad de vida a pesar del bajo porciento de respuesta global

d) La supervivencia media es la misma con quimioterapia que con mejor soporte médico en etapa
IV

e) Por su corta perspectiva de vida no está justificado usar quimioterapia

OM221. El tratamiento multimodal en cáncer pancreático debe considerarse por lo siguiente:

a) En pacientes con adenocarcinoma de páncreas localmente avanzado con radioterapia y


quimioterapia adyuvante han ofrecido prolongar la supervivencia

b) A pesar de todos los estudios publicados, la quimioterapia adyuvante no modifica la


supervivencia en etapas avanzadas

c) Con el tratamiento multimodal en etapas localmente avanzadas se ha incrementado la


supervivencia media hasta en 40 meses

d) El tratamiento secuencial cirugía-radioterapia-quimioterapia ha proporcionado los mejores


resultados
e) La combinación concomitante radioterapia-quimioterapia sólo ofrece mayor morbimortalidad

OM222. La función de la quimioterapia en cáncer de esófago es la siguiente:

a) Es nula, pues los agentes de quimioterapia sólo ofrecen entre 10 y 20% de respuestas

b) No prolongan supervivencia ni mejorar calidad de vida

c) Dado que tiene mal nivel de actividad al diagnóstico ningún paciente es candidato a
quimioterapia pues llega a producir alta toxicidad

d) En general ofrece mejor supervivencia y disminuye síntomas en más del 50% de los pacientes
comparativamente con mejor soporte médico

e) Los agentes más activos son carboplatino, etopósido y 5FU

OM223. El abordaje terapéutico inicial en pacientes con cáncer esofágico localmente avanzado es:

a) Cirugía radical y de acuerdo al hallazgo histológico ofrecer radioterapia adyuvante

b) Cirugía radical sin tratamiento adyuvante

c) Quimioterapia neoadyuvante y ulteriormente cirugía radical

d) Radioterapia como tratamiento único

e) Quimioterapia y radioterapia concomitante y de acuerdo a evolución, observación o cirugía


complementaria

OM224. En relación con el tratamiento multimodal en cáncer de esófago, es cierto lo siguiente:

a) La supervivencia en etapas IIa y IIb es del 70 y 80%, siendo útil sólo la esofagectomía

b) La principal modalidad de tratamiento es la cirugía, pues del 40-50% se diagnostican en etapas


tempranas

c) Dado que al momento del diagnóstico entre 50 y 75% de los pacientes considerados
candidatos a cirugía tendrán metástasis ganglionares deben iniciar tratamiento con quimioterapia
y radioterapia

d) La quimioterapia neoadyuvante ha ofrecido mejor porciento de supervivencia


comparativamente con sólo cirugía

e) La combinación quimioterapia y radioterapia ofrece casi los mismos resultados que sólo
radioterapia con supervivencia a 2 años del 50 y 45%, respectivamente (etapa localmente
avanzada)
OM225. En relación con el tratamiento del cáncer de canal anal, elija la aseveración correcta:

a) La resección radical está indicada en pacientes con etapas I y II con iguales resultados que la
combinación radioterapia y quimioterapia

b) Todos los pacientes deberán someterse a resección abdomino-perineal complementaria

c) La combinación 5FU-mitomicina y radioterapia ofrece mayores porcientos de respuesta con 80


a 90% de respuestas completas, comparativamente con la combinación cisplatino-5FU y
radioterapia con 50 a 70%

d) En pacientes con T3 y T4 sometidos a quimioterapia y radioterapia, el 50% requerirán


resección abdomino-perineal de salvamento

e) Con radioterapia sola se logran respuesta completas 20 a 40% de los casos

OM226. ¿Cuál es la utilidad de la quimioterapia en pacientes con cáncer de pulmón de células no pequeñas
etapas III y IV?

a) Sólo ofrece mejoría de los síntomas sin incremento en la supervivencia

b) En caso de utilizarse concomitante con radioterapia, se puede ocasionar neumonitis hasta en


40% de los pacientes

c) Ninguna

d) Cuando se utiliza de manera neoadyuvante, puede incrementar el porciento de resecabilidad y


doblar el tiempo de supervivencia

e) Mejora la calidad de vida y discretamente incrementa la supervivencia

OM227.¿Cuál es la mejor conducta terapéutica en pacientes con Cáncer de pulmón de células no pequeñas
etapas III-IV?

a) Mejor soporte médico

b) Quimioterapia y radioterapia combinada (concomitante o secuencial)

c) Quimioterapia sola

d) Radioterapia seguida de cirugía

e) Inmunoterapia

OM228. Una de las aseveraciones siguientes es cierta en relación al tratamiento con quimioterapia en
pacientes con cáncer de pulmón de células no pequeñas avanzado:

a) Los principales predictores de respuesta a quimioterapia son los niveles de DHL y albúmina

b) La QT basada en platino, continúa siendo la columna vertebral


c) El carboplatino ofrece mayores porcientos de respuesta que el cisplatino, con igual
supervivencia

d) La combinación cisplatino-etopósido continúa siendo el esquema estándar

e) De las nuevas combinaciones, la que ha demostrado ser superior es cisplatino-vinorelbine

OM229. Existen nuevas terapias de biología molecular en el cáncer de pulmón, elija la correcta:

a) La combinación quimioterapia y C225 (anticuerpo mononuclear del EGFR) produce sinergismo


en la inhibición del crecimiento neoplásico

b) El anti-Her 2/neu deberá ser utilizado en casi todos los pacientes, pues se sobre-expresa hasta
en el 60% de los casos

c) La mutación RAS ocurre en el 20% de los pacientes de CPCNP y tiene un impacto de buen
pronóstico cuando está presente

d) Iressa o tavcera son inhibidores de la tirosin-quinasa que producen sinergismo con varios
agentes citotóxicos, pero no se han utilizado por su alta toxicidad hematológica y cardiaca

e) No tienen algún beneficio, el tratamiento seguirá siendo con quimioterapia

OM230. En relación al tratamiento del cáncer de pulmón de células pequeñas, es cierto que:

a) La combinación de quimioterapia VAC sigue siendo el de primera línea

b) Usar radioterapia concomitante con quimioterapia desde el primer ciclo puede ofrecer hasta
20% de supervivencia a 5 años

c) Utilizando radioterapia secuencial posterior a 4 o 6 ciclos de quimioterapia, se obtiene 20% de


supervivencia a 5 años

d) La radioterapia a SNC profiláctica está indicada en pacientes con respuesta parcial y total,
aplicándose a los 3 meses de haberse terminado la quimioterapia

e) El beneficio de la radioterapia a SNC profiláctica es la reducción del 50% de la presencia de


metástasis cerebrales

OM231. ¿Qué lugar ocupa el cáncer de mama como causa de cáncer en la mujer a nivel mundial?

a) Primero

b) Segundo

c) Tercero

d) Cuarto
e) Quinto

OM232. ¿Cuáles síndromes familiares han sido identificados en cáncer de mama?

a) Síndrome de cáncer de mama-ovario

b) Síndrome de Li-Fraumeni

c) Enfermedad de Cowden

d) Todos los anteriores

e) Ninguno de los anteriores

OM233. Los siguientes factores aumentan el riesgo de cáncer de mama, excepto:

a) Menarca temprana

b) Menopausia tardía

c) Primer embarazo después de los 30 años

d) Mastopatia fibroquística

e) Hiperplasia ductal o lobulillar atípica

OM234. Los genes BRCA 1 y 2 son:

a) Son oncogenes de transmisión autosómica recesiva

b) Son genes supresores

c) Su presencia indica la posibilidad de cáncer de mama en el 10%

d) Están relacionados con el Síndrome de Li Fraumeni

e) Las portadoras siempre desarrollaran cáncer de mama

OM235. El tamoxifén utilizado como adyuvante en cáncer de mama durante 5 años, ¿En qué porciento
reduce la tasa anual de recurrencia y mortalidad, respectivamente?

a) 7 y 5

b) 12 y 9

c) 47 y 26
d) 62 y 35

e) 90 y 80

OM236. ¿Por cuánto tiempo se recomienda dar el tamoxifén adyuvante?

a) 1 año

b) 2 años

c) 5 años

d) 10 años

e) 15 años

OM237. ¿Cuál es el riesgo absoluto de desarrollar cáncer endometrial en mujeres tratadas con tamoxifén?

a) 2 casos por 100 pacientes por año

b) 2 casos por 1000 pacientes por año

c) 20 casos por 100 pacientes por año

d) 20 casos por 1000 pacientes por año

e) 20 casos por 100000 pacientes por año

OM238. ¿Cuál es el efecto colateral más frecuente del tamoxifén?

a) Bochornos

b) Toxicidad ocular

c) Eventos tromboembólicos

d) Trombocitopenia

e) Cáncer de endometrio

OM239. En relación con el tamoxifén es cierto lo siguiente, excepto:

a) Tiene efecto agonista en mama y endometrio

b) En mujeres mayores de 50 años el riesgo relativo de cáncer de endometrio es 2-3 veces mayor

c) Se ha observado reducción de las lipoproteínas de poca densidad y colesterol total


d) En las pacientes tratadas como terapia adyuvante hay una reducción en la incidenacia de
cáncer de mama contralateral del 15%

e) Hay una reducción en la incidencia de cáncer de mama contralateral del 39%

OM240. ¿Cuál es el beneficio otorgado con quimioterapia adyuvante?

a) Reduce el riesgo anual de recurrencia

b) Reduce el riesgo anual de muerte

c) a y b son correctas

d) Ninguna es correcta

OM241. ¿Cuál es la mediana de sobrevida en pacientes con cáncer de mama metastásico?

a) 10 años

b) 5 años

c) 2 años

d) 6 meses

e) 3 meses

OM242. ¿Qué porciento de pacientes con cáncer de mama sobreexpresan HER2neu?

a) 100

b) 45 a 60

c) 25 a 30

d) 5 a 10

e) 0 a 3

OM243. En relación con el estudio comparativo de tamoxifén versus placebo como quimioprevención, una
de las aseveraciones siguientes es correcta:

a) Hubo una reducción en la incidencia de cáncer invasor del 20% para el grupo de tamoxifén

b) Ninguna diferencia estadísticamente significativa en cuanto a la reducción de cáncer invasor


c) El porciento anual de cáncer invasor se redujo a 3.4 de 1000 para el grupo de tamoxifén contra
6.8 de 1000 en el placebo

d) El riesgo relativo para cáncer de endometrio fue de 3.5 para el grupo de tamoxifén

OM244. ¿A qué grupo farmacológico pertenece el trastuzumab?

a) Alcaloides de la Vinca

b) Taxanes

c) Alquilantes

d) Anticuerpos monoclonales

e) Antracíclicos

OM245. El trastuzumab está aprobado para el tratamiento de:

a) Linfomas no Hodgkin CD20+

b) Cáncer de pulmón de células no pequeñas

c) Enfermedad de Hodgkin

d) Mieloma Múltiple

e) Cáncer de mama metastásico HER2neu+

OM246. Es el evento adverso serio más importante observado al combinar trastuzumab con antraciclinas:

a) Neutropenia

b) Vómito

c) Disfunción cardiaca

d) Diarrea

e) Fiebre

OM247. ¿Cuáles son las tasas de respuesta a quimioterapia de primera línea en cáncer de mama
metastásico?

a) 10-20%

b) 20-25%
c) 35-40%

d) 50-70%

e) 90-100%

OM248. En relación al tumor del sito placentario las aseveraciones siguientes son correctas, excepto:

a) Es más frecuentes después de embarazos no molares

b) Por inmunohistoquímica existen pocas células teñidas para hormona gonadotropina coriónica

c) Los valores séricos de la fracción beta de HGC usualmente son muy altos

d) Es relativamente resistente a quimioterapia

e) La terapia de elección es la cirugía

OM249. Uno de los factores de mal pronóstico en la enfermedad trofoblástica gestacional es:

a) Antecedente de mola hidatiforme

b) Metástasis vaginales

c) Antecedente de embarazo a término

d) Fracción beta de hormona GC en orina > de 10, 000

e) Metástasis pulmonares bilaterales de más de 5 en cada campo

OM250. ¿Cuál es la quimioterapia de primera línea más utilizada en la enfermedad trofoblástica gestacional
metastásica de mal pronóstico?

a) Actinomicina más Metotrexate

b) Metotrexate –5FU-cfa

c) Actinomicina D

d) EMA-CO

e) Dosis altas de QT

OM251. ¿Cuál es la tasa de recaída de la enfermedad trofoblástica gestacional metastásica de pobre


pronóstico?

a) 1%
b) 2%

c) 13%

d) 50%

e) 100%

OM252. ¿Qué lugar ocupa el cáncer de cérvix como causa de muerte por cáncer en México?

a) Primero

b) Segundo

c) Cuarto

d) Noveno

e) Décimo

OM253. ¿Cuál es el tipo histológico más frecuentemente encontrado en cáncer de cérvix?

a) Carcinoma epidermoide

b) Adenocarcinoma

c) Carcinoma endometroide

d) Carcinoma de células claras

OM254. El estadio Ib2 de cáncer de cérvix es:

a) Confinado a cérvix mayor de 6 cm

b) Confinado a cérvix mayor de 4 cm

c) Microinvasor

d) Infiltración incipiente de tercio superior de vagina

e) Infiltración incipiente a parametrios sin fijarlos

OM255. La recomendación actual para el manejo del cáncer de cérvix estadio Ib2, IIa-b y IIIa es:

a) Quimioterapia neoadyuvantes con esquemas que contengan platino

b) Quimioterapia adyuvante posterior a RT o cirugía


c) Sólo radioterapia y/o cirugía según la etapa

d) Radioterapia-quimioterapia concomitante con esquemas que contengan platino

e) Ninguna de las anteriores

OM256. La quimioterapia neoadyuvante seguida de RT comparada con sólo RT en cáncer de cervix ha


logrado:

a) Más muertes en el brazo de QT neoadyuvante

b) Mayor supervivencia total en el brazo de QT neoayuvante más RT

c) Ninguna diferencia en ambos brazos

d) Un beneficio en intervalo libre de enfermedad sólo en los estadios II

e) Ninguna de las anteriores

OM257. Los siguientes son mecanismos de resistencia a el metotrexate, excepto:

a) Sobreexpresión de la dehidrofolato-reductasa

b) Defectos en la poliglutamación

c) Alteración de las proteínas transportadoras

d) Sobreexpresión de la timidilatosintetasa

e) Sobreexpresión de la alquil-guanina-alquil transferasa

OM258. En relación al acetato de ciproterona la aseveración siguiente es correcta:

a) Es un antiandrógeno esteroideo

b) Es un agonista-antagonista de LHRH

c) Es un antiandrógeno no esteroideo

d) Es un progestágeno

OM259. Con respecto a letrozol, cuál de las aseveraciones siguientes es correcta:

a) Es un inhibidor de aromatasa esteroideo

b) Es un inhibidor de gonadotropinas coriónicas


c) Su toxicidad más frecuentes son fenómenos tromboembólicos

d) Pertenece al grupo de los imidazoles

e) Es un inhibidor de aromatasa de segunda generación

OM260. El tamoxifén produce un riesgo relativo de cáncer de endometrio en mujeres mayores de 50 años,
¿Cuántas veces?

a) 0.5

b) 1.2

c) 2.5

d) 3.8

e) 5

OM261. En relación con el goserelin todas aseveraciones siguientes son correctas, excepto:

a) Es un agonista de LH-RH

b) Inhibe la secreción de gonadotropinas coriónicas por acción directa en ovarios

c) Inhibe la secreción de testosterona y estrógenos a niveles de castración

d) Bajas dosis producen liberación de gonadotropinas

e) Su dosis es de 3.6 mg SC c/28 días

OM262. En relación con anastrozol y letrozol, lo correcto es la aseveración siguiente:

a) Son antiestrógenos que actúan a nivel de receptores

b) Son inhibidores de aromatasas que se unen irreversiblemente a la enzima

c) Manejo de primera línea para cáncer de mama metastásico en mujeres premenopáusicas y


posmenopáusicas

d) Son inhibidores de aromatasa que se unen reversiblemente a la enzima

e) Bloquean la síntesis de cortisol

OM263. El porciento de producción de azoospermia comparando MOPP con AVBD es de:


a) 10-15

b) 35-10

c) 40-3

d) 100-35

OM264. Los fármacos siguientes son considerados de alto riesgo de carcinogénesis, excepto:

a) Cisplatino

b) Adriamicina

c) Etopósido

d) Melfalán

e) Mecloretamina

OM265. En qué difiere la leucemia inducida por inhibidores de topoisomerasa II de las inducidas por
alquilantes:

a) Buena respuesta a la quimioterapia

b) Son leucemias linfocíticas agudas

c) Se asocian con una translocación en 14q18

d) El intervalo de aparición es más corto que por alquilantes

e) El intervalo de aparición es más largo que por alquilantes

OM266. La toxicidad más frecuente por acetato de megestrol es:

a) Disnea

b) Anorexia

c) Prurito

d) Bochornos

e) Diarrea
OM267. En relación con la presencia de glutation S transferasa en los tumores epidermoides de cabeza y
cuello, una las aseveraciones siguientes es correcta:

a) Factor pronóstico independiente

b) Ninguna relación con los tumores epidermoides de cabeza y cuello

c) Factor pronóstico dependiente de otros

d) Sin ningún valor pronóstico

OM268. Las alteraciones cromosómicas más frecuentes encontradas en tumores de cabeza y cuello son:

a) 3p,5p,8p,9p,18q y 21q

b) 5p,18q,21q 7p

c) 3p ,2q.9p

d) 3p,21q

e) Ninguna de las anteriores

OM269. Cuál es el porciento de HPV 16 detectado en tumores de cabeza y cuello:

a) 10

b) 25

c) 40

d) 50

e) 60

OM270. Cuál es el porciento de enfermedad oculta ganglionar en pacientes con tumores de cabeza y cuello,
con cuello clínicamente negativo?

a) 5

b) 10

c) 20 a 30

d) 40

e) 50
OM271. Síndrome(s) relacionado(s) con tumores de cavidad bucal:

a) Anemia de Fanconi

b) Xeroderma pigmentoso

c) Ataxia telangiectasia

d) Sólo a y b

e) Todas las anteriores

OM272. Porciento de supervivencia para pacientes con tumores de piso de boca EC IV a 5 años:

a) 10%

b) 32%

c) 66%

d) 80%

OM273. Porciento de metástasis ocultas en tumores T1 de piso de boca:

a) 5

b) 12

c) 30

d) 47

e) 60

OM274. En relación con la supervivencia de los tumores avanzados de cabeza y cuello (epidermoides), ésta
se ha prolongado debido a:

a) Quimioterapia neoayuvante

b) Quimioterapia adyuvante

c) Qumioterapia secuencial

d) Quimioterapia-radioterapia concomitante

e) Ninguna de las anteriores


OM275. Con respecto a la quimioterapia neoadyuvante en los tumores de cabeza y cuello, la respuesta
correcta es:

a) Ha logrado aumentar la supervivencia total

b) Su indicación más precisa es para preservar órganos

c) Ha logrado aumentar la supervivencia total y el intervalo libre de enfermedad

d) Sólo hay beneficio en los tumores pequeños

OM276. Con el manejo de QT simultáneo a RT hiperfraccionada versus RT hiperfraccionada, produce


mucositis en el porciento siguiente:

a) 20-10

b) 50-50

c) 60-30

d) 75-75

e) 100-20

OM277. Medida que produce una reducción de Xerostomia de más del 20% en los pacientes tratados con RT
más QT simultánea:

a) Colutorios de factores estimulantes de colonias

b) Amifostina

c) Saliva artificial

d) Colutorios con bicarbonato

OM278. Existe predisposición a sarcomas de tejidos blandos asociados con:

a) Neurofibromatosis y poliposis adenomatosa familiar

b) Síndrome de Gardner´s

c) Síndrome de Li-Fraumeni

d) Enfemedad de Von Recklinhausen

e) Todas las anteriores

OM279. ¿Cuál es el sarcoma de partes blandas más frecuente?


a) Tumor desmoide

b) Linfangiosarcoma

c) Rabdomiosarcoma

d) Histiocitoma fibrosos maligno

OM280. ¿Cuál es el sarcoma de partes blandas que con mayor frecuencia da metástasis linfáticas?

a) Sarcoma sinovial

b) Rabdomiosarcoma

c) Sarcoma epiteliode

d) Histiocitoma fibroso maligno

OM281. Factores pronósticos en sarcomas de tejidos blandos:

a) Edad, tamaño del tumor, grado histológico

b) Etapa clínica

c) Sitio de localización

d) Todas las anteriores

OM282. Los sarcomas de la mama representan el 1%, ¿Cuál es el manejo más adecuado?

a) Mastectomía simple

b) Mastectomía más disección radical de axila

c) Mastectomía +RT +QT

d) RT

OM283. ¿Cuál es el porciento de metástasis pulmonares de sarcomas de extremidades y tronco?

a) 10

b) 15

c) 20

d) 40
e) 50

OM284. Tipo histológico de histiocitoma fibroso maligno:

a) Mixoide

b) Células gigantes

c) Angiomatoide

d) Pleomorfico

e) Todas las anteriores

OM285. ¿Cuál es el manejo actual de los sarcomas del estroma gastroinestinal metastásico?

a) Quimioterapia

b) Radioterapia

c) Cirugía

d) Inhibidores de tirosin-quinasa

OM286. En los sarcomas de cabeza y cuello la limitante quirúrgica es:

a) La localización del tumor

b) El margen inadecuado

c) Los bordes positivos

d) Todas las anteriores

OM287. Sarcoma más frecuente en la edad pediátrica:

a) Rabdomiosarcoma embrionario

b) Rabdomiosarcoma alveolar

c) Rabdomiosarcoma botroide

d) Rabdomiosarcoma paratesticular

OM288. ¿Cuál es el tumor óseo más frecuente entre la 1ª y 2ª década de la vida?


a) Osteosarcoma

b) Sarcoma de Ewing

c) Condrosarcoma

d) Linfoma primario de hueso

OM289. El osteosarcoma es un tumor de la niñez y adolescencia; sin embargo, cuando se presenta en adultos de
más de 40 años se asocia a factores predisponentes de:

a) Enfermedad de Paget

b) RT a hueso

c) Displasia fibrosa poliostática

d) Todas las anteriores

OM290. En los tumores óseos la quimioterapia adyuvante ha mejorado la supervivencia de los pacientes en:

a) 20%

b) 30%

c) 55 a 80%

d) 100%

OM291. La(s) vía(s) de diseminacion más frecuente(s) de los osteosarcomas es(son):

a) Hematógena

b) Linfática

c) Por continuidad

d) Linfática y hematógena

OM292. Contraindicación absoluta para cirugía conservadora en osetosarcoma:

a) Fractura patológica

b) Afectación de paquete vasculonervioso

c) Infiltración a tejidos blandos


d) Edad menor de doce años

OM293. Ventaja de la quimioterapia preoperatoria en osteosarcoma de inducción:

a) Control de la enfermedad micrometastásica

b) Disminución del tamaño tumoral

c) Posibilidad de preservar el miembro

d) Proporciona tiempo para la realización de una endoprótesis

e) Todas las anteriores

OM294. Limitante de las altas dosis de MTX es?

a) Mielosupresión

b) Mucositis

c) Toxicidada renal

d) Todas las anteriores

OM295. Las siguinetes son drogas muy activas para osteosarcoma, excepto:

a) Ifosfamida

b) Platino

c) Doxorrubicina

d) Methotrexate

e) Paclitaxel

OM296. En el osteosarcoma paraosteal se tiene que hacer el diagnóstico diferencial con:

a) Osteocondroma

b) Miositis osificante

c) Osteosarcoma osteogénico

d) Todas las anteriores


OM297. ¿Cuál es el sitio más común de localización del condrosarcoma?

a) Pelvis

b) Fémur

c) Costilla

d) Columna vertebral

CASOS CLÍNICOS ONCOLOGÍA QUIRÚRGICA

CQ1. Femenino de 45 años que acude por sangrado de vías urinarias bajas, disuria, poliuria y dolor en bajo
vientre.

CQ1-1. El estudio inmediato para determinar el origen de la hematuria es:

a) Urografía excretora

b) Ultrasonido pélvico

c) TAC

d) Cistoscopía

e) RNM

CQ1-2. El estudio de biopsia reporta adenocarcinoma de células transicionales con un tumor en cuello
vesical de 3 cm. La conducta a seguir es:

a) Quimioterapia intravesical

b) BCG intravesical

c) Estudios de extensión

d) RTU del tumor

e) RT a vejiga

CQ1-3. El reporte de patología indica infiltración a la muscular profunda, los estudios de extensión son
negativos. La conducta a seguir es:

a) Quimioterapia sistémica

b) Quimioterapia intravesical
c) Cistectomía radical más RT posoperatoria a pelvis

d) BCG intravesical

e) Vigilancia

CQ1-4. La etapa clínica (Marshall-Jawett) a la que corresponde es:

a) B1

b) C

c) A

d) B2

CQ1-5. Dado el diagnóstico, la etapificación y el tratamiento efectuado, la sobrevida de este caso a 5 años es
del:

a) 60 a 80%

b) 16 a 53%

c) 41 a 70%

d) 10%

CQ2. Masculino de 60 años con molestias urinarias bajas ocasionales. Exámenes rutinarios reportan PSA de
15 ng/mL.

CQ2-1. Al tacto rectal se palpa nódulo de 1 cm en lóbulo izquierdo de próstata. La conducta a seguir es:

a) RTUP

b) US transrectal+biopsia

c) Biopsia transrectal

d) Biopsia transperineal

CQ2-2. Nódulo único prostático, positivo para adenocarcinoma, suma de Gleason (2+3) 5. La conducta es:

a) RTUP

b) Hormonoterapia
c) Estudios de extensión

d) Radioterapia

CQ2-3. La terapéutica a seguir en enfermedad localizada es:

a) RTU+radioterapia

b) Doble bloqueo androgénico

c) Radioterapia+quimioterapia

d) Prostatectomía radical

CQ2-4. En qué etapa clínica se encuentra el presente caso:

a) T1 C

b) T2 A

c) T1 B

d) T2 B

CQ2-5. En esta etapa clínica la sobrevida a 5 años es de:

a) 68-95%

b) 50-60%

c) 35-55%

d) 100%

CQ3. Masculino de 50 años con hematuria de dos meses de evolución. A la exploración se palpa masa sobre
flanco e hipocondrio izquierdos. Una urografía excretora demostró deformidad del sistema pielocaliceal
izquierdo.

CQ3-1. Clínica y radiológicamente se tiene el diagnóstico de un tumor renal. El paso siguiente sería:

a) Laparotomía exploradora

b) Punción diagnóstica

c) TAC abdominal y pélvica


d) Resonancia magnética de abdomen

e) Lumbotomía exploradora

CQ3-2. Se demostró tumor renal compatible con malignidad y deformidad vesical por probable tumor. El
siguiente paso sería:

a) Laparotomía exploradora

b) Cistoscopía

c) Arteriografía renal selectiva

d) Ultrasonido renal

e) Lumbotomía exploradora

CQ3-3. Al paciente se le realiza cistoscopía que resulta negativa. Los estudios de extensión negativos. El
siguiente paso es:

a) Laparotomía exploradora

b) Lumbotomía exploradora

c) Punción renal para biopsia

d) Nueva urografía excretora

e) Quimioterapia preoperatoria

CQ3-4. Se reseca un tumor renal, por nefrectomía, con resultado de adenocarcinoma de células claras, sin
invasión a la cápsula renal. No se encontró enfermedad extrarenal. La conducta es:

a) RT posoperatoria al lecho renal

b) QT posoperatoria

c) Vigilancia

d) QT y RT posoperatoria

e) Hormonoterapia

CQ3-5. El pronóstico es:

a) 50% a 5 años

b) 66% a 5 años
c) 30% a 5 años

d) 95% a 5 años

e) 100% a 5 años

CQ4. Femenino de 60 años, diabética, con problemas de enfermedad articular degenerativa sin otros
antecedentes de importancia, acude por presentar hematuria franca de cuatro meses de evolución que se ha
ido acentuando paulatinamente, acompañada de ligero dolor en hipogastrio, pérdida de peso de 5
kilogramos, aproximadamente.

CQ4-1. En el protocolo de estudio de este paciente qué examen o exámenes solicitaría en la primera
consulta:

a) TAC de abdomen y pelvis

b) Resonancia magnética nuclear de abdomen y pelvis

c) Urograma excretor

d) Cistoscopía más examen general de orina

e) Urograma excretor, cistoscopía, citología, EGO

CQ4-2. Los estudios solicitados demuestran una lesión exofítica en la cara posterior de la vejiga, el reporte
de citología es de carcinoma de células transicionales. La biopsia confirma tumor de células transicionales
que infiltra hasta la lámina propia. Su conducta a seguir sería:

a) Resección transuretral vesical

b) RTUV más inmunoterapia

c) Cistectomía parcial más inmunoterapia

d) Cistectomía radical más radioterapia

e) Radioterapia más inmunoterapia

CQ4-3. Si usted decide dar inmunoterapia, qué esquema es el más utilizado en nuestro medio:

a) Mitomicina C. 60 mg semanal x 3

b) BCG semanal por 6

c) BCG mensual por 6

d) M-VAC mensual por 6 ciclos

e) No necesitan de inmunoterapia estas lesiones

CQ4-4. La posibilidad de tener una segunda lesión en el tracto urinario es:


a) 2 al 8%

b) 10 al 20%

c) 22 al 30%

d) 35 al 40%

e) Más del 50%

CQ4-5. ¿Qué probabilidad de recurrencia tiene este paciente a 5 años?

a) 5-10%

b) 11 al 20%

c) 21 al 30%

d) 31 al 40%

e) No recurren

CQ5. Masculino de 59 años de edad con antecedente de tabaquismo positivo durante 30 años una cajetilla
diaria. Su padecimiento lo inició hace un mes con astenia, adinamia, tos persistente no productiva y
síntomas debidos a hipertensión arterial.

CQ5-1. El estudio inicial que está indicado es:

a) Telerradiografía de tórax

b) TAC torácica

c) Broncoscopía

d) Baciloscopía

e) Tomografía lineal pulmonar

CQ5-2. Se descubren varios nódulos pulmonares bilaterales pequeños, periféricos. El estudio que solicitaría
para orientar el diagnóstico es:

a) TAC torácica

b) Lavado bronquial

c) TAC abdomino-pélvica

d) Rastreo óseo
CQ5-3. Se descubre un tumor renal izquierdo que por TAC, mide 7 x 6 cm. Cuál es la conducta a seguir:

a) Nefrectomía simple

b) Nefrectomía radical

c) Biopsia dirigida por TAC al tumor renal

d) Nefrectomía endoscópica

CQ5-4. El estudio histopatológico del riñón reporta carcinoma de células claras con patrón sarcomatoide,
Furhman 3. El tratamiento a seguir será:

a) Metastasectomía pulmonar

b) Radioterapia a lecho quirúrgico

c) Quimioterapia a base de 5FU+Vinblastina

d) Aplicación de interferón alfa o interleucina 2

e) Tamoxifén

CQ5-5. La sobrevida a 5 años en este caso de tumor renal con metástasis pulmonares bilaterales es de
aproximadamente:

a) 0-10%

b) 20%

c) 50%

d) 70%

e) 80%

CQ6. Paciente masculino de 65 años que acude con sangrado de tubo digestivo alto, por lo que se efectúa
esofagogastroscopía que reporta úlcera antral cuya biopsia es positiva para adenocarcinoma gástrico tipo
intestinal.

CQ6-1. El paciente tiene anemia, pérdida de peso, dolor abdominal y rebote positivo. La conducta a seguir
es:

a) Exámenes de extensión de enfermedad


b) Laparotomía exploradora

c) Quimioterapia neoadyuvante

d) Radioterapia

e) Estudios para abdomen agudo

CQ6-2. Con el diagnóstico de patología y en ausencia de medicación quirúrgica inmediata, el seguimiento


debe ser:

a) Laparotomía exploradora

b) Estudios de extensión de enfermedad

c) Quimioterapia neoadyuvante

d) Laparoscopía con toma de biopsias

e) Tratamiento médico

CQ6-3. La enfermedad está localizada al órgano, por lo que la conducta a seguir es:

a) Quimioterapia neoadyuvantes

b) Cirugía laparoscópica

c) Radioterapia más quimioterapia

d) Gastrectomía subtotal D1

e) Gastrectomía total D2

CQ6-4. El reporte de la pieza operatoria dice adenocarcinoma tipo intestinal, que infiltra hasta la serosa con
ganglios negativos. La etapa clínica correspondiente es:

a) EC II (T3, N0, M0)

b) EC III A (T4, N0, M0)

c) EC I (T1, N0, M0 )

d) EC IV (T3, N0, M1)

CQ6-5. Con el diagnóstico, tratamiento y etapificación, el caso debe continuar con:

a) Vigilancia
b) Quimioterapia adyuvante

c) Radioterapia

d) Quimioterapia más radioterapia

CQ7. Masculino de 65 años fumador, alcohólico con pérdida de peso y vómito de toda ingesta.
Endoscópicamente se encuentra estrechez de esófago distal a 36 cm de arcada dentaria.

CQ7-1. El reporte histológico más frecuente en tumores de esta localización es:

a) Adenocarcinoma de unión esofagogástrica

b) Carcinoma epidermoide

c) Tumor carcinoide

d) Sarcoma

CQ7-2. Los estudios de extensión de enfermedad son negativos. La TAC de tórax sin evidencia de
crecimientos ganglionares ni infiltración a otras estructuras. La etapa clínica a la que corresponde es:

a) EC I

b) EC III

c) EC II

d) EC IV

CQ7-3. Según los estudios de extensión de enfermedad y la etapa clínica, el tratamiento indicado es:

a) Quimioterapia+radioterapia

b) Radioterapia+gastrostomía

c) Esofagectomía tóracoabdominal derecha

d) Esofagogastrectomía tóracoabdominal izquierda

e) Quimioterapia+esofagogastrectomía tipo Leáis

CQ7-4. Con el diagnóstico, tratamiento y etapa clínica efectuados en este caso, la sobrevida a 5 años es de:

a) 25%
b) 70%

c) 85%

d) 35%

CQ8. Masculino de 55 años con molestias gastrointestinales bajas y pérdida de 15 Kg de peso, agregándose
ictericia de 15 días de evolución.

CQ8-1. Estudio de ultrasonido hepático revela colédoco y vía biliar intrahepática dilatada. TAC sin lesiones
metastásicas o tumorales ni crecimientos ganglionares, anemia e hipoproteinemia. Resto negativo. La
conducta es:

a) Laparotomía exploradora

b) RMN

c) CPRE

d) Derivación percutánea

CQ8-2. La histología es adenocarcinoma periampular. La conducta es:

a) NPT preoperatoria

b) Laparotomía exploradora

c) Derivación percutánea

d) Pancreatectomía total

CQ8-3. Condiciones estables. Con esta localización e histología, lo que procede es:

a) Pancreatectomía total

b) Operación de Whipple

c) Aplicación de Stent+radioterapia+quimioterapia

d) Derivación percutánea+quimioterapia

CQ8-4. La sobrevida con el tratamiento ideal para el adenocarcinoma periampular a 5 años es de:

a) 3%

b) 15%
c) 57%

d) 30%

CQ9. Masculino de 53 años que acude por sangrado de tubo digestivo bajo, que al realizar
rectosigmoidoscopía se identifica a nivel de sigmoides una tumoración ulcerada de bordes infiltrantes de 4 x
3 cm, friable.

CQ9-1. A qué grupo corresponde según la clasificación de Bormann.

a) I

b) II

c) III

d) IV

e) Ninguna (no se aplica para colon)

CQ9-2. Usted solicitaría colonoscopía complementaria porque:

a) Es un estudio de rutina

b) La posibilidad de encontrar pólipos sincrónicos es hasta del 70%

c) La posibilidad de encontrar un segundo primario sincrónico de colon es del 2 al 8%

d) Sirve para marcar únicamente el sitio de resección marginal

e) No es necesario porque ya tiene el diagnóstico

CQ9-3. Qué tratamiento propondría al paciente:

a) Colectomía total

b) Sigmoidectomía

c) Hemicolectomía izquierda ampliada

d) Resección con márgenes de 1 cm distal y proximal

e) Radioterapia a dosis de 70 Gy
CQ9-4. Durante la laparotomía exploradora se identifica una lesión metastásica de 2 x 2 cm ubicada en el
segmento III del hígado, su conducta sería:

a) Cerrar al paciente sin realizar ningún procedimiento porque el pronóstico ya está dado

b) Resecar únicamente el primario y dejar la metástasis para destruirla con quimioterapia

c) Resecar el primario más lobectomía izquierda

d) Resecar el primario y colocar un catéter en la arteria hepática para dar quimioterapia


intraarterial

e) Resecar el primario y alcoholizar la lesión metastásica

CQ9-5. El reporte de patología de la pieza quirúrgica refiere adenocarcinoma moderadamente diferenciado que
abarca hasta la serosa de 4 x 3 cm, ulcerado, márgenes libres, 20 ganglios negativos. La lesión hepática es una
metástasis de carcinoma de colon. Su conducta con relación a este paciente sería:

a) Observación

b) Radioterapia adyuvante

c) Quimioterapia adyuvante

d) Quimioterapia más radioterapia

e) Braquiterapía

CQ10. Paciente femenino de 61 años de edad con padecimiento de un mes de evolución al presentar
ictericia. El US hepático y de vías biliares reportó vesícula y vía biliar intrahepática dilatada.

CQ10-1. El estudio que debe practicarse para demostrar presencia de tumor es:

a) CPRE

b) TAC abdominal

c) Colangiografía percutánea

d) Colangiografía oral

e) SEGD

CQ10-2. La endoscopía demostró tumor en ámpula de Vater, el procedimiento a seguir será:

a) Colocación de stent
b) Colocación de cateter externo de drenaje biliar

c) Toma de biopsia del tumor

d) Laparotomía exploradora para toma de biopsia

e) Nutrición parenteral preoperatoria

CQ10-3. No se logra establecer diagnóstico histológico y se somete a exploración quirúrgica, como


hallazgos se palpa tumoración de aproximadamente 3 cm a la palpación de la segunda porción de duodeno
en ámpula de Vater, la cabeza de páncreas aumentada de consistencia. El procedimiento de elección para
obtener el diagnóstico histológico deberá ser:

a) Wipple

b) Toma de biopsia transduodenal

c) Duodenectomía y toma de biopsia del ámpula de Vater

d) Colecistectomía

e) Colangiografía a través de cístico

CQ10-4. El diagnóstico histopatológico de la biopsia es de adenocarcinoma. Se realiza


pancreatoduodenectomía y la pieza demuestra tumor de 1.5 x 1 cm en ámpula de Vater, todos los ganglios y
páncreas libres de neoplasia. Según la clasificación TNM correspondería a etapa:

a) I

b) II

c) III

d) IV

CQ10-5. Considerando el tamaño del tumor y el reporte histopatológico la sobrevida a 5 años es de


aproximadamente:

a) 0-10 %

b) 10-20%

c) 30-50%

d) 40-60%

e) 70-80%
CQ11. Femenino de 39 años de edad con antecedentes de madre y hermana con cáncer de mama. Inicio de
VSA a los 28 años G:I P:I hace 8 años, se presenta a la consulta por presentar dolor y presencia de nódulo
mamario derecho de aproximadamente dos meses de evolución. A la exploración masa palpable de
aproximadamente 2 cm de diámetro, con bordes regulares y bien definidos.

CQ11-1. Está indicado:

a) Solicitar mastografía

b) Solicitar US mamario bilateral

c) Solicitar mastografía y BAAF

d) Darle medidas generales

e) Citar nuevamente a revisión en un año

CQ11-2. Los resultados son compatibles con mastografía fibroquística, usted recomienda:

a) Vigilancia anual

b) Mastografía anual

c) Autoexploración mensual, exploración clínica semestral y mastogra fía anual

d) Alta

e) Mastectomía profiláctica

CQ11-3. Seis meses después la paciente se presenta con dolor abdominal y aumento de volumen progresivo,
usted piensa en:

a) Colitis nerviosa

b) Probable tumor ovárico

c) Embarazo ectópico

d) Metástasis abdominal de primario en mama

e) Probable tumor hepático

CQ11-4. Ante la posibilidad diagnóstica usted realiza:

a) Mastográfia bilateral

b) TC abdominopélvica

c) US abdominopélvico y laparotomía exploradora


d) Sólo vigilancia

e) Tratamiento medicamentoso

CQ11-5. En esta paciente y sus familiares de primer grado está indicado

a) Búsqueda molecular de mutaciones en el gen p53

b) Exploración física de mama anual

c) Mastografía semestral a todas las mujeres

d) Búsqueda molecular de mutaciones en BRCA1 y BRCA2

e) Búsqueda de sobreamplificación de HER2/neu

CQ12. Paciente masculino de 5 años con el único antecedente familiar de que sus padres son primos en
segundo grado; acude a consulta por movimientos oculares sacádicos desde los tres años, y desde hace
cuatro meses marcha atáxica. A la exploración física, usted corrobora lo anterior y además nota
telangiectasias en ojos y pabellones auriculares.

CQ12-1. Con estos datos usted sospecha que se trate de:

a) Apraxia motora ocular de Cogan

b) Síndrome de Louis-Bar

c) Tricotiodistrofia

d) Síndrome de Angelman

e) Síndrome de Beckwith-Wiedemann

CQ12-2. Para confirmar el diagnóstico usted solicita el estudio siguiente:

a) Cariotipo en médula ósea

b) Análisis cromosómico con bandas Q

c) FISH

d) Cariotipo en sangre periférica con bandas R

e) Evaluación de rupturas cromosómicas espontáneas e inducidas


CQ12-3. La forma de herencia del padecimiento que usted sospecha es:

a) No tiene forma de herencia, se considera esporádico

b) Autosómico dominante

c) Recesivo ligado al X

d) Autosómico recesivo

e) Mitocondrial

CQ12-4. Si usted contara con estudios moleculares, solicitaría busqueda de mutaciones en el gen:

a) ATM

b) RET

c) XPD

d) XPE

e) P53

CQ12-5. Tanto los pacientes como los heterocigotos tienen prohibido exponerse a:

a) Solventes y químicos

b) Radiación ultravioleta

c) Rayos infrarrojos

d) Radiación ionizante

e) Bromuro de etidio

CQ13. Masculino de 11 años de edad, que tiene como antecedente de importancia abuelo y dos tías por rama
materna finadas por cáncer de colon y madre con diagnóstico de poliposis colónica. Acude a la consulta por
presentar desde hace ocho meses aumento de volumen de la mandíbula, acompañado de dolor moderado y desde
hace pocos días dificultad para comer.

CQ13-1. Con estos antecedentes, usted piensa que lo más probable es que se trate de:

a) Osteomielitis de la mandíbula

b) Osteoma mandibular
c) Displasia ósea

d) Osteopetrosis

e) Osteocondromatosis

CQ13-2. A la exploración física usted busca intencionadamente:

a) Dientes supernumerarios y lipomas

b) Alopecia y talla baja

c) Adoncia parcial y polidactilia

d) Asimetría de extremidades superiores

e) Criptorquidia bilateral

CQ13-3. Con estos datos usted integra:

a) Síndrome de Trucot

b) Adenomatosis colónica

c) Síndrome de Sturge-Wever

d) Síndrome de Gardner

e) Displasia cráneo metafisiaria

CQ13-4. Se le realiza al paciente una colonoscopía reportando la presencia de más de 100 pólipos, lo
siguiente que se debe realizar es:

a) Endoscopía para buscar lesiones extracolónicas, enema de bario y serie ósea completa

b) Sólo vigilar al paciente con colonoscopía cada seis meses

c) Realizar cirugía profiláctica

d) Tomografía abdominal

e) Serie esófagogastroduodenal

CQ13-5. En esta entidad el gen causante es:

a) p53

b) RAS
c) RET

d) NF1

e) APC

CQ14. Femenino de 35 años de edad, con antecedente de tener un primo hermano de 37 años con carcinoma
renal con metástasis diseminadas y un hijo con leucemia diagnosticada a los 5 años. Como antecedentes
personales presentó un sarcoma de cúbito izquierdo a los 13 años y ahora acude por nódulo mamario
izquierdo de aproximadamente dos meses de evolución que apareció posterior a un traumatismo directo a la
mama. A la exploración se encuentra tumoración indurada de aproximadamente 3 cm de diámetro en CSE de
mama izquierda y un ganglio ipsilateral de 1 cm.

CQ14-1. Con esta lesión usted sospecha:

a) Mastopatía fibroquística

b) Probable cáncer de mama

c) Carcinoma metastásico

d) Mastitis granulomatosa

e) Lesión benigna

CQ14-2. Se solicitan los estudios de extensión siguientes:

a) Mastografía bilateral, BAAF y biopsia tru-cut

b) Serie ósea metastásica

c) Sólo biopsia de la lesión

d) US mamario bilateral

e) Se mantiene en vigilancia

CQ14-3. Con los antecedentes de la paciente se está ante:

a) Síndrome de cáncer de mama-ovario

b) Síndrome de Werner

c) Síndrome de Cowden

d) Síndrome de Li-Fraumeni

e) Síndrome de Bloom
CQ14-4. Si tuviera acceso a diagnóstico molecular, buscaría mutaciones en el gen:

a) BRCA1

b) W

c) C

d) P53

e) BLM

CQ14-5. El riesgo para la descendencia de los afectados de que el gen se transmita es del:

a) 100%

b) 30%

c) 25%

d) 50%

e) 70%

CQ15. Masculino de 7 años de edad que es referido a la consulta por presentar pancitopenia en estudio,
como antecedente de importancia tiene que desde el nacimiento presenta hipoplasia de radio y pulgar
derecho y duplicación de ureteros del lado derecho.

CQ15-1. A la exploración física se encuentra que tiene talla baja e hiperpigmentación generalizada de la piel.
El diagnóstico más probable de este paciente es:

a) Síndrome mielodisplásico

b) Síndrome de Bloom

c) Síndrome de Aase

d) Anemia de células falciformes

e) Anemia de Fanconi

CQ15-2. Para corroborar el diagnóstico que sospecha, solicita:

a) Biometría hemática completa y química sanguínea

b) Aspirado de médula ósea


c) Biopsia de hueso

d) Biometría hemática completa, cariotipo en sangre periférica y bús queda de aberraciones


cromosómicas espontáneas e inducidas

e) Evaluación de rupturas cromosómicas inducidas por radiación ionizante

CQ15-3. La enfermedad que usted sospecha se hereda en forma:

a) Autosómica dominante

b) No tiene forma de herencia

c) Mitocondrial

d) Autosómica recesiva

e) Dominante ligada al X

CQ15-4. En estos pacientes existe alto riesgo de desarrollar:

a) Leucemia aguda no linfoblástica

b) Anemia crónica

c) Linfoma no Hodgkin

d) Leucemia granulocitica crónica

e) Neoplasias cerebrales

CQ15-5. Los padres del paciente tienen un porciento de riesgo de tener más hijos con el mismo
padecimiento del:

a) 30%

b) 50%

c) 25%

d) 100%

e) 12%

CQ16. Mujer de 79 años, obesa e hipertensa, con prurito vulvar de larga evolución y tres meses con dolor local
acompañado de sangrado leve al rascado. La exploración física revela tumor de 3 cm de diámetro en labio mayor
derecho a 1 cm de la línea media, ulcerado, ingles sin adenopatías.
CQ16-1. Para llegar al diagnóstico usted realiza:

a) Impronta

b) Biopsia incisional

c) Citología pélvico-vaginal

d) Hemivulvectomía

e) Biopsia exisional

CQ16-2. El estudio histopatológico es positivo a malignidad, cuál es la histología más frecuente en esta
patología:

a) Adenocarcinoma

b) Epidermoide

c) Melanoma

d) Sarcoma

e) Verrucoso

CQ16-3. De acuerdo a la FIGO, en qué etapa clasificaría a esta paciente:

a) I

b) II

c) III

d) Iva

e) Ivb

CQ16-4. El tratamiento de elección en esta etapa es:

a) Biopsia escisional

b) Vulvectomía radical

c) Hemivulvectomía

d) Teleterapia
e) Braquiterapia

CQ16-5. De acuerdo a la etapa clínica, cuál es la sobrevida a 5 años:

a) 91%

b) 48%

c) 81%

d) 15%

e) 0-5%

CQ17. Mujer de 60 años, obesa y diabética, con sangrado transvaginal intermitente de cuatro meses de
evolución, sin otra sintomatología acompañante.

CQ17-1. La primera posibilidad diagnóstica a considerar es:

a) Sarcoma uterino

b) Cáncer cervicouterino

c) Adenocarcinoma de endometrio

d) Hiperplasia endometrial atípica

e) Leiomioma celular

CQ17-2. La estirpe histológica más frecuente es:

a) Adenoescamoso

b) Adenocarcinoma endometroide

c) Adenocarcinoma papilar

d) Carcinoma de células claras

e) Adenoacantoma

CQ17-3. A la exploración física se encuentra vagina sin alteraciones, cérvix parcialmente cupulizado,
orificio cervical abierto. Para llegar al diagnóstico usted realizaría:

a) PAP y legrado endometrial


b) PAP y legrado endocervical

c) PAP, legrado endocervical y endometrial

d) PAP y biopsia de cérvix

e) USG transvaginal

CQ17-4. El reporte histopatológico es un adenocarcinoma endometroide moderadamente diferenciado, la


conducta adecuada a seguir es:

a) Histerectomía vaginal

b) Histerectomía tipo III

c) Lavados peritoneales, histerectomía tipo I, muestreo ganglionar

d) Histerectomía tipo II

e) Histerectomía, salpingooforectomía bilateral, omentectomía, apendicectomía

CQ17-5. El reporte histopatológico de la pieza quirúrgica muestra una EC I, cuál es la sobrevida a 5 años:

a) 98%

b) 86%

c) 78%

d) 60%

e) 50%

CQ18. Mujer de 51 años, acude refiriendo seis meses de evolución con molestia abdominal vaga,
intolerancia a la vía oral, pérdida de peso 8 kg y aumento de perímetro abdominal. Se interna y los estudios
muestran: nódulos pulmonares bilaterales, derrame pleural derecho 30%, ascitis e imagen de masa pélvica y
Ca 125 de 15.

CQ18-1. La paciente es candidata a:

a) Laparotomía exploradora

b) Quimioterapia neoadyuvante por ser EC IV

c) Paracentesis diagnóstica y laparoscopía

d) Colocación de sonda de plesurostomía


e) Biopsia dirigida de nódulo pulmonar

CQ18-2. Los hallazgos son: ascitis hemorrágica 4 L, implantes peritoneales de hasta 3 cm, implantes en
serosa de útero y actividad tumoral pélvica, sin identificar ovarios, la conducta a seguir es:

a) Biopsia de implantes peritoneales

b) Histerectomía tipo I, resección de implantes, omentectomía

c) Biopsia del tumor y quimioterapia neoadyuvante

d) Tumorectomía, histerectomía, salpingooforectomía bilateral, resección intestinal por los


implantes

e) Histerectomía, apendicetomía

CQ18-3. Con los hallazgos quirúrgicos, la etapa clínica según la FIGO es:

a) IIIa

b) IIIb

c) IIIc

d) IV

e) Iva

CQ18-4. El tratamiento posterior a la cirugía en esta etapa es:

a) Radioterapia abdomen total

b) Quimioterapia en base a paclitaxel y CDDP

c) Quimioterapia intraperitoneal

d) Metastasectomía pulmonar y quimioterapia

e) Quimioterapia con base en carboplatino y ciclofosfamida

CQ18-5. La sobrevida a 5 años en esta etapa según la FIGO de 1994 es:

a) 5%

b) 14%

c) 30-40%
d) 60%

e) 50%

CQ19. Mujer de 50 años con antecedente de haber recibido radioterapia pélvica por cáncer cervicouterino
hace 10 años. Acude por masa pélvica de crecimiento rápido.

CQ19-1. La primera posibilidad diagnóstica a considerar de acuerdo a sus antecedentes es:

a) Cáncer cervicouterino recurrente

b) Cáncer de ovario epitelial

c) Sarcoma uterino

d) Cáncer de endometrio

e) Cáncer de recto

CQ19-2. En esta patología 50-10% de las pacientes tienen historia de irradiación pélvica por malignidad y se
caracterizan por:

a) Tener tumores en fases avanzadas y agresivos

b) Manifestarse en etapas tempranas

c) Mayor riesgo en mujer blanca, comparado con la negra

d) Incrementa el riesgo con menopausia temprana

e) Ser frecuentes en mujer joven

CQ19-3. La exploración física muestra una mujer con ECOG de 0, tumor pélvico hasta cicatriz umbilical,
móvil, sin ser posible toma de biopsia, la conducta a seguir es:

a) Exenteración anterior

b) Exenteración posterior

c) Histerectomía tipo I, lavados peritoneales, muestreo ganglionar

d) Histerectomía tipo III

e) Toma de biopsia
CQ19-4. Durante la laparotomía hay diseminación peritoneal, cuál es el tipo histológico más frecuente con
este comportamiento:

a) Leiomiosarcoma

b) Sarcoma del estroma endometrial

c) Tumor mixto mulleriano

d) Carcinosarcoma

e) Adenosarcoma

CQ19-5. Cuál es el objetivo de los procedimientos de etapificación en este tipo de neoplasias cuyo
tratamiento de elección es la cirugía:

a) Mejorar sobrevida

b) Conocer extensión de la enfermedad y seleccionar el tratamiento posterior

c) Evitar recurrencia local

d) Citorreducción

e) Prolongar intervalo libre de enfermedad

CQ20. Mujer de 35 años que se presenta a la consulta con historia de dispareunia y sangrado poscoito. La
exploración física muestra cérvix de aproximadamente 3 cm, con lesión periorificial de aproximadamente 1
cm, exofítica, fácilmente sangrante.

CQ20-1. La ruta diagnóstica a seguir es:

a) PAP y biopsia

b) Colposcopía

c) Histerectomía

d) Conización cervical

e) Cepillado endocervical

CQ20-2. El estudio histopatológico es positivo a malignidad, cuál es la histología más frecuente:

a) Epidermoide

b) Adenocarcinoma
c) Adenoescamoso

d) Carcinoma de células claras

e) Carcinoma de células pequeñas

CQ20-3. A la palpación el cérvix se encuentra indurado en aproximadamente 3 cm, palpándose la lesión


periorificial de 1 cm, fondos de saco respetados, parametrios sin alteración. De acuerdo a la FIGO, la etapa
clínica es:

a) Iia

b) Ib1

c) Ib2

d) Ia1

e) Ia2

CQ20-4. De acuerdo a la etapa, el tratamiento de elección en esta paciente es:

a) Radioterapia radical

b) Histerectomía tipo III con salpingooforectomía unilateral

c) Histerectomía tipo I y radioterapia

d) Quimioterapia y radioterapia

e) Cono cervical

CQ20-5. Sin factores pronósticos adversos, cuál es la sobrevida a 5 años en esta paciente:

a) 80-90%

b) 50%

c) 75%

d) 20%

e) 40%

CQ21. Una mujer de 29 años fue operada fuera del Servicio de Oncología por un tumor en ovario derecho. La
neoplasia medía 10 cm, no tenía adherencias, no-enfermedad fuera del ovario, lavado peritoneales negativos. El
tumor no invadía la superficie de la seudocápsula y fue reportado como cistoadenocarcinoma papilar serosos
bien diferenciado. El CA 125 antes de la operación fue de 100. Sólo se resecó el tumor. La mujer es nuligesta.
CQ21-1. Se estudia en la consulta externa: CA 125 de 4, TAC de abdomen negativa para actividad tumoral.
Exploración físicas negativa para actividad tunoral. Estudios de extensión negativa. La conducta es:

a) Laparotomía exploradora

b) QT posoperatoria

c) Vigilancia

d) Laparoscopía

e) QT posoperatoria

CQ21-2. A los dos meses de operada, presenta un tumor quístico en el ovario restante de 3 cm. El
ultrasonido revela que es totalmente quístico. El CA 125 es de 3. Anote la conducta a seguir:

a) Anticonceptivos, por dos o tres meses

b) Laparotomía exploradora

c) Laparoscopía y punción

d) Punción por TAC

e) QT

CQ21-3. En los dos años siguientes tuvo dos hijos sanos. En el último seguimiento no había embarazo. El
CA 125 en 5 y no había actividad tumoral clínica ni radiológicamente. La conducta más correcta es:

a) Completar cirugía

b) Vigilancia

c) Laparoscopía

d) Repetir estudios

e) QT preventiva

CQ21-4. Cuatro años después de la cirugía, el antígeno CA 125 se eleva a 90 y por exploración y ultrasonido
se detecta tumor pélvico de 8 cm. La conducta es:

a) Laparotomía exploradora

b) QT

c) RT
d) Laparoscópica

e) Punción por TAC

CQ21-5. Durante la laparotomía se encuentra tumor pélvico que se resecó y resultó con CA papilar seroso
bien diferenciado. No había actividad tumoral en ninguna otra parte. A los 15 días de operada el CA 125
bajó a 5. La conducta correcta es:

a) Vigilancia

b) RT

c) QT

d) RT+QT

e) Hormonoterapia

CQ22. Paciente del sexo femenino de 60 años de edad con antecedente de hipertensión arterial y diabetes
mellitus, menarca a los 9 años, G-1 P-1, fecha de última menstruación a los 50 años. Última citología
cervicovaginal hace dos años reportada como negativo II. Su padecimiento actual se inicia hace una semana
con hemorragia transvaginal en cantidad moderada. A la exploración física de la paciente se encuentra
obesidad grado 2, A la exploración ginecopélvica se encuentran labios mayores y menores normales, vagina
estrecha sin tumor, cuello uterino con sangrado a través del orificio cervical, sin neoplasia macroscópica,
cuerpo uterino miomatoso y porametrios libres de tumor.

CQ22-1. La mayor probabilidad diagnóstica sería:

a) Enfermedad del trofoblasto

b) Hiperestrogenismo

c) Endometritis

d) Carcinoma endometrial del cuerpo uterino

e) Tuberculosis genital

CQ22-2. El diagnóstico diferencial debe hacerse con todos los siguientes, excepto:

a) Enfermedad gestacional del trofoblasto

b) Hiperestrogenismo

c) Carcinoma cervicouterino

d) Carcinoma endometrial
e) Sarcoma uterino

CQ22-3. El procedimiento diagnóstico debe incluir para el diagnóstico sospechado:

a) Biometría hemática, química sanguínea, radiografía de tórax y biopsia de cuello uterino

b) Biometría hemática, química sanguínea, colposcopía

c) Biometría hemática, química sanguínea, telerradiografía de tórax y cultivos de exudado


cervicovaginal

d) Biometría hemática, química sanguínea, examen general de orina, legrado y biopsia


endometrial

e) Colposcopía, CA 125, tomografía computarizada abdominopélvica

CQ22-4. Los siguientes estudios están indicados para una paciente de quien se sospeche cáncer ginecológico
que afecta el cuerpo uterino, excepto:

a) Colposcopía

b) Citología cervicovaginal

c) Ultrasonograma pélvico

d) Legrado y biopsia endometrial

e) Gammagrama óseo

CQ22-5. Los factores de riesgo que esta paciente presenta: obesidad, diabetes e hipertensión, hemorragia vaginal
posmenopáusica, menarca a los 9 años y última menstruación a los 50 años, G-1, P-1. Son más frecuentes
encontrarlos en:

a) Endometritis

b) Carcinoma de endometrio

c) Carcinoma ovárico

d) Carcinoma de vulva

e) Enfermedad del trofoblasto

CQ23. Paciente del sexo femenino de 60 años de edad con antecedente de hipertensión arterial y diabetes
mellitus, menarca a los 9 años, G:1 P:1, Fecha de última menstruación a los 50 años. Última citología
cervicovaginal hace dos años reportada como negativo II. Su padecimiento actual se inicia hace una semana
con hemorragia transvaginal en cantidad moderada. A la exploración física de la paciente se encuentra
obesidad grado 2, A la exploración ginecopélvica se encuentran labios mayores y menores normales, vagina
estrecha sin tumor, cuello uterino con sangrado a través del orificio cervical, sin neoplasia macroscópica,
Cuerpo uterino miomatoso y porametrios libres de tumor.

CQ23-1. Todos los siguientes son factores de riesgo para carcinoma endometrial, excepto:

a) Obesidad

b) Diabetes mellitus

c) Hemorragia transvaginal posmenopáusica

d) Hipertensión arterial

e) Citología cervicovaginal negativo II

CQ23-2. Se corroboró el diagnóstico cáncer de endometrio, se encontró invasión al tercio interno del
miometrio, grado 2 en su diferenciación histológica. La clasificación FIGO por etapa sería:

a) Etapa I A

b) Etapa I B

c) Etapa II A

d) Etapa II B

e) Etapa III A

CQ23-3. Las histologías menos frecuentes en los procesos malignos del cuerpo uterino son todas las
siguientes, excepto:

a) Tumor mixto mülleriano

b) Rabdomiosarcoma

c) Sarcoma del estroma endometrial

d) Tumores mesodérmicos mixtos

e) Adenocarcinoma

CQ23-4. El tratamiento en esta paciente debe ser:

a) Hormonoterapia

b) Radioterapia

c) Histerectomía total abdominal y salpingooforectomía bilateral, citología peritoneal, más


radioterapia pélvica externa e intracavitaria
d) Quimioterapia más radioterapia

e) Sólo quimioterapia

CQ23-5. La histerectomía, en este caso, debe ser:

a) Extrafascial

b) Subfascial

c) Rutledge 5

d) Rutledge 4

e) Ninguna de las anteriores

CQ24. Femenino de 18 años, menstruante núbil, evolución de tres meses con aumento de volumen del
hemiabdomen inferior, más dolor pungitivo local. Pérdida de 5 kg de peso. Masa palpable en hemiabdomen
inferior derecho de aproximadamente 12 x 10 cm, parcialmente fija. No ascitis. Tacto rectal: ocupación
parcial del parametrio derecho por lesión descrita en abdomen. No infiltración del fondo de Saco de
Douglas.

CQ24-1. Con base en loanterior, el diagnóstico clínico del caso anterior es:

a) Quiste ovárico derecho

b) Tumor germinal benigno

c) Tumor germinal ovárico maligno

d) Neoplasia epitelial maligna

e) Tumor de células de la granulosa

CQ24-2. Los estudios complementarios mostraron: ultrasonido con lesión anexial derecha de componente
predominantemente sólido. Deshidrogenasa elevada. Otros marcadores, negativos. Su diagnóstico clínico:

a) Cáncer seroso papilar

b) Teratoma maduro

c) Tumor de la teca granulosa

d) Disgerminoma

e) Cistadenofibroma
CQ24-3. La laparotomía demostró masa anexial derecha sólida de 14 X 12 cm, con transoperatorio de tumor
germinal. Dos ganglios retroperitoneales de 3 cm con disgerminoma. Anote la clasificación.

a) Ic

b) Iib

c) IIIa

d) IV

e) IIIc

CQ24-4. Anote el tratamiento quirúrgico para esta paciente:

a) Salpingooforectomía derecha y biopsia de ganglios retroperitoneales

b) Panhisterectomía, omentectomía y disección retroperitoneal

c) Panhisterectomía, omentectomía y biopsia de ganglios retroperitoneales

d) Ooforectomía derecha

e) Salpingooforectomía bilateral y biopsia de ganglios retroperitoneales

CQ24-5. El reporte de patología informa: disgerminoma y teratocarcinoma, ganglios con disgerminoma.


Anote conducta de manejo subsecuente.

a) Observación

b) Radioterapia a abdomen y pelvis

c) Quimioterapia con bleomicina, etopósido y platino

d) Quimioterapia con platino y ciclofosfamida

e) Quimioterapia con adriamicina y ciclofosfamida

CQ25. Femenino de 17 años de edad con antecedentes de oligomenorrea; núbil, quien acude a consulta con
un estudio de ultrasonido que muestra masa sólida de 10 cm, en anexo derecho, la que es corroborada al
examen ginecopélvico, sin otros datos positivos que consignar:

CQ25-1. Una búsqueda intencionada en casos como el presente, puede demostrar:

a) Un síndrome de cáncer mamario-ovárico

b) Una disgenesia gonadal


c) Un síndrome de Lynch II

d) Sobreexpresión del gen supresor Rb

e) Trisomía del cromosoma 12

CQ25-2. El diagnóstico clínico del presente caso es compatible con:

a) Tumor epitelial sólido benigno

b) Tumor epitelial sólido maligno

c) Gonadoblastoma

d) Tumor germinal maligno

e) Tumor germinal metastásico

CQ25-3. La paciente es llevada a cirugía, la que demuestra un tumor sólido de ovario derecho de 8 x 10 cm,
con reporte de patología de disgerminoma más teratocarcinoma grado 1. Su conducta quirúrgica es:

a) Rutina quirúrgica de cáncer de ovario

b) Cirugía conservadora

c) Cirugía de citorreducción primaria

d) Laparotomía y biopsia

e) Histerectomía y ooforectomía unilateral

CQ25-4. El reporte definitivo de patología confirma disgerminoma más teratocarcinoma grado 1. Anote su
conducta de manejo.

a) Observación

b) Second look a los 6 meses

c) Esquema de quimioterapia BEP cuatro ciclos

d) Esquema de quimioterapia AC cuatro ciclos

e) Radioterapia profiláctica a retroperitoneo

CQ25-5. Si el reporte definitivo de patología en el presente caso informara: disgerminoma más focos de
carcinoma embrionario. La neoplasia está limitada al ovario. Su conducta subsecuente es:
a) Observación

b) Second look a los seis meses

c) Esquema de quimioterapia BEP cuatro ciclos

d) Esquema de quimioterapia AC 8 ciclos

e) Radioterapia más quimioterapia

CQ26. Femenino de 31 años de edad con antecedentes de tía materna fallecida de cáncer mamario. Cinco años
de VSA, menstruante y G:0 P:0. Al estudiarse por esterilidad un ultrasonido mostró tumor anexial izquierdo de
componente mixto de 9 x 10 cm. La exploración ginecopélvica corroboró el aumento de volumen sin
evidencia de diseminación locorregional.

CQ26-1. Uno de los siguientes estudios de biología molecular podría resultar positivo y estar relacionado
con la historia natural del presente caso.

a) Sobreexpresión de la proteína P-53

b) Sobreexpresión del gen supresor Rb

c) Expresión del cromocoma Phyladelphia

d) Sobreexpresión del Oncogen MYC

e) Sobreexpresión del Gen BRCa1

CQ26-2. Un hallazgo quirúrgico en esta paciente de tumor de 10 cm limitado a un ovario, con


transoperatorio de cáncer seroso, justifica la siguiente conducta en esta paciente.

a) Salpingooforectomía unilateral

b) Salpingooforectomía bilateral

c) Histerectomía y salpingooforectomía del anexo enfermo

d) Laparotomía y biopsia

CQ26-3. El reporte definitivo de patología informa: cáncer seroso papilar grado 1. Anote su conducta de
manejo para el posoperatorio.

a) Observación

b) Panhisterectomía y omentectomía

c) Seis ciclos de quimioterapia esquema BEP

d) Cuatro ciclos de quimioterapia esquema PC


e) Second look a los 6 meses

CQ26-4. El reporte definitivo de patología informa: cáncer seroso papilar grado III. Anote la conducta de
manejo para el posoperatorio.

a) Observación

b) Panhisterectomía más omentectomía

c) Seis ciclos de quimioterapia esquema BEP

d) Seis ciclos de quimioterapia esquema PC

e) Second look a los 6 meses

CQ26-5. La sobrevida a 5 años sin actividad tumoral en el cáncer epitelial de ovario Etapa grado III, es del:

a) 90-100%

b) 70-80%

c) 60-70%

d) 50-60%

e) 40-50%

CQ27. Femenino de 15 años de edad quien ingresa al Servicio con antecedentes de dolor abdominopélvico de
dos meses de evolución, pérdida de 4 kg de peso y laparotomía dos semanas antes por cuadro de abdomen agudo
por ruptura de tumor ovárico izquierdo. El reporte de patología posoperatorio informado por patólogo certificado
informa: tumor de senos endodérmicos.

CQ27-1. La paciente ingresa con cifras de 850 ng de alfafetoproteína y sin hallazgos a la exploración
locorregional que sugieran actividad tumoral. Anote su conducta de manejo:

a) Cirugía de citorreducción primaria

b) Quimioterapia BEP cuatro ciclos

c) Quimioterapia PC cuatro ciclos

d) Quimioterapia AC seis ciclos

e) Laparotomía y cirugía conservadora

CQ27-2. Una laparotomía exploradora por actividad tumoral posquimioterapia, mostró tumor en cuerpo
uterino, en anexo contralateral e implantes de 1 cm en peritoneo parietal. Anote su conducta quirúrgica.
a) Únicamente biopsia

b) Salpingooforectomia y resección de implantes

c) Panhisterectomía, omentectomía y resección de implantes

d) Resección de implantes

e) Histerectomía abdominal simple

CQ27-3. Los hallazgos quirúrgicos descritos en la pregunta 2, sugieren la siguiente etapificación de la FIGO
para el presente caso

a) Ic

b) IIa

c) IIb

d) IIIa

e) IIIb

CQ27-4. Anote la segunda lineal de quimioterapia a emplearse en el presente caso:

a) Platino-taxol 6 ciclos

b) BEP 6 ciclos

c) Gemcitavicine-ciclofosfamida 6 ciclos

d) Platino ciclofosfamida 6 ciclos

e) AC 6 ciclos

CQ27-5. Anote las cifras de seguimiento a 5 años sin actividad tumoral para casos como el de la pregunta 2.

a) 10-20%

b) 30-40%

c) 50-60%

d) 70-80%

e) 90-100%
CQ28. Femenino de 48 años de edad G:II, P:I, menstruante, que ingresa al Servicio con el antecedente de
resección de masa anexial izquierda de 13 x 15 cm tres semanas antes, existe un reporte de patología de médico
certificado, de cáncer endometroide de ovario con invasión capsular. No más datos en relación al acto quirúrgico.
La paciente cuenta con un Karnofsky del 100% y la exploración locorregional evidencia una cicatriz quirúrgica
reciente media infraumbilical. Líquido libre no a tensión y plastrón no bien delimitado en flanco derecho.
Ginecológicamente, como dato positivo, infiltración del Douglas. Estudios de laboratorio de rutina normales y de
extensión negativos para metástasis.

CQ28-1. La presencia, en esta paciente, de implantes tumorales de 3 cm en epiplón, más líquido libre de células
malignas, clasificaría a este caso en etapa:

a) IIc

b) IV

c) IIIc

d) IIIb

e) IIb

CQ28-2. Una vez completo el expediente clínico de esta paciente, su conducta de manejo es:

a) Radioterapia abdominopélvica radical

b) Seis ciclos de quimioterapia

c) Cirugía de citoreducción secundaria

d) Cirugía etapificadora citoreductora

e) Radioterapia-quimioterapia simultáneas

CQ28-3. ¿Qué porciento de casos de cáncer de ovario locorregionalmente avanzado, y con Karnofsky del
90% o más, pueden ser resueltos con cirugías de citorreducción óptima?

a) 10-20%

b) 20-30%

c) 40-50%

d) 60-70%

e) 80-90%

CQ28-4. Anote el esquema de quimioterapia considerado actualmente de elección para pacientes como la
del presente caso:

a) Platino-ciclofosfamida
b) Platino-taxol

c) Adriamicina-ciclofosfamida

d) Gemcitavine-Adriamicina

e) Bleomicina-platino etopósido

CQ28-5. Qué porciento de pacientes con cáncer de ovario E IIIc sometidas a cirugías de citoreducción
óptima más tratamiento complementario satisfactorio, evolucionan 5 años sin actividad tumoral.

a) 80-90%

b) 60-80%

c) 40-50%

d) 20-30%

e) 0-10%

CQ29. Femenino de 42 años de edad, Gesta II, para 0 abortos I, Cesárea I por embarazo gemelar a los 32 años.
Recibió durante dos años tratamiento hormonal no especificado previo a su cesárea. Actualmente sin control
prenatal. Ingresa con aumento de volumen abdominal de 3 meses. La exploración muestra una masa de
aproximadamente 12 x 14 cm en hipogastrio y fosa ilíaca derecha, hallazgo que es corroborado a la
exploración ginecológica. No infiltración del Douglas. Ultrasonido con lesión anexial derecha de 13 x 14 cm
de ecogenicidad mixta.

CQ29-1. Anote el factor de riesgo que en esta paciente pudo estar directamente involucrado con el desarrollo
de su padecimiento.

a) La edad

b) La baja paridad

c) Embarazo tardío

d) Embarazo gemelar

e) Uso de hormonas

CQ29-2. La exploración quirúrgica mostró como datos positivos, tumor anexial derecho de 15 X 15 cm con
vegetaciones en su superficie y adherencias laxas al epiplón. Reporte transoperatorio de cáncer epitelial.
Anote la clasificación según la FIGO:

a) Estadio Ib

b) Estadio Ic
c) Estadio Ia

d) Estadio IIa

e) Estadio IIIb

CQ29-3. De acuerdo a su experiencia, cuáles son los tumores epiteliales malignos que se diagnostican con
mayor frecuencia.

a) Los seroso y los mucinosos

b) Los serosos y los endometroides

c) Los serosos y los de células claras

d) Los mucinosos y los endometroides

e) Los sólidos y los indiferenciados

CQ29-4. El reporte definitivo de patología informa: cáncer resoro papilar grado II con invasión capsular.
Hay otros datos positivos que consignar. Anote su conducta de manejo.

a) Observación

b) Second look a los 6 meses

c) Radioterapia a la pelvis

d) Quimioterapia con etopósido y platino

e) Quimioterapia con platino y ciclofosfamida

CQ29-5. La sobrevida a 5 años sin actividad tumoral en casos como el presente es de:

a) 100%

b) 80%

c) 60%

d) 40%

e) 20%

CQ30. Femenino de 54 años de edad, multípara sin antecedentes de importancia, quien acude a consultar por
crecimiento abdominal de cinco meses de evolución como manifestación clínica relevante. La laparotomía
exploradora demostró tumor ovárico bilateral, el mayor de ellos de 13 X 15 cm, implantes en el peritoneo
pélvico de 0.5 cm y 2 implantes de 3 X 2 cm en corredera parietocólica izquierda. Reporte transoperatorio
de cáncer epitelial de ovario.

CQ30-1. Anote la estadificación clínico-quirúrgica que corresponde a esta paciente.

a) IIc

b) IIIb

c) IIIa

d) IIIc

e) IV

CQ30-2. En esta paciente una cirugía óptima incluiría:

a) Panhisterectomía, omentectomía y resección de implantes

b) Panhisterectomía, omentectomía y apendicectomía

c) Panhisterectomía, omentectomía y estudio citológico de lavado peritoneal

d) Salpingooforectomía, omentectomía y resección de implantes

e) Panhisterectomía y omentectomía

CQ30-3. Anote la primera línea de quimioterapia para el cáncer de ovario, aceptada actualmente a nivel
internacional.

a) Ciclofosfamida-Ifosfamida

b) Platino etopósido

c) Platino-taxol

d) Ciclofosfamida-Adriamicina

e) Platino-ciclofosfamida

CQ30-4. Anote el diagnóstico en esta paciente de recurrencia tumoral locorregional 18 meses después de
completada su primera línea de quimioterapia. Usted consideraría:

a) Suspender tratamiento oncológico

b) 2ª línea de quimioterapia
c) Radioterapia abdominopélvica

d) Cirugía de citorreducción secundaria

e) Second look

CQ30-5. Anote las cifras de seguimiento a 5 años sin actividad tumoral en pacientes como la del presente
caso:

a) 80%

b) 60%

c) 40%

d) 20%

e) 0%

CQ31. Femenino de 42 años de edad, G:II, P:I, A:I, con antecedentes de tratamiento hormonal por infertilidad.
Operada en Centro de 2º nivel por masa ovárica derecha de 12 X 12 cm con reporte definitivo de cáncer seroso
borderline con invasión capsular. Sin contar con mayor información, la paciente acude para opinión respecto a
manejo subsecuente.

CQ31-1. El tratamiento hormonal por infertilidad, constituye un factor de riesgo para el desarrollo de estas
neoplasias:

a) Verdadero b) Falso

CQ31-2. Anote la frecuencia que ocupan las neoplasias epiteliales Borderline en relación al resto de las
neoplasias epiteliales.

a) 1%

b) 10%

c) 25%

d) 40%

e) 50%

CQ31-3. Una vez corroborado el diagnóstico histopatológico, su indicación de manejo para esta paciente es:

a) Observación
b) Second look en 6 meses

c) Radioterapia a la pelvis

d) Primera línea de quimioterapia

e) Laparotomía estadificadora y terapéutica

CQ31-4. La terapéutica complementaria para esta paciente es:

a) Observación

b) Radioterapia abdominopélvica

c) Platino ciclofosfamida

d) Second look a los 6 meses

e) Adriamicina–ciclofosfamida

CQ31-5. La sobrevida a 10 años sin evidencia de enfermedad para casos como el presente es del:

a) 100%

b) 90%

c) 60%

d) 50%

e) 30%

CQ32. Femenino de 52 años de edad sin antecedentes de importancia, quien acude al Servicio para opinión
terapéutica con el antecedente de panhisterectomía y omentectomía un mes antes, por tumor de células de la
glanulosa moderadamente diferenciado, con un implante de 2 x 2 cm en la serosa del cuerpo uterino.
Marcadores tumorales y estudios de extensión negativos. Según informe del cirujano, no encontró evidencia
de actividad tumoral al término de la cirugía.

CQ32-1. Anote la frecuencia relativa de los tumores de las células de la granulosa, dentro de los cánceres
ováricos

a) 50%

b) 30%

c) 10%
d) 5%

e) 60%

CQ32-2. Los tumores de las células de la granulosa se subdividen en juveniles y en adultos.

a) Verdadero b) Falso

CQ32-3. La etapa clínica del presente caso corresponde a:

a) Estadio Ib

b) Estadio Ic

c) Estadio IIa

d) Estadio IIb

e) Estadio IIIa

CQ32-4. La paciente fue operada por cirujano oncólogo. Usted recomienda la conducta de manejo siguiente:

a) Observación

b) Radioterapia abdominopélvica

c) Hormonoterapia. Progestágenos de depósito

d) Quimioterapia esquema: Bleomicina, etopósido-platino

e) Quimioterapia esquema: Platino ciclofosfamida

CQ32-5. Anote la sobrevida de 5 a 10 años sin evidencia de enfermedad, reportada para estos casos en
general.

a) 100%

b) 70-90%

c) 40-60%

d) 30-50%

e) 10-30%
CQ33. Femenino de 64 años de edad, diabética tipo II, obesa, con padecimiento de cinco meses
caracterizado por aumento progresivo del abdomen, náuseas y pérdida de 5 kg, Karnofsky del 70%.
Abdomen globoso por líquido libre en cavidad y masa abdominopélvica mal delimitada, cuya presencia se
corrobora a la exploración ginecológica, la que además muestra infiltración del Douglas. Laboratorio:
glucosa de 250 mg, Ca-125:125 UI y valoración cardiovascular, isquemia subepicárdica con riesgo III-IV.
Ultrasonido con masas de ecogenicidad mixta. Rx del tórax sin metástasis.

CQ33-1. El diagnóstico clínico en este caso sugiere un:

a) Sarcoma uterino etapa III

b) Ca endometrial etapa IV

c) Ca células de la granulosa etapa III

d) Ca epitelial de ovario etapa III

e) Tumor de Krukerberg

CQ33-2. Anote la conducta terapéutica que Ud, elegiría para esta paciente

a) Laparotomía diagnóstica y etapificadora

b) Quimioterapia de inducción

c) Laparoscopia diagnóstica-terapéutica

d) Seis ciclos con 1ª línea de quimioterapia

e) Radioterapia abdominopélvica

CQ33-3. Con qué frecuencia puede realizarse cirugía óptima en pacientes con cáncer de ovario avanzado.

a) 10%

b) 30%

c) 40%

d) 60%

e) 80

CQ33-4. De acuerdo al diagnóstico elegido, este tipo de cáncer de ovario se acompaña con mayor frecuencia
de recurrencias tumorales postratamiento

a) Cáncer mucinoso
b) Tumor limítrofe endometroide

c) Cáncer seroso

d) Cáncer endometroide

e) Tumor de Brenner

CQ33-5. Una evaluación progresiva del antígeno Ca-125 al término del tratamiento oncológico en casos
como el presente, obliga a considerar la conducta de manejo siguiente:

a) Second look

b) Cirugía de intervalo

c) Cirugía de citorreducción secundaria

d) Radioterapia radical a la pelvis

e) Iniciar con segunda línea de quimioterapia

CQ34. Femenino de 47 años de edad, obesa, G:I, P:I, con antecedentes de tratamiento médico por ovarios
poliquísticos, quien acude a consultar por hiperpolimenorrea de cuatro meses de evolución. La exploración
muestra como datos positivos, útero de 8 cm y un reporte de legrado biopsia de hiperplasia endometrial
compleja atípica.

CQ34-1. Cuál de los siguientes medicamentos puede influir en el desarrollo de la patología del presente
caso:

a) Clomifeno

b) Medroxiprogesterona

c) Megestrol

d) Estrógenos conjugados

e) Estrógenos-progesterona

CQ34-2. Con qué frecuencia una hiperplasia como la del presente caso se ve asociada a ovarios
poliquísticos:

a) 100%

b) 80%

c) 50%
d) 20%

e) 0%

CQ34-3. La hiperplasia compleja atípica evoluciona a cáncer endometrial con la frecuencia siguiente:

a) 0%

b) 25%

c) 40%

d) 60%

e) 80%

CQ34-4. Un reporte de patología, en esta paciente, de adenocarcinoma endometroide del endometrio con
extensión a las glándulas del endocérvix clasificaría esta lesión como etapa:

a) Ia

b) IIb

c) IIa

d) Ib

e) Ic

CQ34-5. Cuál de los siguientes tipos histopatológicos del cáncer endometrial, tiene peor pronóstico.

a) Tipo endometroide grado 3

b) Adenocarcinoma grado 2

c) Adenocantoma grado 3

d) Adenonocarcinoma grado 3

e) Adenocarcinoma papilar

CQ35. Mujer de 46 años la cual notó una masa en su muslo derecho hace aproximadamente 4 a 6 meses
previos a su consulta médica. La exploración física mostró tumor blando, movible, bien circunscrito de 6 x 5
cm. Zonas inguinales negativas.

CQ35-1. El diagnóstico preciso del paciente se realiza con el siguiente método:


a) Resonancia magnética

b) Tomografía axial

c) Biopsia incisional

d) Biopsia excisional

e) Citologia por aspiración

CQ35-2. El resultado de patología reporta liposarcoma mixoide. Una de las siguientes alteraciones
cromosómicas es característica del liposarcoma mixoide:

a) Translocación t(12-16)

b) Cromosomas gigantes

c) Trisomia 18

d) Translocación t(X;18)(p11;q11)

e) Translocación t(11-22)(q13;q12)

CQ35-3. La estadificación clínica de la paciente sería:

a) Estadio Ia

b) Estadio Ib

c) Estadio IIa

d) Estadio IIb

e) Estadio III

CQ35-4. El tratamiento más adecuado para la enferma es:

a) Radioterapia preoperatoria

b) Desarticulación coxofemoral

c) Excisión radical amplia+RT

d) Excisión radical amplia+QT

e) Quimioterapia de inducción

CQ35-5. La sobrevida a 5 años del presente caso es del:


a) 10%

b) 30%

c) 50%

d) 80%

e) 100%

CQ36. Un hombre de 21 años fue visto por primera vez por una masa indolora de 10 x 10 cm en la nalga
derecha. Tenía historia de trauma mientras jugaba basquetbol diagnosticado como hematoma infectado, fue
tratado con antibióticos. Cuando la masa no cedió se intentó drenaje y posterior al procedimiento el tumor
fungó por la herida. El diagnóstico histológico de la biopsia mostró liposarcoma pleomórfico.

CQ36-1. Cuál de los factores pronósticos relacionados con el caso le confiere al enfermo mejor pronóstico:

a) Edad joven

b) Histología

c) Localización central

d) Tamaño

e) Ulceración

CQ36-2. Mencione cuál de las siguientes localizaciones es más común para el liposarcoma en general:

a) Cabeza y cuello

b) Extremidades inferiores

c) Extremidades superiores

d) Retroperitoneo

e) Torax

CQ36-3. Cuál de los siguientes métodos de diagnóstico predice con mayor precisión la profundidad de
invasión:

a) Placa simple de pelvis

b) US pélvico

c) Tomografía axial
d) Marcadores tumorales

e) Angiografía

CQ36-4. Durante la cirugía se identifica que la tumoración se encuentra por encima del gluteo mayor. La
conducta a seguir será:

a) Excisión marginal RT

b) Glutectomía+RT posoperatoria

c) Hemipelvectomia

d) Quimioterapia de inducción

e) Excisión+QT adyuvante

CQ36-5. Durante el seguimiento el enfermo presenta recurrencia en el margen profundo. La conducta a


seguir sería:

a) Hemipelvectomía

b) Reexcisión

c) Quimioterapia paliativa

d) Radioterapia paliativa

e) Tumorectomía

CQ37. Mujer de 44 años con lesión en la articulación de tobillo de 4 x 3 cm derecho. La paciente fue
diagnosticada como granuloma piógeno y manejada con antibióticos. La lesión progresó al manejo
conservador y persistió fija, dura y multinodular.

CQ37-1. Cuál de los siguientes métodos diagnósticos es de elección para el diagnóstico histopatológico
correcto:

a) Biopsia por aspiración con aguja fina

b) Tomografía axial

c) Resonancia magnética nuclear

d) US pélvico

e) Biopsia incisional
CQ37-2. El reporte de patología indica sarcoma sinovial monofásico. El diagnóstico histopatológico
diferencial deberá ser en primer lugar con:

a) Liposarcoma mixoide

b) Leiomiosarcoma

c) Fibrosarcoma

d) Angiosarcoma

e) Sarcoma de Kaposi

CQ37-3. Cuál de los siguientes hallazgos cromosómicos es característico del sarcoma sinovial y diagnóstico
diferencial con respecto a otros sarcomas:

a) Trisomía 18

b) Translocación 11-22

c) Cromosomas gigantes

d) Trisomia 18-11

e) Deleción 21

CQ37-4. Durante la cirugía los márgenes fueron reportados como positivos macroscopicamente. Cuál será el
margen adecuado para este caso:

a) Amputación supracondilea

b) Amputación infracondilea

c) Excisión marginal

d) Desarticulación transmetartasiana

e) Desarticulación coxofemoral

CQ37-5. El pronóstico de sobrevida a 5 años en este caso es del:

a) 10%

b) 20%

c) 30%

d) 50%
e) 70%

CQ38. Masculino de 37 años con tumor en muslo derecho de 8 x 8 cm en tercio medio, de 8 meses de
evolución. Su médico tratante lo catalogó como fibroma benigno, una biopsia amplia reveló sarcoma
sinovial. La historia clínica reveló tumor fijo, duro y multinodular. Ingle negativa.

CQ38-1. El estadio clínico del paciente corresponde a:

a) Ia

b) IIb

c) IIa

d) III

e) IV

CQ38-2. El porciento de ganglios afectados en estos casos es de aproximadamente:

a) 5%

b) 10%

c) 15%

d) 30%

e) 50%

CQ38-3. El manejo quirúrgico de este caso correspondería:

a) Excisión amplia

b) Excisión amplia+disección ganglionar

c) Excisión marginal+disección ganglionar

d) Desarticulación coxofemoral

e) Hemipelvectomia

CQ38-4. Cuál de los siguientes factores justificaría (en caso de considerarse así) disección ganglionar:

a) Localización
b) Grado

c) Tamaño

d) Sexo

e) B y c son correctas

CQ38-5. Cuál de los hallazgos clínicos en el paciente le confiere buen pronóstico:

a) Edad

b) Grado de diferencición

c) Histología

d) Tamaño

e) Sexo

CQ39. Femenino de 72 años la cual inicia con aumento de volumen abdominal y masa palpable izquierda.
La TAC abdominal demostró lesión retroperitoneal con componente graso de aproximadamente 20 x 20 cm.

CQ39-1. Los hallazgos tomográficos son orientadores de:

a) Liposarcoma

b) Angiosarcoma

c) Sarcoma de Kaposi

d) Nurosarcoma

e) Sarcoma sinovial

CQ39-2. Durante la exploración quirúrgica se encuentra tumor adherido a organos cercanos (colon y riñon).
La conducta a seguir es:

a) Excisión amplia

b) Resección del tumor con nefrectomia y colectomia

c) Biopsia y RT posquirúrgica

d) RT intraoperatoria y biopsia

e) Tumorectomia marginal
CQ39-3. El enfermo desarrolla, durante el seguimiento, recurrencia retroperitoneal local. La conducta a
seguir es:

a) Quimioterapia paliativa

b) Radioterapia paliativa

c) Exploración y resección de ser posible

d) Quimioterapia intrarterial

e) Manejo conservador

CQ39-4. El resultado final de patología demostró liposarcoma desdiferenciado. De las siguientes aseveraciones
cuál es correcta para este tipo de sarcoma:

a) Tiene buen pronóstico

b) Su recurrencia local es muy baja

c) Son de bajo grado de diferenciación

d) Son de alto grado histológico

e) Metastatizan a ganglios retroperitoneales

CQ39-5. La sobrevida general a 10 años para este tipo de sarcoma es de:

a) 5%

b) 15%

c) 30%

d) 50%

e) 100%

CQ40. Femenino 17 años con un pequeño tumor en paladar duro de 3 x 2 cm fijo sobre el lado izquierdo del
mismo. El cirujano pediatra lo cataloga como tumor benigno de glándulas salivales. La biopsia reveló
sarcoma sinovial monofásico.

CQ40-1. La frecuencia encontrada de sarcoma sinovial en el área de cabeza y cuello es de


aproximadamente:

a) 2%
b) 8%

c) 20%

d) 50%

e) 70%

CQ40-2. El hallazgo de sarcoma sinovial primario en áreas como el paladar, en este caso, sugieren:

a) Su origen en tejido sinovial

b) Su grado de diferenciación

c) Su capacidad de metastizar

d) Su conducta agresiva

e) Su hallazgo común en esta localización

CQ40-3. El hallazgo tomográfico reporta tumor en tejidos blandos del paladar duro izquierdo con involucro dudoso
del antro maxilar. Cuál será su conducta:

a) biopsia

b) Excisión marginal +RT

c) Palatectomía izquierda

d) RT paliativa

e) QT inducción

CQ40-4. El estudio transoperatorio demostró involucro del antro maxilar ipsilateral. Cuál será la conducta:

a) Maxilectomía radical con conservación orbitaria

b) Maxilectomía y sacrificio de la orbita

c) Legrado del antro maxilar

d) Palatectomía únicamente

e) Biopsia y manejo paliativo

CQ40-5. Cuál de los siguientes hallazgos ayuda al diagnóstico correcto del sarcoma en esta localización:

a) Mutación p53
b) CMyc

c) Translocación 11-22

d) Cromosomas gigantes

e) Translocación t 11-18

CQ41. Femenino de 46 años con tumor en la pared anterior del abdomen el cual fue notado 15 años
posterior a cesárea. La paciente fue manejada como tumor desmoide con múltiples excisiones. Actualmente
el tumor mide 15 x 20 cm sobre la región derecha del abdomen.

CQ41-1. Cual de los hallazgos siguientes se relaciona con una conducta agresiva del tumor:

a) Sexo femenino

b) Edad

c) Excisiones múltiples

d) Tamaño tumoral

e) C y d son correctas

CQ41-2. La biopsia reveló sarcoma sinovial bifásico. Una de las siguientes características es común en este
tipo de tumor:

a) Es más común en sexo femenino

b) Nace de un tumor desmoide previo

c) Existe siempre el antecedente de cirugía o trauma

d) Se asocia a cesarea

e) Es común en sexo masculino

CQ41-3. Durante su evolución la enferma desarrolla dos lesiones aisladas en pulmón derecho y recurrencia
local. Cuál es la conducta más apropiada:

a) Quimioterapia sola

b) Radioterapia y excisión

c) Reexcisión local y resección de metástasis pulmonares

d) Tumorectomía local y QT

e) Inmunoterapia
CQ41-4. La sobrevida a 5 años posterior a la resección completa, en el caso de esta enferma, es de:

a) 10%

b) 15%

c) 30%

d) 50%

e) 70%

CQ41-5. Cuál de los siguientes factores se ha asociado con el peor pronóstico en este tipo de sarcomas:

a) Edad joven

b) Tamaño mayor a 5 cm

c) Translocación 11-18

d) P 53

e) Cmyc

CQ42. Masculino de 59 años con lesión en el codo izquierdo. Tumor de 3 x 4 cm dura sin involucro de la
articulación. Inicialmente manejada como lipoma, presenta recurrencia local. La biopsia excisional demostró
liposarcoma de bajo grado.

CQ42-1. Cuál de las siguientes localizaciones es más común para la presentación de los liposarcomas:

a) Cabeza y cuello

b) Retroperitoneo

c) Abdomen

d) Extremidades inferiores

e) Extremidades superiores

CQ42-2. Los siguientes son variedades de liposarcomas de bajo grado, excepto:

a) Liposarcoma tipo lipoma

b) Liposarcoma mixoide
c) Liposarcoma esclerosante

d) Liposarcoma pleomórfico

e) Liposarcoma inflamatorio

CQ42-3. La conducta quirúrgica para este caso sería:

a) Escisión amplia sola

b) Escisión+radioterapia

c) Escisión y linfadenectomía

d) RT preoperatoria

e) QT sola

CQ42-4. La estadificación clínica del paciente corresponde a:

a) Ia

b) Ib

c) II

d) IIIa

e) IV

CQ42-5. Para este grupo de pacientes la sobrevida a 5 años es de:

a) 90%

b) 50%

c) 30%

d) 10%

e) 5%

CQ43. Femenino de 65 años con dolor pélvico crónico. A la exploración física la enferma presenta masa
pélvica fija a 3 cm por debajo de cicatriz umbilical.
CQ43-1. El estudio que predice mejor el involucro a estructuras pélvicas es el siguiente:

a) Placa simple de pelvis

b) TAC pélvica

c) US pélvico transvaginal

d) Us pélvico abdominal

e) Urograma excretor

CQ43-2. El TAC demuestra tumor pélvico que desplaza los ureteros. El diagnóstico diferencial deberá ser
con las siguientes patologías:

a) Cáncer de ovario

b) Cáncer de colon

c) Sarcoma uterino

d) Todas las anteriores

e) Ninguna de las anteriores

CQ43-3. Durante la cirugía se encuentra tumor retroperitoneal con involucro de recto sigmoides. La
conducta a seguir es:

a) Exenteración pélvica posterior

b) Exenteración pélvica total

c) Tumorectomía

d) Colostomía y biopsia

e) Exenteración pélvica anterior

CQ43-4. El resultado definitivo indica liposarcoma pleomorfico de 10 x 15 cm con involucro de recto. La


siguiente conducta a seguir es:

a) Observación

b) Radioterapia pélvica

c) QT sistémica

d) Inmunoterapia
e) Paliación de síntomas

CQ43-5. La sobrevida a 5 años de los sarcomas retro pélvicos con involucro de organos adyacentes es de:

a) 5%

b) 15%

c) 40%

d) 80%

e) 100%

CQ44. Masculino de 35 años sin antecedentes de importancia que acude por tumoración en cara posterior de
pierna derecha de 6 meses de evolución que condiciona dolor e impotencia funcional, mide 6 x 6 cm
aproximadamente, movible, sólido, no pulsatil.

CQ44-1. De los estudios mencionados a continuación, cual sería el primero que usted solicitaría.

a) RX de pierna

b) Gamagrafia

c) TAC

d) Biopsia Tru-Cut

e) Angiografía

CQ44-2. El reporte de patología refiere sarcoma de tejidos blandos moderadamente diferenciado, probable
histiocitoma fibroso maligno. Sobre la base de este reporte usted solicitaría:

a) Rx. de pierna

b) Angiografía

c) Resonancia magnética nuclear

d) Angiotac

e) Gamagrafía

CQ44-3. En los estudios solicitados el paciente no tiene compromiso vascular ni neurológico, confirmado
por la clínica, usted recomendaría:
a) Amputación supracondilea

b) Hemipelvectomía

c) Resección marginal

d) Radioterapia 80 Gy

e) Excisión amplia más radioterapia

CQ44-4. El reporte final de patología indica tumor de 8 x 6 x 5 cm, con márgenes quirúrgicos libres de
lesión, tumor moderadamente diferenciado con permeación vascular y linfática. A que etapa corresponde.

a) IA

b) IB

c) IIA

d) IIB

e) III

CQ45. Masculino de 43 años con tumor en cara anterior de muslo tercio superior, llega con antecedente de
resección del mismo, clínicamente no se palpa tumor residual pero en el reporte de patología la descripción
macroscópica dice múltiples fragmentos de tumor que en conjunto miden 8 x 8 cm y el reporte es de
Dermatofibrosarcoma Protuberans.

CQ45-1. La conducta más adecuada sería:

a) RT adyuvante

b) QT adyuvante

c) Observación

d) Hemipelvectomía externa

e) Reexcisión de cicatriz y RT adyuvante

CQ45-2. El dermatofibrosarcoma protuberans, según la clasificación histopatológica actual, se considera:

a) Tumor fibroso maligno

b) Tumor fibrohistiocítico

c) Tumor fibrohistiocítico intermedio


d) Fibromatosis

e) Tumor fibroso benigno

CQ45-3. De acuerdo a la pregunta anterior, y por el tamaño tumoral y márgenes libres de resección, que
recomendaría al paciente:

a) RT

b) Quimioterapia

c) Compartamentectomía

d) Amputación

e) Ninguno de los anteriores

CQ45-4. Suponiendo que en la revisión de laminillas el patólogo le reporta tumor de Bednar corresponde a:

a) Fibroxantoma atípico

b) Histiocitoma fibroso benigno

c) Histiocitoma fibroso mixoide

d) Dermatofibrosarcoma protuberans pigmentado

e) Rabdomiosarcoma embrionario

CQ45-5. Entonces cuál sería la conducta más apropiada

a) Quimioterapia

b) Radioterapia

c) QT, RT, concomitante

d) Amputación

e) Disección inguinal electiva

CQ46. Paciente femenino de 50 años de edad con un tumor en cara anterior de muslo izquierdo de 10 X 10
cm. Fue biopsiada y el reporte, únicamente sarcoma con este diagnóstico, es referido a usted.

CQ46-1. Mencione el enunciado correcto con respecto a los sarcomas.


a) Los sarcomas son el 10% de los tumores malignos

b) Los sarcomas más frecuentemente se localizan en cabeza y cuello

c) Son el 1% de los casos y el 2% de mortalidad de los tumores malignos

d) Siempre se originan de tejido derivado del mesodermo

e) Se comportan igual que los carcinomas

CQ46-2. Manda a revisión las laminillas y el reporte del patológo es de un Liposarcoma bien diferenciado,
con base en lo anterior cuál es una variedad de liposarcoma bien diferenciado:

a) Liposarcoma mixoide

b) Liposarcoma pleomórfico

c) Hibernoma

d) Liposarcoma esclerosante

e) Liposarcoma de células redondas

CQ46-3. Como el liposarcoma bien diferenciado es un sarcoma de bajo grado con respecto a este
diagnóstico, cuál o cuáles serían los estudios de extensión que solicitaría:

a) Gamagrama óseo y US Hepático

b) Tele de tórax y gamagrama óseo

c) Tele de tórax

d) Tele de tórax y TAC de tórax

e) Tele de tórax y US hepático

CQ46-4. A este paciente únicamente se le practicó biopsia incisional y con el reporte de liposarcoma bien
diferenciado. Qué tratamiento le ofrecería:

a) Tumorectomía marginal+Radioterapia

b) Compartamentectomía+QT

c) Amputación+Quimioterapia

d) Compartamentectomía+RT

e) Compartamentectomía únicamente
CQ46-5. Suponiendo que en éste mismo paciente el tumor se encuentra en cara anterior del brazo entonces,
qué tratamiento le ofrecería:

a) Tumorectomía marginal+RT

b) Compartamentectomía+QT

c) Amputación+QT

d) Compartamentectomía+RT

e) Compartamentectomía únicamente

CQ47. Masculino de 58 años de edad, presenta tumor abdominal de 20 x 15 cm, fijo, no doloroso, niega otra
sintomatología, es referido a su consulta.

CQ47-1. De los siguientes cuál sería el estudio de gabinete más importante:

a) US hepático

b) Gamagrama Óseo

c) Tele de tórax

d) TAC de abdomen

e) Tránsito intestinal

CQ47-2. De los sarcomas de tejidos blandos, qué porciento ocupan los sarcomas del retroperitoneo:

a) Menos del 2

b) 5

c) 15

d) 35

e) 50

CQ47-3. Qué tipo de sarcomas son más frecuentes en retroperitoneo:

a) Liposarcoma y leiomiosarcoma

b) Liposarcoma y Rabdomiosarcoma
c) Angiosarcoma y Sarcoma Sinovial

d) Leiomiosarcoma e Histiocitoma fibroso maligno

e) Liposarcoma y fibrosarcoma

CQ47-4. A qué órgano metastatizan más frecuentemente los sarcomas de retroperitoneo:

a) Riñón

b) Hueso

c) Hígado

d) Pulmón

e) Intestino delgado

CQ47-5. Qué tratamiento le ofrecería a este paciente:

a) Quimioterapia Neoadyuvante y tumorectomía

b) Tumorectomía y QT adyuvante

c) Resección en bloque con órganos adyacentes

d) Resección en bloque con órganos adyacentes, RT intraoperatoria y teleterapia

e) Resección en bloque con órganos adyacentes y teleterapia

CQ48. Femenino de 42 años de edad con antecedentes familiares de leucemia y tumores cerebrales y
personal de cáncer de mama, tratado hace 8 años, mastectomía, QT adyuvante y radioterapia; es referida a su
consultorio con un tumor de 6 X 5 cm en brazo derecho asintomático, el reporte de la biopsia fue
fibrosarcoma.

CQ48-1. Con qué síndrome se asocia lo anterior:

a) No se asocia con ningún síndrome

b) Síndrome de Gorlin

c) Síndrome de Lynch tipo I

d) Síndrome de Li Fraumeni

e) Síndrome de Gardner
CQ48-2. Con base en su respuesta anterior, en qué gene se encuentra el defecto:

a) NF1

b) RB1

c) P53

d) PTC

e) APC

CQ48-3. Si esta paciente tuviera el sarcoma en el miembro superior ipsilateral a la mastectomía y ésta cursa
con linfedema crónico, él diagnostico histopatológico del tumor más probablemente sería:

a) Fibrosarcoma

b) Linfagiosarcoma

c) Rabdomiosarcoma

d) Leiomiosarcoma

e) Liposarcoma

CQ48-4. Con base en esta paciente y a su respuesta anterior, cómo se le conoce a este síndrome:

a) De Lynch

b) De Gorlin

c) De Stewart-Treves

d) De Gardner

e) De Werner

CQ48-5. Además de los factores hereditarios, genéticos y clínicos (linfedema), qué otro factor de riesgo
presenta para la aparición de un sarcoma de tejidos blandos:

a) Mastectomía

b) Quimioterapia

c) Radioterapia

d) Hormonoterapia
e) No tiene otro factor de riesgo

CQ49. Paciente masculino de 59 años de edad con un tumor en tercio proximal de muslo en su cara anterior
de 15 X 10 cm totalmente asintomático, ha sido biopsiado en 2 ocasiones (Tru-cut) con muestra insuficiente
observando sólo necrosis.

CQ49-1. Es referido a su consulta, usted no tiene diagnóstico, que le ofrecería al paciente:

a) Biopsia por aguja fina

b) Biopsia por Tru-cut

c) Biopsia incisional

d) Biopsia incisional con estudio transoperatorio

e) Biopsia dirigida por TAC

CQ49-2. El reporte de patología tiene un diagnóstico de sarcoma alveolar de alto grado, en la clasificación
histológica qué origen tiene este tumor:

a) Tumor fibrohistiocítico

b) Tumor paragangliónico

c) Tumor pluripotencial mesenquimal

d) Tumor misceláneo

e) Tumor inclasificado

CQ49-3. En este paciente y con diagnóstico de sarcoma alveolar, usted cree que el paciente es candidato a
cirugía conservadora, solicita preoperatorios y estudios de extensión y en la TAC de tórax se observan dos
nódulos en el pulmón izquierdo y otros dos en pulmón derecho. Con este nuevo dato qué conducta tomaría
usted ahora con respecto al primario:

a) QT neoadyuvante y amputación

b) Amputación y QT adyuvante

c) RT preoperatorio y amputación

d) Compartamentectomía y radioterapia adyuvante

e) Tumorectomía
CQ49-4. Después del tratamiento anterior, con el primario ya controlado, que le ofrecería al paciente con
respecto a las metástasis pulmonares:

a) Observación

b) Lo considera fuera de tratamiento oncológico

c) Metastasectomía unilateral

d) Metastasectomía bilateral

e) Lobectomía bilateral

CQ49-5. Cómo debe estadificarse este paciente, de acuerdo a la estatificación del 97, de la UICC.

a) EC IIIa

b) EC IIIb

c) EC IV

d) EC IVa

e) ECIVb

CQ50. Masculino de 49 años de edad, con tumor en tercio proximal de muslo en su cara anterior de 15 X 10
cm totalmente asintomático. Se le realiza una biopsia por Tru-cut, con reporte de Sarcoma alveolar con este
diagnóstico es referido a su consulta.

CQ50-1. Qué estudios de extensión y para valorar al primario le solicitaría usted:

a) Tele de tórax y TAC de muslo y pelvis

b) Tele de tórax y TMN de muslo y pelvis

c) Tele de tórax y TAC de tórax

d) TAC de tórax y Tac de muslo y pelvis

e) TAC de tórax y Gamagrama óseo

CQ50-2. Los estudios de extensión fueron negativos, no hay contraindicación para la cirugía y usted cree
que el paciente es candidato a cirugía conservadora. De las siguientes opciones, cual le ofrecería.

a) Tumorectomía+RT adyuvante

b) Compartamentectomía
c) Excisión amplia

d) Compartamentectomía+QT intra arterial

e) Compartamentectomía+RT adyuvante

CQ50-3. El paciente tarda un mes para operarse, usted observa que el tumor progresó pero piensa que puede
ofrecer todavía cirugía conservadora, en el transoperatorio observó ue no se puede realizar tratamiento
conservador, el paciente había aceptado amputación previa a la cirugía de no poder realizarse el tratamiento
conservador. En el tercio proximal del muslo qué estructura anatómica es la frontera para decidir
desarticulación de la cadera vs hemipelvectomía externa.

a) Trocánter-mayor

b) Trocánter-menor

c) Ligamento inguinal

d) Borde superior de acetábulo

e) Espina iliaca antero superior

CQ50-4. Qué tipo de hemipelvectomía realizaría a este paciente:

a) Hemipelvectomía interna tipo II

b) Hemipelvectomía externa con colgajo anterior

c) Hemipelvectomía externa con colgajo posterior

d) Hemipelvectomía ampliada con colgajo posterior

e) Ninguno de los anteriores

CQ50-5. Al momento de la cirugía se encuentra el cordón espermático totalmente infiltrado por el tumor,
que conducta tomaría usted:

a) Considera el tumor irresecable

b) Conserva el testículo dejado tumor macroscópico

c) Hemipelvectomía externa compuesta con colgajo posterior

d) Hemipelvectomía externa compuesta con colgajo anterior

e) No cambia conducta anterior


CQ51. Masculino de 53 años de edad con un tumor de 10 X 8 cm en brazo derecho cara posterior
asintomático, llega a su consulta y le realiza biopsia incisional con diagnostico de Sarcoma Sinovial, los
estudios de extensión son negativos con respecto a los sarcomas.

CQ51-1. Cuál es la piedra angular en el tratamiento.

a) RT preoperatoria

b) QT neoadyuvante

c) Cirugía con márgenes negativos

d) Tumorectomía

e) RT posoperatoria

CQ51-2. Con respecto a las amputaciones en el tratamiento de sarcoma de tejidos blandos, mencione el
enunciado correcto.

a) Es el tratamiento de elección

b) Aumenta el control local y la sobrevida global

c) Disminuye el control local y la sobrevida global

d) Aumenta el control local y no impacta la sobrevida

e) No impacta en control y en sobrevida

CQ51-3. En este paciente usted elige realizar cirugía conservadora, le realiza compartamentectomía
posterior del brazo y el reporte es de sarcoma sinovial de alto grado, tamaño tumoral 9.5 x 9.5 x 6.0 cm, con
márgenes libres, el más cercano a 2.5 cm, usted considera al paciente con:

a) Margen insuficiente y reexcisión

b) Suficientemente tratado

c) Margen insuficiente y amputación

d) Margen suficiente y agrega RT

e) Margen insuficiente y agrega RT

CQ51-4. Si en este paciente usted hubiera decidido administrar RT preoperatorio, cuál sería el enunciado
falso en relación a la RT preoperatoria vs RT posoperatoria.

a) Aumenta las complicaciones locales


b) Menor campo de Radiación

c) Mayor sobrevida

d) Igual recurrencia local

e) Usualmente no se añade un Boost

CQ51-5. Con el reporte histopatológico y el tamaño tumoral usted decide que el paciente es candidato a QT
adyuvante, cuáles serían los agentes más activos contra sarcomas.

a) Adriamicina, ifosfamida y dacarbazina

b) Platino, 5 fluorouracilo y bleomicina

c) Carboplatino, melfalán y dacarbazina

d) Actinomicina D, adriamicina y factor de necrosis tumoral

e) Etopósido, platino y vinblastina

CQ52. Paciente femenino de 30 años de edad con tumor en cara anterior de muslo derecho de 7 X 5 cm.
Tiene antecedentes de resección de 2 tumores con reporte de neurofibromas plexiforme a la exploración
física; lo relevante, ademas del tumor, son varios nódulos cutáneos menores de 1 cm y manchas café con
leche en la piel usted diagnostica neurofibromatosis.

CQ52-1. Cuál es el gene afectado en esta enfermedad.

a) Rb 1

b) P 53

c) PTC

d) NF1

e) TSC1

CQ52-2. Con el diagnóstico de neurofibromatosis, cuál es el sarcoma de tejidos blandos más frecuente
asociado a este padecimiento.

a) Sarcoma sinovial monofásico

b) Rabdomiosarcoma embrionario

c) Tumor maligno de vaina nerviosa periférica


d) Leiomiosarcoma Epiteloide

e) Liposarcoma de células redondas

CQ52-3. Comó afecta el tratamiento de este paciente al hecho que padezca neurofibromatosis.

a) Empeora el pronóstico

b) Mejora el pronóstico

c) No afecta el pronóstico

d) La radioterapia esta contraindicada

e) Mejora el período libre de enfermedad

CQ52-4. Qué otro nombre recibe la neurofibromatosis tipo I.

a) Síndrome de Werner

b) Síndrome de Li Freumeni

c) Enfermedad de Gardner

d) Enfermedad de Von Recklinghausen

e) Enfermedad de Bednar

CQ52-5. El sarcoma de tejidos blandos asociado con la neurofibromatosis desde el punto de vista
histopatológico y embriológico tiene una característica que lo distingue de los demás. Mencione esta
característica.

a) Tumor mesenquimal pluripotencial

b) Se origina de un epitelio

c) Se origina del mesodermo

d) Se origina del ectodermo

e) Se origina del endodermo

CQ53. Paciente masculino de 18 años de edad con un tumor de 5 X 5 cm en región hipotenar totalmente
asintomático.
CQ53-1. Acude a su consulta con el antecedente de dos biopsias con Tru-Cut, que muestran únicamente
necrosis, usted le propone al paciente:

a) Amputación

b) Biopsia excisional

c) Biopsia por Tru-Cut

d) Biopsia aspiración con aguja fina

e) Biopsia incisional con estudio transoperatorio

CQ53-2. El reporte de patología es de sarcomas de células claras. Desde el punto de vista histopatológico,
cómo se clasifica este tumor.

a) Tumor sinovial

b) Tumor Neural

c) Tumor Paraganglionico

d) Tumor misceláneo

e) Tumor de músculo esquelético

CQ53-3. Con qué otro nombre se le conoce a este tumor.

a) Tumor de Dabska

b) Tumor de Bednar

c) Melanoma de partes blandas

d) Plexosarcoma

e) Schwannoma maligno epiteloide

CQ53-4. Usted reexplora al paciente encontrando una adenomegalia axilar ipsilateral de 2.5 x 2 cm le realiza
un BAAF con reporte positivo para sarcoma, cómo estadifica al paciente.

a) ECIIIa

b) ECIIIb

c) ECIV

d) ECIVa
e) ECIVb

CQ53-5. Con respecto a la metástasis ganglionar, qué tratamiento le propone al paciente.

a) Observación

b) Perfusión aislada de la extremidad

c) Resección en el ganglio metástasico

d) Disección baja de axila

e) Disección radical de axila

CQ54. Femenino de 40 años de edad presenta un tumor de mama derecha de 6 X 5 cm totalmente


asintomática, la axila y hueso supraclavicular ipsilateral son negativos, la mastografía muestra una lesión de
bordes bien definidos. Se le realiza una biopsia por Tru-Cut y se identifica hueso y cartílago maduro y
células epiteliales malignas.

CQ54-1. Qué enfermedad diagnóstica en esta paciente.

a) Osteocondroma extraesquelético

b) Carcinosarcoma

c) Osteosarcoma extraesquelético

d) Tumor Phyllodes

e) Carcinoma metaplasico

CQ54-2. En la clasificación histopatológica de los sarcomas de tejidos blandos, cómo se clasifica al tumor
Phylloides.

a) Tumor óseo extraesqueletico

b) Tumor cartilaginoso extraesqueletico

c) Tumor mesenquimal pluripotencial

d) Tumor misceláneo

e) No se clasifica como sarcoma

CQ54-3. En esta misma paciente, si la biopsia hubiese mostrado un tumor Phylloides acompañado de
fibrosarcoma y tejido epitelial benigno, usted considera a este tumor como:
a) Sarcoma de alto grado

b) Sarcoma de bajo grado

c) Tumor Phylloides maligno

d) Tumor Phylloides benigno

e) Carcinoma metaplásico

CQ54-4. De los siguientes tumores de mama ¿Cual no puede producir hueso y/o cartílago?

a) Carcinoma metaplasico

b) Carcinoma Mucinoso

c) Fibroadenoma

d) Adenoma pleomorfico

e) Tumor Phyllodes

CQ54-5. El reporte definitivo de patología muestra un histiocitoma fibroso maligno de alto grado, en mama
clínicamente el tumor está adherido a la fascia del pectoral mayor y mide 6 x 5 cm. Qué tratamiento le
ofrecería a esta paciente:

a) Mastectomía radical modificada

b) Mastectomía radical clásica

c) Mastectomía total, resección de pared y disección axilar

d) Mastectomía total y resección del músculo pectoral mayor y menor en bloque

e) Mastectomía total y resección de pectorales en bloque y disección axilar

CQ54-6. El reporte de patología muestra márgenes libres de tumor, histiocitoma fibroso maligno de alto
grado, tamaño tumoral 7.2 x 6.5 x 6.0 cm. La paciente reune requisitos para tratamiento adyuvante, de ser
así cuál o cuáles son estos:

a) No tiene indicación para Tx adyuvante

b) Si tiene indicación por tamaño tumoral

c) Tiene indicación por tipo histológico

d) Tiene indicación por grado histológico

e) Tiene indicación por tamaño tumoral y grado histológico


CQ55. Masculino de 26 años de edad con diagnóstico de histiocitoma fibroso maligno en antebrazo
izquierdo, el tamaño tumoral es de 7 x 6 cm en cara posterior del mismo por lo demás asintomático. Los
estudios de extensión son negativos.

CQ55-1. Qué tratamiento le propone a este paciente.

a) RT preoperatoria y compartamentectomía

b) Tumorectomía y RT. adyuvante

c) Compartamentectomía y QT. adyuvante

d) Compartamentectomía y RT adyuvante

e) Compartamentectomía y QT-Rt concomitante

CQ55-2. El reporte de patología muestra histiocitoma fibroso maligno de alto grado, tamaño tumoral 7.5 x 6.7 x
5.2 cm, mágenes libres de tumor, usted lo refiere a oncología médica para tratamiento adyuvante según los
metanálisis del 97, con respecto a QT adyuvante. Qué espera para este paciente.

a) Menor sobrevida libre de enfermedad a 10 años

b) Menor sobrevida libre de enfermedad local a 10 años

c) Menor sobrevida global a 10 años

d) Mayor sobrevida global sin significancia estadística a 10 años

e) Mayor sobrevida global estadísticamente significativa a 10 años

CQ55-3. Al seguimiento 18 meses posterior al término del tratamiento detecta subyacente a cicatriz
quirúrgica un nódulo de 1.5 x 1.5 cm, qué conducta tomaría usted.

a)Observación y biopsia en un mes

b) TAC de Tórax

c) BAAF

d) Biopsia incisional

e) Biopsia dirijida por TAC

CQ55-4. El reporte de patología es de histiocitoma fibroso maligno. Qué consideraciones realizaría con
respecto a la recurrencia local.
a) Aumenta metástasis a pulmón

b) Aumenta metástasis ganglionares

c) Disminuye sobrevida global

d) No afecta sobrevida global

e) Aumenta sobrevida global

CQ55-5. Con respecto a la recurrencia local, mencione el enunciado correcto:

a) Casi el 100% de las recurrencias son antes de 12 meses

b) La mayoría de las recurrencias locales son en ganglios regionales

c) El margen microscópico positivo no afecta la recurrencia local

d) Alrededor del 90-95% de las recurrencias locales son antes de 3 años

e) La aparición del tumor después de 18 meses se conoce como persistencia

CQ55-6. Qué tratamiento le ofrecería a este paciente:

a) Amputación supracondilea

b) Quimioterapia paliativa

c) Resección amplia del tumor y colocación del injerto

d) Resección amplia del tumor y colgajo microvascular

e) Lo considera fuera de tratamiento oncológico

CQ56. Femenino de 28 años de edad con padecimiento de 3 meses de evolución. Inició con astenia,
adinamia, pérdida de peso, agregándose petequias, equimosis, disnea progresiva y lipotimias. A la
exploración física se le encontró con marcada palidez de tegumentos y conjuntivas, petequias generalizadas
y equimosis de predominio en extremidades inferiores, en ingle izquierda se palpó un ganglio de 2 cm de
diámetro mayor. Al tacto rectal se detecta induración en pared lateral izquierda de recto la mucosa integra.
Los exámenes de laboratorio reportaron hemoglobina de 5.7 g/dl, hematocrito de 14%, plaquetas de 4
mil/mm3, leucocitos 5,700 miles/mm3.

CQ56-1. ¿Cuál de los estudios siguientes es el indicado para establecer el diagnóstico?

a) TAC abdomino-pélvica y torácica

b) Colonoscopia
c) Aspirado de médula ósea

d) Biopsia de ganglio inguinal

e) Nueva biometría hemática

CQ56-2. Se practicó biopsia de ganglio y reporta carcinoma poco diferenciado con componente de células
claras; el aspirado de médula ósea revela infiltración de células epiteliales y se observan vacuolas e imagen
en anillo de sello en algunas de ellas. El siguiente estudio a solicitar es:

a) Cortes profundos con tinción de PAS

b) Citometría de flujo

c) Inmunohistoquímica

d) Tinción de Mason

e) Citogenética

CQ56-3. La TAC abdómino-pélvica demostró tumor localizado desde el borde del ámpula rectal hasta la
región perineal, además de crecimientos ganglionares en cadenas ilíacas y paraorticos, se practicó biopsia
del tumor y se hizo inmunohistoquímica reportando: desmina positiva, actina focal y débilmente positiva,
vimentina positiva, queratina negativa, S-100 focal y débilmente positiva, HMB-45 negativa, cromogranina
negativa, sinaptofisina negativa. El diagnóstico es:

a) Adenocarcinoma de recto

b) Leiomiosarcoma pararrectal

c) Carcinoma renal

d) Rabdomiosarcoma pararrectal

e) Síndrome de Evans

CQ56-4. El procedimiento terapéutico a seguir es:

a) Resección abdominoperineal

b) Resección abdominoperineal con disección ilioinguinal izquierda

c) Quimioterapia

d) Colostomía

e) Radioterapia preoperatoria
CQ56-5. Considerando la extensión de la enfermedad y con tratamiento adecuado, la sobrevida a 2 años
seria de:

a) 90%

b) 70-80%

c) 50-60%

d) 30-40%

e) 0-20%

CQ57. Paciente de 62 años de edad con padecimiento de 6 meses de evolución con náusea, vómito
posprandial, pérdida de peso. Los exámenes de laboratorio revelan proteínas totales de 6 g hemoglobina de
10.5 g/dL.

CQ57-1. La TAC abdominal demuestra un tumor que comprimela la segunda porción del duodeno y mide
aproximadamente 6 cm de diámetro mayor, sin ningún otro hallazgo. El estudio indicado para complementar
la valoración es:

a) SEGD

b) CPRE

c) TAC dinámica de páncreas

d) Arteriografía selectiva de arteria mesentérica superior

e) Aorto cacografía

CQ57-2. Previa corrección de cifras de hemoglobina y administración de nutrición parenteral. Se somete a


exploración quirúrgica encontrando tumor dependiente de vena cava inferior adherida parcialmente a
duodeno. El procedimiento quirúrgico a realizar será:

a) Whipple

b) Resección del tumor con ligadura de vena cava inferior

c) Resección del tumor con aplicación de injerto en cava y yeyunostomía

d) Gastroyeyunoanastomosis

e) Resección del tumor con ligadura de vena cava inferior, resección de duodeno con cierre
primario, procedimiento de exclusión pilórica y gastroyeyuno anastomosis, además de
colecistectomía y colocación de sonda en t en colédoco
CQ57-3. El reporte histopatológico de la pieza quirúrgica es de leiomiosarcoma el tratamiento a seguir es:

a) Vigilancia

b) RT posoperatoria

c) QT adyuvante

d) QT y RT concomitante

e) Inmunoterapia

CQ57-4. Seis meses después de intervención quirúrgica en un US hepático se encuentran metástasis


hepáticas tres lesiones en lóbulo derecho, el tratamiento indicado será:

a) Lobectomía derecha

b) Segmentectomía

c) Radioterapia

d) Quimioterápia

e) Vigilancia

CQ57-5. La sobrevida a 5 años en pacientes con leiomiosarcoma retroperitoneal resecado completamente


pero con progresión de la enfermedad por la presencia de metástasis hepáticas es de:

a) 0%

b) 5%

c) 10%

d) 20%

e) 30%

CQ58. Paciente femenino de 76 años de edad con antecedente de estreñimiento crónico, agregándose
rectorragia y aparición de tumor perianal, el cual creció rápidamente hasta medir 15 cm de diámetro mayor,
presentaba una zona necrosada fácilmente sangrante además de infección agregada. Al tacto rectal se palpó
tumor que desplazaba recto y ano y obstruía el 40 % de la luz.

CQ58-1. El estudio histopatológico de la biopsia del tumor reporto fibrosarcoma de alto grado. El estudio
indicado para establecer extensión de la actividad local es:

a) TAC abdomino-pélvico
b) Tele radiografía de tórax

c) U.S. hepático

d) Rastreo óseo

e) TAC torácico

CQ58-2. Los estudios sólo revelan tumor pararrectal, se practicó arteriografía la cual demostraba numerosos
vasos de neoformación dependientes de ambas arterias iliacas internas de predominio de la derecha. El
tratamiento a seguir será:

a) Colostomía y tratamiento antimicrobiano

b) RT hemostática

c) Vigilancia y tratamiento sintomático

d) Resección abdominoperineal

e) Colostomía+RT preoperatoria

CQ58-3. Después de resección quirúrgica de un fibrosarcoma perineal, el tratamiento a seguir será:

a) QT adyuvante

b) RT

c) Inmunoterapia

d) RT y QT concomitante

e) Vigilancia estrecha

CQ58-4. Las metástasis a distancia más frecuentes que presentan los fibrosarcoma pélvicos son:

a) Pulmonares

b) Hepáticas

c) Cerebrales

d) Ganglionares

e) A médula ósea
CQ58-5. La incidencia de recurrencia a dos años, en sarcomas pararrectales que han sido resecados
completamente es de:

a) 10%

b) 25%

c) 40%

d) 50%

e) 70%

CQ59. Paciente del sexo masculino de 17 años de edad con aumento de volumen en tercio inferior del
muslo, de 4 meses de evolución. Niega traumatismo. La exploración física revela tumor de 4 cm duro, fijo a
planos profundos.

CQ59-1. Es el estudio básico para los tumores óseos:

a) TAC

b) Gamagrafía ósea

c) Resonancia magnética

d) Radiografía simple

e) Tomografía por emisión de positrones

CQ59-2. El estudio que usted solicitó revela tumor metaepifisiario de fémur, con ruptura de la cortical, e
imagen en rayos de sol, por lo que el diagnóstico más probable es:

a) Tumor de células gigantes

b) Encondroma

c) Osteoblastoma

d) Osteosarcoma

e) Quiste oseo aneurismático

CQ59-3. Con base al diagnóstico que usted ha realizado la conducta terapéutica de elección es:

a) Observación

b) Resección quirúrgica
c) Curetaje oseo

d) Crioterapia

e) Radioterapia

CQ59-4. Qué tratamiento adyuvante indicaría, en caso de confirmar el diagnóstico:

a) Quimioterapia

b) Radioterapia

c) No requiere tratamiento adyuvante

d) Crioterapia

e) QT-RT concomitante

CQ59-5. El diagnóstico de certeza en las lesiones óseas se realiza mediante:

a) Biopsia

b) Biopsia y Rx simple

c) Rx simple

d) TAC

e) Gamagrafía con I131-MIBI

CQ60. Se presenta a usted paciente del sexo masculino de 18 años de edad, el cual presenta una lesión en
tercio distal de fémur, clinica y radiográficamente compatible con osteosarcoma. La radiografía además
presenta fractura del fémur. Los estudios de extensión son negativos.

CQ60-1. La conducta a seguir en este paciente es:

a) Quimioterapia de inducción (6 ciclos)

b) Quimioterapia de inducción (4 ciclos)

c) Cirugía conservadora seguida de quimioterapia sistémica

d) Amputación seguida de quimioterapia sistémica

e) Amputación exclusivamente
CQ60-2. Son contraindicaciones absolutas de cirugía conservadora en osteosarcoma:

a) Tumor de tercio distal de fémur

b) Involucro vascular

c) Tumor de tercio proximal de fémur

d) Tumor de tercio proximal de húmero

e) Involucro de tejidos blandos limitado

CQ60-3. La tasa de recurrencia esperada para la cirugía conservadora y radical:

a) Es mayor para la cirugía conservadora

b) Es mayor para la amputación

c) Es igual

d) Depende de la edad

e) Depende de la reconstrucción utilizada

CQ60-4. Es el tumor maligno más frecuente del hueso:

a) Metastásicos

b) Fibrosarcomas

c) Histiocitoma fibroso maligno

d) Ewing

e) Osteosarcomas

CQ60-5. Es la principal vía de diseminación de los tumores óseos:

a) Hematógena

b) Linfática

c) Transcortical

d) Transmedular

e) Extensión directa
CQ61. Paciente del sexo femenino de 67 años de edad la cual se presenta con tumor en región esternal de 4
x 5 cm, duro y fijo. Los estudios de extensión no revelan enfermedad metastásica.

CQ61-1. Es el tumor maligno más frecuente del esternón:

a) Osteosarcoma

b) Tumor de células gigantes

c) Condrosarcoma

d) Ewing

e) Fibrosarcoma

CQ61-2. Es la variedad de condrosarcoma más frecuente:

a) Periférico

b) Central

c) Mesenquimal

d) Diferenciado

e) Células claras

CQ61-3. Es el tratamiento de elección en esta paciente:

a) Cirugía

b) Quimioterapia

c) Radioterapia

d) Crioterapia

e) Ninguno

CQ61-4. Cuál sería el tratamiento adyuvante en esta paciente:

a) Quimioterapia

b) Radioterapia

c) Inmunoterapia
d) Crioterapia

e) Ninguno

CQ61-5. El tratamiento de los pacientes con condrosarcoma de grados bajo e intermedio:

a) Incluye la resección de la lesión y quimioterapia adyuvante

b) Incluye quimioterapia neoadyuvante y la resección de la lesión

c) Incluye radioterapia y quimioterapia concomitante

d) Incluye sólo la resección de la lesión

e) Únicamente se les administra quimioterapia

CQ62. Paciente del sexo femenino de 57 años de edad la cual cuenta con antecedente de relevancia al haber
sido tratada por cáncer de mama hace cinco años, desconociendo mayor información. Actualmente se
presenta por una lesión radiográfica lítica en cresta ilíaca rápidamente progresiva. La serie ósea metastásica
no revela otro sitio de enfermedad.

CQ62-1. Es el tumor óseo maligno más frecuente:

a) Osteosarcoma

b) Condrosarcoma

c) Sarcoma de Ewing

d) Tumor de células gigantes

e) Metastásico

CQ62-2. Esta paciente es llevada a biopsia donde se reporta condrosarcoma con un patrón histológico
bifásico, la opción diagnóstica es:

a) Condrosarcoma periférico

b) Condrosarcoma central

c) Condrosarcoma células claras

d) Condroblastoma

e) Condrosarcoma mesenquimal
CQ62-3. El tratamiento de esta paciente incluye:

a) Radioterapia

b) Cirugía

c) Cirugía y quimioterapia

d) Cirugía y radioterapia

e) Quimioterapia radioterapia concomitante

CQ62-4. El condrosarcoma mesenquimal tiene predilección por los huesos:

a) Largos

b) Cortos

c) Planos

d) Inmaduros

e) Osteoporóticos

CQ62-5. Es la variedad más frecuente de condrosarcoma:

a) Periférico

b) Central

c) Mesenquimal

d) De células pequeñas

e) Pigmentado

CQ63. Paciente del sexo masculino de 18 años con tumor en rodilla derecha de un año de evolución.
Radiograficamente compatible con malignidad. La TAC revela tumor de 3 x 4 cm, no hay fractura ni
compromiso vascular o nervioso, la función de la rodilla es adecuada.

CQ63-1. El tratamiento inicial de este paciente será:

a) Amputación

b) Radioterapia

c) Quimioterapia de inducción
d) Amputación y quimioterapia adyuvante

e) Amputación y radioterapia

CQ63-2. Son, según su estirpe, los tumores óseos más frecuentes:

a) Hematopoyéticos

b) Condrogénicos

c) Osteogénicos

d) Fibrogénicos

e) Vasculares

CQ63-3. Es la principal vía de diseminación de los tumores óseos:

a) Hematógena

b) Linfática

c) Transcortical

d) Transmedular

e) Extensión directa

CQ64. Paciente del sexo femenino de 37 años de edad con lesión en fémur radiográficamente compatible
con osteosarcoma parosteal.

CQ64-1. En relación con la variedad periosteal de los osteosarcomas, la aseveración correcta es:

a) Es de origen centromedular

b) Es la variedad de osteosarcoma más frecuente

c) La localización más frecuente es tibia

d) Siempre son de bajo grado

e) El tratamiento es la resección marginal exclusivamente

CQ64-2. Con relación al osteosarcoma parosteal la aseveración correcta es:

a) Es de origen centromedular
b) Es la variedad de osteosarcoma más frecuente

c) Siempre es de bajo grado

d) Tiene mejor pronóstico que el osteosarcoma clásico

e) Los osteosarcomas parosteales de bajo grado requieren quimioterapia adyuvante

CQ64-3. Con relación al osteosarcoma de superficie de alto grado la aseveración correcta es:

a) Es de mejor pronóstico que los parosteales

b) El sitio más frecuente es el fémur

c) No requiere de quimioterapia en ninguna de sus modalidades

d) Es en pacientes ancianos

e) Es una lesión epifisiaria

CQ64-4. El tratamiento de esta paciente incluye:

a) Resección quirúrgica

b) Quimioterapia de inducción y amputación

c) Quimioterapia de inducción y cirugía conservadora

d) Radioterapia

e) Ninguno

CQ64-5. El pronóstico de esta paciente sería:

a) Peor que con la variedad clásica

b) Mejor que con la variedad clásica

c) Igual que con la variedad clásica

d) No hay comparación

e) Es letal en un año

CQ65. Paciente del sexo masculino de 16 años de edad el cual presenta una lesión metaepifisiaria expansiva,
lítica y excéntrica en tercio distal de húmero.
CQ65-1. Los diagnósticos diferenciales de esta lesión no incluyen, excepto:

a) Quiste óseo aneurismático

b) Tumor de células gigantes

c) Osteosarcoma telangiectásico

d) Adamantinoma

e) Ninguno

CQ65-2. En relación al quiste óseo aneurismático la aseveración correcta es:

a) Radiográficamente son lesiones blásticas, densas y corticales

b) Ocurren en pacientes jóvenes antes del cierre epifisiario

c) En un tercio de los casos ocurren simultáneamente con otras le siones neoplásicas

d) El tratamiento habitual es la amputación

e) Ninguna es correcta

CQ65-3. Es el tratamiento con menor tasa de recurrencia local del quiste óseo anuerismático:

a) Legrado e injerto óseos

b) Legrado y crioterapia

c) Aspiración

d) Termoterapia

e) Ninguno

CQ65-4. Tratamiento de elección para lesiones inaccesibles quirúrgicamente:

a) Crioterapia

b) Braquiterapia

c) Quimioterapia

d) Teleterapia

e) Ninguno
CQ65-5. El principal periodo de recurrencia es dentro de los:

a) Primeros doce meses

b) Primeros dos años

c) Primeros cinco años

d) Primeros diez años

e) Hasta en los primeros 20 años

CQ66. Paciente del sexo femenino de 57 años con lesión pélvica que en la imagen radiográfica es
compatible con condrosarcoma. Los estudios de extensión no revelan enfermedad en otros sitios de la
economía. La paciente tiene dolor importante en una escala visual análoga lo refiere de 9-10. La función es
limítrofe.

CQ66-1. El principal estudio, después de la placa simple, que ayudaría a usted a la toma de decisión
quirúrgica es:

a) TAC

b) Angiografía

c) USG

d) Centellografía ósea

e) Ninguno

CQ66-2. Es la variedad más frecuente de condrosarcoma:

a) Periférico

b) Central

c) Mesenquimal

d) De células pequeñas

e) Pigmentado

CQ66-3. En relación al condrosarcoma, la aseveración correcta es:

a) Los condrosarcoma centrales tienen un grado menor que las lesiones periféricas
b) La supervivencia a 10 años para los de bajo grado es de 17%

c) La supervivencia a 10 años para los de alto grado es de 87%

d) El régimen de platino-etopósido es una buena opción adyuvante con respuesta de hasta el


70%

e) Ninguna

CQ66-4. Para esta paciente la resección periacetabular de la pelvis sería un tratamiento adecuado, debido a
sus estudios, este tipo de cirugía se conoce como:

a) Hemipelvectomía externa

b) Hemipelvectomía interna

c) Desarticulación coxofemoral

d) Hemicorporectomía

e) Ninguna

CQ66-5. La supervivencia de esta paciente, reportándose un condrosarcoma central de bajo grado, es de :

a) 20 %

b) 40 %

c) 60 %

d) 80 %

e) 0 %

CQ67. Paciente del sexo masculino de 21 años de edad con tumor óseo femoral de tres meses de evolución.

CQ67-1. El primer estudio en solicitar sería:

a) Tomografía computada de la lesión

b) Radiografía simple

c) Centellografía ósea

d) Biometría hemática

e) Ninguno
CQ67-2. Son, según su estirpe, los tumores óseos más frecuentes:

a) Hematopóyeticos

b) Condrogénicos

c) Osteogénicos

d) Fibrogénicos

e) Vasculares

CQ67-3. Es la principal vía de diseminación de los tumores óseos:

a) Hematógena

b) Linfática

c) Transcortical

d) Transmedular

e) Extensión directa

CQ67-4. En la comparación del tratamiento local de los sarcomas óseos, cirugía conservadora vs
amputación, los resultados de la mayoría de los estudios indican:

a) Una ventaja para la cirugía radical (amputación) en términos de control local y supervivencia

b) Una ventaja para la cirugía conservadora en términos de control local y supervivencia

c) No hay diferencia entre ambos procedimientos en términos de recurrencia local ni


supervivencia

d) La cirugía conservadora sólo está indicada en caso de paliación

e) La cirugía radical sólo está reservada para los pacientes mayores de 60 años

CQ67-5. Es el tumor maligno primario óseo más frecuente:

a) Osteosarcoma

b) Condrosarcoma

c) Fibrosarcoma

d) Tumor de células gigantes


e) Tumor de Ewing

CQ68. Se le consulta como experto oncólogo, acerca de algunas lesiones óseas, por un médico radiólogo,
quien le interconsulta:

CQ68-1. La localización del adamantinoma es:

a) Diafisiaria

b) Metafisiaria

c) Epifisiaria

d) Metaepifisiaria

e) Ninguna

CQ68-2. La localización del osteoma osteide es:

a) Diafisiaria

b) Metafisiaria

c) Epifisiaria

d) Metaepifisiaria

e) Ninguna

CQ68-3. La localización del quiste óseo aneurismático es:

a) Diafisiaria

b) Metafisiaria

c) Epifisiaria

d) Metaepifisiaria

e) Ninguna

CQ68-4. La localización del condroblastoma es:

a) Diafisiaria

b) Metafisiaria
c) Epifisiaria

d) Metaepifisiaria

e) Ninguna

CQ68-5. La localización de displasia fibrosa es:

a) Diafisiaria

b) Metafisiaria

c) Epifisiaria

d) Metaepifisiaria

e) Ninguna

CQ69. Femenino de 37 años que se presenta por mastalgia de larga evolución, agudizada en los últimos 2
meses posterior a cambio de contraceptivo. En la exploración física usted sólo encuentra una placa de
mastopatía en el cuadrante superoexterno de la mama izquierda de 2 x 2 cm y una adenopatía axilar en nivel
I izquierdo de 1 x 1 cm.

CQ69-1. El primer estudio que solicita es:

a) Mastografía bilateral

b) Ultrasonido mamario bilateral

c) Mastografía y ultrasonido bilateral

d) Resonancia magnética

e) Biopsia por aspiración con aguja fina

CQ69-2. El estudio le reporta un área de microcalcificaciones en el cuadrante inferoexterno de la mama derecha de


8 x 8 mm, las cuales se hallan en medio de una placa de mastopatía que le impide definir detalles. El siguiente que
solicita es:

a) Gammagrama con sesta MIBI

b) Ultrasonido Doppler

c) Resonancia magnética

d) Cono de compresión
e) Cono de magnificación

CQ69-3. Este estudio le confirma que se trata de microcalcificaciones sospechosas de malignidad. Su


siguiente estudio es:

a) Biopsia guiada por estereotaxia

b) Citología por aspiración guiada por ultrasonido

c) Biopsia guiada por resonancia

d) Biopsia con mamotome guiada por mastografía

e) Gammagrama con sestaMIBI

CQ69-4. El reporte histopatológico es de hiperplasia ductal atípica. La posibilidad de que se encuentra un


carcinoma adyacente a la lesión es de:

a) 10%

b) 20%

c) 30%

d) 50%

e) 70%

CQ69-5. Usted le propone a la paciente:

a) Mastectomía total

b) Lumpectomía con disección axilar

c) Vigilancia estrecha

d) Excisión amplia del área

e) Tamoxifén

CQ70. Femenino de 32 años que se presenta para determinar su riesgo de presentar cáncer de mama. Como
antecedentes, una hermana con cáncer de mama a los 35 años, madre con cáncer de mama bilateral a los 39
años, tía abuela con cáncer epitelial de ovario a los 56 años.

CQ70-1. Usted considera que la paciente tiene antecedentes familiares que la colocan en el grupo de:

a) Alto riesgo para cáncer familiar


b) Mediano riesgo para cáncer esporádico

c) Bajo riesgo para cáncer familiar

d) Mediano riesgo para cáncer familiar

e) No tiene riesgo

CQ70-2. Ante esto usted evalúa la posibilidad de solicitar determinación de:

a) Mutaciones en p53

b) Mutaciones en BRCA1

c) Mutaciones en ATM

d) Mutaciones en BRCA2

e) Mutaciones en Rad 51

CQ70-3. En tanto tiene los resultados usted decide manejarla de acuerdo a su riesgo estimado, por lo que le
indica:

a) Mastografía bilateral

b) US transvaginal y CA 125

c) Autoexploración y ultrasonido mamario bilateral

d) Mastografía, US transvaginal y CA 125

e) Tamoxifén

CQ70-4. Ella le solicita información acerca de la posibilidad de mastectomía profiláctica, por lo que usted le
indica que el porciento de reducción del riesgo de cáncer de mama con este abordaje es de:

a) 100%

b) 10%

c) 50%

d) Mayor a 89%

e) Menor al 14%

CQ70-5. Usted debe indagar antecedentes étnicos:


a) Rusos

b) Alemanes

c) Negros

d) Judíos sefaradíes

e) Judíos ashkenazi

CQ71. Se presenta una paciente de 43 años que refiere la presencia de una masa en mama izquierda desde
hace 5 días. A la exploración física usted encuentra una masa dura de 1.5 x 2 cm en el cuadrante
superointerno. En la axila localiza una adenopatía de 1 x 1 cm en nivel I.

CQ71-1. De primera instancia usted piensa que se trata de un carcinoma con TNM:

a) T2, N0, Mx

b) T1b, N1, Mx

c) T1a, N0, Mx

d) T1c, N1, Mx

e) T3, N0, Mx

CQ71-2. La etapa clínica a la que pertenece es:

a) I

b) IIA

c) IIB

d) IIIA

e) IV

CQ71-3. Para corroborar su diagnóstico en ese momento decide:

a) Solicitar mastografía

b) Tomar biopsia por aspiración con aguja fina

c) Programar la excisión de la lesión

d) Dejar en vigilancia
e) Solicitar resonancia magnética

CQ71-4. Confirman un carcinoma canalicular. Decide realizar como estudios de extensión:

a) Tele de tórax

b) CA 15-3

c) Tomografía abdominal

d) Tele de tórax, ultrasonido hepático

e) Tele de tórax, ultrasonido hepático, gammagrama óseo

CQ71-5. En la mastografía se reportan microcalcificaciones difusas en 3 cuadrantes de la mama izquierda.


Usted propone:

a) Tamoxifén

b) Mastectomía radical clásica

c) Segmentectomía y disección radical de axila

d) Segmentectomía y ganglio centinela

e) Mastectomía radical modificada

CQ72. Femenino de 37 años en puerperio tardío que se presenta con cuadro de 10 días de evolución con
aumento de volumen de mama derecha, eritema, dolor leve. A la exploración física encuentra mama
eritematosa, ligero aumento en la temperatura local, zona nodular de 3 x 4 cm en región retroareolar,
adenopatía axilar de 1 x 1 cm.

CQ72-1. Su primera impresión es que se trata de:

a) Mastitis aguda con absceso retroareolar

b) Carcinoma inflamatorio

c) Linfangitis mamaria

d) Linfoma mamario

e) Mastitis granulomatosa

CQ72-2. Para confirmar su sospecha decide:


a) Solicita TAC de tórax

b) Solicita mastografía

c) Realiza biopsia por aspiración con aguja fina

d) Programa mastectomía radical clásica

e) Inicia manejo con antibióticos

CQ72-3. Se encuentra con una masa firme, decide entonces:

a) Solicitar mastografía

b) Iniciar antibióticos

c) Programa excisión de la lesión

d) Tomar biopsia incisional de piel

e) Dejar en observación

CQ72-4. Le reportan carcinoma canalicular infiltrante en linfaticos subdermicos. Su etapificación TNM es:

a) T2, N1, Mx

b) T4b, N1, Mx

c) T4d, N1, M1

d) T4d, N1, Mx

e) T2, N0, Mx

CQ72-5. Usted le indica a la paciente que debe tratarse de primera instancia con:

a) Tamoxifén

b) Mastectomía radical clásica

c) Mastectomía radical modificada

d) Quimioterapia

e) Radioterapia
CQ73. Femenino de 29 años que se presenta con 20 semanas de embarazo y masa en mama izquierda notada
por autoexploración 2 semanas antes. A la exploración física se encuentra mamas turgentes y
multinodulares, con nódulo dominante en cuadrante supero externo izquierdo de 4 x 6 cm, adenopatía axilar
de 1 x 1 cm.

CQ73-1. Su primera impresión es que puede tratarse de:

a) Galactocele

b) Carcinoma

c) Absceso

d) Mastitis

e) Mama accesoria

CQ73-2. Para confirmar su diagnóstico decide:

a) Solicitar mastografía

b) Solicitar ultrasonido

c) Tomar biopsia por aspiración con aguja fina

d) Tomar biopsia Tru-Cut

e) Iniciar antibióticos

CQ73-3. El reporte del estudio es de carcinoma canalicular infiltrante SBR 8. Usted lo etapifica como:

a) T3, N1, Mx

b) T4b, N1, Mx

c) T2, N2, Mx

d) T3, N2, Mx

e) T1, N1, Mx

CQ73-4. Usted decide manejarla con:

a) Vigilancia hasta el parto

b) Aborto terapéutico
c) Quimioterapia

d) Mastectomía radical modificada

e) Mastectomía radical clásica

CQ73-5. Posterior a este primer abordaje:

a) Quimioterapia

b) Radioterapia

c) Cirugía conservadora

d) Tamoxifén

e) Mastectomía radical modificada

CQ74. Femenino de 45 años que se presenta con una mastografía de control anual que presenta
microcalcificaciones poco definidas en un acúmulo de aproximadamente 2 cm en el cuadrante superoexterno
de la mama izquierda. A la exploración física usted sólo palpa áreas de mastopatía sin masas dominantes y
no hay adenomegalias.

CQ74-1. Su primer abordaje es:

a) Tomar biopsia por aspiración con aguja fina

b) Solicitar ultrasonido mamario

c) Programar segmentectomía

d) Solicitar cono de magnificación

e) Iniciar tamoxifén

CQ74-2. El estudio demuestra calcificaciones sospechosas de malignidad. Requiere determinar su estirpe


histológica. Usted considera como primera opción:

a) Segmentectomía

b) Excisión del área previo marcaje por estereotaxia

c) Mastectomía radical modificada

d) Biopsia por aspiración guiada por ultrasonido

e) Observación por 3 meses


CQ74-3. Al obtener la biopsia debe:

a) Cortarla

b) Tomar mastografía de la pieza para corroborar se retiró la lesión

c) Solicitar cultivo del tejido

d) Ponerla en formol y enviarla a estudio definitivo

e) Tomar una impronta

CQ74-4. El reporte definitivo es de carcinoma canalicular in situ de tipo cribiforme. Su propuesta a seguir
es:

a) Mastectomía radical modificada

b) Mastectomía radical clásica

c) Segmentectomía

d) Segmentectomía y disección radical de axila

e) Observación

CQ74-5. Como adyuvancia usted sugiere:

a) Tamoxifén

b) Quimioterapia

c) Radioterapia

d) Observación

e) Exemestane

CQ75. Femenino de 57 años a la que le está practicando una segmentectomía con disección radical de axila
por un tumor T2, N1, M0, tratado con quimioterapia neoadyuvante.

CQ75-1. Usted debe resecar con el segmento de mama los conductos galactóforos regionales. El número
total de estos es de:

a) 5
b) 7

c) 10 a 20

d) 25

e) 50

CQ75-2. Se dispone a resecar los ganglios axilares. El límite posterior está dado por:

a) Serrato anterior

b) Borde anterior del dorsal ancho

c) Escápula

d) Borde lateral del pectoral mayor

e) Paquete toraco-dorsal

CQ75-3. Se encuentra algunos ganglios altamente sugestivos de malignidad en el área de los ganglios de
Rotter. Estos se encuentran:

a) Entre el paquete toraco-dorsal y el nervio torácico largo

b) Sobre la vena mamaria externa

c) En el borde lateral del pectoral mayor

d) Entre el pectoral mayor y el menor

e) Cerca del ligamento de Halsted

CQ.75-4 Se presenta un accidente y tiene una laceración longitudinal de 1 cm en la vena axilar. Usted
procede a:

a) Ligarla

b) Repararla

c) Colocar un parche muscular

d) Pinzar y colocar ligadura

e) Llamar a un cirujano vascular

CQ75-5. Encuentra una adenopatía firmemente adherida al nervio respiratorio de Bell o torácico largo, por
lo que decide sacrificarlo para extirpar el ganglio. El defecto funcional que presentará es:
a) Debilidad a la abducción del brazo

b) Atrofia del pectoral mayor

c) Insuficiencia respiratoria

d) Hipo

e) Escápula alada

CQ76. Femenino de 19 años que se presenta con masa de crecimiento rápido, de 3 meses de evolución, con
erosión de la piel y sangrado profuso recurrente. A la exploración física encuentra masa violácea de 15 x 12
cm que prácticamente sustituye la mama izquierda, con un área ulcerada de 4 x 4 cm con sangrado profuso
al tacto, franca necrosis a la vista, cuatro adenopatías axilares blandas de 1 x 1 cm, no hay fijación a planos
profundos.

CQ76-1. Su primera impresión es que se trata de un carcinoma, al cual etapifica como:

a) T4b, N1, Mx

b) T4d, N2, Mx

c) T3, N1, Mx

d) T4c, N1, Mx

e) T2, N1, Mx

CQ76-2. Decide tomar biopsia para determinar su estirpe presentando sangrado profuso. Ante la posibilidad
de que tome sólo necrosis del tumor, reseca una adenopatía axilar que en estudio histopatológico se reporta
como hiperplasia. El reporte de la biopsia probablemente será de:

a) Carcinosarcoma

b) Tumor phyllodes

c) Angiosarcoma

d) Carcinoma metaplásico

e) Fibroadenoma gigante juvenil

CQ76-3. La biopsia reporta un angiosarcoma de alto grado. Su abordaje terapéutico es:

a) Mastectomía radical clásica

b) Mastectomía radical modificada


c) Quimioterapia

d) Radioterapia

e) Quimioterapia con radioterapia concomitantes

CQ76-4. La mejor adyuvancia probada en estos casos es:

a) Quimioterapia

b) Radioterapia

c) Tamoxifén

d) Ninguna

e) Quimioterapia con radioterapia concomitantes

CQ76-5. Los primeros sitios de diseminación de este tumor son:

a) Hígado

b) Pulmón

c) Pulmón y hueso

d) Ganglios axilares

e) Hueso y páncreas

CQ77. Paciente sexo masculino de 36 años de edad el cual presenta una lesión pigmentada de 1.3 cm en
tórax anterior en la línea media de cuatro años de evolución. La lesión fue escindida totalmente hace un mes
y el reporte de patología informa: Melanoma de diseminación superficial, ulcerado con patrón de
crecimiento vertical, Clark IV e índice de Breslow de 1.0 mm, con márgenes a 5 mm de la lesión. La
exploración física no revela lesión, sólo una cicatriz de 1.5 cm en tórax en región esternal. Ambas axilas y el
cuello son negativos.

CQ77-1. La decisión de tratamiento de este paciente incluye:

a) Excisión de la cicatriz y ganglio centinela

b) Sólo ganglio centinela

c) Excisión de la cicatriz y disección electiva axilar

d) Sólo disección electiva axilar


e) Excisión de la lesión y disecciones electivas axilar y cervical

CQ77-2. La estadificación de este paciente, con refrencia al T, sería:

a) T1 a

b) T2 a

c) T1 b

d) T2 b

e) T3 a

CQ77-3. Además de los estudios preoperatorios de este paciente, los estudios de extensión necesarios
incluyen:

a) Tomografía tórax y cuello

b) Linfocentellografía, pruebas de función hepática y DHL

c) Linfocentellografia, tele de tórax, pruebas de función hepática y DHL

d) Téle de tórax y DHL

e) Tomografía de tórax, cuello y craneo, así como linfocentellografía

CQ77-4. En el supuesto de que los ganglios fueran negativos con cualquiera de las opciones de tratamiento,
como tratamiento adyuvante usted indicaría:

a) Interferon alfa 2b a altas dosis

b) Interferon alfa 2b a bajas dosis

c) Protocolo de Vacunas BCG

d) Ningún tratamiento adyuvante

e) Sólo radioterapia al primario

CQ77-5. Cuál sería la supervivencia esperada a 5 años para este paciente:

a) 20%

b) 40%

c) 50%
d) 85%

e) 100%

CQ78. Paciente del sexo femenino de 64 años de edad, se presenta con una lesión plantar de 3 cm de
diámetro compatible clínicamente con Melanoma, se toma biopsia incisional la que reporta un melanoma
acral lentiginoso con Breslow de 3.6 mm ulcerado. La exploración física revela adenopatía inguinal dura,
movible de 1.5 cm.

CQ78-1. Los estudios de extensión necesarios para esta paciente incluyen:

a) Ninguno

b) Tele de tórax, pruebas de función hepática, DHL, USG hepático y tomografía de pelvis

c) Tele de tórax, pruebas de función hepática, DHL, y tomografía tórax y craneo

d) Sólo tele de tórax

e) Sólo tomografía pélvica

CQ78-2. La conducta terapéutica óptima, con relación al primario, sería:

a) Resección de la lesión con márgenes de 5 cm

b) Resección de la lesión con márgenes de 2 cm

c) Resección de la lesión con márgenes de 1 cm y radioterapia posoperatoria

d) Resección de la lesión con márgenes de 5 cm y radioterapia posoperatoria

e) Resección de la lesión con márgenes de 2 cm y radioterapia posoperatoria

CQ78-3. La conducta terapéutica óptima, con relación a la ingle, sería:

a) Radioterapia 50-70 Gys

b) Disección inguinofemoral

c) Disección inguinopélvica

d) Ganglio centinela

e) Quimioterapia-radioterapia concomitante
CQ78-4. La supervivencia de este paciente a 5 años sería aproximadamente del:

a) 5-10%

b) 30-40%

c) 70-80%

d) 80-90%

e) 100%

CQ78-5. En el seguimiento de este paciente se reporta DHL de 589 U/L, usted:

a) Solicita estudios de extensión y con base en los resultados actuaría

b) Solicita un perfil de hepatitis, situación común en melanoma

c) Solicita una angiografía hepática, porque la frecuencia de hemangiomas hepáticos es alta en


los pacientes con melanoma

d) No le da relevancia

e) Le solicita gamagrafia hepática

CQ79. Paciente del sexo masculino de 45 años de edad a quién se le resecó una lesión de 2 cm en antebrazo
derecho con reporte de melanoma acral lentiginoso con Breslow de 1.8 mm no ulcerado, márgenes de
resección a 2 cm de la lesión. La exploración física revela exclusivamente una cicatriz de 4 cm y la axila es
negativa. Los estudios de extensión son negativos. Fue llevado a ganglio centinela y el reporte es de ganglio
centinela con metástasis por melanoma en 1/1 ganglio.

CQ79-1. La conducta a seguir es:

a) Radioterapia axilar

b) Interferón alfa 2b a altas dosis

c) Interferón alfa 2b a bajas dosis

d) Disección axilar complementaria

e) Ninguno

CQ79-2. La posibilidad de que el ganglio centinela fuera positivo en este caso era del:

a) 10%

b) 20%
c) 70%

d) 80%

e) 90%

CQ79-3. La supervivencia de este paciente a 5 años es de:

a) 20%

b) 40%

c) 60%

d) 80%

e) 100%

CQ79-4. El marcador inmunohistoquímico de mayor utilidad en melanoma, debido a sus altos índices de
sensibilidad y especificidad es:

a) Proteína S-100

b) HLA

c) Catepsina D

d) Citoqueratinas

e) HMB-45

CQ79-5. La posibilidad de que mediante RT-RCP un ganglio previamente reportado como negativo a HE e
inmunohistoquímica en los pacientes que recurren en el área linfática evaluada con el ganglio centinela es
de:

a) 17%

b) 32%

c) 50%

d) 82%

e) 100%
CQ80. Paciente del sexo femenino de 62 años de edad, se presenta con adenopatía de 2 cm en nivel IV de
cuello derecho. Fue llevada a biopsia excisional con reporte de ganglio linfático con metástasis por
melanoma. La exploración física no revela lesión dérmica alguna.

CQ80-1. Además del laboratorio básico, los estudios de extensión que usted solicitaría incluyen:

a) TAC de cabeza y cuello, triple endoscopia, tele de tórax y ultrasonografia hepática

b) Tele de tórax y USG hepático

c) Triple endoscopía

d) Triple endoscopía y USG hepático

e) TAC cabeza y cuello, triple endoscopía y USG pélvico

CQ80-2. Los estudios de extensión que usted solicitó, no revelan otro sitio de enfermedad, por lo que usted
trataría a esta enferma de la siguiente manera:

a) Disección radical clásica de cuello derecho

b) Disección radical modificada de cuello derecho

c) Disección supraomohioidea bilateral

d) Disección supraomohioidea de cuello derecho

e) Disección posterolateral bilateral

CQ80-3. El reporte de la disección le revela metástasis en 5/24 ganglios linfáticos, usted agregaría a su
tratamiento:

a) Radioterapia

b) Interferón

c) Vacunas

d) Quimioterapia

e) Ninguno

CQ80-4. En general el pronóstico de los pacientes con melanoma de piel de la cabeza y cuello es peor que el
de las extremidades, pero en qué localización de la cabeza y cuello se observa el peor pronóstico:

a) Cuero cabelludo
b) Pabellón auricular

c) Cuello

d) Mentón

e) Párpado

CQ80-5. El margen quirúrgico es de suma importancia, pero la función y cosmética es una limitante
relevante en cabeza y cuello, por lo que se ha sugerido su uso en melanoma;

a) Radioterapia

b) Cirugía micrográfica de Mohs

c) Crioterapia

d) Vacunas

e) Ninguna

CQ81. Paciente del sexo femenino que fue llevada a resección de melanoma plantar y ganglio centinela, el
cual fue positivo por lo que se llevó a disección complementaria ganglionar, el reporte de patología informa
metástasis en 3 de 17 ganglios linfáticos.

CQ81-1. La estadificación con relación al N sería:

a) N1 a

b) N1 b

c) N2 a

d) N2 b

e) N3

CQ81-2. Son indicaciones de radioterapia adyuvante al área ganglionar, tras disección ganglionar:

a) Extensión extracapsular

b) Más de dos ganglios linfáticos afectados

c) Un ganglio metástasico

d) Más de cinco ganglios afectados

e) No se recomienda su uso
CQ81-3. La frecuencia de enfermedad local y metastásica en pacientes con lesiones con Breslow mayores de
4 mm es aproximadamente del:

a) 60 y 70%

b) 10 y 20%

c) 10 y 70%

d) 30 y 90%

e) 3 y 7%

CQ81-4. Es el sitio más común de presentación en el melanoma de mucosas de cabeza y cuello:

a) Cavidad nasal

b) Paladar

c) Trígono retromolar

d) Mucosa oral

e) Nasofaringe

CQ81.5. Es la supervivencia a cinco años esperada para este paciente:

a) 20%

b) 40%

c) 60%

d) 80%

e) 100%

CQ82. Una mujer de 33 años de edad se presenta con el antecedente familiar de melanoma y a la
exploración física se encuentran múltiples nevos melanocíticos adquiridos en número mayor a 100, todos de
aspecto benigno.

CQ82-1. El riesgo de desarrollar melanoma en esta paciente es:

a) Igual a la población general


b) Menor a la población general

c) 100 veces mayor

d) 20 veces mayor

e) No se relacionan

CQ82-2. Es factor de riesgo para desarrollar melanoma:

a) Tipo de piel V y VI de Fitzpatrick

b) Nevo congénito gigante

c) Nevo azul

d) Bloqueadores solares

e) Ninguno

CQ82-3. Es característica clínica asociada con un mayor riesgo de malignidad en una lesión pigmentada:

a) Simetría

b) Bordes bien definidos

c) Color homogéneo

d) Diámetro mayor de 6 mm

e) Ninguno

CQ82-4. Entre los nevos que deben ser resecados por el riesgo de malignización, aunque clínicamente sean
benignos, se incluyen:

a) Los nevos en áreas expuestas al sol

b) Los nevos en sitios de difícil autoexploración

c) Los nevos en extremidades

d) Los nevos en cabeza y cuello

e) Todos los nevos deben escindirse

CQ82-5. La frecuencia de seguimiento en estos pacientes deberá realizarse:


a) Anual

b) Semestral

c) Trimestral

d) Bianual

e) No requiere seguimiento

CQ83. Paciente del sexo masculino de 56 años de edad quien se presenta tras haberle sido resecada una
lesión encefálica con reporte de metástasis de melanoma. La TAC posoperatoria no revela ninguna otra
lesión.

CQ83-1. Los estudios de extensión mínimos incluyen:

a) Tele de tórax, ultrasonografia hepática y pélvica

b) Tele de tórax, ultrasonografia hepática y DHL

c) Sólo tele de tórax

d) Sólo TAC cráneo

e) Sólo USG hepático

CQ83-2. El tratamiento de elección en este paciente sería:

a) Radioterapia a cráneo total

b) Resección del lecho quirúrgico y radioterapia adyuvante

c) Resección del lecho quirúrgico y quimioterapia

d) Quimioterapia

e) Resección del lecho quirúrgico

CQ83-3. Es el agente quimioterápico con mayor efectividad en lesiones encefálicas por melanoma:

a) Platino

b) Ciclofosfamida

c) DTIC

d) Tomozolamida
e) Interferón alfa 2b

CQ83-4. Llevando a cabo una conducta terapéutica adecuada cuál sería la expectativa de vida:

a) 5 años

b) 3 años

c) 1 año

d) 6 a 9 meses

e) 3 meses

CQ83-5. En el caso de lesiones metastásicas encefálicas resecables y únicas el tratamiento de elección sería:

a) Teleterapia

b) Radiocirugía

c) Cirugía

d) Quimioterapia

e) QT-RT concomitante

CQ84. En un paciente del sexo femenino de 24 años de edad con antecedente de resección quirúrgica de un
nevo con reporte de melanoma in situ en muslo derecho.

CQ84-1. El margen adecuado de resección en las lesiones in situ es de:

a) 5 cm

b) 3 cm

c) 2 cm

d) 1 cm

e) 0.5 cm

CQ84-2. Es el principal factor pronóstico en las lesiones delgadas:

a) Clark
b) Breslow

c) Ulceración

d) Regresión

e) Índice mitótico

CQ84-3. La supervivencia a 5 años sería de:

a) 99%

b) 70%

c) 50%

d) 30%

e) 10%

CQ84-4. Se consideran nevos atípicos aquellos que:

a) Son más de 50

b) Son asimétricos, heterogéneos y de diámetro mayor de 6 mm

c) Son más de 100

d) Son simétricos, homogéneos y menores de 6 mm

e) Son congénitos

CQ84-5. Es el riesgo de malignización de un nevo en la clasificación de Kraemer D2:

a) 0%

b) 20%

c) 40%

d) 80%

e) 100%

CQ85. Paciente del sexo masculino de 45 años de edad con lesión pigmentada de 0.3 cm subungueal en
índice derecho, de bordes mal definidos, heterogénea y asimétrica.
CQ85-1. La conducta a seguir sería:

a) Observación

b) Biopsia

c) Amputación

d) Solicitar marcadores tumorales

e) Ninguno

CQ85-2. En el supuesto de tener un reporte previo de melanoma acral lentiginoso con Breslow de 3.1 mm,
no ulcerado, usted:

a) Realizaría la amputación de la falange distal

b) Realizaría la amputación de la falange media y distal

c) Realizaría amputación metacarpofalángica

d) Tomaría nueva biopsia

e) Ninguno

CQ85-3. La exploración física no revela adenopatías axilares, usted:

a) Realizaría ganglio centinela

b) Realizaría disección electiva axilar

c) La observaría

d) Daría radioterapia electiva

e) Ninguno

CQ85-4. En este caso y sin evidencia de afección ganglionar que tratamiento adyuvante recomendaría:

a) Quimioterapia

b) Radioterapia

c) Interferón a bajas dosis

d) Interferón a altas dosis

e) Ninguno
CQ85-5. En el seguimiento, este paciente desarrolla múltiples lesiones en tránsito en la extremidad afectada,
los estudios de extensión no revelan enfermedad en otros sitios usted indicaría:

a) Interferón a altas dosis

b) Perfusión aislada de extremidad con interferón y DTIC

c) Perfusión aislada de extremidad con melfalán y factor de necrosis tumoral

d) Perfusión aislada de extremidad con interferón y factor de necrosis tumoral

e) Radioterapia

CQ86. Paciente del sexo masculino con lesión pigmentada y biopsia que reporta melanoma nodular en tórax
con Breslow de 2.1 mm, Clark IV, ulceración y datos de regresión.

CQ86-1. Es el tipo más común de melanoma en México:

a) Diseminación superficial

b) Lentigo maligno

c) Acral lentiginoso

d) Nodular

e) Neurotrópico

CQ86-2. Son indicaciones de realización de ganglio centinela:

a) Breslow menor de 1 mm y Clark V

b) Breslow menor de 1 mm y Clark II

c) Melanoma in situ

d) Breslow manor de 1 mm sin ulceración

e) Ninguno

CQ86-3. El margen de resección en este paciente es de:

a) 2 cm

b) 4 cm
c) 5 cm

d) 0.5 cm

e) 1 cm

CQ86-4. La linfocentellografía preoperatoria le revela como sitios de drenaje primario la axila e ingle
derechas, usted:

a) Realizaría disección electiva axilar e inguinofemoral

b) Realizaría ganglio centinela de ambos drenajes

c) Realizaría sólo la disección de la axila

d) Realizaría sólo la disección de la ingle

e) Lo dejaría en observación

CQ86-5. Son los pacientes a los que la disección electiva ha demostrado un beneficio:

a) Mayores de 60 años, con lesiones ulceradas y Breslow entre 1 y 4 mm

b) Menores de 60 años con lesiones ulceradas y Breslow entre 1 y 4 mm

c) Menores de 60 años sin lesiones ulceradas y Breslow entre 1 y 2 mm

d) Mayores de 60 años con lesiones ulceradas y Breslow entre 1 y 4 mm

e) Mayores de 60 años sin lesiones ulceradas y Breslow entre 1 y 2 mm

CQ87. Un paciente de 65 años de edad acude a la consulta con algunos meses de evolución presentando
crecimiento lentamente progresivo de ganglios linfáticos del cuello y una biometría hemática que reporta
hemoglobina de 11.2 g/dl, leucocitos de 43 X 109/L con 88% de linfocitos y plaquetas de 106 x 109/L. A
pesar de todo ha continuado haciendo su vida diaria normal, niega fiebre, aunque ocasionalmente tiene
diaforesis nocturna profusa y ha perdido unos 3 K en los últimos 8 a 10 meses, aparentemente porque ha
estado vigilando lo que come. Al examen físico se confirma la presencia de ganglios linfáticos cervicales de
1.5 a 2 cm bilaterales. No se encuentran otras alteraciones.

CQ87-1. Cuál sería su diagnóstico.

a) Leucemia linfocítica aguda

b) Leucemia granulocítica crónica

c) Leucemia linfocítica crónica


d) Leucemia mielocítica aguda

e) Reacción leucemoide

CQ87-2. Qué tratamiento propondría a este paciente.

a) Vigilancia

b) Fludarabina

c) Melfalán y prednisona

d) Clorambucil

e) 6 mercaptopurina

CQ87-3. Algunos meses después de que el paciente se encuentra bajo su cuidado, se presenta quejándose de
debilidad extrema en la última semana. Una biometría hemática muestra hemoglobina de 6.7 g/dl, leucocitos
de 49 x 109/L y plaquetas de 106 x 109/L. Cuál sería su principal sospecha diagnóstica.

a) Progresión de la enfermedad

b) Anemia hemolítica autoinmune

c) Toxicidad medicamentosa

d) Síndrome de Richter

e) Segunda neoplasia

CQ87-4. Cómo trataría ahora a este paciente.

a) Prednisona

b) Aumentar dosis de quimioterapia

c) Transfusión

d) Disminuir dosis de quimioterapia

e) Cambiar de agentes quimioterapéuticos

CQ87-5. En un paciente con este diagnóstico, qué medida es útil para prevenir infecciones.

a) Antibióticos profilácticos

b) Inmunizaciones
c) Gama globulina intravenosa

d) Inmunizaciones y antibióticos profilácticos

e) Inmunizaciones y gama globulina intravenosa

CQ88. Una mujer de 60 años, lo consulta debido a que desde hace algunos meses ha notado distensión abdominal
progresiva, saciedad temprana y fatiga. Una biometría hemática mostró Hemoglobina de 13.1 g/dL, 86 x 109/L con
(65% de segmentados, 4% de bandas, 3% de metamielocitos, 4% de eonsinófilos, 6% de basófilos, 2% de
monocitos y 18% de linfocitos), y 786 x 109/L plaquetas. El examen físico muestra exclusivamente bazo a 15 cm
por debajo del reborde costal.

CQ88-1. Cual sería su diagnóstico clínico.

a) Leucemia neutrofílica crónica

b) Leucemia granulocítica crónica

c) Síndrome de Richter

d) Leucemia mielocítica aguda

e) Linfoma esplénico

CQ88-2. Cuál de las siguientes pruebas sería útil para confirmar su sospecha diagnóstica:

a) Biopsia de bazo

b) Fosfatasa alcalina de los leucocitos

c) Deshidrogenasa láctica

d) Citometría de flujo de sangre o médula ósea

e) Mieloperoxidasa de los leucocitos

CQ88-3. En caso de confirmar su sospecha diagnóstica, cuál sería el medicamento de elección para tratar a
esta paciente:

a) Imatinib

b) Clorambucil

c) Melfalán

d) Fludarabina
e) Prednisona

CQ88-4. En qué circunstancia indicaría transplante alogénico de médula ósea para esta paciente.

a) Ante la primera evidencia de progresión

b) Siempre que tenga un donador compatible

c) Una vez lograda la remisión

d) Un año después de haber iniciado el tratamiento

e) Nunca

CQ88-5. Cuál es la complicación más frecuente en pacientes como ésta.

a) Progresión a fase blástica

b) Conversión a síndrome de Richter

c) Ruptura esplénica

d) Leucostasis

e) Trombosis venosa profunda

CQ89. Un paciente de 18 años de edad se presenta con 15 días de fiebre, diaforesis nocturna y ataque al
estado general. El examen físico lo muestra pálido y algunos ganglios cervicales se encuentran aumentados
de tamaño. Los exámenes de laboratorio muestran hemoglobina de 5 .3 g/dl, leucocitos 28.5 x 109/L y
plaquetas 23 x 109/L. Un aspirado de médula ósea revela 80% de blastos. La tinción de PAS es positiva y la
de mieloperoxidasa es negativa. Por citometría de flujo los blastos son CD10, CD19, CD34, DLA DR y TdT
positivos.

CQ89-1. Cuál es su impresión diagnóstica:

a) Leucemia aguda M3

b) Leucemia aguda pre B temprana

c) Leucemia aguda T

d) Leucemia aguda M4

e) Leucemia aguda pre T

CQ89-2. Cuál de los siguientes estudios es indispensable en este paciente:


a) Ultrasonido testicular bilateral

b) Determinación de hierro sérico

c) Serología para el VIH

d) Cariotipo

e) Serología para EBV

CQ89-3. Qué medicamentos utilizaría en el tratamiento inicial de este paciente.

a) Prednisona, vincristina y doxorrubicina

b) Etopoósido, ciclofosfamida y prednisona

c) Citarabina, asparaginasa y ciclofosfamida

d) Etopoósido, citarabina y prednisona

e) Doxorrubicina, etopoósido y prednisona

CQ89-4. Porciento de pacientes con esta enfermedad que sobreviven a largo plazo.

a) Menos del 10

b) 20 a 40

c) 40 a 60

d) 60 a 80

e) Más del 80

CQ89-5. Cuál de las siguientes alteraciones citogenéticas representaría un factor pronóstico desfavorable en
este paciente.

a) T(15;17)

b) T(8;21)

c) T(9;22)

d) Inv 16

e) T(8;14)
CQ90. Un paciente de 21 años de edad tiene unos días de fiebre, tos y ortopnea. Muestra ingurgitación
yugular bilateral y taquicardia. La biometría hemática revela 13.1 g/dl de hemoglobina, leucocitos 26 x
109/L y plaquetas 96 x 109 /L. La radiografía de tórax muestra ensanchamiento mediastinal. La médula ósea
muestra 65% de blastos.

CQ90-1. Qué marcadores de superficie esperaría que los blastos de la médula ósea de este paciente
presentaran.

a) CD19 y CD20

b) CD10 y CD15

c) CD3 y CD7

d) CD22 y CD23

e) Inmunoglobulinas

CQ90-2. Le reportan que el ácido úrico de este paciente se encuentra elevado, qué otros exámenes de
laboratorio necesita.

a) Enzimas hepáticas

b) Gasometría arterial

c) Electrólitos séricos

d) Deshidrogenasa láctica

e) Examen general de orina

CQ90-3. Qué medidas serían de utilidad en este paciente.

a) Transfusión de plaquetas

b) Hidratación y alcalinización de orina

c) Vigilar tensión arterial

d) Citorreducción rápida

e) Administración de oxígeno

CQ90-4. Cómo trataría el síndrome de compresión de vena cava superior.

a) Radioterapia y qimioterapia concomitantes

b) Quimioterapia
c) Radioterapia

d) Cirugía

e) Cirugía y quimioterapia

CQ90-5. Cuál es la complicación inmediata más frecuente en estas circunstancias.

a) Ruptura esofágica

b) Insuficiencia renal aguda

c) Síndrome de coagulación intravascular diseminada

d) Lisis tumoral

e) Insuficiencia respiratoria progresiva del adulto

CQ91. Un paciente de 36 años tiene algunos días de fatiga, palidez progresiva y fiebre. Al examen físico se
encuentra con palidez generalizada de tegumentos. No hay ganglios linfáticos aumentados de tamaño y
tampoco se encuentra hepatomegalia o esplenomegalia. La biometría hemática muestra hemoglobina de 8
g/dl, leucocitos de 175 x 109 /L con 70% de blastos y plaquetas de 36 x 109 /L. La médula ósea muestra
blastos con cuerpos de Auer prominentes. La tinción de mieloperoxidasa es intensamente positiva en los
blastos.

CQ91-1. Qué exámenes de laboratorio son indispensables en la evaluación de este paciente.

a) Examen general de orina

b) Enzimas hepáticas

c) Pruebas de coagulación

d) Ácido láctico en suero

e) Gasometría arterial

CQ91-2. Cuál de las siguientes alteraciones citogenéticas establecería el diagnóstico de leucemia aguda
promielocítica.

a) T(9;22)

b) T(8;21)

c) T(8;14)

d) T(15;18)
e) T(15;17)

CQ91-3. Qué agentes utilizaría para obtener remisión en este paciente, de no confirmarse el diagnóstico de
leucemia aguda promielocítica.

a) Vincristina y prednisona

b) Ciclofosfamida y L-asparaginasa

c) Mitoxantrona y vincristina

d) Citarabina y daunorrubicina

e) Melfalán y prednisona

CQ91-4. Cuál de los siguientes representa un factor de mal pronóstico en este paciente.

a) La tinción de mieloperoxidasa

b) La cuenta de leucocitos

c) La edad

d) La cifra de hemoglobina

e) La cuenta de plaquetas

CQ91-5. Qué tratamiento de consolidación, en este grupo de pacientes, ha demostrado prolongar la duración
de la remisión.

a) Dosis altas de citarabina

b) Dosis altas de etopoósido

c) Dosis altas de antracíclicos

d) Mantenimiento por 2 años

e) Mantenimiento por 3 años

CQ92. Masculino de 45 años de edad que ingresa al servicio por presentar obstrucción nasal, epistaxis
recurrente, tumor en fosa nasal derecha, dolor, pérdida de peso, astenia, adinamia de seis meses de
evolución. A la exploración física presenta asimetría facial, borramiento del surco nasogeniano, mucosa
ulcerada e infiltración de piel y partes blandas. La TAC demuestra actividad tumoral en fosa nasal derecha
nasofaringe con infiltración ósea al antro maxilar derecho, fosa infratemporal, fosa pterigomaxilar y pared
medial de la órbita derechos.
CQ92-1. El paciente se clasifica como:

a) Etapa II

b) Etapa 0

c) Etapa III

d) T2 N0 M0

e) Etapa IV

CQ92-2. Una biopsia incisional de fosa nasal reporta carcinoma epidermoide moderadamente diferenciado.
Anote su conducta de manejo.

a) Cirugía radical

b) Cirugía citó reductora

c) Quimioterapia de inducción platino y 5 FU 3 ciclos

d) Radioterapia

e) Cirugía citó reductora+radioterapia

CQ92-3. El paciente presenta respuesta parcial (60%) al término de la quimioterapia y a la exploración física
se palpa adenopatía de 3 cm en hemicuello derecho nivel II. Anote su conducta terapéutica:

a) Únicamente biopsia y corroborar diagnostico histológico

b) Radioterapia de consolidación

c) Maxilectomía radical y disección radical de cuello

d) Radioterapia y quimioterapia

e) Maxilectomía radical y disección supraomohoidea

CQ92-4. Los hallazgos quirúrgicos demuestran actividad tumoral con erosión ósea del piso y cara medial de
la orbita, cara posterior del antro maxilar. Anote su conducta.

a) Resección limitada del maxilar

b) Maxilectomía con exenteración orbitaria

c) Maxilectomía incluyendo piso de la orbita con preservación de globo ocular


d) Maxilectomía medial

e) Cirugía Degloving

CQ92-5. Los hallazgos patológicos reportan bordes quirúrgicos positivos y 3 ganglios con ruptura capsular,
anote su conducta.

a) Vigilancia

b) Radioterapia a lecho quirúrgico

c) Quimioterapia y radioterapia concomitante

d) Ampliar márgenes quirúrgicos y radioterapia

e) Radioterapia a lecho quirúrgico y cuello

CQ93. Paciente de 52 años de edad que presenta dificultad para la deglución y fonación, halitosis, perdida
de peso de 10 kg y dolor en piso de boca de 10 meses de evolución. A la exploración presenta tumor
ulcerado y exofítico en piso de boca con infiltración al borde lateral derecho de la lengua, de 4 cm y
adenopatía de 2 cm nivel I de hemicuello derecho. La biopsia reporta carcinoma epidermoide.

CQ93-1. Anote su conducta:

a) Quimioterapia con 5FU/platino y radioterapia concomitante

b) Radioterapia preoperatoria

c) Cirugía radical y disección radical de cuello

d) Biopsia del primario y disección radical de cuello

e) Cirugía radical y disección supraomohoidea

CQ93-2. Según la pregunta 1 sugiere la etapificación siguiente:

a) T2 N1 M0

b) Etapa II

c) T2 N2 M0

d) Etapa III

e) T3 N2 M0
CQ93-3. El paciente concluye tratamiento con respuesta completa y a los 4 meses presenta lesión nodular en
piso de boca de 0.5 cm. Anote su conducta:

a) Quimioterapia de segunda línea

b) Tratamiento paliativo

c) Sobredosis con radioterapia

d) Biopsia excisional

e) Draquiterapia

CQ93-4. Se documenta recurrencia. Anote su conducta terapéutica:

a) Cirugía de rescate

b) Inmunoterapia

c) Sobredosis de RT

d) Inmunoterapia de segunda línea

e) Vigilancia

CQ93-5. Anote las cifras de sobrevida a 5 años sin actividad tumoral, en una lesión como la descrita.

a) 20%

b) 45%

c) 60-70%

d) 80-90%

e) 50%

CQ94. Mujer de 35 años que presenta aumento de volumen en región preauricular, con crecimiento
progresivo, dolor de moderada intensidad de 6 meses de evolución y a la exploración presenta tumor de
consistencia sólida de 3 cm en glándula parótida y adenopatía de 1.5 cm nivel II.

CQ94-1. Qué estudios de extensión son esenciales para el diagnóstico:

a) Sialografía, Baaf, telerradiografía de tórax

b) Tac, Usg y Baaf


c) Tac, Baaf, tele tórax

d) Usg y Sialografía

e) Biopsia por Tru-cut y Tac

CQ94-2. Este paciente corresponde a la etapa clínica siguiente:

a) Etapa I

b) T2 N0 M0

c) T3 N1 M0

d) T2 N1 MO

e) T2 N2 MO

CQ94-3. La biopsia por aspiración reporta carcinoma mucoepidermoide de alto grado. Anote su conducta
terapéutica:

a) Nodulectomía

b) Parotidectomía total y DSOH

c) Parotidectomía total y DRM de cuello

d) Parotidectomía superficial y DSOMH

e) Parotidectomía Total y Radioterapia a cuello

CQ94-4. Los hallazgos de patología reportan carcinoma mucoepidermoide de alto grado y dos ganglios
positivos con ruptura capsular. Anote su conducta:

a) Radioterapia

b) Quimioterapia

c) Radioterapia + quimioterapia

d) Únicamente vigilancia

e) Braquiterapia

CQ94-5. Anote la cifra de sobrevida a 5 años sin actividad tumoral:

a) 25%
b) 50%

c) 70%

d) 92%

e) 10%

CQ95. Masculino de 34 años de edad sin antecedentes patológicos ni familiares de importancia, acude por
presentar masa central en cuello de 6 meses de evolución más adenopatías cervicales de lado izquierdo en
niveles III, IV y V. El resto del examen físico normal.

CQ95-1. Cuál de los siguientes métodos enumerados a continuación escogería como el ideal para
diagnóstico.

a) TAC de cuello

b) Gamagrafía de tiroides

c) BAF de la lesión o de una adenopatía

d) Nasofibrolaringoscopía

e) Esofagograma

CQ95-2. Al realizar los estudios completos se demuestra que es una masa dependiente de la glándula
tiroides. La biopsia del ganglio linfático reporta cáncer medular de tiroides. Cuál tratamiento aconsejaría
usted.

a) Tiroidectomía subtotal

b) Hemitiroidectomía

c) Tiroidectomía total más disección central ganglionar

d) Tiroidectomía total más disección central más disección radical modificada de lado izquierdo

e) Tiroidectomía total más disección radical modificada bilateral de cuello más disección central

CQ95-3. El reporte definitivo de la pieza quirúrgica es de cáncer medular de tiorides de 10 x 8 x 6 cm. Con
metástasis a 3 ganglios. El paso siguiente será:

a) Observación y controles con calcitonina

b) Radioterapia adyuvante

c) Yodo131 adyuvante
d) Quimioterapia

e) Ninguna

CQ95-4. Uno de los marcadores que le puede servir para normar la conducta en estos pacientes y sus
familiares es:

a) Determinar la proteína p53

b) Determinar el oncogen MYC

c) Determinar protooncogén RET

d) Determinar BRCA1

e) Determinar Paratohormona

CQ95-5. Si el marcador genético buscado es positivo en el paciente que conducta se debe seguir en los
familiares.

a) Ninguna

b) Observación

c) Si es positiva en el familiar realizar tiroidectomía total antes de los 30 años

d) Realizar hemitiroidectomía antes de los 8 años

e) Realizar en los hijos con el marcador positivo tiroidectomía total antes de los seis años de edad

CQ96. Masculino de 61 años de edad. Diabético e hipertenso. Fumador desde los 20 años de edad a razón de
dos cajetillas diarias. Acude por disfonía de seis meses de evolución. Al estudio de laringoscopia se
identifica un tumor que infiltra cuerda vocal izquierda, comisura anterior y tercio anterior de cuerda vocal
derecha. En la TAC se aprecia destrucción del cartílago tiroides. Ganglios negativos.

CQ96-1. Según la AJCC a qué etapa clínica corresponde:

a) I

b) II

c) III

d) IV

e) No es clasificable
CQ96-2. Qué tratamiento sería el más indicado:

a) Cirugía conservadora de laringe

b) Laringectomía total

c) Radioterapia

d) Quimioterapia más radioterapia

e) Laringectomía total más radioterapia

CQ96-3. En qué porciento este paciente tendrá ganglios positivos al cuello:

a) Menos del 10

b) 11 al 20

c) 21 al 30

d) 31 al 40

e) 41 o más

CQ96-4. El manejo del cuello sería con:

a) Observación

b) Radioterapia 40Gy

c) Disección radical clásica de cuello

d) Disección radical modificada bilateral de cuello

e) Disección selectiva de los niveles II-III-IV

CQ96-5. La sobrevida de estos pacientes a 5 años es:

a) 90 a 95%

b) 70 a 89%

c) 50 a 69%

d) 30 al 49%

e) Menos del 20%


CQ97. Paciente masculino de 62 años de edad con antecedentes de tabaquismo, tiene un area de
leucoplaquia en borde izquierdo de lengua, de 2 cm de diámetro con elevación verrugosa, adenomegalia de 1
cm de diámetro en región submaxilar izquierda.

CQ97-1. La exploracion clínica inicial debe incluir:

a) Broncoscopía, esofagoscopía y nasofaringoscopía

b) Palpación de la lesión y biopsia

c) Aspirado con aguja fina de Adenomegalia Submaxilar

d) Aspirado con aguja fina de la lesión

e) Inspección y tomografia axial de piso de boca y lengua

CQ97-2. La leucoplaquia con hiperplasia verrugosa se asocia a carcinoma en:

a) 40% de casos aproximadamente

b) 5% de casos aproximadamente

c) Menos de 90% de casos aproximadamente

d) Raramente

e) 10% de casos, aproximadamente

CQ97-3. El tratamiento de este paciente debe consistir en:

a) Carotenos

b) Nistatina y aseo bucal

c) Dieta con cítricos y carotenos

d) Eliminación de factores irritativos

e) Biopsia Excisional bajo anestesia con estudio transoperatorio

CQ97-4. En caso de reportar carcinoma en el estudio definido, con tumor in situ de 1 cm de extensión se
debe:

a) Realizar excisión amplia de la zona (1cm de margen)


b) Administrar radioterapia posoperatoria

c) Practicar disección supraomohioidea del cuello

d) Mantener en observación

e) Practicar excisión amplia y disección supraomohioidea

CQ97-5. En caso de reportar carcinoma invasor en el estudio definitivo se debe:

a) Realizar excisión amplia y disección supraomohioidea del cuello, con estudio transoperatorio

b) Realizar disección supraomohioidea del cuello y estudio transoperatorio

c) Administrar radioterapia posoperatoria

d) Realizar disección modificada del cuello

e) Observación

CQ98. Femenino de 85 años de edad con diagnóstico de carcinoma escamoso de piso de boca. La paciente
tiene una lesión de 6 cm de diámetro con fijación a cuerpo mandibular. Presenta además adenomegalia
yugulo digástrica de 1 cm de diámetro, móvil.

CQ98-1. En esta paciente se debe completar su estudio clínico con:

a) Resonancia magnética nuclear

b) Broncoscopía

c) Pruebas de función respiratoria, valoración cardiovascular con ecocardiograma para valorar


fracción de eyección

d) Prueba de funcion respiratoria

e) Resonancia magnética nuclear y broncoscopía

CQ98-2. El tratamiento quirúrgico en este caso sería:

a) Resección de piso de boca, mandibulectomía marginal de ser técnicamente posible, disección


del cuello

b) Resección de piso de boca, mandibulectomía segmentaria y disección del cuello

c) Resección de piso de boca, hemimandibulectomía y disección del cuello

d) Resección de piso de boca y biopsia de adenomegalia cervical


e) Mandibulotomía lateral, resección de piso de boca y cierre con placas de titanio. Disección del
cuello

CQ98-3. El tratamiento reconstructivo se realizará en caso de reconstrucción mandibular con:

a) Colgajo libre de cresta ilíaca

b) Colgajo de peroné

c) Colgajo de pectoral mayor

d) Placa de titanio y colgajo pectoral mayor

e) Colgajo libre de escápula

CQ98-4. El tratamiento adyuvante en este caso:

a) Depende de los hallazgos patológicos posoperatorios

b) Está indicado de manera absoluta con radioterapia

c) Consiste en quimioterapia

d) No está indicado

e) Está indicado de manera absoluta con radioterapia sólo al cuello

CQ98-5. El pronóstico de sobrevida libre de enfermedad a 5 años en esta paciente es:

a) 100%

b) 90%

c) 70%

d) 40%

e) 30%

CQ99. Femenino de 28 años de edad con tumor en piso de boca sin ulceraciones de 3 cm de diámetro, de
crecimiento progresivo en tres meses; hay trismus y dolor discreto a la palpacion de la lesión. La tomografia
axial con dentascan no muestra invasión mandibular, aunque la lesión por clínica está parcialmente fija a la
mandíbula.

CQ99-1. Dentro de la historia clínica son de importancia los antecedentes siguientes:


a) Historia de HIV

b) Historia de tabaquismo y alcoholismo

c) Historia de alcoholismo

d) Historia de dentaduras mal aplicadas

e) Antecedentes familiares

CQ99-2. En consulta usted realizaría:

a) Biopsia por sacabocado

b) Biopsia por aguja fina

c) Laringoscopía directa

d) Nasofaringoscopía

e) Biopsia incisional con anestesia local

CQ99-3. Obtiene un resultado de carcinoma escamoso el procedimiento a realizar es:

a) Resección de cuello supraomohioidea

b) Mandibulotomía lateral y resección de piso de boca, disección radical de cuello

c) Mandibulotomía media y resección marginal de mandíbula, disección supraomohioidea del


cuello, sección de pterigoideo Interno

d) Disección radical del cuello y resección de piso de boca

e) Radioterapia

CQ99-4. La radioterapia+quimioterapia simultánea en este caso:

a) Demuestra sobrevida superior a cirugía+radioterapia adyuvante

b) Permite preservar estética y función (Trismus)

c) Tiene morbilidad intermedia comparada con otros métodos de tratamiento

d) Se administraría sólo en caso de negativa del paciente a un procedimiento quirúrgico

e) Se administraría previo a procedimiento quirúrgico para reducir extensión de cirugía


CQ99-5. Se recibe reporte de carcinoma adenoideo quístico, el tratamiento es:

a) Radioterapia

b) Quimioterapia+cirugía

c) Resección de piso de boca, resección de pterigoideos, por mandibulotomía media

d) Disección del cuello, resección de piso de boca, resección de pterigoideos por mandibulotomía
media

e) Mandibulectomía+resección de piso de boca y disección del cuello

CQ100. Paciente masculino de 72 años de edad, cardiópata y neumópata, con lesión de 2 cm en borde
izquierdo de lengua, con placa de eritroleucoplaquia en la superficie. Cuello sin adenomegalias.

CQ100-1. En este paciente son antecedentes de importancia:

a) Historia de alcoholismo

b) Historia de tabaquismo

c) Historia de herpes bucal

d) Historia de alcoholismo y tabaquismo

e) Historia de HIV

CQ100-2. En este paciente se recomienda para completar estudio diagnóstico:

a) Nasofaringolaringoscopía

b) Esofagoscopía

c) Laringoscopía y broncoscopía

d) Triple endoscopía

e) Laringoscopía indirecta

CQ100-3. Se recomienda completar estudios de pruebas de función respiratoria, valoración cardiológica y


ecocardiograma. Los resultados son limítrofes y el resultado de biopsia es carcinoma escamoso. Se
recomienda:

a) Tratamiento con radioterapia

b) Tratamiento con cirugía y radioterapia adyuvante


c) Quimioterapia y radioterapia secuenciales

d) Quimioterapia y radioterapia simultaneas

e) Tratamiento con cirugía

CQ100-4. En caso de un procedimiento quirúrgico cual estaría indicado:

a) Hemiglosectomía

b) Hemiglosectomía y disección modificada del cuello tipo III

c) Hemiglosectomía y disección supraomohioidea con estudio transoperatorio+disección completa


del cuello Tipo III

d) Resección amplia de la lesion

e) Disección bilateral de cuello y glosectomía

CQ100-5. El tratamiento con radioterapia adyuvante estaría indicado en caso de:

a) Todos los casos

b) En caso de 1 metástasis ganglionar en la disección del cuello supraomohioidea

c) En caso de tumor con penetración de más de 2 mm

d) En caso de 2 o más metástasis en la disección supraomohioidea

e) Sólo en caso de márgenes positivos en tumor primario

CQ101. Paciente femenino de 58 años de edad con tumoración de 3 cm en piso de boca, con dolor local, la
biopsia reporta hiperplasia melanocítica atípica.

CQ101-1. Usted practica:

a) Nueva biopsia

b) Excisión amplia de la lesión

c) Tumorectomía

d) Resección de tumor y hemiglosectomía

e) Administra radioterapia
CQ101-2. Si usted tuviera diagnóstico de melanoma de mucosa el manejo del cuello en esta enferma sería:

a) Disección supraomohioidea

b) Disección radical del cuello

c) Observación

d) Radioterapia

e) Excisión de glandula submaxilar

CQ101-3. El sitio más frecuente de melanoma de cavidad bucal es:

a) Piso de boca

b) Base de lengua

c) Paladar

d) Encía

e) Trígono retromolar

CQ101-4. El crecimiento de un melanoma de mucosa es:

a) Superficial

b) Vertical

c) Exofítico

d) Verrucoso

e) Ulcerativo

CQ101-5. La sobrevida en este paciente a 5 años se espera de:

a) 100 %

b) 25 %

c) 10 %

d) 60 %

e) 80 %
CQ102. Paciente masculino de 28 años de edad, tabaquismo y alcoholismo negados, tiene lesion ulcerada de
5 cm en borde derecho de lengua con invasion a piso de boca incipiente tiene adenomegalia cervical derecha
de 1 cm; la biopsia se reporta positiva a carcinoma escamoso.

CQ102-1. El padecimiento en un paciente joven y sin antecedentes de tabaquismo y alcoholismo.

a) Tiene un mejor pronóstico que en pacientes de más de 45 años

b) Tiene un pronóstico peor que en pacientes de más de 45 años

c) Tiene un pronóstico igual que en pacientes de más de 45 años

d) Tiende a presentarse con enfermedad Metastásica

e) Tiene tendencia a recurrencia local baja

CQ102-2. El manejo inicial debe realizarse con:

a) Quimioterapia

b) Cirugía

c) Radioterapia

d) Quimioterapia+radioterapia

e) Interleucinas

CQ102-3. El tratamiento del cuello en este caso debe ser:

a) Disección radical unilateral (derecha)

b) Disección supraomohioidea del cuello derecho

c) Disección radical modificada tipo III del cuello derecho y disección supraomohioidea izquierda

d) Disección radical modificada tipo III derecha

e) Observación

CQ102-4. La posibilidad de metástasis en este caso es a nivel ipsilateral:

a) Más del 90%

b) 0%

c) 15%
d) 5%

e) 35%

CQ102-5. El procedimiento reconstructivo recomendable en este paciente, posterior a resección de piso de


boca y hemiglosectomía derecha es:

a) Aplicación de colgajo nasogeniano y cierre primario de lengua

b) Aplicacioón de colgajo pectoral mayor+

c) Injerto libre de piel

d) Colgajo microvascular radial

e) Colgajo microvascular de recto anterior

CQ103. Paciente femenino de 58 años de edad con lesión de 5 cm en piso de boca derecho.El reporte
patológico es de carcinoma escamoso poco diferenciado. Se somete a radioterapia como tratamiento inicial
(primario y cuello), obteniendo respuesta del 100%. Ocho meses después se presenta con recurrencia
tumoral de 2 cm en piso de boca derecho, adherido a la mandíbula. Tiene adenomeglia de 1 cm ipsilateral en
cuello. No evidencia de enfermedad metastásica. La biopsia es positiva para carcinoma escamoso.

CQ103-1. La estadificación original en este caso con adenomegalias negativas cervicales es:

a) T2N0M0

b) T3 N0 M0

c) T1 N1a M0

d) T2 N1a M0

e) T3 N1a M0

CQ103-2. La supervivencia libre de enfermedad en este grupo de enfermos se encuentra en:

a) 80% a 5 años

b) 90% a 5 años

c) 60% a 5 años

d) 40% a 5 años

e) 30% a 5 años
CQ103-3. La recurrencia en este caso debe manejarse con:

a) Resección del tumor primario y disección radical modificada del cuello, ipsilateral

b) Resección del tumor primario y disección supraomohioidea del cuello

c) Resección del tumor primario y excisión de adenomegalia cervical

d) Resección del tumor primario y disección supraomohioidea bilateral

e) Resección del tumor primario y disección radical modificada del cuello bilateral (Tipo III)

CQ103-4. La ortopantomografía y la TAC con dentascan no demuestran invasión mandibular, usted:

a) Realiza resección de tumor primario y hemimandibulectomía

b) Realiza resección de tumor primario y mandibulectomía marginal

c) Realiza resección de tumor primario y mandibulectomía segmentaria

d) Realiza sólo la resección del tumor primario

e) Realiza resección de tumor primario y de acuerdo a los hallazgos transoperatorios una


mandibulectomía segmentaria o marginal

CQ103-5. La reconstrucción en caso de realizar mandibulectomía segmentaria (defecto de 4 cm) es con:

a) Colgajo microvascular de radial

b) Colgajo pediculado de pectoral mayor

c) Colgajo de trapecio

d) Colgajo de cresta ilíaca

e) Colgajo de peroné

CQ104. Masculino de 70 años de edad, con antecedentes de tabaquismo y alcoholismo positivos. Se


presenta con historia de 1 año de evolución con tumor de piso de boca de 12 cm de diámetro, izquierdo, con
infiltración a base de lengua y trígono retromolar. El enfermo tiene adenomegalia cervical izquierda de 4 cm
de diámetro, parcialmente fija a planos profundos. Invasión mandibular no demostrada, aunque la lesión se
encuentra fija al ramus mandibular. El paciente es sometido a biopsia con reporte de carcinoma escamoso
moderadamente diferenciado. Estudios negativos a enfermedad metastásica.

CQ104-1. Este paciente porta un estadio de la clasificación T, N, M:


a) T3 N2A M0

b) T4 N2B M0

c) T4 N2A M0

d) T4 N3 M0

e) T4 N1 M0

CQ104-2. En este enfermo el pronóstico es determinado por:

a) La velocidad clínica del crecimiento tumoral, extensión primaria del padecimiento y calidad del
tratamiento oncológico

b) Determinación de P 53 en células tumorales

c) Criterios de Jacobson

d) Clasificación T, N, M.

e) Extensión primaria del padecimiento y calidad del tratamiento oncológico

CQ104-3. Si el tumor en este paciente rebasa la línea media en base de lengua usted propondría:

a) Administración de quimioterapia neoadyuvante y consolidación de acuerdo a respuesta

b) Administración de quimioterapia y radioterapia simultáneas

c) Cirugía resectiva y radioterapia adyuvante

d) Radioterapia

e) Manejo con interleucinas y consolidación de acuerdo a respuesta

CQ104-4. La respuesta al tratamiento es del 90% quedando un residual de 2 cm en base de lengua, y en


cuello de 1 cm, usted:

a) Propone glosectomía total y disección del cuello izquierdo supraomohioidea

b) Propone hemiglosectomía, resección parcial de base de lengua. y disección del cuello


izquierda por vía transbucal

c) Propone mandibulotomía media, hemiglosectomía con resección parcial de base de lengua y


disección del cuello izquierda

d) Propone resección de base de lengua, local , transoral y disección supraomohioidea izquierda

e) Propone mandibulotomía lateral, resección de la mitad izquierda de base de lengua y disección


modificada del cuello izquierdo
CQ104-5. Usted administra quimioterapia neoadyuvante en este enfermo, con una respuesta del 90%, en
tumor primario (residual en base de lengua) y regional. La cirugía que propondría es:

a) Glosectomía total, resección de piso de boca izquierdo, con disección modificada del cuello
izquierdo, laringectomía y reconstrucción

b) Hemiglosectomía izquierda, resección de piso de boca izquierdo y disección modificada del


cuello izquierdo

c) Mandibulotomía lateral, resección de mitad izquierda de base de lengua y disección modificada


del cuello izquierdo

d) Glosectomía total, resección de piso de boca izquierdo, con disección supraomohioidea del
cuello izquierdo, laringectomía y reconstrucción

e) Resección parcial de base de lengua, transbucal, y disección supraomohioidea del cuello


izquierdo

CQ105. Un varón de 17 años sin antecedentes relevantes se presenta con historia de seis meses de evolución
caracterizada por epistaxis y obstrucción facial. La rinoscopía revela un tumor polipoideo rojo-violáceo que
obstruye la fosa nasal derecha. El cuello es clínicamente negativo a actividad tumoral.

CQ105-1. El diagnóstico clínico que usted establece es:

a) Papiloma invertido

b) Pólipo antrocoanal

c) Hemangiopericitoma

d) Rabdomiosarcoma

e) Nasoangiofribroma

CQ105-2. El estudio inicial más útil en el diagnóstico y evaluación de extensión de la enfermedad es:

a) La biopsia incisional en el consultorio

b) La TC con medio de contraste

c) La IRM

d) La angiografía

e) Las placas de senos paranasales


CQ105-3. El estudio solicitado muestra un tumor vascular de bordes empujantes limitado a la fosa nasal y
nasofaringe, el tratamiento incluye:

a) Retirar la exposición a alergenos

b) Radioterapia preoperatoria y cirugía

c) Maxilectomía medial

d) Quimioterapia de inducción y consolidación de acuerdo a respuesta

e) Excisión con o sin embolización preoperatoria

CQ105-4. Si el diagnóstico histopatológico correspondiera a rabdomiosarcoma embrionario usted


recomendaría el siguiente tratamiento:

a) Quimioterapia neoadyuvante y consolidación de acuerdo a la respuesta

b) Cirugía y radioterapia

c) Radioterapia radical

d) Radioterapia y cirugía

e) Cirugía solamente

CQ106. Una mujer de 49 años portadora de artritis reumatoide acude con historia de 8 meses de evolución
caracterizada por crecimiento no doloroso difuso de parótida derecha. La exploración corrobora crecimiento
bilateral predominantemente derecho. No hay adenopatías cervicales pero sí deformidad en las
articulaciones de las falanges.

CQ106-1. El estudio diagnóstico que usted solicita es:

a) Biopsia por aspiración con aguja fina

b) Biopsia incisional

c) Parotidectomía superficial con estudio transoperatorio

d) Biopsia de la mucosa oral

e) Imagen por resonancia de parótidas

CQ106-2. El reporte del estudio solicitado reporta: “compatible con lesión linfoepitelial”. El tratamiento
subsecuente consiste:

a) Radioterapia posoperatoria
b) Tratamiento antiinflamatorio

c) Ningún tratamiento

d) Completar parotidectomía radical y radioterapia posoperatoria

e) Quimioterapia adyuvante

CQ107. Un varón de 60 años se presenta a su consulta con un padecimiento de 4 años de evolución,


caracterizado por obstrucción nasal persistente unilateral. La exploración física revela un tumor de aspecto
polipoideo en la cara lateral de la fosa nasal. La TC muestra una lesión que refuerza en forma moderada con
el medio de contraste, ubicada en la pared medial del seno maxilar. Erosiona la pared medial del seno
maxilar y moldea el hueso de las celdillas etmoidales anteriores.

CQ107-1. El diagnóstico más probable es:

a) Carcinoma sinunasal indiferenciado

b) Papiloma invertido

c) Nasoangiofibroma

d) Carcinoma epidermoide

e) Melanoma de mucosas

CQ107-2. La biopsia incisional transnasal corrobora su impresión clínica, el tratamiento propuesto por usted
es el siguiente:

a) Maxilectomía total con exenteración orbitaria

b) Maxilectomía medial y etmoidectomía

c) Radioterapia preoperatoria y maxilectomía medial

d) Interferón a altas dosis

e) Excisión previa embolización

CQ108. Un varón de piel blanca y de 76 años de edad, campesino y portador de enfermedad de Alzheimer
acude con un tumor ubicado en la región parótidea izquierda que ha crecido en 3 meses hasta alcanzar un
tamaño de 5 x 4 cm. Es de consistencia dura y está fijo a planos subyacentes. Hay paresia de la rama
temporal y zigomática del nervio facial. Tiene múltiples queratosis actínicas en la piel y cicatrices. Tiene en
la región temporal cerca de la comisura externa un área con piel atrófica y telangiectasias. El cuello no
muestra adenopatías. Usted realiza una biopsia por aspiración con aguja fina que reporta: “maligno:
carcinoma”.
CQ108-1. Ud. establece el siguiente diagnóstico:

a) Carcinoma mucoepidermoide de alto grado

b) Carcinoma epidermoide metastásico a ganglios parotideos probable primario en piel

c) Tumor mixto maligno

d) Carcinoma de células escamosas primario de la parótida

e) Linfoma de parótida

CQ108-2. La TC muestra un tumor irregular poco definido confinado a la parótida, usted propone el
tratamiento siguiente:

a) Parotidectomía total con resección del nervio facial

b) Parotidectomía total luego de parotidectomía superficial más estudio transoperatorio,


identificación del nervio facial, probable resección de la rama temporal y zigomática y
reconstrucción nerviosa inmediata, tarsorrafía y disección supraomohioidea

c) Parotidectomía total con resección del nervio facial, disección radical clásica del cuello y
radioterapia posoperatoria

d) Radioterapia paliativa

e) Quimioterapia de inducción y consolidación de acuerdo a respuesta

CQ108-3. El reporte de patología es corrovorado a carcinoma, márgenes de resección libres y 18 ganglios sin
metástasis, el manejo subsecuente consiste en:

a) Radioterapia al hecho quirúrgico (área preparotidea)

b) Radioterapia al sitio del primario y el cuello

c) Quimioterapia adyuvante

d) Ningún tratamiento

e) Quimiorradioterapia posoperatoria

CQ109. Una mujer de 25 años acude a su consulta con un padecimiento de 6 meses de evolución
caracterizado por obstrucción nasal progresiva, rinorrea hialina y sanguinolenta, recientemente se agrega
epifora y deformidad de la región maxilar y malar derecha. Niega diplopía; sin embargo, el globo ocular está
desplazado hacia arriba y en sentido lateral. La TC muestra un tumor dependiente del antro maxilar que
rompe la pared anterior e involucra tejidos blandos, rompe la pared posterolateral y se extiende a la fosa
infratemporal y fosa pterigopalatina, rompe en piso de la órbita pero no parece haber involucro de grasa
periorbitaria ni recto inferior. La pared medial del antro también está rota y el tumor ocupa la fosa nasal
hasta contactar con el tabique nasal. El cuello es clínica y tomográficamente negativo, la tele de tórax es
normal, usted practica una biopsia transnasal y es reportado una carcinoma adenoideoquístico grado II.

CQ109-1. En estas condiciones el tumor corresponde al estadio:

a) Etapa clínica I por T1, N0, M0

b) Etapa clínica II por T2, N0, M0

c) Etapa clínica III por T3, N0, M0

d) Etapa clínica IV por T4, N0, M0

e) No es estadificable

CQ109-2. La propuesta de tratamiento incluye:

a) Maxilectomía radical con exenteración orbitaria y reconstrucción con colgajo de recto


abdominal microvascularizado

b) Maxilectomía total con preservación del contenido orbitario

c) Radioterapia radical

d) Quimioterapia de inducción y consolidación según respuesta

e) Quimioterapia y radioterapia concomitantes

CQ109-3. Posterior al tratamiento existe una respuesta parcial, el tumor ahora está confinado a la
infraestructura. Cuál es el tratamiento que usted propone.

a) Maxilectomía radical con exenteración orbitaria

b) Maxilectomía parcial con preservación de la órbita

c) Maxilectomía total con exploración del contenido orbitario

d) Quimioterapia paliativa

e) Ningún tratamiento

CQ110. Una mujer de 50 años, acude a usted con un tumor en apariencia submucoso que desplaza en
sentido medial la pared lateral de la orofaringe y deforma el paladar blando. El tumor es por demás
asintomático y tiene 4 años de evolución con lento crecimiento. El resto de la exploración física es
irrelevante.
CQ110-1. Usted considera el diagnóstico siguiente:

a) Cáncer de amígdala

b) Schwanoma del espacio parafaringeo

c) Tumor del lóbulo profundo de la parótida, probablemente adenoma pleomorfo

d) Paraganglioma

e) Cualquiera de los 3 últimos

CQ110-2. El estudio inmediato de mayor utilidad en el diagnóstico es:

a) Biopsia incisional transoral

b) Biopsia dirigida con aguja de núcleo

c) TC contrastada

d) Angiografía

e) Gamagrama con galio

CQ110-3. En el supuesto que haya solicitado un estudio de imagen, usted encuentra un tumor en el espacio
preestiloioideo de bordes empujantes que refuerza moderadamente con el medio de contraste, que involucra
el túnel estilo mandibular y que desplaza los vasos hacia atrás. Su primera consideración sería:

a) Tumor del lóbulo profundo de la parótida, probablemente un adenoma pleomorfo

b) Paraganglioma

c) Schwanoma

d) Carcinoma metastático

e) Linfoma

CQ110-4. Si un tumor de características semejantes, con bordes nítidos se encontrara en el espacio


posestiloioideo, su primera consideración sería:

a) Tumor del lóbulo profundo de la parótida, probablemente un adenoma pleomorfo

b) Paraganglioma

c) Schwanoma

d) Carcinoma metastático
e) Linfoma

CQ110-5. Si el mismo tumor reforzara en forma importante con el medio de contraste y desplaza la carótida
en sentido anterior y lateral, usted consideraría inicialmente:

a) Tumor del lóbulo profundo de la parótida, probablemente un adenoma pleomorfo

b) Paraganglioma

c) Schwanoma

d) Carcinoma metastático

e) Linfoma

CQ111. Una mujer de 30 años, acude con un padecimiento de 4 meses de evolución, ella ha tenido dolor en
la región parietooccipital, ocasionalmente obstrucción nasal de predominio derecho, escurrimiento hialino y
sanguinolento a través de las narinas y recientemente se agregó amaurosis derecha. Niega otras
manifestaciones en el interrogatorio dirigido. La rinoscopia no muestra mayores datos, el cuello es negativo.

CQ111-1. El diagnóstico más probable es:

a) Cáncer de nasofaringe

b) Meningioma en el canal óptico

c) Tumor de celdillas etmoidales anteriores

d) Cancer del seno esfenoidal

e) Sinusitis y absceso cerebral

CQ111-2. Cual de los siguientes estudios indicaría el objeto de documentar en forma precisa la extensión de
la enfermedad:

a) Nasofibroscopía con toma de biopsias

b) IRM de cráneo

c) TAC de senos paranasales

d) USG del globo ocular y fluoroangiografía de retina

e) Tomografía con emisión de positrones


CQ111-3. Asumamos que el tumor penetró la fosa media, involucrando el seno cavernoso, probablemente
tendría además las siguientes manifestaciones:

a) Oftalmoplejía e hipoestesia en territorio de la segunda rama del trigémino

b) Deterioro de las funciones mentales superiores

c) Desprendimiento de retina

d) Dolor sinusal

e) Edema de papila

CQ111-4. El tratamiento que usted propondría, sobre la base de los datos señalados previamente, sería:

a) Cirugía

b) Braquiterapia

c) Teleterapia

d) Radio cirugía

e) Quimioterapia y radioterapia concomitantes

CQ112. Un varón de 47 años se presenta con una lesión ulcerada en paladar duro. Él señala que desde hace
7 años tenía una lesión en el mismo sitio que no producía dolor sólo sensación de cuerpo extraño, ésta se
había mantenido estable pero recientemente creció rápidamente y desarrolló una lesión ulcerada central.
Observa una lesión predominantemente submucosa pero con úlcera central limpia de 3 x 4 cm. El resto de la
exploración es irrelevante.

CQ112-1. El diagnóstico clínico más probable es:

a) Carcinoma epidermoide de paladar

b) Tumor maligno de glándulas salivales menores

c) Melanoma amelánico

d) Linfoma angiocéntrico

e) Sarcoma

CQ112-2. Señala que hace 5 años se practicó biopsia con reporte de “tumor benigno: adenoma pleomorfo”
razón por lo que ignoró la lesión, ahora tiene una biopsia reciente con diagnóstico de carcinoma de células
ductales, usted considera que el diagnóstico es consistente con la entidad siguiente:
a) Adenoma pleomorfo

b) Tumor mixto maligno

c) Carcinoma ex adenoma pleomorfo

d) Carcinoma en adenoma pleomorfo

e) Carcinosarcoma

CQ112-3. De acuerdo a su diagnóstico, el tratamiento inmediato que recomendaría sería el siguiente:

a) Palatectomía

b) Radioterapia

c) Quimioterapia

d) Interferón Alfa2 B

e) Anticuerpos anti receptor para el factor de crecimiento epidérmico y radioterapia concurrente

CQ112-4. Si la biopsia fue generosa y el reporte de patología fuera de tejido necrótico, le sugeriría el
diagnóstico de:

a) Linfoma angiocéntrico

b) Proceso infeccioso

c) Tumor benigno

d) Melanoma

e) Sarcoma

CQ113. Un varón de 58 años acude con un padecimiento de seis meses de evolución caracterizado por
obstrucción nasal progresiva y ocasional escurrimiento hialino teñido con sangre a través de la narina
izquierda. Ha acudido con otro médico quién indicó una TC de fosas nasales, senos paranasales y cráneo.
Existe una lesión en el extremo superior de la fosa nasal, que parece erosiona la lámina cribosa.

CQ113-1. Usted realiza una biopsia que reporta neoplasia de células pequeñas y redondas que forma
seudorrosetas. En estas condiciones usted favorece el diagnóstico siguiente:

a) Melanoma de mucosas

b) Rabdomiosarcoma
c) Estesioneuroblastoma

d) Craneofaringioma

e) Linfoma

CQ113-2. Con el fin de confirmar el diagnóstico, usted solicita realizar las siguientes tinciones de
inmunohistoquímica:

a) Cromogranina y sinaptofisina

b) HMB 45 y S100

c) ALC y CD 45

d) Vimentina y mioglobina

e) Citoqueratina

CQ113-3. Si el tumor se encuentra confinado a fosas nasales corresponde a un estadio Kadish:

a) A

b) B

c) C

d) D

e) Ninguno de los anteriores

CQ113-4. En estas condiciones el tratamiento inicial de elección es:

a) Resección craneofacial

b) Radioterapia

c) Quimioterapia de inducción

d) Quimioterapia y radioterapia

e) Cirugía endoscópica seguida de radioterapia

CQ114. Un varón de 80 años, diabético y con antecedente de infarto agudo del miocardio hace 5 meses, se
presenta con una lesión de dos años de evolución a nivel del ángulo de la mandíbula de 2 cm de diámetro. El
tumor es parcialmente móvil y renitente. No hay lesiones en cavidad oral u orofaringe, la
nasofibrolaringoscopía es normal. Se practica una biopsia por aspiración con aguja fina con reporte de:
cistadenoma papilar linfomatoso. Indica un gammagrama con pertenectato de tecnecio que reporta una zona
hipercaptante en el sitio correspondiente al tumor en cuestión.

CQ114-1. A este tumor se le conoce comúnmente con el siguiente epónimo:

a) Tumor de Godwin

b) Tumor de Warthin

c) Tumor de Askin

d) Tumor de células de Paget

e) Tumor Gorlin–Goltz

CQ114-2. El riesgo de bilateralidad es de:

a) 1 %

b) 10 %

c) 20 %

d) 30 %

e) 40 %

CQ114-3. Usted recomendaría el siguiente tratamiento:

a) Radioterapia

b) Parotidectomía radical con excisión del nervio facial

c) Quimioterapia y radioterapia concomitantes

d) Observación

e) Tratamiento con esteroides

CASOS CLÍNICOS ONCOLOGÍA MÉDICA

CM1. Paciente de 18 años de edad con antecedentes de importancia: criptorquidea derecha resto sin
importancia, Su padecimiento de 6 meses de evolución con tumor en testículo derecho, es manejado como
orquitis, hace 3 meses presenta tos y disnea de medianos esfuerzos asi como dolor lumbar, Exploración física:
Tumor en testículo derecho de 15 cm, en abdomen gran masa que ocupa su totalidad, Exámenes de laboratorio:
DHL 5000, ast 89, alt 104, fa 145, depuración de creatinina de 120 mL x min. resto normal, tomografia
abdominal con gran masa retroperitoneal, hidronefrosis derecha y tres metástasis hepáticas no mayores de 3
cm cada una, Rx de tórax con gran masa en mediastino y múltiples metástasis pulmonares, AFP 12,000,
fracción beta 5000.

CM1-1. Porciento aproximado en el sitio del testículo no descendido con más posibilidades de desarrollar
cáncer:

a) Inguinal 5

b) Abdominal 5

c) Abdominal e inguinal 10

d) Abdominal 20

e) Inguinal 40

CM1-2. En relación a la clasificación del grupo colaborador internacional de tumores germinales se clasifica
como:

a) Intermedio riesgo por AFP mayor de 10,000 pero menor de 20,000

b) Alto riesgo por Fracción Beta de más de 1000

c) Alto riesgo por metástasis pulmonares

d) Alto riesgo por AFP mayor de 10 000 y DHL mayor de 10 veces su valor normal

CM1-3. Que manejo da usted:

a) Orquiectomía y de acuerdo a resultado histológico tratamiento con QT

b) Quimioterapia con carboplatino ante la hidronefrosis

c) No espero resultado de histología y administro QT dosis estándar

d) Ante tal volumen tumoral y esperarando lisis tumoral severa lo hidrato al menos 48 h e inicio
QT sustituyendo platino por carboplatino

CM1-4. Si el paciente recibe dos ciclos de quimioterapia lográndose sólo estabilización, cuál sería la
conducta a seguir:

a) Dosis altas con trasplante de celulas tallo periféricas

b) Esquema de segunda línea con taxanos más platino

c) Esquema con ifosfamida (VIP o VeIP) con dosis de ifosfamida de 800 mg /m 2 días 1–5
d) Esquema con Ifosfamida (VIP O VeIP) con dosis de ifosfamida de 1.2 gr/m 2 por 5 días

e) Sólo manejo paliativo con etopósido bucal ante el fracaso a manejo de primera línea

CM1-5. El porciento de supervivencia libre de enfermedad a 5 años de este paciente con la quimioterapia de
primera línea es del:

a) 60

b) 80

c) 10

d) 90

e) 40

CM2. Femenino de 65 años de edad con 2 hijas de 40 y 45 años, con antecedente de ser diabetica de larga
evolución. Su padecimeinto de un año de evolución con la presencia de tumoración en glandula mamaria
izquierda y retracción del pezón, se realiza mastografia que al ser sospechosa se realiza BAAF con informe de
cancer ductal infiltrante, se porcede a mastectomía radical modificada obteniendose informe histopatológico de:
primario de 2 x 2 cm, SBR suma de 3, receptores de estrógenos 34 fml y progesterona de 25 fm, Her 2 Neu por
inmunohistoquímica (+), 25 ganglios negativos, P53 negativo, exámenes de laboratorio y Rx de tórax normales.

CM2-1. Riesgo de las hijas de tener cáncer de mama:

a) Tres veces más que la población sin antecedentes

b) Ocho veces más que la población sin antecedentes

c) Igual que la población sin antecedentes

d) Cinco veces más que la población sin antecedentes

e) Ninguna de las anteriores

CM2-2. El sistema de gradacion de Bloom–Richardson (SBR) modificado por Nottingham toma en cuenta:

a) Invasión vascular y linfática, pleomorfismo nuclear

b) Invasion neural, vascular y linfatica

c) Formación tubular, pleomorfismo nuclear y número de mitosis

d) Número de mitosis, pleomorfismo nuclear e invasion vascular

e) Formación tubular, numero de mitosis y formación de conductos


CM2-3. En esta paciente el factor pronóstico con mayor valor predicitivo es:

a) Tamaño del tumor

b) SBR

c) Ganglios negativos

d) P54

e) Receptores hormonales

CM2-4. La conducta posterior es:

a) Radioterapia más tamoxifén por 3 años

b) Tamoxifén por 5 años

c) Ningún tratamiento

d) CMF x 6 ciclos y tamoxifén por 5 años

e) AC x 4 ciclos y tamoxifén por 3 años

CM2-5. Estudio más importante de seguimiento en esta paciente:

a) Ca 15.3

b) Rx de tórax

c) Ultrasonido hepático

d) Mastografía

e) Gamagrama óseo

CM3. Masculino de 56 años, sin antecedentes importantes. El padecimiento actual lo inicia con cuadro de
constipación de seis meses de evolución, acude a urgencias por dolor abdominal intenso de tres días de
evolución. A la exploración física con plastrón en cuadrante inferior derecho y palidez de tegumentos. En
estudios de laboratorio solo con Hb de 10.2, Rx de Tórax y abdomen normales. Es sometido a laparotomía
exploradora encontrando un tumor en ciego perforado. Se le efectuó hemicolectomía derecha, disección
ganglionar y lavado peritoneal. El informe de patología es de adenocarcinoma poco diferenciado que infiltra
pared de ciego hasta serosa con metástasis a 1 de 25 ganglios linfáticos.

CM3-1. De acuerdo a la clasificación de Astler–Coler modificada la etapa es:


a) B2

b) C2

c) C 1

d) B3

e) A

CM3-2. De los siguientes, qué manejo adyuvante propone usted:

a) 5FU 425 mg/m2 días 1-5, ac. Folínico 20 mg/m2 días 1-5 cada 28 días x 6 ciclos

b) 5FU 600/m2 días 1-5, ac. folínico 200 mg/m2 días 1-5 más levamisol x 6 ciclos

c) Radioterapia más 5FU 600/m2 semanal

d) Oxaliplatin 120 mg/m2 día 1, 5FU 600 x m2 días 1-5 más leucovorín 200 x m2 x 6 ciclos

CM3-3. Beneficio esperado del tratamiento adyuvante en este paciente:

a) Reducir la mortalidad en el 5%

b) Reducir la mortalidad en el 50%

c) Sólo aumentar el intervalo libre de enfemedad

d) Reducir la mortalidad en el 25%

e) Ninguna de las anteriores

CM3-4. Factor de riesgo más importante para la recurrencia:

a) Edad < de 40 años

b) Presencia de BCL 2

c) Etapa

d) Grado de diferenciación

e) Todos los anteriores

CM3-5. Dos años después de haber terminado terapia adyuvante el paciente presenta metastasis pulmonares y
usted elige como manejo sistémico oxaliplatino como monodroga que dosis recomendaría y que efecto de
toxicidad lo caracteriza:
a) 200 x m2 --- Neutropenia grado 3-4

b) 130 x m2 ---- Parestesias peribucales y disfagia

c) 130 X m2 ---- Nefrotoxicidad y emesis grado 3

d) 180 x m2----- Neurotoxicidad en guante y calcetín

CM4. Femenino de 60 años sin antecedente de importancia. Su padeciemiento actual lo inicia hace cuatro
meses con la presencia de molestias vagas en hipogastrio, hace un mes nota creciemiento abdominal
progresivo e incremento del dolor.Exploración física: paciente adelgazada, abdomen globoso a expensas de
liquido de ascitis, exámenes de laboratorio normales excepto DHL de 580 y Hb de 10.0, Rx de tórax, normal
, TAC de abdomen solo muestra líquido de ascitis, Ca 125 de 800 ng/mL. Se realiza Laparotomía
exploradora detectando en ovario der. tumor de 15 cm, ovario izq de 8 cm, múltiples implantes algunos de 1
cm y los más grandes de 2 cm, se realiza histerectomía, SOB, omentectomía dejando residual menor de 2
cm. Posoperatorio sin complicaciones, informe de patología: Carcinoma de células claras de ambos ovarios,
con infiltración al omento.

CM4-1. En relación con la clasificación de la FIGO, la etapa es:

a) III c

b) III b

c) IV a

d) III a

CM4-2. Porciento de frecuencia del tumor epitelial de células claras:

a) 20

b) 40

c) 1

d) 6

CM4-3. En relación al tumor de células claras todo es correcto, excepto:

a) Es un tumor raro

b) De comportamiento más agresivo

c) La respuesta a la quimioterapia es la misma que para el resto de epiteliales

d) La respuesta a la quimioterapia es menor que para el resto de los tumores epiteliales


e) La cirugía es la piedra angular del tratamiento

CM4-4. En relación al Ca125, una de las siguientes aseveraciones es correcta:

a) Sólo se eleva en padecimientos malignos

b) En los tumores mucinosos se eleva en < del 20%

c) Tiene valor fuertemente predictivo

d) No es útil como método de tamizaje en pacientes de alto riesgo

CM4-5. La QT más adecuada para esta paciente es:

a) Carboplatino 350 mg x m2 más paclitaxel 175 mg en infusión de 24 h

b) Platino a 120 mg/m2 más paclitaxel 175 mg x m2 en infusión de 24 h

c) Platino 70 mg/m2-Adriamicina 50 mg/m2.ciclofosfamida 750 mgxm2

d) Carboplatino calculado con AUC de 7-7.5 más paclitaxel 175 mg /m 2 infusión de 3 h

e) Carboplatino calculado con AUC de 3-4 más paclitaxel 175 mg/m 2 infusión de 3 h

CM5. Mujer de 65 años, con tumor mamario bilateral: mama derecha con tumor de 6 cm y conglomerado
axilar móvil de 3 cm; mama izquierda con tumor 8 cm, piel ulcerada y conlgomerado axilar 3 cm. Además,
dolor en cadera y región lumbar. Hiporexia, hipostenia, mareo, náusea y vómito, estreñiminento y oliguria.
Estudios de urgencia: leucocitos 3,,600 plaquetas 80,000. Hb 8.9 g/dl; Hto 26%, creatinina 3.8 mg/dl; Sodio
125 mEq/dl; Potasio 6.0 mg/dl, Calcio sérico 14.5 mg/dl; ALT, AST, Fosfatasa alcalina BT y BI todas
elevadas >2.5 su valor máximo normal. BAAF de mama: positivo para carcinoma.

CM5-1. Primera conducta a seguir con esta paciente:

a) Solicitar biopsia incisional de mama, tele de tórax, rastreo óseo

b) Iniciar tratamiento con FAC

c) Iniciar tratamiento con tamoxifén

d) Iniciar tratamiento con corticoides, hidratación parenteral

e) Iniciar tratamiento con CMF

CM5-2. La infiltración tumoral del cáncer mamario a médula ósea:


a) Sólo se presenta cuando hay metástasis óseas

b) Se presenta en 30-60% cuando hay enfermedad diseminada

c) La manifestación en sangre más común es trombocitopenia

d) El aspirado de médula ósea es el estudio más sensible y específico para su diagnóstico

CM5-3. De acuerdo a los valores de pruebas de función hepática, los siguientes fármacos requieren ajuste
en su dosis si decide prescribirlos a esta enferma, excepto:

a) Epirubicina

b) Vinorelbine

c) Doxorrubicina

d) Ciclofosfamida

e) Capecitabine

CM5-4. El resultado de la biopsia incisional informa de un carcinoma canalicular infiltrante con índice de
SBR III, receptores estrogénicos negativos y HER2 positivo. Cuál de las siguientes aseveraciones es
correcta:

a) El Her 2 positivo tiene un valor pronóstico favorable

b) La determinación de HER 2 de mayor utilidad se realiza en sangre periférica

c) El tratamiento con anticuerpos anti HER2 está indicado en HER2 positivo una cruz

d) El HER 2 está relacionado con riesgo familiar elevado de cáncer mamario

e) El HER2 se sobreexpresa en cáncer gástrico, ovario y pulmón

CM5-5. El Trastuzumab (Anticuerpo anti HER2) no debe usarse en forma simultánea con uno de los
siguientes fármacos, ya que se observó:

a) Con doxorubicina porque aumenta la cardiotoxicidad

b) Con Paclitaxel porque incrementa la neurotoxicidad

c) Con tamoxifén porque tiene un efecto antagónico con antiestrógenos

d) Con CMF porque incrementa la toxicidad renal del metotrexate

e) Con Docetaxel porque potencia en forma no predecible la mielotoxicidad


CM6. Masculino de 16 años, deportista que inicia con dolor en región costal derecha, sin antecedentes
oncológicos, sano hasta el momento actual, se revisa por médico del deporte quien realiza Rx de tórax donde
muesta una lesión expansiva en arco costal derecho. A la exploración ECOG 1, con tumoración que afecta
partes blandas de aproximadamente 10 cm, no dolorosa, sin repercusión en la función respiratoria.

CM6-1. Qué estudios de laboratorio y gabinete solicitaría de inicio:

a) BH,QS,PFH,FA,DHL. Rx de tórax y lateral, Tomografía de Tórax, GAMAGRAMA ÓSEO

b) Exámenes de laboratorio completos, gamagrama óseo

c) Rx de tórax y resonancia magnética de tórax

d) TAC de tórax y exámenes de laboratorio completos

CM6-2. Los exámenes de laboratorio sólo muestran elevación de FA y DHL discretamente elevada, RX de tórax
PA y lateral muestran una lesión expansiva del 10 arco costal y derrame pleural derecho. La TAC de tórax
muestra gran tumoración que destruye el arco costal infiltra partes blandas y penetra la pleura parietal con
desplazamiento de la glándula hepática, Gamagrama óseo normal. En orden de frecuencia cuáles son sus
primeras consideraciones diagnósticas:

a) Sarcoma de Ewing, Condrosarcoma, Sarcoma de partes blandas

b) Rabdomiosarcoma, Sarcoma de Ewing, Sarcoma oseteogénico

c) Sarcoma de Ewing.,Tumor de Askin.Sarcoma de partes blandas

d) Sarcoma de partes blandas, Tumor de Askin, Condrosarcoma

CM6-3. Se realiza biopsia y el informe de patología es de Sarcoma de Ewing, ¿Qué tratamiento propondría?

a) Cirugía con resección del arco costal,pleura l

b) QT de inducción o neoadyuvante

c) RT

d) QT/RT

CM6-4. Qué fármacos son los más activos en este padecimiento:

a) Dacarbazina, Ciclofosfamida, Adriamicina

b) Ifosfamida, Doxorubicina,Ciclofosfamida,Vincrisina

c) Ifosfamida, epirubicina, Bleomicina

d) Adriamicina, actinomicina D, methotrexate,dacarbazina


CM6-5. El paciente se somete a cirugía posterior a un tratamiento de quimioterapia, encontrando una gran
tumoracion que infiltra pleura, ¿que tratamiento propone usted?

a) Radioterapia

b) Esquema de rescate con quimioterapia

c) Esquema con quimioterapia a dosis altas más trasplante con c. Tallo periféricas

d) Resección de la tumoración con lobectomía

CM7. Masculino de 50 a con antecedente de tabaquismo y alcoholismo positivo intenso,inició su


padecimiento hace aprox 3 meses al presentar nodulación a nivel del frenillo de la lengua con crecimiento
progresivo indoloro, hasta alcanzar dimensiones de aproximadamente 2 cm, de aspecto vegetante y
fácilmente sangrante, acompañado de dolor leve, acude con dentista quien da manejo sintomático sin
mejoría decidiéndose tomar biopsia con reporte de: carcinoma epidermoide moderadamente diferenciado, se
realizan estudios destacando una TAC que muestra una gran lesión de piso de boca con infiltración a
mandíbula. A la exploración ECOG 1 adoncia casi total con pésima higiene bucal, se observa lesión
vegetante en piso de boca de aproximadamente 5 cm ulcerada, sangrante, fija, con cuello negativo.

CM7-1. La etafipificación clínica de este paciente sería:

a) T4, N0,M0

b) T3,N0,M0

c) T2c,N0,M0

d) T3 b NO M1

CM7-2.¿Qué tratamiento propone con base a ser el único que ha logrado porlongar la supervivencia?

a) Cirugia radical más RTadyuvante

b) Cirugia radical más QT adyuvante

c) Qumioterapia neoadyuvante

d) Quimioterapia más radioterapia concomitante

CM7-3. ¿Qué porciento de probabilidad tiene este paciente de tener enfermedad subclínica en cuello?

a) 5%

b) 30%
c) 70%

d) 100%

CM7-4. En su programa de tratamiento usted incluye amifostina ya que su objetivo fundamental es:

a) Evitar la nefrotoxicidad por platino

b) Evitar la neurotoxicidad por platino

c) Disminuir la posibilidad de mucositis y xerostomía

d) Disminuir la posibilidad de ataxia

e) Aumentar la posibilidad de ataxia

CM7-5. El paciente no responde al tratamiento propuesto por usted, ¿Qué fármacos dan mejores resultados
como terapia de segunda línea paliativa?

a) Vinorelbina y vincristina

b) Antraciclinas

c) Ciclofosfamida e ifosfamida

d) Taxanos

CASOS CLÍNICOS ONCOLOGÍA PEDIÁTRICA

CP1. Recién nacido masculino que presenta desde el nacimiento lesiones dérmicas generalizadas redondas,
de color violáceo, con bordes bien definidos y hepatomegalia que ocupa casi la totalidad del abdomen no
hay otra sintomatología, el examen físico confirma los hallazgos.

CP1-1. El diagnóstico de mayor probabilidad en este paciente es:

a) Hepatoblastoma

b) Leucemia aguda linfoblástica

c) Neuroblastoma

d) Tumor de células germinales

e) Rabdomiosarcoma
CP1-2. Debido a la hepatomegalia masiva del paciente se esperaría encontrar la siguiente complicación
clínica.

a) Síndrome de lisis tumoral

b) Síndrome de compresión de vena cava inferior

c) Síndrome de compresión medular

d) Insuficiencia respiratoria

e) Hipercalcemia

CP1-3. Qué porciento ocupan las neoplasias malignas congénitas dentro del total de cáncer en la niñez.

a) 1-3

b) 5-8

c) 10-15

d) 15-20

e) 20-25

CP1-4. Se realiza tomografía computada de tórax y abdomen en la que se corrobora el gran tamaño del
hígado, con un patrón de densidad homogéneo y no existe tumor primario, la biometría hemática no reporta
alteraciones, se toma biopsia de lesión dérmica en la cual reportan neoplasia maligna de células pequeñas
redondas y azules, qué resultados de inmunohistoquímica esperaría encontrar para confirmar el diagnóstico.

a) MIC2, proteína S100, CD 45

b) Sinaptofisina, neurofilamento, enolasa neurona específica

c) Vimentina, actina, desmina

d) Neurofilamento, CD 30, sinaptofisina

e) Actina, MIC2, enolasa neurona específica

CP1-5. La inmunohistoquímica es positiva, la conducta terapéutica a seguir en este paciente es:

a) Biopsia hemática, radioterapia y resección del tumor en segunda intención

b) Iniciar inmunoterapia y ATRA con radioterapia

c) Vigilancia con Ultrasonido cada mes

d) Iniciar quimioterapia y en caso de falta de respuesta radioterapia


e) Inmunoterapia y trasplante de médula ósea

CP2. Paciente femenino de 7 años de edad con distención abdominal, ataque al estado general, pédida de
peso e ictericia. Antecedente de madre con hepatitis B durante el embarazo, la exploración física muestra
paciente caquéctica, ictericia generalizada, abdomen con ascitis hígado a 10-10-8 cm por debajo de borde
costal.

CP2-1. La primera posibilidad diagnóstica en la paciente es:

a) Leucemia aguda linfoblástica

b) Hepatoblastoma

c) Hepatocarcinoma

d) Tumor de células germinales

e) Rabdomiosarcoma de vías biliares

CP2-2. La posibilidad de desarrollo de neoplasias malignas en hijos de madres infectadas por hepatitis B
tiene una latencia de:

a) 1 a 2 años

b) 2 a 4 años

c) 5 a 7 años

d) 10 a 20 años

e) 20 a 30 años

CP2-3. La tomografía de tórax y abdomen muestra múltiples zonas hiperdensas redondeadas de predominio en el
lóbulo hepático derecho la conducta a seguir es:

a) Toma de biopsia, inicio de quimioterapia neoadyuvante y resección del tumor residual

b) Toma de biopsia, radioterapia e inmunoterapia

c) Quimioterapia neoadyuvante, vacuna de la hepatitis B y radioterapia

d) Transplante de hígado

e) Quimioterapia de inducción a la remisión y profilaxis a sistema nervioso central


CP2-4. La sobrevida libre de enfermedad a 5 años en pacientes con respuesta completa y control local del
tumor es de:

a) 85-90%

b) 75-80%

c) 30-35%

d) 10-20%

e) 5-10%

VERSIÓN EN LINEA DEL LIBRO:

ONCOLOGÍA PARA CERTIFICACIÓN


Guía para presentar el examen
del Consejo Mexicano de Oncología
2002
Esta obra tiene como objetivo fundamental servir como un instrumento para todos aquellos oncólogos médicos,
quirúrgicos y pediatras que pretenden presentar el examen de certificación que aplica el Consejo Mexicano de
Oncología. La obra se estructuró con el apoyo y colaboración de destacados médicos especialistas de diferentes
áreas de la Oncología Mexicana. La selección de los reactivos se realizó en diferentes categorías, la primera mide
el conocimiento básico teórico que todo oncólogo debe tener, independientemente de la subespecialidad que
posea; las restantes, están constituidas por reactivos de corte básico aplicado y clínico enfocados a la aplicación
específica de conocimientos en el proceso diagnóstico y en el plan de tratamiento, conociendo a profundidad los
efectos secundarios, secuelas y complicaciones que suelen presentarse en la práctica diaria.

ONCOLOGÍA PARA CERTIFICACIÓN, una de las primeras obras en su género, contribuirá sin duda a elevar la
calidad académica de los especialistas de la oncología en México.

Derechos Reservados
© 2002
ISBN: 968-36-8970-1

PATROCINADOR DE LA PUBLICACIÓN

CONSEJO MEXICANO DE ONCOLOGÍA, A.C.


TUXPAN 59 PH, COL. ROMA 06760, MÉXICO DF

UNIVERSIDAD NACIONAL AUTÓNOMA DE MÉXICO


FACULTAD DE ESTUDIOS SUPERIORES IZTACALA
Av. de los Barrios s/n Los Reyes Iztacala, Tlalnepantla,
54090, Estado de México, México.

Ninguna parte de esta obra puede ser reproducida o trasmitida,


mediante ningún sistema o método, electrónico o mecánico,
sin el consentimiento escrito de los Editores.

APOYO TÉCNICO

MC RAMIRO JESÚS SANDOVAL


Coordinación de la edición y supervisión

MC JOSÉ JAIME ÁVILA VALDIVIESO


Cuidado de la edición, corrección de estilo
y formación editorial

MTRA. GLORIA A. GARCÍA MIRANDA


LIC. NICOLÁS H. GARCÍA RENTERÍA
Corrección de estilo y lectura de pruebas

BIÓL. VALENTÍN MORENO COLÍN


Elaboración de portada.

IMPRESO Y HECHO EN MÉXICO

RESPUESTAS

GE1. ( c ) EP26. ( a ) GN31. ( b ) GN81. ( e ) GN131. ( d )

GE2. ( a ) EP27. ( c ) GN32. ( a ) GN82. ( d ) GN132. ( d )

GE3. ( b ) EP28. ( b ) GN33. ( c ) GN83. ( c ) GN133. ( d )

GE4. ( e ) EP29. ( e ) GN34. ( c ) GN84. ( b ) GN134. ( d )


GE5. ( d ) EP30. ( a ) GN35. ( d ) GN85. ( e ) GN135. ( a )

GE6. ( a ) EP31. ( e ) GN36. ( e ) GN86. ( c ) GN136. ( e )

GE7. ( b ) EP32. ( b ) GN37. ( a ) GN87. ( c ) GN137. ( c )

GE8. ( c ) EP33. ( e ) GN38. ( a ) GN88. ( d ) GN138. ( e )

GE9. ( d ) EP34. ( b ) GN39. ( d ) GN89. ( b ) GN139. ( b )

GE10. ( a ) EP35. ( d ) GN40. ( b ) GN90. ( b ) GN140. ( e )

GE11. ( c ) EP36. ( d ) GN41. ( e ) GN91. ( e ) GN141. ( e )

GE12. ( b ) EP37. ( b ) GN42. Células GN92. ( b ) GN142. ( a )


claras
GE13. ( a ) EP38. ( c ) GN93. ( c ) GN143. ( c )
GN43. Seroso
GE14. ( c ) EP39. ( a ) papilar GN94. ( c ) GN144. ( a )

GE15. ( c ) EP40. ( c ) GN44. GN95. ( c ) GN145. ( e )


Adenoescamoso
GE16. ( e ) EP41. ( a ) GN96. ( d ) GN146. ( e )
GN45. ( d )
GE17. ( c ) EP42. ( a ) GN97. ( b ) GN147. ( d )
GN46. ( c )
GE18. ( e ) EP43. ( c ) GN98. ( a ) GN148. ( d )
GN47. ( d )
GE19. ( b ) EP44. ( b ) GN99. ( e ) GN149. ( e )
GN48. ( e )
GE20. ( e ) EP45. ( c ) GN100. ( d ) GN150. ( e )
GN49. ( c )
GE21. ( e ) GN1. ( d ) GN101. ( b ) GN151. ( e )
GN50. ( c )
GE22. ( a ) GN2. ( b ) GN102. ( c ) GN152. ( d )
GN51. ( b )
GE23. ( b ) GN3. ( d ) GN103. ( c ) GN153. ( e )
GN52. ( d )
GE24. ( e ) GN4. ( c ) GN104. ( b ) GN154. ( d )
GN53. ( b )
GE25. ( e ) GN5. ( e ) GN105. ( c ) GN155. ( d )
GN54. ( d )
EP1. ( b ) GN6. ( a ) GN106. ( b ) GN156. ( b )
GN55. ( c )
EP2. ( c ) GN7. ( e ) GN107. ( c ) GN157. ( e )
GN56. ( c )
EP3. ( e ) GN8. ( a ) GN108. ( d ) GN158. ( a )
GN57. ( a )
EP4. ( b ) GN9. ( d ) GN109. ( d ) GN159. ( d )
GN58. ( b )
EP5. ( c ) GN10. ( b ) GN110. ( a ) GN160. ( a )
GN59. ( c )
EP6. ( b ) GN11. ( c ) GN111. ( b ) GN161. ( d )
EP7. ( c ) GN12. ( e ) GN60. ( d ) GN112. ( d ) GN162. ( c )

EP8. ( c ) GN13. ( d ) GN61. ( d ) GN113. ( c ) GN163. ( d )

EP9. ( c ) GN14. ( c ) GN62. ( a ) GN114. ( e ) GN164. ( b )

EP10. ( e ) GN15. ( d ) GN63. ( a ) GN115. ( a ) GN165. ( b )

EP11. ( d ) GN16. ( a ) GN64. ( a ) GN116. ( b ) GN166. ( c )

EP12. ( d ) GN17. ( c ) GN65. ( b ) GN117. ( c ) GN167. ( e )

EP13. ( e ) GN18. ( b ) GN66. ( c ) GN118. ( c ) GN168. ( c )

EP14. ( a ) GN19. ( 3 ) GN67. ( d ) GN119. ( d ) GN169. ( c )

EP15. ( e ) GN20. ( 1 ) GN68. ( e ) GN120. ( a ) GN170. ( e )

EP16. ( c ) GN21. ( 4 ) GN69. ( a ) GN121. ( e ) GN171. ( b )

EP17. ( b ) GN22. ( 2 ) GN70. ( e ) GN122. ( c ) GN172. ( c )

EP18. ( d ) GN23. ( 1 ) GN71. ( c ) GN123. ( d ) GN173. ( e )

EP19. ( a ) GN24. ( 3 ) GN72. ( b ) GN124. ( e ) GN174. ( d )

EP20. ( e ) GN25. ( 4 ) GN73. ( d ) GN125. ( c ) GN175. ( a )

EP21. ( e ) GN26. ( 2 ) GN74. ( a ) GN126. ( e ) GN176. ( c )

EP22. ( d ) GN27. ( b ) GN75. ( b ) GN127. ( b ) GN177. ( e )

EP23. ( a ) GN28. ( d ) GN76. ( d ) GN128. ( d ) GN178. ( e )

EP24. ( e ) GN29. ( a ) GN77. ( b ) GN129. ( b ) GN179. ( d )

EP25. ( d ) GN30. ( c ) GN78. ( d ) GN130. ( a ) GN180. ( b )

GN79. ( a )

GN80. ( b )

GN181. ( e ) GN231. ( f ) GN281. ( e ) GN331. ( e ) GN381. ( d )

GN182. ( d ) GN232. ( d ) GN282. ( e ) GN332. ( c ) GN382. ( d )

GN183. ( v ) GN233. ( b ) GN283. ( d ) GN333. ( c ) GN383. ( b )


GN184. ( v ) GN234. ( a ) GN284. ( a ) GN334. ( c ) GN384. ( e )

GN185. ( f ) GN235. ( c ) GN285. ( b ) GN335. ( b ) GN385. ( b )

GN186. ( v ) GN236. ( e ) GN286. ( a ) GN336. ( d ) GN386. ( d )

GN187. ( v ) GN237. ( c ) GN287. ( b ) GN337. ( a ) GN387. ( c )

GN188. ( c ) GN238. ( e ) GN288. ( a ) GN338. ( d ) GN388. ( e )

GN189. ( c ) GN239. ( d ) GN289. ( b ) GN339. ( e ) GN389. ( c )

GN190. ( c ) GN240. ( c ) GN290. ( e ) GN340. ( d ) GN390. ( e )

GN191. ( a ) GN241. ( a ) GN291. ( d ) GN341. ( a ) GN391. ( b )

GN192. ( a ) GN242. ( b ) GN292. ( b ) GN342. ( b ) GN392. ( b )

GN193. ( a ) GN243. ( e ) GN293. ( e ) GN343. ( e ) GN393. ( d )

GN194. ( b ) GN244. ( a ) GN294. ( b ) GN344. ( d ) GN394. ( a )

GN195. ( c ) GN245. ( c ) GN295. ( c ) GN345. ( d ) GN395. ( e )

GN196. ( d ) GN246. ( c ) GN296. ( a ) GN346. ( c ) GN396. ( a )

GN197. ( d ) GN247. ( c ) GN297. ( e ) GN347. ( b ) GN397. ( c )

GN198. ( c ) GN248. ( b ) GN298. ( d ) GN348. ( b ) GN398. ( a )

GN199. ( d ) GN249. ( c ) GN299. ( d ) GN349. ( b ) GN399. ( a )

GN200. ( d ) GN250. ( c ) GN300. ( e ) GN350. ( c ) GN400. ( c )

GN201. ( a ) GN251. ( a ) GN301. ( c ) GN351. ( b ) GN401. ( c )

GN202. ( a ) GN252. ( d ) GN302. ( b ) GN352. ( a ) GN402. ( d )

GN203. ( e ) GN253. ( d ) GN303. ( b ) GN353. ( a ) GN403. ( d )

GN204. ( d ) GN254. ( c ) GN304. ( e ) GN354. ( c ) GN404. ( d )

GN205. ( d ) GN255. ( d ) GN305. ( b ) GN355. ( b ) GN405. ( c )

GN206. ( c ) GN256. ( e ) GN306. ( e ) GN356. ( c ) GN406. ( c )

GN207. ( c ) GN257. ( c ) GN307. ( b ) GN357. ( a ) GN407. ( d )

GN208. ( a ) GN258. ( a ) GN308. ( e ) GN358. ( c ) GN408. ( d )

GN209. ( a ) GN259. ( d ) GN309. ( c ) GN359. ( c ) GN409. ( d )

GN210. ( d ) GN260. ( a ) GN310. ( e ) GN360. ( b ) GN410. ( d )


GN211. ( c ) GN261. ( c ) GN311. ( d ) GN361. ( c ) GN411. ( e )

GN212. ( a ) GN262. ( c ) GN312. ( c ) GN362. ( b ) GN412. ( d )

GN213. ( b ) GN263. ( a ) GN313. ( d ) GN363. ( c ) GN413. ( c )

GN214. ( e ) GN264. ( c ) GN314. ( c ) GN364. ( d ) GN414. ( d )

GN215. ( d ) GN265. ( b ) GN315. ( e ) GN365. ( e ) GN415. ( c )

GN216. ( b ) GN266. ( b ) GN316. ( b ) GN366. ( c ) GN416. ( d )

GN217. ( c ) GN267. ( c ) GN317. ( e ) GN367. ( b ) GN417. ( c )

GN218. ( b ) GN268. ( d ) GN318. ( d ) GN368. ( e ) GN418. ( c )

GN219. ( d ) GN269. ( c ) GN319. ( a ) GN369. ( a ) GN419. ( d )

GN220. ( b ) GN270. ( b ) GN320. ( c ) GN370. ( c ) GN420. ( b )

GN221. ( c ) GN271. ( b ) GN321. ( c ) GN371. ( a ) GN421. ( d )

GN222. ( b ) GN272. ( b ) GN322. ( a ) GN372. ( b ) GN422. ( d )

GN223. ( c ) GN273. ( a ) GN323. ( d ) GN373. ( c ) GN423. ( a )

GN224. ( e ) GN274. ( a ) GN324. ( b ) GN374. ( e ) GN424. ( c )

GN225. ( a ) GN275. ( b ) GN325. ( b ) GN375. ( b ) GN425. ( b )

GN226. ( e ) GN276. ( b ) GN326. ( c ) GN376. ( c ) GN426. ( a )

GN227. ( e ) GN277. ( a ) GN327. ( d ) GN377. ( b ) GN427. ( c )

GN228. ( e ) GN278. ( a ) GN328. ( c ) GN378. ( b ) GN428. ( c )

GN229. ( c ) GN279. ( a ) GN329. ( c ) GN379. ( a ) GN429. ( a )

GN230. ( d ) GN280. ( d ) GN330. ( c ) GN380. ( e ) GN430. ( b )

GN431. ( c ) GN481. ( c ) MA4. ( d ) MA54. ( d ) MA104. ( e )

GN432. ( e ) GN482. ( a ) MA5. ( b ) MA55. ( a ) MA105. ( a )


GN433. ( c ) GN483. ( a ) MA6. ( a ) MA56. ( c ) MA106. ( d )

GN434. ( c ) GN484. ( a ) MA7. ( c ) MA57. ( c ) MA107. ( b )

GN435. ( e ) GN485. ( c ) MA8. ( a ) MA58. ( c ) MA108. ( b )

GN436. ( d ) GN486. ( a ) MA9. ( b ) MA59. ( b ) MA109. ( b )

GN437. ( c ) GN487. ( d ) MA10. ( c ) MA60. ( c ) MA110. ( e )

GN438. ( b ) GN488. ( c ) MA11. ( d ) MA61. ( a ) MA111. ( d )

GN439. ( a ) GN489. ( e ) MA12. ( e ) MA62. ( b ) MA112. ( c )

GN440. ( a ) GN490. ( b ) MA13. ( e ) MA63. ( b ) MA113. ( b )

GN441. ( b ) GN491. ( c ) MA14. ( d ) MA64. ( d ) MA114. ( b )

GN442. ( c ) GN492. ( a ) MA15. ( e ) MA65. ( a ) MA115. ( a )

GN443. ( d ) GN493. ( a ) MA16. ( d ) MA66. ( c ) MA116. ( c )

GN444. ( a ) GN494. ( a ) MA17. ( e ) MA67. ( c ) MA117. ( b )

GN445. ( c ) GN495. ( a ) MA18. ( b ) MA68. ( e ) MA118. ( d )

GN446. ( c ) GN496. ( b ) MA19. ( a ) MA69. ( d ) MA119. ( e )

GN447. ( a ) GN497. ( a ) MA20. ( b ) MA70. ( b ) MA120. ( b )

GN448. ( b ) GN498. ( d ) MA21. ( b ) MA71. ( c ) MA121. ( e )

GN449. ( a ) GN499. ( e ) MA22. ( e ) MA72. ( b ) MA122. ( b )

GN450. ( a ) GN500. ( e ) MA23. ( b ) MA73. ( c ) MA123. ( d )

GN451. ( c ) GN501. ( a ) MA24. ( a ) MA74. ( c ) MA124. ( d )

GN452. ( e ) GN502. ( a ) MA25. ( b ) MA75. ( b ) MA125. ( d )

GN453. ( d ) GN503. ( b ) MA26. ( b ) MA76. ( d ) MA126. ( e )

GN454. ( d ) GN504. ( a ) MA27. ( e ) MA77. ( b ) MA127. ( d )

GN455. ( b ) GN505. ( d ) MA28. ( c ) MA78. ( b ) MA128. ( d )

GN456. ( b ) GN506. ( c ) MA29. ( a ) MA79. ( d ) MA129. ( b )

GN457. ( c ) GN507. ( c ) MA30. ( c ) MA80. ( c ) MA130. ( b )

GN458. ( b ) GN508. ( d ) MA31. ( c ) MA81. ( a ) MA131. ( d )

GN459. ( d ) GN509. ( d ) MA32. ( e ) MA82. ( c ) MA132. ( c )


GN460. ( d ) GN510. ( d ) MA33. ( c ) MA83. ( a ) MA133. ( b )

GN461. ( a ) GN511. ( e ) MA34. ( d ) MA84. ( e ) MA134. ( e )

GN462. ( d ) GN512. ( d ) MA35. ( d ) MA85. ( d ) MA135. ( c )

GN463. ( c ) GN513. ( c ) MA36. ( b ) MA86. ( e ) MA136. ( b )

GN464. ( a ) GN514. ( c ) MA37. ( c ) MA87. ( a ) MA137. ( a )

GN465. ( c ) GN515. ( c ) MA38. ( c ) MA88. ( a ) MA138. ( b )

GN466. ( c ) GN516. ( b ) MA39. ( e ) MA89. ( b ) MA139. ( e )

GN467. ( c ) GN517. ( c ) MA40. ( d ) MA90. ( b ) MA140. ( b )

GN468. ( a ) GN518. ( c ) MA41. ( c ) MA91. ( c ) MA141. ( d )

GN469. ( c ) GN519. ( b ) MA42. ( d ) MA92. ( d ) MA142. ( a )

GN470. ( b ) GN520. ( b ) MA43. ( c ) MA93. ( e ) MA143. ( c )

GN471. ( b ) GN521. ( c ) MA44. ( c ) MA94. ( d ) MA144. ( b )

GN472. ( c ) GN522. ( b ) MA45. ( b ) MA95. ( b ) MA145. ( d )

GN473. ( a ) GN523. ( b ) MA46. ( c ) MA96. ( b ) MA146. ( c )

GN474. ( e ) GN524. ( c ) MA47. ( e ) MA97. ( b ) MA147. ( b )

GN475. ( d ) GN525. ( b ) MA48. ( e ) MA98. ( c ) MA148. ( d )

GN476. ( d ) GN526. ( c ) MA49. ( c ) MA99. ( e ) MA149. ( c )

GN477. ( e ) GN527. ( a ) MA50. ( b ) MA100. ( e ) MA150. ( a )

GN478. ( c ) MA1. ( c ) MA51. ( d ) MA101. ( d ) MA151. ( b )

GN479. ( b ) MA2. ( d ) MA52. ( d ) MA102. ( d ) MA152. ( d )

GN480. ( e ) MA3. ( c ) MA53. ( b ) MA103. ( d ) MA153. ( c )

MA154. ( b ) MA204. ( d ) DG45. ( d ) DG95. ( b ) DG145. ( c )


MA155. ( a ) MA205. ( d ) DG46. ( c ) DG96. ( a ) DG146. ( c )

MA156. ( c ) MA206. ( e ) DG47. ( c ) DG97. ( b ) DG147. ( e )

MA157. ( c ) MA207. ( e ) DG48. ( c ) DG98. ( d ) DG148. ( a )

MA158. ( b ) MA208. ( d ) DG49. ( c ) DG99. ( e ) DG149. ( e )

MA159. ( d ) MA209. ( d ) DG50. ( d ) DG100. ( b ) DG150. ( c )

MA160. ( b ) DG1. ( c ) DG51. ( d ) DG101. ( c ) DG151. ( a )

MA161. ( d ) DG2. ( d ) DG52. ( c ) DG102. ( d ) DG152. ( c )

MA162. ( e ) DG3. ( b ) DG53. ( b ) DG103. ( d ) DG153. ( b )

MA163. ( a ) DG4. ( c ) DG54. ( e ) DG104. ( a ) DG154. ( e )

MA164. ( c ) DG5. ( b ) DG55. ( b ) DG105. ( c ) DG155. ( b )

MA165. ( d ) DG6. ( a ) DG56. ( d ) DG106. ( b ) DG156. ( e )

MA166. ( b ) DG7. ( a ) DG57. ( d ) DG107. ( c ) DG157. ( c )

MA167. ( d ) DG8. ( a ) DG58. ( c ) DG108. ( d ) DG158. ( d )

MA168. ( a ) DG9. ( d ) DG59. ( a ) DG109. ( c ) DG159. ( c )

MA169. ( c ) DG10. ( d ) DG60. ( a ) DG110. ( e ) DG160. ( d )

MA170. ( a ) DG11. ( b ) DG61. ( b ) DG111. ( d ) DG161. ( b )

MA171. ( e ) DG12. ( a ) DG62. ( d ) DG112. ( d ) DG162. ( c )

MA172. ( c ) DG13. ( b ) DG63. ( d ) DG113. ( c ) DG163. ( e )

MA173. ( d ) DG14. ( a ) DG64. ( c ) DG114. ( d ) DG164. ( c )

MA174. ( d ) DG15. ( b ) DG65. ( e ) DG115. ( c ) DG165. ( c )

MA175. ( c ) DG16. ( d ) DG66. ( e ) DG116. ( e ) DG166. ( c )

MA176. ( d ) DG17. ( a ) DG67. ( a ) DG117. ( c ) LL1. ( b )

MA177. ( e ) DG18. ( a ) DG68. ( c ) DG118. ( c ) LL2. ( d )

MA178. ( c ) DG19. ( e ) DG69. ( d ) DG119. ( c ) LL3. ( a )

MA179. ( a ) DG20. ( a ) DG70. ( b ) DG120. ( e ) LL4. ( b )

MA180. ( d ) DG21. ( c ) DG71. ( e ) DG121. ( a ) LL5. ( d )

MA181. ( c ) DG22. ( a ) DG72. ( b ) DG122. ( d ) LL6. ( e )


MA182. ( b ) DG23. ( d ) DG73. ( d ) DG123. ( b ) LL7. ( a )

MA183. ( e ) DG24. ( b ) DG74. ( e ) DG124. ( a ) LL8. ( e )

MA184. ( d ) DG25. ( b ) DG75. ( c ) DG125. ( b ) LL9. ( b )

MA185. ( a ) DG26. ( e ) DG76. ( a ) DG126. ( c ) LL10. ( c )

MA186. ( c ) DG27. ( d ) DG77. ( b ) DG127. ( c ) LL11. ( b )

MA187. ( c ) DG28. ( b ) DG78. ( d ) DG128. ( d ) LL12. ( a )

MA188. ( d ) DG29. ( c ) DG79. ( b ) DG129. ( e ) LL13. ( c )

MA189. ( b ) DG30. ( a ) DG80. ( c ) DG130. ( a ) LL14. ( d )

MA190. ( d ) DG31. ( b ) DG81. ( b ) DG131. ( e ) LL15. ( e )

MA191. ( c ) DG32. ( c ) DG82. ( c ) DG132. ( b ) LL16. ( b )

MA192. ( d ) DG33. ( b ) DG83. ( c ) DG133. ( c ) LL17. ( a )

MA193. ( e ) DG34. ( d ) DG84. ( c ) DG134. ( e ) LL18. ( b )

MA194. ( b ) DG35. ( c ) DG85. ( c ) DG135. ( b ) LL19. ( e )

MA195. ( b ) DG36. ( e ) DG86. ( c ) DG136. ( d ) LL20. ( c )

MA196. ( d ) DG37. ( e ) DG87. ( c ) DG137. ( d ) LL21. ( c )

MA197. ( d ) DG38. ( e ) DG88. ( a ) DG138. ( b ) LL22. ( a )

MA198. ( e ) DG39. ( e ) DG89. ( b ) DG139. ( a ) LL23. ( c )

MA199. ( b ) DG40. ( b ) DG90. ( b ) DG140. ( e ) LL24. ( b )

MA200. ( d ) DG41. ( b ) DG91. ( d ) DG141. ( e ) LL25. ( d )

MA201. ( d ) DG42. ( d ) DG92. ( d ) DG142. ( d ) LL26. ( e )

MA202. ( c ) DG43. ( b ) DG93. ( c ) DG143. ( d ) LL27. ( a )

MA203. ( a ) DG44. ( c ) DG94. ( d ) DG144. ( c ) LL28. ( e )


LL29. ( e ) LL79. ( c ) UR1. ( d ) UR51. ( d ) UR101. ( e )

LL30. ( e ) LL80. ( d ) UR2. ( a ) UR52. ( c ) UR102. ( b )

LL31. ( a ) LL81. ( b ) UR3. ( d ) UR53. ( e ) UR103. ( d )

LL32. ( b ) LL82. ( d ) UR4. ( d ) UR54. ( b ) UR104. ( b )

LL33. ( d ) LL83. ( e ) UR5. ( d ) UR55. ( d ) UR105. ( c )

LL34. ( d ) LL84. ( c ) UR6. ( d ) UR56. ( e ) UR106. ( b )

LL35. ( c ) LL85. ( c ) UR7. ( e ) UR57. ( b ) UR107. ( d )

LL36. ( e ) LL86. ( e ) UR8. ( e ) UR58. ( a ) UR108. ( e )

LL37. ( c ) LL87. ( d ) UR9. ( d ) UR59. ( d ) UR109. ( d )

LL38. ( b ) LL88. ( c ) UR10. ( c ) UR60. ( c ) UR110. ( e )

LL39. ( b ) LL89. ( d ) UR11. ( c ) UR61. ( c ) CC1. ( d )

LL40. ( c ) LL90. ( a ) UR12. ( e ) UR62. ( b ) CC2. ( c )

LL41. ( b ) LL91. ( d ) UR13. ( d ) UR63. ( e ) CC3. ( e )

LL42. ( b ) LL92. ( d ) UR14. ( e ) UR64. ( c ) CC4. ( d )

LL43. ( c ) LL93. ( b ) UR15. ( b ) UR65. ( a ) CC5. ( b )

LL44. ( e ) LL94. ( d ) UR16. ( e ) UR66. ( d ) CC6. ( e )

LL45. ( e ) LL95. ( b ) UR17. ( c ) UR67. ( b ) CC7. ( e )

LL46. ( a ) LL96. ( c ) UR18. ( a ) UR68. ( c ) CC8. ( a )

LL47. ( b ) LL97. ( d ) UR19. ( e ) UR69. ( a ) CC9. ( c )

LL48. ( d ) LL98. ( c ) UR20. ( a ) UR70. ( a ) CC10. ( b )

LL49. ( c ) LL99. ( d ) UR21. ( e ) UR71. ( e ) CC11. ( c )

LL50. ( e ) LL100. ( a ) UR22. ( c ) UR72. ( a ) CC12. ( c )

LL51. ( d ) LL101. ( c ) UR23. ( b ) UR73. ( b ) CC13. ( d )

LL52. ( d ) LL102. ( b ) UR24. ( a ) UR74. ( d ) CC14. ( a )

LL53. ( c ) LL103. ( e ) UR25. ( c ) UR75. ( e ) CC15. ( b )

LL54. ( d ) LL104. ( a ) UR26. ( a ) UR76. ( b ) CC16. ( d )

LL55. ( c ) LL105. ( c ) UR27. ( c ) UR77. ( d ) CC17. ( c )


LL56. ( e ) LL106. ( d ) UR28. ( e ) UR78. ( c ) CC18. ( b )

LL57. ( c ) LL107. ( c ) UR29. ( a ) UR79. ( e ) CC19. ( d )

LL58. ( c ) LL108. ( c ) UR30. ( e ) UR80. ( a ) CC20. ( b )

LL59. ( c ) LL109. ( d ) UR31. ( e ) UR81. ( c ) CC21. ( e )

LL60. ( a ) LL110. ( b ) UR32. ( c ) UR82. ( a ) CC22. ( c )

LL61. ( e ) LL111. ( b ) UR33. ( c ) UR83. ( d ) CC23. ( c )

LL62. ( d ) LL112. ( d ) UR34. ( d ) UR84. ( d ) CC24. ( b )

LL63. ( d ) LL113. ( e ) UR35. ( d ) UR85. ( c ) CC25. ( c )

LL64. ( b ) LL114. ( b ) UR36. ( d ) UR86. ( a ) CC26. ( a )

LL65. ( c ) LL115. ( a ) UR37. ( c ) UR87. ( c ) CC27. ( b )

LL66. ( b ) LL116. ( e ) UR38. ( d ) UR88. ( c ) CC28. ( c )

LL67. ( c ) LL117. ( d ) UR39. ( c ) UR89. ( d ) CC29. ( d )

LL68. ( d ) LL118. ( a ) UR40. ( d ) UR90. ( d ) CC30. ( a )

LL69. ( d ) LL119. ( c ) UR41. ( c ) UR91. ( a ) CC31. ( b )

LL70. ( c ) LL120. ( e ) UR42. ( d ) UR92. ( c ) CC32. ( e )

LL71. ( b ) LL121. ( c ) UR43. ( d ) UR93. ( d ) CC33. ( d )

LL72. ( c ) LL122. ( d ) UR44. ( b ) UR94. ( a ) CC34. ( c )

LL73. ( b ) LL123. ( c ) UR45. ( e ) UR95. ( d ) CC35. ( a )

LL74. ( b ) LL124. ( d ) UR46. ( a ) UR96. ( c ) CC36. ( d )

LL75. ( c ) LL125. ( a ) UR47. ( b ) UR97. ( e ) CC37. ( c )

LL76. ( a ) LL126. ( c ) UR48. ( b ) UR98. ( b ) CC38. ( e )

LL77. ( b ) LL127. ( e ) UR49. ( e ) UR99. ( d ) CC39. ( c )

LL78. ( d ) LL128. ( b ) UR50. ( b ) UR100. ( b ) CC40. ( e )


CC41. ( e ) CC91. ( d ) CC141. ( a ) CC191. ( c ) CC241. ( a ) CC291. ( e )

CC42. ( c ) CC92. ( e ) CC142. ( b ) CC192. ( b ) CC242. ( d ) CC292. ( b )

CC43. ( c ) CC93. ( a ) CC143. ( c ) CC193. ( e ) CC243. ( v ) CC293. ( e )

CC44. ( e ) CC94. ( c ) CC144. ( c ) CC194. ( a ) CC244. ( v ) CC294. ( c )

CC45. ( b ) CC95. ( e ) CC145. ( e ) CC195. ( d ) CC245. ( v ) CC295. ( b )

CC46. ( a ) CC96. ( a ) CC146. ( d ) CC196. ( b ) CC246. ( v ) CC296. ( d )

CC47. ( b ) CC97. ( a ) CC147. ( c ) CC197. ( a ) CC247. ( f ) CC297. ( d )

CC48. ( c ) CC98. ( c ) CC148. ( d ) CC198. ( d ) CC248. ( a ) CC298. ( b )

CC49. ( b ) CC99. ( c ) CC149. ( a ) CC199. ( b ) CC249. ( e ) CC299. ( d )

CC50. ( b ) CC100. ( a ) CC150. ( c ) CC200. ( c ) CC250. ( c ) CC300. ( e )

CC51. ( a ) CC101. ( b ) CC151. ( b ) CC201. ( b ) CC251. ( c ) CC301. ( c )

CC52. ( b ) CC102. ( d ) CC152. ( b ) CC202. ( c ) CC252. ( b ) CC302. ( a )

CC53. ( c ) CC103. ( a ) CC153. ( a ) CC203. ( a ) CC253. ( a ) CC303. ( e )

CC54. ( d ) CC104. ( d ) CC154. ( c ) CC204. ( a ) CC254. ( b ) CC304. ( c )

CC55. ( c ) CC105. ( b ) CC155. ( d ) CC205. ( c ) CC255. ( c ) CC305. ( b )

CC56. ( d ) CC106. ( b ) CC156. ( d ) CC206. ( c ) CC256. ( b ) CC306. ( c )

CC57. ( a ) CC107. ( b ) CC157. ( b ) CC207. ( b ) CC257. ( b ) CC307. ( b )

CC58. ( c ) CC108. ( d ) CC158. ( e ) CC208. ( c ) CC258. ( a ) CC308. ( a )

CC59. ( d ) CC109. ( d ) CC159. ( c ) CC209. ( b ) CC259. ( b ) CC309. ( c )

CC60. ( c ) CC110. ( d ) CC160. ( d ) CC210. ( b ) CC260. ( a ) CC310. ( b )

CC61. ( b ) CC111. ( e ) CC161. ( a ) CC211. ( a ) CC261. ( b ) CC311. ( c )

CC62. ( d ) CC112. ( e ) CC162. ( d ) CC212. ( b ) CC262. ( c ) CC312. ( c )

CC63. ( c ) CC113. ( a ) CC163. ( c ) CC213. ( b ) CC263. ( b ) CC313. ( c )

CC64. ( b ) CC114. ( c ) CC164. ( e ) CC214. ( c ) CC264. ( c ) CC314. ( e )

CC65. ( d ) CC115. ( d ) CC165. ( d ) CC215. ( e ) CC265. ( b ) CC315. ( b )

CC66. ( b ) CC116. ( a ) CC166. ( e ) CC216. ( c ) CC266. ( b ) CC316. ( b )


CC67. ( b ) CC117. ( e ) CC167. ( d ) CC217. ( b ) CC267. ( d ) CC317. ( b )

CC68. ( c ) CC118. ( a ) CC168. ( d ) CC218. ( c ) CC268. ( c ) CC318. ( e )

CC69. ( d ) CC119. ( b ) CC169. ( d ) CC219. ( a ) CC269. ( a ) CC319. ( d )

CC70. ( b ) CC120. ( a ) CC170. ( b ) CC220. ( c ) CC270. ( b ) CC320. ( a )

CC71. ( d ) CC121. ( c ) CC171. ( c ) CC221. ( d ) CC271. ( b ) CC321. ( c )

CC72. ( d ) CC122. ( b ) CC172. ( b ) CC222. ( a ) CC272. ( b ) CC322. ( b )

CC73. ( a ) CC123. ( b ) CC173. ( e ) CC223. ( b ) CC273. ( c ) CC323. ( e )

CC74. ( e ) CC124. ( b ) CC174. ( a ) CC224. ( a ) CC274. ( a ) CC324. ( e )

CC75. ( b ) CC125. ( c ) CC175. ( b ) CC225. ( e ) CC275. ( c ) CC325. ( d )

CC76. ( b ) CC126. ( d ) CC176. ( a ) CC226. ( c ) CC276. ( a ) CC326. ( d )

CC77. ( d ) CC127. ( d ) CC177. ( a ) CC227. ( e ) CC277. ( b ) CC327. ( d )

CC78. ( d ) CC128. ( b ) CC178. ( b ) CC228. ( b ) CC278. ( d ) CC328. ( e )

CC79. ( b ) CC129. ( b ) CC179. ( c ) CC229. ( a ) CC279. ( c ) CC329. ( b )

CC80. ( c ) CC130. ( a ) CC180. ( d ) CC230. ( c ) CC280. ( b ) CC330. ( a )

CC81. ( a ) CC131. ( a ) CC181. ( d ) CC231. ( d ) CC281. ( a ) CC331. ( a )

CC82. ( d ) CC132. ( e ) CC182. ( b ) CC232. ( a ) CC282. ( b ) CC332. ( d )

CC83. ( c ) CC133. ( e ) CC183. ( b ) CC233. ( f ) CC283. ( b ) CC333. ( e )

CC84. ( c ) CC134. ( b ) CC184. ( e ) CC234. ( v ) CC284. ( a ) CC334. ( b )

CC85. ( c ) CC135. ( d ) CC185. ( b ) CC235. ( v ) CC285. ( b ) CC335. ( e )

CC86. ( c ) CC136. ( c ) CC186. ( a ) CC236. ( f ) CC286. ( b ) CC336. ( b )

CC87. ( c ) CC137. ( e ) CC187. ( e ) CC237. ( v ) CC287. ( a ) CC337. ( d )

CC88. ( a ) CC138. ( d ) CC188. ( e ) CC238. ( a ) CC288. ( b ) CC338. ( c )

CC89. ( c ) CC139. ( b ) CC189. ( d ) CC239. ( c ) CC289. ( b ) CC339. ( e )

CC90. ( b ) CC140. ( c ) CC190. ( b ) CC240. ( b ) CC290. ( b ) CC340. ( c )


TB1. ( d ) TB51. ( a ) TB101. ( c ) MX9. ( a ) MX59. ( e ) TO38. ( d )

TB2. ( c ) TB52. ( c ) TB102. ( c ) MX10. ( c ) MX60. ( d ) TO39. ( c )

TB3. ( a ) TB53. ( b ) TB103. ( d ) MX11. ( b ) MX61. ( d ) TO40. ( b )

TB4. ( b ) TB54. ( e ) TB104. ( e ) MX12. ( b ) MX62. ( c ) PA1. ( c )

TB5. ( b ) TB55. ( d ) TB105. ( e ) MX13. ( d ) MX63. ( c ) PA2. ( c )

TB6. ( c ) TB56. ( c ) TB106. ( e ) MX14. ( c ) MX64. ( d ) PA3. ( a )

TB7. ( b ) TB57. ( a ) TB107. ( a ) MX15. ( a ) MX65. ( d ) PA4. ( d )

TB8. ( a ) TB58. ( d ) TB108. ( b ) MX16. ( a ) MX66. ( e ) PA5. ( e )

TB9. ( c ) TB59. ( a ) TB109. ( d ) MX17. ( b ) MX67. ( e ) PA6. ( b )

TB10. ( a ) TB60. ( a ) TB110. ( d ) MX18. ( e ) MX68. ( c ) PA7. ( c )

TB11. ( b ) TB61. ( e ) TB111. ( d ) MX19. ( c ) MX69. ( a ) PA8. ( a )

TB12. ( c ) TB62. ( b ) TB112. ( d ) MX20. ( d ) MX70. ( d ) PA9. ( c )

TB13. ( b ) TB63. ( e ) TB113. ( c ) MX21. ( a ) MX71. ( c ) PA10. ( e )

TB14. ( d ) TB64. (c ) TB114. ( d ) MX22. ( d ) TO1. ( b ) PA11. ( b )

TB15. ( b ) TB65. ( a ) TB115. ( e ) MX23. ( b ) TO2. ( b ) PA12. ( c )

TB16. ( a ) TB66. ( e ) TB116. ( e ) MX24. ( b ) TO3. ( e ) PA13. ( c )

TB17. ( d ) TB67. ( a ) TB117. ( e ) MX25. ( b ) TO4. ( a ) PA14. ( d )

TB18. ( a ) TB68. ( b ) TB118. ( a ) MX26. ( a ) TO5. ( e ) PA15. ( d )

TB19. ( c ) TB69. ( b ) TB119. ( e ) MX27. ( b ) TO6. ( b ) PA16. ( e )

TB20. ( e ) TB70. ( a ) TB120. ( c ) MX28. ( a ) TO7. ( d ) PA17. ( b )

TB21. ( b ) TB71. ( a ) TB121. ( d ) MX29. ( d ) TO8. ( a ) PA18. ( a )

TB22. ( e ) TB72. ( e ) TB122. ( d ) MX30. ( c ) TO9. ( c ) PA19. ( c )

TB23. ( b ) TB73. ( d ) TB123. ( c ) MX31. ( d ) TO10. ( d ) PA20. ( c )

TB24. ( c ) TB74. ( a ) TB124. ( c ) MX32. ( c ) TO11. ( c ) PA21. ( c )

TB25. ( b ) TB75. ( b ) TB125. ( e ) MX33. ( e ) TO12. ( a ) PA22. ( b )


TB26. ( e ) TB76. ( d ) TB126. ( d ) MX34. ( b ) TO13. ( b ) PA23. ( b )

TB27. ( c ) TB77. ( d ) TB127. ( d ) MX35. ( c ) TO14. ( c ) PA24. ( b )

TB28. ( c ) TB78. ( b ) TB128. ( c ) MX36. ( a ) TO15. ( a ) PA25. ( c )

TB29. ( d ) TB79. ( d ) TB129. ( e ) MX37. ( b ) TO16. ( b ) PA26. ( a )

TB30. ( b ) TB80. ( d ) TB130. ( a ) MX38. ( c ) TO17. ( e ) PA27. ( a )

TB31. ( b ) TB81. ( e ) TB131. ( e ) MX39. ( c ) TO18. ( d ) PA28. ( e )

TB32. ( e ) TB82. ( d ) TB132. ( d ) MX40. ( a ) TO19. ( b ) PA29. ( c )

TB33. ( c ) TB83. ( c ) TB133. ( d ) MX41. ( d ) TO20. ( c ) PA30. ( a )

TB34. ( e ) TB84. ( e ) TB134. ( b ) MX42. ( d ) TO21. ( e ) PA31. ( e )

TB35. ( d ) TB85. ( b ) TB135. ( a ) MX43. ( c ) TO22. ( a ) PA32. ( c )

TB36. ( a ) TB86. ( c ) TB136. ( e ) MX44. ( e ) TO23. ( d ) PA33. ( e )

TB37. ( b ) TB87. ( e ) TB137. ( e ) MX45. ( e ) TO24. ( d ) PA34. ( a )

TB38. ( c ) TB88. ( d ) TB138. ( d ) MX46. ( a ) TO25. ( d ) PA35. ( c )

TB39. ( a ) TB89. ( e ) TB139. ( d ) MX47. ( e ) TO26. ( e ) PA36. ( e )

TB40. ( d ) TB90. ( e ) TB140. ( c ) MX48. ( e ) TO27. ( a ) PA37. ( b )

TB41. ( c ) TB91. ( b ) TB141. ( e ) MX49. ( a ) TO28. ( d ) PA38. ( c )

TB42. ( d ) TB92. ( b ) TB142. ( c ) MX50. ( e ) TO29. ( e ) PA39. ( a )

TB43. ( a ) TB93. ( d ) MX1. ( d ) MX51. ( e ) TO30. ( e ) PA40. ( a )

TB44. ( c ) TB94. ( c ) MX2. ( e ) MX52. ( b ) TO31. ( d ) PA41. ( a )

TB45. ( b ) TB95. ( b ) MX3. ( a ) MX53. ( c ) TO32. ( c ) PA42. ( e )

TB46. ( c ) TB96. ( a ) MX4. ( c ) MX54. ( b ) TO33. ( e ) PA43. ( d )

TB47. ( a ) TB97. ( c ) MX5. ( b ) MX55. ( b ) TO34. ( d ) PA44. ( d )

TB48. ( c ) TB98. ( e ) MX6. ( b ) MX56. ( b ) TO35. ( e ) PA45. ( b )

TB49. ( b ) TB99. ( b ) MX7. ( c ) MX57. ( d ) TO36. ( e ) PA46. ( a )

TB50. ( d ) TB100. ( a ) MX8. ( b ) MX58. ( a ) TO37. ( e ) PA47. ( b )


PA48. ( d ) IN4. ( d ) IN54. ( a ) IN104. ( a ) IN154. ( c ) IN204. ( b )

PA49. ( a ) IN5. ( b ) IN55. ( c ) IN105. ( b ) IN155. ( b ) IN205. ( b )

PA50. ( e ) IN6. ( b ) IN56. ( e ) IN106. ( a ) IN156. ( c ) IN206. ( a )

PA51. ( a ) IN7. ( c ) IN57. ( d ) IN107. ( c ) IN157. ( a ) IN207. ( a )

PA52. ( c ) IN8. ( d ) IN58. ( b ) IN108. ( c ) IN158. ( c ) IN208. ( c )

PA53. ( a ) IN9. ( d ) IN59. ( c ) IN109. ( a ) IN159. ( e ) IN209. ( a )

PA54. ( a ) IN10. ( e ) IN60. ( b ) IN110. ( a ) IN160. ( a ) IN210. ( b )

PA55. ( e ) IN11. ( c ) IN61. ( d ) IN111. ( d ) IN161. ( e ) IN211. ( c )

PA56. ( c ) IN12. ( c ) IN62. ( c ) IN112. ( b ) IN162. ( d ) IN212. ( e )

PA57. ( c ) IN13. ( a ) IN63. ( c ) IN113. ( a ) IN163. ( d ) IN213. ( b )

PA58. ( c ) IN14. ( b ) IN64. ( a ) IN114. ( a ) IN164. ( a ) IN214. ( c )

PA59. ( c ) IN15. ( e ) IN65. ( d ) IN115. ( d ) IN165. ( c ) IN215. ( b )

PA60. ( a ) IN16. ( e ) IN66. ( b ) IN116. ( a ) IN166. ( a ) IN216. ( c )

PA61. ( e ) IN17. ( c ) IN67. ( c ) IN117. ( b ) IN167. ( d ) IN217. ( d )

PA62. ( b ) IN18. ( a ) IN68. ( c ) IN118. ( e ) IN168. ( c ) IN218. ( c )

PA63. ( d ) IN19. ( e ) IN69. ( d ) IN119. ( d ) IN169. ( b ) IN219. ( c )

PA64. ( a ) IN20. ( e ) IN70. ( b ) IN120. ( b ) IN170. ( c ) IN220. ( e )

PA65. ( b ) IN21. ( c ) IN71. ( c ) IN121. ( d ) IN171. ( c ) IN221. ( b )

PA66. ( b ) IN22. ( d ) IN72. ( a ) IN122. ( c ) IN172. ( d ) IN222. ( d )

PA67. ( a ) IN23. ( b ) IN73. ( b ) IN123. ( e ) IN173. ( c ) IN223. ( c )

PA68. ( b ) IN24. ( c ) IN74. ( e ) IN124. ( b ) IN174. ( d ) IN224. ( c )

PA69. ( d ) IN25. ( e ) IN75. ( a ) IN125. ( a ) IN175. ( b ) IN225. ( d )

PA70. ( c ) IN26. ( d ) IN76. ( b ) IN126. ( c ) IN176. ( c ) IN226. ( c )

PA71. ( c ) IN27. ( a ) IN77. ( b ) IN127. ( b ) IN177. ( c ) IN227. ( a )


PA72. ( d ) IN28. ( c ) IN78. ( e ) IN128. ( a ) IN178. ( c ) IN228. ( b )

PA73. ( a ) IN29. ( c ) IN79. ( a ) IN129. ( a ) IN179. ( a ) IN229. ( e )

PA74. ( e ) IN30. ( c ) IN80. ( c ) IN130. ( d ) IN180. ( a ) IN230. ( b )

PA75. ( a ) IN31. ( d ) IN81. ( b ) IN131. ( a ) IN181. ( b ) IN231. ( b )

PA76. ( b ) IN32. ( e ) IN82. ( c ) IN132. ( d ) IN182. ( d ) IN232. ( b )

PA77. ( e ) IN33. ( e ) IN83. ( c ) IN133. ( a ) IN183. ( a ) IN233. ( c )

CR1. ( e ) IN34. ( c ) IN84. ( a ) IN134. ( c ) IN184. ( a ) IN234. ( c )

CR2. ( b ) IN35. ( d ) IN85. ( e ) IN135. ( a ) IN185. ( b ) IN235. ( d )

CR3. ( c ) IN36. ( a ) IN86. ( b ) IN136. ( d ) IN186. ( b ) IN236. ( e )

CR4. ( c ) IN37. ( b ) IN87. ( b ) IN137. ( a ) IN187. ( d ) IN237. ( c )

CR5. ( d ) IN38. ( e ) IN88. ( b ) IN138. ( b ) IN188. ( c ) IN238. ( b )

CR6. ( b ) IN39. ( b ) IN89. ( a ) IN139. ( a ) IN189. ( c ) IN239. ( b )

CR7. ( d ) IN40. ( e ) IN90. ( d ) IN140. ( b ) IN190. ( c ) IN240. ( c )

CR8. ( c ) IN41. ( e ) IN91. ( c ) IN141. ( c ) IN191. ( a ) IN241. ( d )

CR9. ( b ) IN42. ( d ) IN92. ( e ) IN142. ( a ) IN192. ( d ) IN242. ( e )

CR10. ( b ) IN43. ( e ) IN93. ( d ) IN143. ( e ) IN193. ( d ) IN243. ( d )

CR11. ( a ) IN44. ( c ) IN94. ( a ) IN144. ( d ) IN194. ( a ) IN244. ( e )

CR12. ( d ) IN45. ( d ) IN95. ( b ) IN145. ( c ) IN195. ( a ) IN245. ( b )

CR13. ( b ) IN46. ( b ) IN96. ( a ) IN146. ( d ) IN196. ( a ) IN246. ( c )

CR14. ( e ) IN47. ( a ) IN97. ( d ) IN147. ( b ) IN197. ( c ) IN247. ( c )

CR15. ( a ) IN48. ( a ) IN98. ( b ) IN148. ( c ) IN198. ( c ) IN248. ( d )

CR16. ( d ) IN49. ( a ) IN99. ( c ) IN149. ( e ) IN199. ( b ) IN249. ( d )

CR17. ( d ) IN50. ( a ) IN100. ( b ) IN150. ( a ) IN200. ( c ) IN250. ( c )

IN1. ( d ) IN51. ( b ) IN101. ( d ) IN151. ( d ) IN201. ( a ) IN251. ( d )

IN2. ( c ) IN52. ( b ) IN102. ( b ) IN152. ( b ) IN202. ( d ) IN252. ( b )

IN3. ( d ) IN53. ( b ) IN103. ( a ) IN153. ( d ) IN203. ( a ) IN253. ( b )


IN254. ( a ) IN304. ( c ) RO26. ( a ) OM42. ( e ) OM92. ( a ) OM142. ( a )

IN255. ( a ) IN305. ( f ) RO27. ( d ) OM43. ( c ) OM93. ( e ) OM143. ( c )

IN256. ( d ) IN306. ( b ) RO28. ( e ) OM44. ( c ) OM94. ( c ) OM144. ( e )

IN257. ( a ) IN307. ( c ) RO29. ( a ) OM45. ( d ) OM95. ( e ) OM145. ( e )

IN258. ( e ) IN308. ( a ) RO30. ( d ) OM46. ( d ) OM96. ( c ) OM146. ( e )

IN259. ( b ) IN309. ( a ) RO31. ( d ) OM47. ( c ) OM97. ( e ) OM147. ( b )

IN260. ( e ) IN310. ( a ) RO32. ( b ) OM48. ( a ) OM98. ( c ) OM148. ( c )

IN261. ( a ) IN311. ( e ) RO33. ( c ) OM49. ( b ) OM99. ( d ) OM149. ( b )

IN262. ( e ) IN312. ( e ) RO34. ( c ) OM50. ( e ) OM100. ( c ) OM150. ( b )

IN263. ( b ) IN313. ( a ) OM1. ( b ) OM51. ( d ) OM101. ( a ) OM151. ( b )

IN264. ( e ) IN314. ( a ) OM2. ( c ) OM52. ( e ) OM102. ( e ) OM152. ( b )

IN265. ( d ) IN315. ( e ) OM3. ( d ) OM53. ( b ) OM103. ( d ) OM153. ( b )

IN266. ( c ) IN316. ( c ) OM4. ( b ) OM54. ( a ) OM104. ( a ) OM154. ( d )

IN267. ( e ) IN317. ( e ) OM5. ( b ) OM55. ( a ) OM105. ( d ) OM155. ( d )

IN268. ( c ) IN318. ( a ) OM6. ( d ) OM56. ( b ) OM106. ( d ) OM156. ( c )

IN269. ( b ) IN319. ( a ) OM7. ( a ) OM57. ( a ) OM107. ( a ) OM157. ( d )

IN270. ( c ) IN320. ( a ) OM8. ( c ) OM58. ( a ) OM108. ( c ) OM158. ( d )

IN271. ( d ) IN321. ( c ) OM9. ( c ) OM59. ( c ) OM109. ( a ) OM159. ( e )

IN272. ( e ) IN322. ( d ) OM10. ( a ) OM60. ( e ) OM110. ( b ) OM160. ( f )

IN273. ( e ) IN323. ( b ) OM11. ( c ) OM61. ( b ) OM111. ( a ) OM161. ( b )

IN274. ( e ) IN324. ( a ) OM12. ( b ) OM62. ( c ) OM112. ( c ) OM162. ( a )

IN275. ( e ) IN325. ( a ) OM13. ( b ) OM63. ( c ) OM113. ( c ) OM163. ( c )

IN276. ( b ) IN326. ( a ) OM14. ( a ) OM64. ( b ) OM114. ( d ) OM164. ( e )


IN277. ( e ) IN327. ( e ) OM15. ( c ) OM65. ( d ) OM115. ( a ) OM165. ( c )

IN278. ( c ) IN328. ( d ) OM16. ( c ) OM66. ( a ) OM116. ( d ) OM166. ( e )

IN279. ( b ) RO1. ( d ) OM17. ( c ) OM67. ( b ) OM117. ( a ) OM167. ( c )

IN280. ( e ) RO2. ( c ) OM18. ( a ) OM68. ( c ) OM118. ( c ) OM168. ( c )

IN281. ( d ) RO3. ( e ) OM19. ( c ) OM69. ( c ) OM119. ( b ) OM169. ( c )

IN282. ( a ) RO4. ( b ) OM20. ( e ) OM70. ( e ) OM120. ( e ) OM170. ( a )

IN283. ( b ) RO5. ( e ) OM21. ( c ) OM71. ( d ) OM121. ( a ) OM171. ( b )

IN284. ( a ) RO6. ( e ) OM22. ( a ) OM72. ( c ) OM122. ( d ) OM172. ( d )

IN285. ( d ) RO7. ( c ) OM23. ( b ) OM73. ( d ) OM123. ( e ) OM173. ( b )

IN286. ( e ) RO8. ( a ) OM24. ( c ) OM74. ( a ) OM124. ( b ) OM174. ( c )

IN287. ( c ) RO9. ( d ) OM25. ( c ) OM75. ( b ) OM125. ( e ) OM175. ( d )

IN288. ( c ) RO10. ( d ) OM26. ( d ) OM76. ( b ) OM126. ( c ) OM176. ( b )

IN289. ( c ) RO11. ( a ) OM27. ( c ) OM77. ( c ) OM127. ( c ) OM177. ( c )

IN290. ( d ) RO12. ( a ) OM28. ( a ) OM78. ( c ) OM128. ( d ) OM178. ( c )

IN291. ( b ) RO13. ( d ) OM29. ( b ) OM79. ( a ) OM129. ( d ) OM179. ( c )

IN292. ( c ) RO14. ( d ) OM30. ( e ) OM80. ( e ) OM130. ( d ) OM180. ( a )

IN293. ( d ) RO15. ( a ) OM31. ( d ) OM81. ( e ) OM131. ( b ) OM181. ( a )

IN294. ( b ) RO16. ( c ) OM32. ( a ) OM82. ( b ) OM132. ( e ) OM182. ( e )

IN295. ( a ) RO17. ( b ) OM33. ( e ) OM83. ( a ) OM133. ( f ) OM183. ( d )

IN296. ( c ) RO18. ( d ) OM34. ( c ) OM84. ( c ) OM134. ( a ) OM184. ( e )

IN297. ( e ) RO19. ( b ) OM35. ( e ) OM85. ( d ) OM135. ( c ) OM185. ( d )

IN298. ( d ) RO20. ( b ) OM36. ( a ) OM86. ( c ) OM136. ( e ) OM186. ( c )

IN299. ( b ) RO21. ( c ) OM37. ( a ) OM87. ( e ) OM137. ( d ) OM187. ( c )

IN300. ( b ) RO22. ( d ) OM38. ( d ) OM88. ( d ) OM138. ( c ) OM188. ( c )

IN301. ( b ) RO23. ( a ) OM39. ( a ) OM89. ( a ) OM139. ( d ) OM189. ( a )

IN302. ( b ) RO24. ( b ) OM40. ( c ) OM90. ( b ) OM140. ( c ) OM190. ( e )

IN303. ( d ) RO25. ( e ) OM41. ( e ) OM91. ( e ) OM141. ( e ) OM191. ( d )


OM192. ( c ) OM242. ( c ) OM292. ( b ) CQ10-2. ( c ) CQ20-2. ( a ) CQ30-2. ( a )

OM193. ( f ) OM243. ( c ) OM293. ( e ) CQ10-3. ( b ) CQ20-3. ( b ) CQ30-3. ( c )

OM194. ( e ) OM244. ( d ) OM294. ( d ) CQ10-4. ( a ) CQ20-4. ( b ) CQ30-4. ( d )

OM195. ( g ) OM245. ( e ) OM295. ( e ) CQ10-5. ( d ) CQ20-5. ( a ) CQ30-5. ( d )

OM196. ( h ) OM246. ( c ) OM296. ( d ) CQ11-1. ( c ) CQ21-1. ( c ) CQ31-1. ( a )

OM197. ( i ) OM247. ( d ) OM297. ( a ) CQ11-2. ( c ) CQ21-2. ( a ) CQ31-2. ( b )

OM198. ( j ) OM248. ( c ) CQ1-1. ( d ) CQ11-3. ( b ) CQ21-3. ( b ) CQ31-3. ( e )

OM199. ( d ) OM249. ( c ) CQ1-2. ( d ) CQ11-4. ( b ) CQ21-4. ( a ) CQ31-4. ( c )

OM200. ( b ) OM250. ( d ) CQ1-3. ( c ) CQ11-5. ( ) CQ21-5. ( c ) CQ31-5. ( b )

OM201. ( a ) OM251. ( c ) CQ1-4. ( d ) CQ12-1. ( b ) CQ22-1. ( d ) CQ32-1. ( d )

OM202. ( c ) OM252. ( a ) CQ1-5. ( c ) CQ12-2. ( e ) CQ22-2. ( a ) CQ32-2. ( a )

OM203. ( d ) OM253. ( a ) CQ2-1. ( b ) CQ12-3. ( d ) CQ22-3. ( d ) CQ32-3. ( c )

OM204. ( a ) OM254. ( b ) CQ2-2. ( d ) CQ12-4. ( a ) CQ22-4. ( e ) CQ32-4. ( d )

OM205. ( b ) OM255. ( d ) CQ2-3. ( d ) CQ12-5. ( ) CQ22-5. ( b ) CQ32-5. ( b )

OM206. ( d ) OM256. ( a ) CQ2-4. ( b ) CQ13-1. ( b ) CQ23-1. ( e ) CQ33-1. ( d )

OM207. ( d ) OM257. ( e ) CQ2-5. ( b ) CQ13-2. ( a ) CQ23-2. ( b ) CQ33-2. ( b )

OM208. ( b ) OM258. ( a ) CQ3-1. ( c ) CQ13-3. ( d ) CQ23-3. ( e ) CQ33-3. ( c )

OM209. ( e ) OM259. ( d ) CQ3-2. ( b ) CQ13-4. ( a ) CQ23-4. ( d ) CQ33-4. ( b )

OM210. ( d ) OM260. ( c ) CQ3-3. ( a ) CQ13-5. ( e ) CQ23-5. ( a ) CQ33-5. ( c )

OM211. ( a ) OM261. ( b ) CQ3-4. ( c ) CQ14-1. ( b ) CQ24-1. ( c ) CQ34-1. ( d )

OM212. ( a ) OM262. ( d ) CQ3-5. ( b ) CQ14-2. ( a ) CQ24-2. ( d ) CQ34-2. ( d )

OM213. ( b ) OM263. ( d ) CQ4-1. ( e ) CQ14-3. ( d ) CQ24-3. ( e ) CQ34-3. ( b )


OM214. ( c ) OM264. ( a ) CQ4-2. ( b ) CQ14-4. ( d ) CQ24-4. ( a ) CQ34-4. ( c )

OM215. ( c ) OM265. ( d ) CQ4-3. ( b ) CQ14-5. ( d ) CQ24-5. ( c ) CQ34-5. ( e )

OM216. ( c ) OM266. ( a ) CQ4-4. ( a ) CQ15-1. ( e ) CQ25-1. ( b ) CQ35-1. ( c )

OM217. ( c ) OM267. ( a ) CQ4-5. ( c ) CQ15-2. ( d ) CQ25-2. ( d ) CQ35-2. ( a )

OM218. ( e ) OM268. ( a ) CQ5-1. ( a ) CQ15-3. ( d ) CQ25-3. ( b ) CQ35-3. ( b )

OM219. ( b ) OM269. ( b ) CQ5-2. ( c ) CQ15-4. ( a ) CQ25-4. ( a ) CQ35-4. ( c )

OM220. ( c ) OM270. ( c ) CQ5-3. ( b ) CQ15-5. ( c ) CQ25-5. ( c ) CQ35-5. ( d )

OM221. ( a ) OM271. ( e ) CQ5-4. ( d ) CQ16-1. ( b ) CQ26-1. ( e ) CQ36-1. ( a )

OM222. ( d ) OM272. ( b ) CQ5-5. ( a ) CQ16-2. ( b ) CQ26-2. ( a ) CQ36-2. ( b )

OM223. ( e ) OM273. ( b ) CQ6-1. ( e ) CQ16-3. ( b ) CQ26-3. ( a ) CQ36-3. ( e )

OM224. ( c ) OM274. ( d ) CQ6-2. ( b ) CQ16-4. ( b ) CQ26-4. ( d ) CQ36-4. ( b )

OM225. ( d ) OM275. ( b ) CQ6-3. ( d ) CQ16-5. ( c ) CQ26-5. ( b ) CQ36-5. ( a )

OM226. ( e ) OM276. ( d ) CQ6-4. ( a ) CQ17-1. ( c ) CQ27-1. ( b ) CQ37-1. ( e )

OM227. ( b ) OM277. ( b ) CQ6-5. ( b ) CQ17-2. ( b ) CQ27-2. ( c ) CQ37-2. ( c )

OM228. ( b ) OM278. ( e ) CQ7-1. ( b ) CQ17-3. ( c ) CQ27-3. ( e ) CQ37-3. ( d )

OM229. ( a ) OM279. ( d ) CQ7-2. ( c ) CQ17-4. ( c ) CQ27-4. ( a ) CQ37-4. ( b )

OM230. ( b ) OM280. ( b ) CQ7-3. ( c ) CQ17-5. ( b ) CQ27-5. ( c ) CQ37-5. ( e )

OM231. ( a ) OM281. ( d ) CQ7-4. ( d ) CQ18-1. ( a ) CQ28-1. ( c ) CQ38-1. ( b )

OM232. ( d ) OM282. ( c ) CQ8-1. ( c ) CQ18-2. ( b ) CQ28-2. ( d ) CQ38-2. ( c )

OM233. ( d ) OM283. ( c ) CQ8-2. ( a ) CQ18-3. ( d ) CQ28-3. ( c ) CQ38-3. ( b )

OM234. ( b ) OM284. ( e ) CQ8-3. ( b ) CQ18-4. ( b ) CQ28-4. ( b ) CQ38-4. ( e )

OM235. ( c ) OM285. ( d ) CQ8-4. ( c ) CQ18-5. ( b ) CQ28-5. ( d ) CQ38-5. ( a )

OM236. ( c ) OM286. ( d ) CQ9-1. ( c ) CQ19-1. ( c ) CQ29-1. ( e ) CQ39-1. ( a )

OM237. ( b ) OM287. ( c ) CQ9-2. ( c ) CQ19-2. ( a ) CQ29-2. ( b ) CQ39-2. ( b )

OM238. ( a ) OM288. ( a ) CQ9-3. ( b ) CQ19-3. ( c ) CQ29-3. ( b ) CQ39-3. ( c )

OM239. ( e ) OM289. ( d ) CQ9-4. ( c ) CQ19-4. ( c ) CQ29-4. ( e ) CQ39-4. ( d )

OM240. ( c ) OM290. ( c ) CQ9-5. ( c ) CQ19-5. ( b ) CQ29-5. ( b ) CQ39-5. ( c )


OM241. ( c ) OM291. ( a ) CQ10-1. ( a ) CQ20-1. ( a ) CQ30-1. ( d ) CQ40-1. ( b )

CQ40-2. ( a ) CQ50-3. ( b ) CQ60-1. ( d ) CQ70-3. ( c ) CQ80-3. ( a ) CQ90-3. ( b )

CQ40-3. ( c ) CQ50-4. ( c ) CQ60-2. ( b ) CQ70-4. ( d ) CQ80-4. ( a ) CQ90-4. ( b )

CQ40-4. ( a ) CQ50-5. ( c ) CQ60-3. ( c ) CQ70-5. ( e ) CQ80-5. ( b ) CQ90-5. ( d )

CQ40-5. ( e ) CQ51-1. ( c ) CQ60-4. ( a ) CQ71-1. ( d ) CQ81-1. ( c ) CQ91-1. ( c )

CQ41-1. ( e ) CQ51-2. ( d ) CQ60-5. ( a ) CQ71-2. ( b ) CQ81-2. ( a ) CQ91-2. ( e )

CQ41-2. ( a ) CQ51-3. ( d ) CQ61-1. ( c ) CQ71-3. ( b ) CQ81-3. ( a ) CQ91-3. ( d )

CQ41-3. ( c ) CQ51-4. ( c ) CQ61-2. ( b ) CQ71-4. ( e ) CQ81-4. ( a ) CQ91-4. ( b )

CQ41-4. ( c ) CQ51-5. ( a ) CQ61-3. ( a ) CQ71-5. ( e ) CQ81-5. ( b ) CQ91-5. ( a )

CQ41-5. ( b ) CQ52-1. ( d ) CQ61-4. ( e ) CQ72-1. ( a ) CQ82-1. ( d ) CQ92-1. ( c )

CQ42-1. ( d ) CQ52-2. ( c ) CQ61-5. ( d ) CQ72-2. ( c ) CQ82-2. ( b ) CQ92-2. ( c )

CQ42-2. ( d ) CQ52-3. ( c ) CQ62-1. ( e ) CQ72-3. ( d ) CQ82-3. ( d ) CQ92-3. ( c )

CQ42-3. ( a ) CQ52-4. ( d ) CQ62-2. ( e ) CQ72-4. ( d ) CQ82-4. ( b ) CQ92-4. ( b )

CQ42-4. ( a ) CQ52-5. ( d ) CQ62-3. ( c ) CQ72-5. ( d ) CQ82-5. ( a ) CQ92-5. ( e )

CQ42-5. ( a ) CQ53-1. ( e ) CQ62-4. ( c ) CQ73-1. ( a ) CQ83-1. ( a ) CQ93-1. ( a )

CQ43-1. ( b ) CQ53-2. ( b ) CQ62-5. ( b ) CQ73-2. ( d ) CQ83-2. ( a ) CQ93-2. ( a )

CQ43-2. ( d ) CQ53-3. ( c ) CQ63-1. ( c ) CQ73-3. ( a ) CQ83-3. ( d ) CQ93-3. ( d )

CQ43-3. ( a ) CQ53-4. ( c ) CQ63-2. ( b ) CQ73-4. ( c ) CQ83-4. ( d ) CQ93-4. ( a )

CQ43-4. ( b ) CQ53-5. ( e ) CQ63-3. ( a ) CQ73-5. ( e ) CQ83-5. ( c ) CQ93-5. ( a )

CQ43-5. ( c ) CQ54-1. ( e ) CQ64-1. ( c ) CQ74-1. ( d ) CQ84-1. ( e ) CQ94-1. ( c )

CQ44-1. ( d ) CQ54-2. ( e ) CQ64-2. ( d ) CQ74-2. ( b ) CQ84-2. ( b ) CQ94-2. ( d )

CQ44-2. ( d ) CQ54-3. ( c ) CQ64-3. ( b ) CQ74-3. ( b ) CQ84-3. ( a ) CQ94-3. ( c )


CQ44-3. ( e ) CQ54-4. ( b ) CQ64-4. ( a ) CQ74-4. ( c ) CQ84-4. ( b ) CQ94-4. ( a )

CQ44-4. ( d ) CQ54-5. ( d ) CQ64-5. ( b ) CQ74-5. ( c ) CQ84-5. ( e ) CQ94-5. ( a )

CQ45-1. ( e ) CQ54-6. ( e ) CQ65-1. ( d ) CQ75-1. ( c ) CQ85-1. ( b ) CQ95-1. ( c )

CQ45-2. ( c ) CQ55-1. ( d ) CQ65-2. ( c ) CQ75-2. ( b ) CQ85-2. ( b ) CQ95-2. ( e )

CQ45-3. ( a ) CQ55-2. ( d ) CQ65-3. ( b ) CQ75-3. ( d ) CQ85-3. ( a ) CQ95-3. ( a )

CQ45-4. ( d ) CQ55-3. ( c ) CQ65-4. ( d ) CQ75-4. ( b ) CQ85-4. ( e ) CQ95-4. ( c )

CQ45-5. ( b ) CQ55-4. ( d ) CQ65-5. ( b ) CQ75-5. ( e ) CQ85-5. ( c ) CQ95-5. ( e )

CQ46-1. ( c ) CQ55-5. ( d ) CQ66-1. ( a ) CQ76-1. ( a ) CQ86-1. ( d ) CQ96-1. ( d )

CQ46-2. ( d ) CQ55-6. ( d ) CQ66-2. ( b ) CQ76-2. ( c ) CQ86-2. ( a ) CQ96-2. ( e )

CQ46-3. ( d ) CQ56-1. ( d ) CQ66-3. ( a ) CQ76-3. ( a ) CQ86-3. ( a ) CQ96-3. ( d )

CQ46-4. ( d ) CQ56-2. ( c ) CQ66-4. ( b ) CQ76-4. ( d ) CQ86-4. ( c ) CQ96-4. ( e )

CQ46-5. ( d ) CQ56-3. ( a ) CQ66-5. ( d ) CQ76-5. ( c ) CQ86-5. ( c ) CQ96-5. ( d )

CQ47-1. ( d ) CQ56-4. ( c ) CQ67-1. ( b ) CQ77-1. ( a ) CQ87-1. ( c ) CQ97-1. ( b )

CQ47-2. ( c ) CQ56-5. ( e ) CQ67-2. ( b ) CQ77-2. ( a ) CQ87-2. ( a ) CQ97-2. ( a )

CQ47-3. ( a ) CQ57-1. ( e ) CQ67-3. ( a ) CQ77-3. ( c ) CQ87-3. ( b ) CQ97-3. ( e )

CQ47-4. ( c ) CQ57-2. ( e ) CQ67-4. ( c ) CQ77-4. ( e ) CQ87-4. ( a ) CQ97-4. ( e )

CQ47-5. ( d ) CQ57-3. ( b ) CQ67-5. ( a ) CQ77-5. ( d ) CQ87-5. ( c ) CQ97-5. ( a )

CQ48-1. ( d ) CQ57-4. ( d ) CQ68-1. ( a ) CQ78-1. ( b ) CQ88-1. ( b ) CQ98-1. ( d )

CQ48-2. ( c ) CQ57-5. ( a ) CQ68-2. ( a ) CQ78-2. ( b ) CQ88-2. ( b ) CQ98-2. ( a )

CQ48-3. ( b ) CQ58-1. ( a ) CQ68-3. ( d ) CQ78-3. ( c ) CQ88-3. ( a ) CQ98-3. ( c )

CQ48-4. ( c ) CQ58-2. ( d ) CQ68-4. ( c ) CQ78-4. ( b ) CQ88-4. ( e ) CQ98-4. ( b )

CQ48-5. ( c ) CQ58-3. ( b ) CQ68-5. ( a ) CQ78-5. ( a ) CQ88-5. ( a ) CQ98-5. ( e )

CQ49-1. ( d ) CQ58-4. ( b ) CQ69-1. ( c ) CQ79-1. ( d ) CQ89-1. ( b ) CQ99-1. ( b )

CQ49-2. ( d ) CQ58-5. ( c ) CQ69-2. ( e ) CQ79-2. ( b ) CQ89-2. ( c ) CQ99-2. ( b )

CQ49-3. ( d ) CQ59-1. ( d ) CQ69-3. ( d ) CQ79-3. ( d ) CQ89-3. ( a ) CQ99-3. ( c )

CQ49-4. ( d ) CQ59-2. ( d ) CQ69-4. ( d ) CQ79-4. ( e ) CQ89-4. ( b ) CQ99-4. ( d )

CQ49-5. ( c ) CQ59-3. ( b ) CQ69-5. ( d ) CQ79-5. ( d ) CQ89-5. ( ) CQ99-5. ( c )


CQ50-1. ( d ) CQ59-4. ( a ) CQ70-1. ( a ) CQ80-1. ( a ) CQ90-1. ( c ) CQ100-1. ( d )

CQ50-2. ( e ) CQ59-5. ( b ) CQ70-2. ( c ) CQ80-2. ( b ) CQ90-2. ( c ) CQ100-2. ( d )

CQ100-3. ( e ) CQ13-1. ( c )

CQ100-4. ( c ) CQ13-2. ( a )

CQ100-5. ( b ) CQ13-3. ( a )

CQ101-1. ( a ) CQ13-4. ( a )

CQ101-2. ( c ) CQ14-1. ( b )

CQ101-3. ( c ) CQ14-2. ( b )

CQ101-4. ( b ) CQ14-3. ( d )

CQ101-5. ( b ) CM1-1. ( d )

CQ102-1. ( c ) CM1-2. ( d )

CQ102-2. ( b ) CM1-3. ( c )

CQ102-3. ( d ) CM1-4. ( d )

CQ102-4. ( ) CM1-5. ( e )

CQ102-5. ( d ) CM2-1. ( a )

CQ103-1. ( b ) CM2-2. ( c )

CQ103-2. ( c ) CM2-3. ( e )

CQ103-3. ( a ) CM2-4. ( b )

CQ103-4. ( e ) CM2-5. ( d )

CQ103-5. ( e ) CM3-1. ( b )

CQ104-1. ( c ) CM3-2. ( a )

CQ104-2. ( e ) CM3-3. ( d )
CQ104-3. ( b ) CM3-4. ( c )

CQ104-4. ( a ) CM3-5. ( b )

CQ104-5. ( a ) CM4-1. ( b )

CQ105-1. ( a ) CM4-2. ( d )

CQ105-2. ( b ) CM4-3. ( c )

CQ105-3. ( e ) CM4-4. ( b )

CQ105-4. ( a ) CM4-5. ( d )

CQ106-1. ( a ) CM5-1. ( d )

CQ106-2. ( b ) CM5-2. ( b )

CQ107-1. ( b ) CM5-3. ( d )

CQ107-2. ( b ) CM5-4. ( e )

CQ108-1. ( b ) CM5-5. ( a )

CQ108-2. ( b ) CM6-1. ( a )

CQ108-3. ( a ) CM6-2. ( c )

CQ109-1. ( d ) CM6-3. ( b )

CQ109-2. ( e ) CM6-4. ( b )

CQ109-3. ( c ) CM6-5. ( a )

CQ110-1. ( e ) CM7-1. ( a )

CQ110-2. ( c ) CM7-2. ( d )

CQ110-3. ( a ) CM7-3. ( b )

CQ110-4. ( c ) CM7-4. ( c )

CQ110-5. ( b ) CM7-5. ( d )

CQ111-1. ( c ) CP1-1. ( c )

CQ111-2. ( c ) CP1-2. ( d )

CQ111-3. ( a ) CP1-3. ( a )

CQ111-4. ( e ) CP1-4. ( b )

CQ112-1. ( b ) CP1-5. ( d )
CQ112-2. ( d ) CP2-1. ( c )

CQ112-3. ( a ) CP2-2. ( c )

CQ112-4. ( a ) CP2-3. ( a )

Vous aimerez peut-être aussi